You are on page 1of 402

Visit For More : www.LearnEngineering.

in

1 Differential calculus

.in
1.1 Representation of a function.

ng
Definition. A function f is a rule that assigns to each element x in a set D exactly

eri
one element, called f (x), in a set E.
The set D is called the domain of the function. The number f (x) is the value of

e
f at x. The range of f is the set of all possible values of f (x) as x varies throughout
gin
the domain. A symbol that represents an arbitrary number in the domain of a
function f is called an independent variable. A symbol that represents a number
in the range of f is called a dependent variable.
En

Example. The area A of a circle depends on the radius r of the circle. The rule
that connects r and A is given by the equation A = πr2 . With each positive number
arn

r, there is associated one value of A and we say that A is a function of r. Here, r is


the independent variable and A is the dependent variable.
We can think of a function as a machine. If x is in the domain of the function
Le

f , then when x enters the machine, it is accepted as an input and the machine
produces an output f (x) according to the rule of the function. Thus, we can think
w.

of the domain as the set of all possible inputs and the range as the set of all possible
outputs.
input x → f → f (x) output
ww

Another way to represent a function is by an Arrow diagram. Each arrow


connects an element of D to an element of E. The arrow indicates that f (x) is

Visit For More : www.LearnEngineering.in


Visit For More : www.LearnEngineering.in

2 Engineering Mathematics - I

associated with x, f (a) is associated with a and so on.

x f (x)
a f (a)
y f (y)

.in
The most common method for visualizing a function is its graph. If f is a

ng
function with domain D, then its graph is the set of ordered pairs {(x, f (x))/x ∈ D}.
In other words, the graph of f consists of all points (x, y) in the coordinate plane

eri
such that y = f (x), and x is in the domain of f .
Y
Y
(x, f (x))
e
gin
range y = f (x)
f (x)
f (2)
f (1) X
En

X 0 Domain
0 1 2 x
arn

The graph of a function f gives us a useful picture of the behavior of a function.


Since the y−coordinate of any point (x, y) on the graph is y = f (x), we can read the
value of f (x) from the graph as being the height of the graph above the point x. The
Le

graph of f also allows us to picture the domain of f on the x−axis and its range on
the y−axis.
w.

Example 1.1. Sketch the graph of f (x) = 2x + 1 and find the domain and range.
Solution.
ww

Y
−1
x 0
2 1
y 1 0
X
−1/2

y = 2x + 1

Visit For More : www.LearnEngineering.in


Visit For More : www.LearnEngineering.in

Differential calculus 3

The equation of the graph is y = 2x + 1. This represents a line with slope 2 and
y−intercept 1. With these informations, we can sketch the portion of the graph of
f . The expression 2x + 1 is defined for all real numbers, and hence the domain of
f is the set of all real numbers, which we denote by R. The graph shows that the
range is also R.

.in
Example 1.2. Sketch the graph of g(x) = x2 and find the domain and range.
Solution. The equation of the graph is y = x2 .

ng
Y

eri
x −2 −1 0 1 2
8
y 4 1 0 1 4
6

e 4
gin
2
y = x2
X
−2 −1 0 1 2
En

From the tabular column we can plot the points (−2, 4), (−1, 1), (0, 0), (1, 1) and (2, 4)
and join them to produce the graph, which represents a parabola. The domain of
arn

g is R. The range of g consists of all values of g(x), that is, all numbers of the form
x2 . But x2 ≥ 0 for all numbers x and any positive number y is a square. Hence, the
range of g is {y : y ≥ 0} = [0, ∞].
Le

The graph of a function is a curve in the xy−plane. But whether all curves in
the xy−plane represent the graph of some function. This can be answered by the
w.

vertical line test.


The vertical line test. A curve in the xy−plane is the graph of a function of
ww

x if and only if no vertical line intersects the curve more than once.
Consider the following graphs.

Visit For More : www.LearnEngineering.in


Visit For More : www.LearnEngineering.in

4 Engineering Mathematics - I

Y Y
(a, c)
x=a x=a

(a, b)

.in
(a, b)

X X
a 0 a
0

ng
In the first graph, each vertical line x = a intersects the curve only once, at

eri
(a, b), then exactly one function value is defined by f (a) = b. But in the second
graph, the line x = a intersects the curve twice at (a, b) and (a, c), that is, the

e
function assigns two different values to a. Hence, the first curve represents the
gin
graph of a function while the second graph is not.
As an illustration consider the parabola x = y2 − 4. The curve that represents
this equation is give below.
En

This curve does not represent the


Y
graph of a function of x because, we can
arn

see that every vertical line intersects


x = y2 − 4
the parabola twice. This parabola is the
union of the graphs of two functions of
Le

(−4, 0)

0 X x. Rewriting the equation as y2 = x + 4



we get y = ± x + 2.
w.
ww

Hence, the upper and lower halves of the parabola are the graphs of the
√ √
functions f (x) = x + 4 and g(x) = − x + 4. The separate graphs are given below.

Visit For More : www.LearnEngineering.in


Visit For More : www.LearnEngineering.in

Differential calculus 5


Y
Y y= x+4

(−4, 0)
(−4, 0) 0 X
0 X

.in
y=− x+4

ng
If we reverse the roles of x and y, then we get the equation x = h(y) = y2 − 4
definitely define x as a function of y with y as the independent variables and x as

eri
the dependent variable and in this case, the parabola represent the graph of the
function h.

Piecewise difined functions. e


gin

1 − x , if x ≤ −1



Example 1.3. Consider the function f defined by f (x) =  .


 x2

 , if x > −1
En

Y
For all x ≤ −1.
arn

6
x -3 -2 -1
5
y=1−x 4 3 2
4
Le

For all x > −1. 3


x -1 0 1 2 3 2
w.

y = x2 1 0 1 4 9 1

The combined graph of the function f is X


−4 −3 −2 −1 0 1 2 3 4
ww

as follows

Notice that the point (−1, 1) in y = x2 is not included and there is a discontinuity
between the graph for all x ≤ 1 (i.e.y = 1 − x) and the graph for all x > 1 (i.e. y = x2 ).

Visit For More : www.LearnEngineering.in


Visit For More : www.LearnEngineering.in

6 Engineering Mathematics - I

Example 1.4. Consider the absolute value function f (x) = |x|, which is defined as

x , if x ≥ 0



follows f (x) =  . The graph of f (x) consists of two branches namely


−x , if x < 0


y = x for all x ≥ 0 and y = −x for all x < 0.

.in
The combined graph of f is given below.
y = x (x ≥ 0)

ng
x 0 1 2 3 Y
y 0 1 2 3
3 y = |x|

eri
2
y = −x (x < 0) 1
x -3 -2 -1 0 X
y 3 2 1 0
e −4 −3 −2 −1 0 1 2 3 4
gin
Notice that the point (0, 0) is not included in the graph of y = −x (x < 0).

En






 x , if 0 ≤ x ≤ 1



Example 1.5. Consider the function f (x) =  , if 1 < x ≤ 2 .


2−x



arn


0 , if x > 2

y = 2 − x (1 ≤ x ≤ 2) y = 0 (x > 2)
y = x (0 ≤ x ≤ 1) x 1 2 x 2 3 4 5
Le

x 0 1
y 1 0 y 0 0 0 0
y 0 1
w.

The graph of f is given below.


ww

1
X
0 1 2 3 4 5

Visit For More : www.LearnEngineering.in


Visit For More : www.LearnEngineering.in

Differential calculus 7

Example 1.6. Consider the greatest integer function defined by f (x) = [x].
By definition [x]= Largest integer that is less than or equal to x
= max{m ∈ Z; m ≤ x}.
Example. [2.7] = 2, [−1.3] = −2, [4] = 4

.in
The graph of the function is as follows

The open dots are not included at the

ng
Y
corresponding points.
5 The Examples 1.3 to 1.6 indicate that

eri
4
there is jump from one value to the
3
2 next. Hence, the above functions are
1
ecalled piecewise functions. The function
gin
X discussed in Example 1.6 is called step
−2 −1 0 1 2 3 4 5
−1 function.
−2
En

symmetry
Even function. If a function f satisfies f (−x) = f (x) for every number x in its
arn

domain, then f is called an even function.


Le

Example. f (x) = x2 is even, Y


since f (−x) = (−x)2 = x2 = f (x).
Geometrically, tha graph of an even
w.

function is symmetric about the y−axis.


f (−x) f (x)
Consider the curve of y = x2 − 4.
ww

0 X
−x x

Visit For More : www.LearnEngineering.in


Visit For More : www.LearnEngineering.in

8 Engineering Mathematics - I

Odd function. If f statifies f (−x) = − f (x) for every number x in the domain, then
f is called an odd function.
Example. Consider f (x) = x3
f (−x) = (−x)3 = −x3 = − f (x).
∴ f (x) = x3 is an odd function.

.in
Y

f (x)
−x

ng
X
0 x
− f (x)

eri
The graph of the odd function is symmetric about the origin.

Example 1.7. Determine the nature of the following functions.


(i) f (x) = x5 − x. (ii) g(x) = 1 + x4 . e (iii) h(x) = x + x2 .
gin
Solution.
(i) f (x) = x5 − x.
En

f (−x) = (−x)5 − (−x) = −x5 + x = −(x5 − x) = − f (x).


∴ f (x) is odd.
(ii) g(x) = 1 + x4 .
arn

g(−x) = 1 + (−x)4 = 1 + x4 = g(x).


∴ g(x) is even.
Le

(iii) h(x) = x + x2 .
h(−x) = (−x) + (−x)2 = −x + x2 .
h(−x) , h(x) and h(−x) , −h(x)
w.

∴ h(x) is neither even nor odd.


ww

Increasing and decreasing functions. A function f is called increasing on an


interval I, if f (x1 ) < f (x2 ) whenever x1 < x2 in I.
It is called decreasing on I, if f (x1 ) > f (x2 ) whenever x1 < x2 in I.

Visit For More : www.LearnEngineering.in


Visit For More : www.LearnEngineering.in

Differential calculus 9

1.2 Limit of a function

Definition. Intuitive definition of a limit. Suppose f (x) is defined. When x is


near the number a, then we write lim f (x) = L. This means that the limit of f (x),
x→a
as x approaches a equals L.

.in
lim f (x) = L can also be represented as f (x) → L as x → a.
x→a
One sided limits. The left hand limit of f (x) as x approaches a is written as
lim f (x) = L which means that the limit of f (x) as x approaches a from the left

ng
x→a−
is equal to L if we can make the values of f (x) arbitrarily close to L by taking x
sufficiently close to a with x less than a.

eri
Similarly, if we require that x greater than a, we get the right hand limit of
f (x) as x approaches a is equal to L and we write lim f (x) = L.

e x→a+

Result. lim f (x) = L if and only if lim f (x) = L and lim f (x) = L.
gin
x→a x→a− x→a+

Infinite limits.
(i) Let f ba a function defined on both sides of a, except possibly at a itself. Then
En

lim f (x) = ∞ means that the value of f (x) can be made arbitrarily large by taking x
x→a
sufficiently close to a, but not equal to a.
arn

(ii) Let f be a function defined on both sides of a, expect possibly at a itself. Then
lim f (x) = −∞ means that the values of f (x) can be made arbitrarily large negative
x→a
by taking x sufficiently close to a, but not equal to a.
Le

1
Example. lim 2 = ∞.
x→0 x !
1
lim − 2 = −∞.
w.

x→0 x
Asymptote. The vertical line x = a is called a vertical asymptote of the curve
y = f (x) if atleast one of the following statements is true:
ww

lim f (x) = ∞, lim f (x) = ∞, lim f (x) = ∞.


x→a x→a− x→a+
lim f (x) = −∞, lim f (x) = −∞, lim f (x) = −∞.
x→a x→a− x→a+
2x
Example. (i) For the function f (x) = , x = 3 is a vertical asymptote.
x−3

Visit For More : www.LearnEngineering.in


Visit For More : www.LearnEngineering.in

10 Engineering Mathematics - I

π
(ii) For the curve f (x) = tan x, x =
is a vertical asymptote.
2
Laws of limits. Suppose that c is a constant and the limits lim f (x) and lim g(x)
x→a x→a
exist, then the following laws are true.

1. lim f (x) + g(x) = lim f (x)+ lim g(x).


 
x→a x→a x→a
 
2. lim f (x) − g(x) = lim f (x) − lim g(x).

.in
x→a x→a x→a
 
3. lim c f (x) = c lim f (x).
x→a x→a

ng
4. lim f (x)g(x) = lim f (x) × lim g(x).
 
x→a x→a x→a
" # lim f (x)
f (x)

eri
5. lim = x→a . if lim g(x) , 0.
x→a g(x) lim g(x) x→a
x→a

  n
6. lim f (x) n = lim f (x) , where n is a positive integer.

x→a x→a
e
gin
7. lim c = c.
x→a

8. lim x = a.
x→a
En

9. lim xn = an , where n is a positive integer


x→a
√n √n
10. lim x= a, where n is a positive integer and a > 0.
x→a
arn

p
n
q
11. lim f (x) = n lim f (x).
x→a x→a

Direct substitution property. If f is a polynomial or a rational function and a


Le

is in the domain of f , then lim f (x) = f (a).


x→a
Theorem. If f (x) ≤ g(x), when x is near a and the limits of f and g both exist as x
w.

approaches a, then lim f (x) ≤ lim g(x).


x→a x→a
The Squeeze Theorem. If f (x) ≤ g(x) ≤ h(x), when x is near a and
ww

lim f (x) = lim h(x) = L, then lim g(x) = L.


x→a x→a x→a
The above theorem is sometimes called Sandwich Theorem or the Pinching
Theorem.
✎ ☞
Worked Examples
✍ ✌

Visit For More : www.LearnEngineering.in


Visit For More : www.LearnEngineering.in

Differential calculus 11

Example 1.8. Find lim (2x2 − 3x + 4).


x→5

Solution. lim (2x2 − 3x + 4) = lim 2x2 − lim 3x + lim 4


x→5 x→5 x→5 x→5

= 2 lim x2 − 3 lim x + 4
x→5 x→5
2
= 2 × 5 − 3 × 5 + 4 = 50 − 15 + 4 = 39.

.in
x3 + 2x2 − 1
Example 1.9. Find lim .
x→−2 5 − 3x

ng
lim x3 + 2x2 − 1
x3 + 2x2 − 1 x→−2
Solution. lim =
x→−2 5 − 3x lim 5 − 3x
x→−2

eri
lim x3 + lim 2x2 − lim 1
x→−2 x→−2 x→−2
=
lim 5 − lim 3x
x→−2 x→−2

e
lim x3 + 2 lim x2 − 1
gin
x→−2 x→−2
=
lim 5 − 3 lim x
x→−2 x→−2
−8 + 2(−2)2
−1
=
5 − 3(−2)
En

−8 + 8 − 1 1
= =− .
5+6 11
x2 − 1
arn

Example 1.10. Find lim .


x→1 x−1
x2 − 1 (x + 1)(x − 1)
Solution. lim = lim = lim (x + 1) = 1 + 1 = 2.
x→1 x − 1 x→1 x−1 x→1
Le


 x + 1 , ifx , 1




Example 1.11. Find lim f (x) where f (x) =  .
x→1
w.


, ifx = 1

π

Solution. f is defined at x = 1 and g(1) = π.
But the value of a limit as x approahces 1 does not depend on the value
ww

of the function at 1.
Since g(x) = x + 1 for x , 1, we have
lim g(x) = lim (x + 1) = 2.
x→1 x→1

Visit For More : www.LearnEngineering.in


Visit For More : www.LearnEngineering.in

12 Engineering Mathematics - I

(3 + x)2 − 9
Example 1.12. Evaluate lim .
x→0 x

(3 + x)2 − 9 (3 + x − 3)(3 + x + 3)
Solution. lim = lim
x→0 x x→0 x
x(6 + x)
= lim
x→0 x

.in
= lim (6 + x) = 6.
x→0

x2 + 9 − 3
Example 1.13. Find lim .

ng
x→0 x2
Solution. On rationalizing the numerator we get
√ √ √

eri
x2 + 9 − 3 x2 + 9 − 3 x2 + 9 + 3
lim = lim × √
x→0 x2 x→0 x2 x2 + 9 + 3
2
x +9−9
= lim √
x→0 x2 x2 + 9 + 3
e
gin
x2
= lim √
x→0 x2 x2 + 9 + 3
1 1 1
= lim √ = = .
x→0 x2 + 9 + 3 3 + 3 6
En

Example 1.14. Show that lim |x| = 0.


x→0

, ifx ≥ 0
arn

x




Solution. We know that |x| =   .
−x , ifx < 0


Since |x| = −x for x < 0, we have lim |x| = lim −x = 0.
x→0− x→0−
Le

Also |x| = x for x > 0, we have lim |x| = lim x = 0.


x→0+ x→0+
∴ lim |x| = lim |x| = 0.
x→0− x→0+
w.

Hence, lim |x| = 0.


x→0
ww

|x|
Example 1.15. Prove that lim does not exist.
x→0 x

x , ifx ≥ 0




Solution. We know that |x| = 
 .
−x , ifx < 0

Visit For More : www.LearnEngineering.in


Visit For More : www.LearnEngineering.in

Differential calculus 13

|x| x
Now, lim = lim = 1.
x→0+ x x→0+ x
|x| −x
lim = lim = −1.
x→0− x x→0− x
|x| |x| |x|
Since lim , lim , lim does not exist.
x→0+ x x→0− x x→0 x
√
 x − 4 , ifx > 4

.in

Example 1.16. If f (x) =  , determine whether lim f (x) exists.

 x→4
8 − 2x , ifx < 4



Solution. We have f (x) = x − 4 for x > 4.

ng

∴ lim f (x) = lim x − 4 = 0.
x→4+ x→4+
Also, f (x) = 8 − 2x for x < 4.

eri
∴ lim f (x) = lim (8 − 2x) = 0.
x→4− x→4−
Since lim f (x) = lim f (x) = 0, we have lim f (x) exists and lim f (x) = 0.
x→4+ x→4− x→4 x→4

e
Example 1.17. Show that for the greatest integer function [x], lim [x] does not
gin
x→3
exist.
Solution. By definition [x] = 3 for 3 ≤ x < 4.
En

∴ lim [x] = lim 3 = 3.


x→3+ x→3+
Also [x] = 2 for 2 ≤ x < 3.
∴ lim [x] = lim 2 = 2.
arn

x→3− x→3−
Since lim [x] , lim [x], lim [x] does not exist.
x→3+ x→3− x→3
Le

1.3 Continuity

Definition. A function f is continuous at a number a, if lim f (x) = f (a).


w.

x→a
f is discontinuous at a if it is not continuous at a.
Note. For proving a function f to be continuous, we have to prove the following.
ww

1. f (a) must be defined.

2. lim f (x) exists.


x→a

Visit For More : www.LearnEngineering.in


Visit For More : www.LearnEngineering.in

14 Engineering Mathematics - I

3. lim f (x) = f (a).


x→a

x2 − x − 2
Example. Consider f (x) = .
x−2
Here, f (x) is not defined at x = 2.
∴ f is not continuous at x = 2.

x2 −x−2
, ifx , 2

.in

 x−2

If we define f (x) as f (x) = 
 .
, ifx = 2

1

Then

ng
(i) f (2) is defined and f (2) = 1.
x2 − x − 2 (x − 2)(x + 1)

eri
(ii) lim f (x) = lim = lim = lim (x + 1) = 3
x→2 x→2 x−2 x→2 x−2 x→2

(iii) Since f (2) = 1 , lim f (x), f (x) is not continuous at x = 2.


x→2

e
Definition. A function f is said to be continuous from the right at a number a if
gin
lim f (x) = f (a), and f is said to be continuous from the left at a if lim f (x) = f (a).
x→a+ x→a−
Example. Consider the greatest integer function f (x) = [x]. We notice that
lim f (x) = lim [x] = n = f (n)
En

x→n+ x→n+
lim f (x) = lim [x] = n − 1 , f (n).
x→n− x→n−
∴ f (x) = [x] is continuous from the right but not continuous from the
arn

left.
Definition. A function f is continuous on an interval if it is continuous at every
number in the interval.
Le

Theorem. If f and g are continuous at a and c is a constant, then the following


functions are also continuous at a.
w.

f
1. f + g 2. f − g 3. c f 4. f g 5. g(a) , 0.
g
Theorem.
ww

(a) Any polynomial is continuous everywhere. i.e., it iscontinuous on R = (−∞, ∞).

(b) Any rational function is continuous wherever it is defined, i.e., it is continuous


on its domain.

Visit For More : www.LearnEngineering.in


Visit For More : www.LearnEngineering.in

Differential calculus 15

Theorem. The following types of functions are continuous at every number in


their domains.

1. Polynomials.

2. Rational functions.

.in
3. Root functions.

4. Trigonometric functions.

ng
5. Inverse trigonometric functions.

eri
6. Exponential functions.

7. Logarithmic functions.

e
gin
Theorem. If f is continuous at b, and lim g(x) = b, then lim f (g(x)) = f (b).
  x→a x→a
i.e., lim f (g(x)) = f lim g(x) .
x→a x→a
Theorem. If g is continuous at a and f is continuous at g(a), then the composite
En

function f og given by ( f og)(x) = f (g(x)) is continuous at a.


The intermediate value theorem. Suppose that f is continuous on the closed
arn

interval [a, b] and let N be any number between f (a) and f (b), where f (a) , f (b).
Then there exists a number c in (a, b) such that f (c) = N.
Result. The intermediate value theorem is useful in the location of the roots
Le

of the given equation.

Example 1.18. Find the domain where the fnction f is continuous. Also find the
w.

numbers  at which the function f is discontinous where


1 + x2

 , if x ≤ 0
ww








f (x) =  , if 0 < x ≤ 2 . [A.U. Dec. 2015]


2−x



(x − 2)2 x

, if x > 2


Solution. The function f changes its value at x = 0, and x = 2.

Visit For More : www.LearnEngineering.in


Visit For More : www.LearnEngineering.in

16 Engineering Mathematics - I

lim f (x) = lim (1 + x2 ) = 1.


x→0− x→0−
lim f (x) = lim (2 − x) = 2.
x→0+ x→0+
Since, lim f (x) = lim f (x), f is not continuous at x = 0.
x→0− x→0+
lim f (x) = lim (2 − x) = 0.
x→2− x→2−
lim f (x) = lim (x − 2)2 = 0.

.in
x→2+ x→2+
Since, lim f (x) = lim f (x) = 0 = f (2), f (x) is continuous at x = 2.
x→2− x→2+
∴ The only number at which the function is discontinuous is at x = 0.

ng
The domain of continuity of f is {(−∞, 0) ∪ (0, ∞).
The graph of f is given below

eri
The combined graph of the function f is
For all x ≤ 0.
as follows
y = f (x) = 1 + x2 .
e
gin
Y
x -3 -2 -1 0 10
y 10 5 2 1 9
8
En

For all 0 < x ≤ 2. 1 + x2


7
y = f (x) = 2 − x.
6
arn

x 0 1 2 5
y 2 1 0 4
For all x > 2. 3
Le

2 (x − 2)2
y = f (x) = (x − 2)2 . 2−x
1
x 2 3 4 X
w.

y 0 1 4
−4 −3 −2 −1 0 1 2 3 4
ww

Notice that the point (−1, 1) in y = x2 is not included and there is a discontinuity
between the graph for all x ≤ 1 (i.e.y = 1 − x) and the graph for all x > 1 (i.e. y = x2 ).

Example 1.19. Show that there is a root of the equation 4x3 − 6x2 + 3x − 2 between

Visit For More : www.LearnEngineering.in


Visit For More : www.LearnEngineering.in

Differential calculus 17

1 and 2.

Solution.Let f (x) = 4x3 − 6x2 + 3x − 2.

f (1) = 4(1)3 − 6(1)2 + 3(1) − 2 = 4 − 6 + 3 − 2 = −1 < 0.

.in
f (2) = 4(2)3 − 6(2)2 + 3(2) − 2

= 4 × 8 − 6 × 4 + 3 × 2 − 2 = 32 − 24 + 6 − 2 = 12 > 0.

ng
The function changes its sign between 1 and 2.

eri
Y
Since f (x) is a polynomial, which is
continuous in its domain the graph of
y = f (x) must cross the x−axis atleast
e f (1)
gin
at one point say x = c between x = 1
and x = 2 such that f (c) = 0, which is c 2
X
the root of the equation f (x) = 0. This 1
f (2)
En

proves that the given equation f (x) = 0


has a root between 1 and 2.
arn

Example 1.20. Prove that the equation x3 − 15x + 1 = 0 has atmost one real root
in the interval [−2, 2]. [A.U.Nov.2016]
Le

Solution. Let f (x) = x3 − 15x + 1


w.

f (−2) = −8 + 30 + 1 = 23 = +ve.
f (−1) = −1 + 15 + 1 = 15 = +ve. + + +
ww

f (0) = 1 = +ve.
f (1) = 1 − 15 + 1 = −13 = −ve. −2 −1 0 1 2
f (2) = 8 − 30 + 1 = −21 = −ve. − −

Visit For More : www.LearnEngineering.in


Visit For More : www.LearnEngineering.in

18 Engineering Mathematics - I

Hence, f (0) > 0 > f (1).


f changes sign between 0 and 1.
∴ By intermediate value theorem, there is a number c between 1 and 2
such that f (c) = 0.
∴ The given equation has atleast one root c in the interval (0, 1).

.in
Since f (x) changes sign in the interval [−2, 2] only once between 0 and
1, y = f (x) crosses the x axis only once.
i.e., f (x) attains 0 only once in [−2, 2]

ng
Hence, there is atmost one real root in the interval [−2, 2].

eri
1.4 Derivatives and differentiation rules

e
Definition. The tangent line to the curve y = f (x) at the point P(a, f (a)) is the line
gin
f (x) − f (a)
through P with slope m = lim .
x→a x−a
En

Definition. The derivative of a function f at a number a, denoted by f ′ (a) is


f (a + h) − f (a)
defined as f ′ (a) = lim , if the limit exists.
h→0 h
arn

Note. Let x = a+h. As h → 0, x → a. The equivalent definition for the derivative


f (x) − f (a)
is f ′ (a) = lim .
x→a x−a
✎ ☞
Worked Examples
Le

✍ ✌

Example 1.21. Find an equation to the tangent line to the parabola y = x2 at the
w.

point P(1, 1).


Solution. Given a = 1, f (a) = 1, f (x) = x2 .
ww

f (x) − f (1)
Slope m = lim
x→1 x−1
x2 − 1 (x − 1)(x + 1)
= lim = lim = lim (x + 1) = 1 + 1 = 2.
x→1 x − 1 x→1 x−1 x→1

Visit For More : www.LearnEngineering.in


Visit For More : www.LearnEngineering.in

Differential calculus 19

Equation of the tangent line through (1, 1) is

y − y1 = m(x − x1 ).

y − 1 = 2(x − 1)

y = 2x − 2 + 1

.in
y = 2x − 1.

ng
Example 1.22. Find an equation to the tangent line to the hyperbola y = at the
x
point (3, 1).

eri
Solution. Given a = 3, f (a) = f (3) = 1.

f (a + h) − f (a)
We have m = lim
h→0 h
e
f (3 + h) − f (3)
gin
= lim
h→0 h
3
−1
= lim 3+h
h→0 h
En

3−3−h
= lim
h→0 h(3 + h)
−h −1 −1
= lim = lim = .
h→0 h(3 + h) h→0 (3 + h) 3
arn

∴ The equation of the tangent line is


Le

y − y1 = m(x − x1 ).
−1
y−1= (x − 3)
3
w.

3y − 3 = −x + 3

x + 3y − 3 − 3 = 0
ww

x + 3y − 6 = 0.

Example 1.23. Find the derivative of the function f (x) = x2 − 8x + 9 at a number

Visit For More : www.LearnEngineering.in


Visit For More : www.LearnEngineering.in

20 Engineering Mathematics - I

a. Find also the equation the tangent line at the point (3, −6).
f (a + h) − f (a)
Solution. f ′ (a) = lim
h→0 h
(a + h) − 8(a + h) + 9 − (a2 − 8a + 9)
2
= lim
h→0 h
a + h + 2ah − 8a − 8h + 9 − a2 + 8a − 9
2 2
= lim

.in
h→0 h
h2 + 2ah − 8h
= lim
h→0 h

ng
= lim (h + 2a − 8) = 2a − 8.
h→0

We have m = f ′ (a) = f ′ (3) = 2 × 3 − 8 = −2.

eri
∴ The equation of the tangent line is

y − y1 = m(x − x1 ).

e
gin
y + 6 = −2(x − 3)

= −2x + 6

2x + y = 0.
En

Derivative of a function. Let f (x) be a given function. The derivative of f (x) at


f (x + h) − f (x)
any variable point x is defined by f ′ (x) = lim .
arn

h→0 h
Example 1.24. If f (x) = x3 − x, find a formula for f ′ (x).

Solution.Given f (x) = x3 − x.
Le

f (x + h) − f (x)
f ′ (x) = lim
h→0 h
(x + h)3 − (x + h) − (x3 − x)
w.

= lim
h→0 h
x + 3x h + 3xh2 + h3 − x − h − x3 + x
3 2
= lim
ww

h→0 h
2 2
3x h + 3xh + h − h3
= lim
h→0 h
= lim 3x2 + 3xh + h2 − 1 = 3x2 − 1.
h→0

Visit For More : www.LearnEngineering.in


Visit For More : www.LearnEngineering.in

Differential calculus 21


Example 1.25. If f (x) = x, find the derivative of f . State the domain of f ′ .


Solution.Given f (x) = x.
f (x + h) − f (x)
f ′ (x) = lim
h→0 h
√ √
x+h− x
= lim

.in
h→0 h
√ √ √ √
x+h− x x+h+ x
= lim × √ √
h→0 h x+h+ x

ng
x+h−x
= lim  √ √ 
h→0 h x+h+ x

eri
h
= lim  √ √ 
h→0 h x+h+ x
1 1 1
= lim  √ √ = √ √ = √ .
h→0 x+h+ x
e
x+ x 2 x
gin
From this we notice that f ′ (x) exists if x > 0.
∴ The domain of f ′ is (0, ∞).
En

Example 1.26. Where is the function f (x) = |x| differentiable. [A.U. Nov. 2015]

x if x ≥ 0



arn


Solution. We have that |x| =  .
−x if x < 0


Consider x > 0. Then |x| = x. We choose h small enough such that x + h > 0.
∴ |x + h| = x + h.
Le

∴ When x > 0,
w.

f (x + h) − f (x)
f ′ (x) = lim
h→0 h
|x + h| − |x|
= lim
ww

h→0 h
x+h−x h
= lim = lim = lim 1 = 1.
h→0 h h→0 h h→0

∴ f is differentiable for x > 0.

Visit For More : www.LearnEngineering.in


Visit For More : www.LearnEngineering.in

22 Engineering Mathematics - I

Consider x = 0.

f (0 + h) − f (x)
f ′ (0) = lim
h→0 h
|0 + h| − |x|
= lim
h→0 h
|h|
= lim [if it exists]
h→0 h

.in
Let us find the left and right limits.
|h| h
lim = lim = lim 1 = 1.

ng
h→0+ h h→0+ h h→0+
|h| −h
lim = lim = lim −1 = −1.
h→0− h h→0− h h→0
Since the two limits are different, f ′ (0) does not exist.

eri
∴ f is not differentiable at x = 0.
Consider x < 0.
In this case |x| = −x. e
gin
Choose h small so that x + h < 0.
∴ |x + h| = −(x + h)
∴ For x < 0,
En

f (x + h) − f (x)
f ′ (x) = lim
h→0 h
arn

−(x + h) − (−x)
= lim
h→0 h
−x − h + x −h
= lim = lim = lim −1 = −1.
h→0 h h→0 h h→0
Le

∴ f is differentiable for x < 0.


Combining all the three cases we obtain that f is differentiable at all x except 0.
w.


1 ifx > 0





∴ f (x) = 
 .
−1 ifx < 0


ww

Theorem. If f is differentiable at a, then f is continuous at a.


Proof. Given that f is differentiable at a.
f (x) − f (a)
f ′ (x) = lim exits. (1)
x→a x−a

Visit For More : www.LearnEngineering.in


Visit For More : www.LearnEngineering.in

Differential calculus 23

f (x) − f (a)
Now, f (x) − f (a) = (x − a)
x−a
Taking limits as x → a both sides we get
f (x) − f (a)
lim f (x) − f (a) = lim (x − a)
x→a x→a x−a
f (x) − f (a)
= lim × lim (x − a)
x→a x−a x→a

.in
= f ′ (a) × 0 [by (1)]

lim f (x) − f (a) = 0 (2)


x→a

ng
Also lim f (x) = lim f (a) + ( f (x) − f (a))
 
x→a x→a

= lim + lim ( f (x) − f (a))

eri
x→a x→a

= f (a) + 0 [by (1)]

lim f (x) = f (a)


x→a
e
gin
∴ f (x) is continuous at a.
Result. The converse of the above theorem is false.
i.e., If a function is continuous at a need not be differentiable at a.
En

For example consider the function f (x) = |x|.


lim f (x) = lim |x| = 0 = f (0).
x→a x→a
arn

But by example 1.26, f (x) = |x| is not differentiable at 0.


Higher derivatives. If a function f is differentiable, then its derivative f ′ is also
a function. Hence, f ′ may have a derivative of its own denoted by ( f ′ )′ = f ′′ . f ′′ is
Le

called !the second derivative of f . The second derivative of y = f (x) is denoted by


d dy d2 y
= 2 . In a similar way we can have the third derivative of f denoted by
dx dx dx
w.

d d2 y d3 y
!
( f ′′ )′ = f ′′′ . If y = f (x) then we have y′′′ = f ′′′ (x) = = . If this process is
dx dx2 dx3
continued we can get in general, the nth derivative of f denoted by f (n) is obtained
ww

dn y
by differentiating f n times. Generally if y = f (x) then we write y(n) = f (n) (x) = n .
dx
Rules on differentiation

1. The sum rule. If f and g are both differentiable, then

Visit For More : www.LearnEngineering.in


Visit For More : www.LearnEngineering.in

24 Engineering Mathematics - I

d   d   d 
g(x) .

f (x) + g(x) = f (x) +
dx dx dx
Proof. Let F(x) = f (x) + g(x). Then,

F(x + h) − F(x)
F ′ (x) = lim
h→0 h
f (x + h) + g(x + h) − [ f (x) + g(x)]
= lim
h→0 h

.in
[ f (x + h) − f (x)] + [g(x + h) − g(x)]
= lim
h→0 h
[ f (x + h) − f (x)]
= lim + lim [g(x + h) − g(x)]h

ng
h→0 h h→0

= f ′ (x) + g′ (x).

eri
2. Constant multiple rule. If c is a constant and f is a differentiable function,
d  d 
then

f (x) .

c f (x) = c
dx dx
Proof. Let F(x) = c f (x). Then, e
gin
F(x + h) − F(x)
F ′ (x) = lim
h→0 h
c f (x + h) − c f (x)
= lim
En

h→0 h
c[ f (x + h) − f (x)]
= lim
h→0 h
arn

[ f (x + h) − f (x)]
= c lim
h→0 h
d
= c f ′ (x) = c
 
f (x) .
dx
Le

3. The difference rule. If f and g are both differentiable, then


d   d   d 
g(x) .

f (x) − g(x) = f (x) −
w.

dx dx dx
Proof. We have f (x) − g(x) = f (x) + (−)g(x).

d  d 
ww

 
f (x) − g(x) = f (x) + (−1)g(x)
dx dx
d   d  
= f (x) + (−1)g(x) [by sum rule]
dx dx
d   d  
= f (x) + (−1) g(x) [by constant multiple rule]
dx dx

Visit For More : www.LearnEngineering.in


Visit For More : www.LearnEngineering.in

Differential calculus 25

d   d  
= f (x) − g(x) .
dx dx

4. The product rule. If f and g are both differentiable, then


d   d   d  
f (x) × g(x) = f (x) g(x) + g(x) f (x) .
dx dx dx

f (x + h)g(x + h) − f (x)g(x)

.in
Proof. ( f (x)g(x))′ = lim
h→0 h
f (x + h)g(x + h) − f (x + h)g(x) + f (x + h)g(x) − f (x)g(x)
= lim
h→0 h

ng
f (x + h){g(x + h) − g(x)} + g(x){ f (x + h) − f (x)}
= lim
h→0 h
( )
g(x + h) − g(x) f (x + h) − f (x)

eri
= lim f (x + h) + g(x)
h→0 h h
g(x + h) − g(x) f (x + h) − f (x)
= lim f (x + h) lim + lim g(x) lim
h h
h→0 h→0

e
( f (x)g(x))′ = f (x)g′ (x) + g(x) f ′ (x).
h→0 h→0
gin
5. Quotient rule.  If f d and g are both differentiable, then
d
g(x) dx f (x) − f (x) dx g(x)
" #
d f (x)
En

= 2
.
dx g(x) [g(x)]

!′  f (x+h) f (x) 
f (x)  g(x+h) − g(x) 
arn



 

Proof. = lim 

g(x) h

h→0 

 


( )
f (x + h)g(x) − g(x + h) f (x)
= lim
h→0 hg(x + h)g(x)
Le

( )
f (x + h)g(x) − f (x)g(x) + f (x)g(x) − g(x + h) f (x)
= lim
h→0 hg(x + h)g(x)
( )
w.

g(x)[ f (x + h) − f (x)] − f (x)[g(x + h) − g(x)]


= lim
h→0 hg(x + h)g(x)
f (x+h)− f (x)
− f (x) g(x+h)−g(x)
 
 g(x)

h h

ww

 
= lim 

g(x + h)g(x)

h→0 
 

f (x + h) − f (x) g(x + h) − g(x)
lim g(x) − lim f (x)
h→0 h h→0 h
=
lim g(x + h)g(x)
h→0

Visit For More : www.LearnEngineering.in


Visit For More : www.LearnEngineering.in

26 Engineering Mathematics - I

f (x + h) − f (x) g(x + h) − g(x)


lim g(x) lim − lim f (x) lim
h→0 h→0 h h→0 h→0 h
=
g(x)g(x)
g(x) f ′ (x) − f (x)g′ (x)
= .
[g(x)]2

Derivatives of simple functions

.in
d
1. If c is a constant, prove that (c) = 0.
dx
Proof. Let f (x) = c.

ng
f (x + h) − f (x)
f ′ (x) = lim
h→0 h

eri
c−c 0
= lim = lim = 0.
h→0 h h→0 h

d n
2. If n is a positive integer, then prove that
e dx
(x ) = nxn−1 .
gin
Proof. Let f (x) = xn .

f (x + h) − f (x)
f ′ (x) = lim
h→0 h
En

(x + h)n − xn
= lim
h→0 h
x + C1 xn−1 h + nC2 xn−2 h2 + nC3 xn−3 h3 + · · · + hn − xn
n n
= lim
arn

h→0 h
n−1 n(n−1) n−2 2 n(n−1)(n−2) n−3 3
nx h + 2 x h + 6 x h + · · · + hn
= lim
h→0 h
n−1 n(n − 1) n−2 1 n(n − 1)(n − 2) n−3 2
x h + · · · + hn−1
Le

= lim nx + x h +
h→0 2 6
n(n − 1) n−2 1 n(n − 1)(n − 2) n−3 2
= lim nxn−1 + lim x h + lim x h + · · · + lim hn−1
h→0 h→0 2 h→0 6 h→0
w.

= nxn−1 + 0 + 0 + · · · + 0 = nxn−1 .
ww

d n
General rule. If n ia any real number, then (x ) = nxn−1 .
dx

Visit For More : www.LearnEngineering.in


Visit For More : www.LearnEngineering.in

Differential calculus 27

d x
3. Derivative of e x . Prove that (e ) = e x .
dx
eh − 1
Definition of the number e. e is a number such that lim = 1.
h→0 h
Proof. Let f (x) = e x .
f (x + h) − f (x)
f ′ (x) = lim

.in
h→0 h
e x+h − e x
= lim
h→0 h

ng
e e − ex
x h
= lim
h→0 h
h
!
x e −1
= lim e

eri
h→0 h
eh − 1
!
x
= lim e × lim = ex × 1 = ex .
h→0 h→0 h

Two important limits e


gin
sin θ
1. Prove that lim = 1.
θ→0 θ
En

D
Proof. Consider a circle with center
O and radius 1 unit. Let arc(AB) B
arn

subtends an angle θ at the centre E


π
and assume that 0 < θ < . Draw θ
2 A
BC perpendicular to OA. Let the 0 C
Le

tangents at A and B meet at E.


Produce AE and OB to intersect at D.
w.

Now we have arcAB = θ.


BC BC
ww

From ∆OAB, sin θ = OB = 1 .


∴ |BC| = sin θ.
From the figure we have
|BC| < |AB| < arcAB.

Visit For More : www.LearnEngineering.in


Visit For More : www.LearnEngineering.in

28 Engineering Mathematics - I

∴ sin θ < θ.
sin θ
⇒ < 1. (1)
θ
Again form the figure,
arcAB < |AE| + |EB|
∴ θ = arcAB < |AE| + |EB| < |AB| + |ED| = |AD| = |OA| tan θ = tan θ.

.in
∴ θ < tan θ.
sin θ
i.e., θ <

ng
cos θ
θ cos θ < sin θ
sin θ

eri
cos θ < <1 [by(1)]
θ

Taking limit as θ → 0 we get


lim cos θ < lim
sin θ
< lim 1 e
gin
θ→0 θ→0 θ θ→0
sin θ
1 < lim <1
θ→0 θ
sin θ
By squeeze theorem lim = 1.
θ→0 θ
En

cos θ − 1
2. Prove that lim = 0.
arn

θ→0 θ

cos θ − 1 cos θ − 1 cos θ + 1


Proof. lim = lim ×
θ→0 θ θ→0 θ cos θ + 1
Le

2
cos θ − 1 − sin2 θ
= lim = lim
θ→0 θ(cos θ + 1) θ→0 θ(cos θ + 1)
sin θ sin θ 0
= − lim × lim = −1 × = 0.
w.

θ→0 θ θ→0 (cos θ + 1) 1+1

cos θ − 1
∴ lim = 0.
ww

θ→0 θ

Derivatives of trigonometric functions

d
1. Prove that (sin x) = cos x.
dx

Visit For More : www.LearnEngineering.in


Visit For More : www.LearnEngineering.in

Differential calculus 29

Proof. Let f (x) = sin(x)


f (x + h) − f (x)
f ′ (x) = lim
h→0 h
sin(x + h) − sin(x)
= lim
h→0 h
sin(x) cos(h) + cos(x) sin(h) − sin(x)
= lim
h→0 h

.in
sin(x)(cos(h) − 1) + cos(x) sin(h)
= lim
h→0 h
!
sin(x)(cos(h) − 1) cos(x) sin(h)

ng
= lim +
h→0 h h
!
(cos(h) − 1) sin(h)
= lim sin(x) + lim cos(x)
h→0 h h→0 h

eri
!
(cos(h) − 1) sin(h)
= sin(x) lim + cos(x) lim
h→0 h h→0 h

e
= sin(x) × 0 + cos(x) × 1 = 0 + cos(x) = cos(x).
gin
d
2. Prove that (cos x) = − sin x.
dx
Proof. Let f (x) = cos(x)
En

f (x + h) − f (x)
f ′ (x) = lim
h→0 h
cos(x + h) − cos(x)
= lim
arn

h→0 h
cos(x) cos(h) − sin(x) sin(h) − cos(x)
= lim
h→0 h
cos(x)(cos(h) − 1) − sin(x) sin(h)
= lim
Le

h→0 h
!
cos(x)(cos(h) − 1) sin(x) sin(h)
= lim −
h→0 h h
w.

!
(cos(h) − 1) sin(h)
= lim cos(x) − lim sin(x)
h→0 h h→0 h
ww

= cos(x) × 0 − sin(x) × 1 = − sin(x).

f ′ (x) = − sin x.

d
3. Prove that (tan x) = sec2 x.
dx

Visit For More : www.LearnEngineering.in


Visit For More : www.LearnEngineering.in

30 Engineering Mathematics - I

Proof. Let f (x) = tan(x)


d d
cos x dx (sin x) − sin x dx (cos x)
!
d d sin x
(tan x) = = 2
dx dx cos x cos x
cos x × cos x − sin x × (− sin x)
=
cos2 x
2
cos2 x + sin x 1
= = = sec2 x.

.in
cos2 x cos2 x
d
4. Prove that (cosecx) = −cosecx cot x.
dx

ng
d d
sin x dx (1) − 1 × dx (sin x)
!
d d 1
Proof. (cosecx) = =
dx dx sin x sin2 x

eri
sin x × 0 − 1 × (cos x)
=
sin2 x
− cos x −1 cos x
= = × = −cosecx cot x.
2
sin x
e
sin x sin x
gin
d
5. Prove that (sec x) = sec x tan x.
dx
d d
cos x dx (1) − 1 × dx (cos x)
!
d d 1
En

Proof. (sec x) = = 2
dx dx cos x cos x
cos x × 0 − 1 × (− sin x)
=
cos2 x
arn

sin x sin x 1
= 2
= × = sec x tan x.
cos x cos x cos x
d
6. Prove that (cot x) = −cosec2 x.
Le

dx
d d
d d  cos x  sin x dx (cos x) − cos x × dx (sin x)
Proof. (cot x) = =
dx dx sin x sin2 x
w.

sin x × (− sin x) − cos x × (cos x)


=
sin2 x
− sin2 x − cos2 x −(sin2 x + cos2 x)
ww

1
= 2
= 2
= − 2 = −cosec2 x.
sin x sin x sin x

The chain rule. If g is differentiable at x and f is differentiable at g(x), then the


composite function F = f og defined by F(x) = f (g(x)) is differentiable at x and F ′ is

Visit For More : www.LearnEngineering.in


Visit For More : www.LearnEngineering.in

Differential calculus 31

given by the product F ′ (x) = f ′ (g(x)) × g′ (x).


Proof. Let y = F(x) = f (g(x))
Let u = g(x) ⇒ y = f (u)
Let ∆x be a small change in x, then ∆u is the corresponding change in u.
∆u = g(x + ∆x) − g(x).

.in
Then the corresponding change in y is given by
∆y = f (u + ∆u) − f (u)

ng
dy ∆y
Now, = lim
dx ∆x→0 ∆x
∆y ∆u
= lim ×

eri
∆x→0 ∆u ∆x
∆y ∆u
= lim × lim
∆x→0 ∆u ∆x→0 ∆x
∆y ∆u
= lim
∆x→0 ∆u
× lim
∆x→0 ∆x e
[∆u → 0 as ∆x → 0]
gin
dy du
= ×
du dx
d d
= ( f (u)) × (g(x)) = f ′ (u) × g′ (x)
du dx
En

d
[F(x)] = f ′ (g(x)) × g′ (x)
dx
F ′ (x) = f ′ (g(x)) × g′ (x)
arn

✎ ☞
Worked Examples
✍ ✌
1 + cos 2x
Le

Example 1.27. Evaluate limπ . [A.U.Jan. 2015]


x→ 2 (π − 2x)2

1 + cos 2x 2 cos2 x
w.

Solution. limπ = lim


x→ 2 (π − 2x)2 x→ π2 (π − 2x)2
π
Let −x=y
ww

2
π
As x → , y → 0
2
1 + cos 2x 2 cos2 x
∴ limπ = limπ 
x→ 2 (π − 2x)2 x→ 2
2
4 π−x 2

Visit For More : www.LearnEngineering.in


Visit For More : www.LearnEngineering.in

32 Engineering Mathematics - I

 
1 cos2 π2 − y
= lim
2 y→0 y2
1 sin2 y
= lim 2
2 y→0 y
!2
1 sin y
= lim
2 y→0 y

.in
!2
1 sin y 1 1
= lim = × 12 = .
2 y→0 y 2 2

ng
dy
Example 1.28. If y = x8 + 12x5 − 4x4 + 10x3 − 6x + 5, find .
dx

eri
Solution. Given, y = x8 + 12x5 − 4x4 + 10x3 − 6x + 5
dy d  8
=
dx dx e
x + 12x5 − 4x4 + 10x3 − 6x + 5

gin
d 8 d d d d d
= (x ) + (12x5 ) − (4x4 ) + (10x3 ) − (6x) + (5)
dx dx dx dx dx dx
d d d d
= 8x8−1 + 12 (x5 ) − 4 (x4 ) + 10 (x3 ) − 6 (x) + 0
dx dx dx dx
En

= 8x7 + 12 × 5(x5−1 ) − 4 × 4(x4−1 ) + 10 × 3(x3−1 ) − 6 × 1

= 8x7 + 60x4 − 16x3 + 30x2 − 6.


arn

dy d2 y
Example 1.29. If y = e x − x, find and 2 .
dx dx
Le

Solution. Given y = e x − x.

dy
= ex − 1
w.

dx
d2 y
= ex .
dx2
ww

dy
Example 1.30. If y = x2 sin x, find .
dx
Solution. Given y = x2 sin x.

Visit For More : www.LearnEngineering.in


Visit For More : www.LearnEngineering.in

Differential calculus 33

Applying the product rule we obtain

dy d  2 
= x sin x
dx dx
d d
= x2 (sin x) + sin x (x2 )
dx dx
= x2 × (cos x) + sin x × (2x)

.in
= x2 cos x + 2x sin x.

sec x dy

ng
Example 1.31. If f (x) = , find . For what values of x, does the graph of
1 + tan x dx
f , has a horizontal tangent.
sec x
Solution. Given f (x) = .

eri
1 + tan x
By the quotient rule, we have
d d
(1 + tan x) dx (sec x) − sec x dx (1 + tan x)
f ′ (x) =
(1 + tan x)2e
gin
 
(1 + tan x)(sec x tan x) − sec x 0 + sec2 x
=
(1 + tan x)2
sec x tan x + sec x tan2 x − sec3 x
En

=
(1 + tan x)2
sec x(tan x + tan2 x − sec2 x)
=
(1 + tan x)2
arn

sec x(tan x − (sec2 x − tan2 x))


=
(1 + tan x)2
sec x(tan x − 1)
Le

=
(1 + tan x)2
sec x(tan x − 1)
f ′ (x) = .
(1 + tan x)2
w.

The points where the graph of f has a horizontal tangent are given by f ′ (x) = 0
ww

sec x(tan x − 1)
i.e., =0
(1 + tan x)2
⇒ sec x(tan x − 1) = 0

But sec x , 0

Visit For More : www.LearnEngineering.in


Visit For More : www.LearnEngineering.in

34 Engineering Mathematics - I

∴ tan x − 1 = 0
π
i.e., tan x = 1 ⇒ x = nπ + , n is an integer.
4

Example 1.32. Find the points on the curve y = x4 − 6x2 + 4 where the tangent line
is horizontal.
dy

.in
Solution. Horizontal tangent occur at points where = 0.
dx

i.e.,4x3 − 6 × 2x = 0

ng
4x3 − 12x = 0

4x(x2 − 3) = 0

eri

i.e.,x = 0 or x2 − 3 = 0 ⇒ x2 = 3 ⇒ x = ± 3.

When x = 0, y = 4.
e
gin
The point is (0, 4).
√  √ 4  √ 2
When x = 3, y = 3 − 6 3 + 4 = 9 − 18 + 4 = −5.

The point is ( 3, −5).
En

√  √ 4  √ 2
When x = − 3, y = − 3 − 6 − 3 + 4 = 9 − 18 + 4 = −5.

The point is (− 3, −5).
√ √
arn

The given curve has horizontal tangents at the points (0, 4), ( 3, −5) and (− 3, −5).

Example 1.33. At what point on the cure y = e x , is the tangent line parallel to the
Le

line y = 2x.
Solution. Given y = e x ⇒ y′ = e x .
slope of the tangent at x = e x .
w.

Since the tangent line is parallel to the line y = 2x,


Slope of the tangent = 2.
ww

i.e., e x = 2 ⇒ x = log 2.
When x = log 2, y = elog 2 = 2.
∴ The required point is (log 2, 2).

Visit For More : www.LearnEngineering.in


Visit For More : www.LearnEngineering.in

Differential calculus 35

Example 1.34. If f (x) = xe x find f (n) (x).


Solution. Given f (x) = xe x .

d
f ′ (x) = xe x

dx
d x d
=x e + e x (x) = xe x + e x .1 = e x (x + 1).
dx dx

.in
d
f ′′ (x) = (x + 1)e x

dx
d x d
= (x + 1) e + e x (x + 1)

ng
dx dx
= (x + 1)e x + e x .1 = e x (x + 1 + 1) = e x (x + 2).
d

eri
f ′′′ (x) = (x + 2)e x

dx
d x d
= (x + 2) e + e x (x + 2)
dx dx

e
= (x + 2)e x + e x .1 = e x (x + 2 + 1) = e x (x + 3).
gin
Applying this process successively n times we get
En

f (n) (x) = e x (x + n).

x2 + x − 2 dy
Example 1.35. If y = 3
, find .
x +6 dx
arn

x2 + x − 2
Solution. Given y = .
x3 + 6
d d
(x3 + 6) dx (x2 + x − 2) − (x2 + x − 2) dx (x3 + 6)
Le

dy
=
dx (x3 + 6)2
(x3 + 6)(2x + 1) − (x2 + x − 2)(3x2 )
=
w.

(x3 + 6)2
2x + x + 12x + 6 − 3x4 − 3x3 + 6x2
4 3
=
(x3 + 6)2
ww

−x4 − 2x3 + 6x2 + 12x + 6


= .
(x3 + 6)2

ex
Example 1.36. Find the equation of the tangent line to the curve y = at the
1 + x2

Visit For More : www.LearnEngineering.in


Visit For More : www.LearnEngineering.in

36 Engineering Mathematics - I

e
point (1, ).
2
ex
Solution. Given y =
1 + x2
ex
!
dy d
=
dx dx 1 + x2
d d
(1 + x2 ) dx (e x ) − e x dx (1 + x2 )
=

.in
(1 + x2 )2
(1 + x2 )e x − e x 2x
=
(1 + x2 )2

ng
e x (1 + x2 − 2x) e x (1 − x)2
= =
(1 + x2 )2 (1 + x2 )2
!
dy e(1 − 1)

eri
e ! = = 0.
dx 1, (1 + 1)2
2
∴ Slope of the tangent m = 0.
Equation of the tangent line is e
gin
y − y1 = m(x − x1 )
e
y − = 0(x − 1)
2
En

e
y − = 0.
2

Example 1.37. Find F ′ (x) if F(x) = x2 + 1.
arn

√ √
Solution. Let y = x2 + 1, let u = x2 + 1 and y = u.
dy du
NowF ′ (x) = × .
du dx
Le

1 −1 1 x
= u 2 × 2x = √ x = √ .
2 u 2
x +1
dy
w.

Example 1.38. Find if (i) y = sin(x2 ) (ii) y = sin2 x.


dx
Solution. (i) Given y = sin(x2 ).
ww

Let u = x2 . Then y = sin u.


dy dy du
Now = × .
dx du dx
= cos u × 2x = cos(x2 )2x = 2x cos(x2 ).

Visit For More : www.LearnEngineering.in


Visit For More : www.LearnEngineering.in

Differential calculus 37

(ii) Given y = sin2 x


Let u = sin x. Then y = u2 .

dy dy du
Now = × .
dx du dx
= 2u cos x = 2 sin x cos x = sin 2x.

.in
Example 1.39. Differentiate the following functions

ng
(iv) y = esin x .
3 100
(i) y = (x − 1) .

eri
(v) y = sin(cos(tan x)).
1
(ii) y = √ .
3
x2 + x + 1 (vi) y = esec 3x .

e x−2
!9
gin
(iii) y = (2x + 1)5 (x3 − x + 1)4 . (vii) y = .
2x + 1

Solution. (i) Given y = (x3 − 1)100


En

Let u = x3 − 1. Then y = u100 .

dy dy du
Now = × .
arn

dx du dx
= 100u99 × 3x2 = 300x2 (x3 − 1)99 .

1
Le

(ii) Given y = √
3
x2 +x+1
−1
Let u = x2 + x + 1. Then y = u 3 .
w.

dy dy du
Now = × .
dx du dx
−1 −1 −1
= u 3 × (2x + 1)
ww

3
−1 −4 −1 −(2x + 1)
= u 3 × (2x + 1) = 4 × (2x + 1) = 4
.
3 3u 3 3(x2 + x + 1) 3

(iii) Given y = (2x + 1)5 (x3 − x + 1)4

Visit For More : www.LearnEngineering.in


Visit For More : www.LearnEngineering.in

38 Engineering Mathematics - I

−1
Let u = x2 + x + 1. Then y = u 3 .

dy d  
Now = (2x + 1)5 (x3 − x + 1)4
dx dx
d  3  d  
= (2x + 1)5 (x − x + 1)4 + (x3 − x + 1)4 (2x + 1)5
dx dx
   
= (2x + 1) 4(x − x + 1) (3x − 1) + (x − x + 1)4 5(2x + 1)4 × 2
5 3 3 2 3

.in
= 4(2x + 1)5 (x3 − x + 1)3 (3x2 − 1) + 10(x3 − x + 1)4 (2x + 1)4

= 2(2x + 1)4 (x3 − x + 1)3 {2(2x + 1)(3x2 − 1) + 5(x3 − x + 1)}

ng
= 2(2x + 1)4 (x3 − x + 1)3 {12x3 − 4x + 6x2 − 2 + 5x3 − 5x + 5)}

= 2(2x + 1)4 (x3 − x + 1)3 {17x3 + 6x2 − 9x + 3)}.

eri
(iv) Given y = esin x
dy
= esin x cos x. e
gin
dx
(v) Given y = sin (cos(tan x)).

dy
= cos (cos(tan x)) (− sin(tan x)) sec2 x
En

dx
= − cos (cos(tan x)) sin(tan x) sec2 x.
arn

(vi) Given y = esec 3x .

dy
Le

= esec 3x × sec 3x × tan 3x × 3 = 3 sec 3x tan 3xesec 3x .


dx

!9
x−2
w.

(vii) Given y = .
2x + 1
ww

!8 !
dy x−2 (2x + 1) × 1 − (x − 2) × 2
=9
dx 2x + 1 (2x + 1)2
!8
(x − 2)8
!
x−2 2x + 1 − 2x + 4
=9 = 45 .
2x + 1 (2x + 1)2 (2x + 1)10

Visit For More : www.LearnEngineering.in


Visit For More : www.LearnEngineering.in

Differential calculus 39

1.5 Implicit differentiation, logarithmic differentiation and


Inverse trigonometric function.

Whenever an equation containing two variables x and y, it is not always


dy
possible to express y interns of x and it is not easy to find directly. In such
dx
case, we differentiate both sides with respect to x and then solve the resultant

.in
dy
equation for .
✎ ☞ dx
Worked Examples

ng
✍ ✌
dy
Example 1.40. If x2 + y2 = 25, find . Find also the equation of the tangent to the
dx

eri
circle at the point (3, 4).
Solution. Given x2 + y2 = 25.
Differentiating both sides w.r.to x we get
e
gin
dy
2x + 2y =0
dx
dy
x+y =0
dx
En

dy dy −x
y = −x ⇒ = .
dx dx y
dy −3
arn

At (3, 4) = .
dx 4
−3
Slope of the tangent at (3, 4) = m = .
4
Equation of the tangent is
Le

−3
y−4= (x − 3)
4
w.

4y − 16 = −3x + 9

3x + 4y = 16 + 9
ww

3x + 4y = 25.

dy
Example 1.41. If x3 +y3 = 6xy, find . Find the tangent to the Folium of Descartes
dx
x3 + y3 = 6xy at the point (3, 3). At what point in the first quadrant is the tangent

Visit For More : www.LearnEngineering.in


Visit For More : www.LearnEngineering.in

40 Engineering Mathematics - I

line horizontal.
Solution. Given x3 + y3 = 6xy.
Differentiating both sides w.r.to x we get
" #
2 2 dy dy
3x + 3y =6 x +y×1
dx dx
" #
2 dy dy dy

.in
2
x +y = 2 x + y = 2x + 2y
dx dx dx
dy dy
y2 − 2x = 2y − x2
dx dx

ng
dy 2
(y − 2x) = 2y − x2
dx
dy 2y − x2

eri
= .
dx y2 − 2x
dy 2 × 3 − 32
At(3, 3) = = −1.
dx 32 − 2 × 3
e
gin
Slope of the tangent at (3, 3) = m = −1 .
The equation of the tangent is y − 3 = −1(x − 3) = −x + 3 ⇒ x + y = 6.
dy
The points at which the tangent line is horizontal are given by = 0.
dx
En

2y − x 2
i.e., 2 =0
y − 2x
i.e., 2y − x2 = 0
arn

x2
i.e., y = .
2
x2
Substituting y = in the equation of the curve we obtain
2
Le

!3
3 x2 x2
x + = 6x ×
2 2
w.

x6
!
x3 + = 3x3
8
x6
− 2x3 = 0
ww

8
3
!
3 x
x −2 =0
8
 
x3 x3 − 16 = 0

Visit For More : www.LearnEngineering.in


Visit For More : www.LearnEngineering.in

Differential calculus 41

x3 = 0 or x3 − 16 = 0

x3 = 0 or x3 = 16

But x , 0
4
∴ x3 = 16 = 24 ⇒ x = 2 3 .
4  4 2 8 8 5
when x = 2 3 , y = 12 2 3 = 12 2 3 = 2 3 −1 = 2 3 .

.in
4 5
∴ In the first quadrant, the tangent line is horizontal at (2 3 , 2 3 ).

ng
dy
Example 1.42. Find if sin(x + y) = y2 cos x.
dx
Solution. Given sin(x + y) = y2 cos x.

eri
Differentiating both sides w.r.to x we get
!
dy dy
cos(x + y) 1 + = y2 (− sin x) + cos x × 2y
dx
dy e dy
dx
gin
cos(x + y) + cos(x + y) = −y2 sin x + 2y cos x
dx dx
dy dy 2
cos(x + y) − 2y cos x = −y sin x − cos(x + y)
dx dx
dy
En

 
(cos(x + y) − 2y cos x) = − y2 sin x + cos(x + y)
dx  
2
dy − y sin x + cos(x + y)
=
arn

dx (cos(x + y) − 2y cos x)
y2 sin x + cos(x + y)
= .
2y cos x − cos(x + y)
Le

dy
Example 1.43. If sin y = x sin(a + y) find .
dx
Solution. Given sin y = x sin(a + y)
sin y
w.

∴x=
sin(a + y)
Differentiating with respect to y we get
ww

dx sin(a + y) cos y − sin y cos(a + y) sin(a + y − y) sin(a)


= 2
= 2
=
dy sin (a + y) sin (a + y) sin2 (a + y)
dy sin2 (a + y)
∴ = .
dx sin(a)

Visit For More : www.LearnEngineering.in


Visit For More : www.LearnEngineering.in

42 Engineering Mathematics - I

p √ dy 1
Example 1.44. If x 1 + y + y 1 + x = 0, prove that =− .
p √ p dx √ (1 + x)2
Solution. Given x 1 + y + y 1 + x = 0 ⇒ x 1 + y = −y 1 + x
squaring both sides we get

x2 (1 + y) = y2 (1 + x)

x2 + x2 y = y2 + y2 x

.in
x2 + x2 y − y2 − y2 x = 0

ng
x2 − y2 + x2 y − y2 x = 0

(x − y)(x + y) + xy(x − y) = 0

eri
(x − y)(x + y + xy) = 0

∴ x − y = 0 or x + y + xy = 0

i.e.,x = y or x + y + xy = 0
e
gin
But x , y.
∴ x + y + xy = 0 (1)
En

Differentiating both sides w.r.to x we get


!
dy dy
1+ + x +y =0
dx dx
arn

dy dy
1+ +x +y=0
dx dx
dy
(1 + x) = −y − 1
dx
Le

dy −(y + 1)
= .
dx (1 + x)
−x
w.

But form (1) we have y(1 + x) = −x ⇒ y = .


x
1+x
dy 1 − 1+x 1+x−x 1
∴ =− =− 2
=− .
dx 1+x (1 + x) (1 + x)2
ww

r q √ dy
Example 1.45. If y = x + x + x + · · · ∞, find .
r dx
q √
Solution. Given y = x + x + x + · · · ∞.

Visit For More : www.LearnEngineering.in


Visit For More : www.LearnEngineering.in

Differential calculus 43


y= x+y
y2 = x + y.
Differentiating w.r.to x we get
dy dy
2y =1+
dx dx
dy
(2y − 1) = 1
dx

.in
dy 1
= .
dx (2y − 1)
r

ng
q
dy
Example 1.46. If y = sin x + sin x + sin x + · · · ∞, find .
r dx
q √
Solution. Given y = sin x + sin x + sin x + · · · ∞.

eri
p
y = sin x + y.
Squaring on both sides we get
y2 = sin x + y.
e
gin
Differentiating w.r.to x we get
dy dy
2y = cos x +
dx dx
dy dy
En

2y − = cos x
dx dx
dy
(2y − 1) = cos x
dx
dy cos x
arn

= .
dx (2y − 1)

Example 1.47. Find y′′ if x4 + y4 = 16.


Solution. Given x4 + y4 = 16
Le

Differentiating w.r.to x we get

dy
w.

4x3 + 4y3 =0
dx
dy
x3 + y3 =0
ww

dx
3 dy
y = −x3
dx
dy x3
=− .
dx y3

Visit For More : www.LearnEngineering.in


Visit For More : www.LearnEngineering.in

44 Engineering Mathematics - I

Again differentiating w.r.to x


dy
d2 y y3 × 3x2 − x3 × 3y2
=− dx
dx2 y6
! !
2 2 dy 2 2 dy
−3x y y − x −3x y y − x
dx dx
= =
y6 y6

.in
−x3
!!
2
−3x y − x 3  
y −3x2 y4 + x4 −3x2 × 16 −48x2
= = = = .

ng
y4 y7 y7 y7
Derivative of Inverse Trigonometric functions

eri
dy
1. Find if y = sin−1 x.
dx
Solution. Given y = sin−1 x.
sin y = x.
e
gin
Differentiating w.r.to x
dy
cos y =1
dx
dy 1 1 1 1
= = p = q = √ .
En

dx cos y cos2 y 2 1 − x 2
1 − sin y

dy
2. If y = cos−1 x, find.
arn

dx
Solution. Given y = cos−1 x.
cos y = x.
Differentiating w.r.to x
Le

dy
− sin y =1
dx
dy −1 −1 −1 −1
= = q = p = √ .
w.

dx sin y 2 1 − cos 2y 1 − x 2
sin y

dy
3. If y = tan−1 x, find
ww

.
dx
Solution. Given y = tan−1 x.
tan y = x.
Differentiating w.r.to x

Visit For More : www.LearnEngineering.in


Visit For More : www.LearnEngineering.in

Differential calculus 45

dy
sec2 y =1
dx
dy 1 1 1
= = = .
dx sec y 1 + tan y 1 + x2
2 2

dy
4. If y = cot−1 x, find.
dx
Solution. Given y = cot−1 x.

.in
cot y = x.
Differentiating w.r.to x
dy

ng
−cosec2 y =1
dx
dy −1 −1 −1
= = = .
dx cosec y 1 + cot y 1 + x2
2 2

eri
dy
5. If y = sec−1 x, find.
dx
Solution. Given y = sec−1 x.
sec y = x. e
gin
Differentiating w.r.to x
dy
sec y tan y =1
dx
dy 1 1 1 1
En

= = p = p = √ .
dx sec y tan y x tan2 y x sec2 y − 1 x x2 − 1

dy
6. If y = cosec−1 x, find.
arn

dx
Solution. Given y = cosec−1 x.
cosecy = x.
Le

Differentiating w.r.to x
dy
−cosecy cot y =1
dx
dy −1 −1 −1 −1
w.

= = p = p = √ .
dx cosecy cot y x cot y x cosec y − 1 x x2 − 1
2 2

✎ ☞
Worked Examples
ww

✍ ✌
1 √ 
Example 1.48. Find the derivatives of (i) −1 (ii) x tan−1 x.
sin x
1 
−1 −1

Solution. (i) Let y = = sin x .
sin−1 x

Visit For More : www.LearnEngineering.in


Visit For More : www.LearnEngineering.in

46 Engineering Mathematics - I

dy  −2 1 1
= (−1) sin−1 x √ = 2 √ .
dx 1 − x2 sin−1 x

1 − x2
√ 
(ii) Let y = x tan−1 x .

dy 1 −1 √ x .1 = x −1 √ x .
   
1
= (x) √ 2 × 2√ x
+ tan 2(1+x) + tan
dx 1 + ( x)
√ 
Example 1.49. Differentiate w.r.to x (i) sin−1 x (ii) (1 + x2 ) tan−1 x

.in
 cos x 
(iii) sec−1 (x4 ) (iv) tan−1 .
√1 + sin x
Solution. (i) Let y = sin−1 ( x).
dy 1 1 1 1

ng
dx
= q  √ 2 × 2 √ x = 2 √ x √1 − x = 2 √ x(1 − x) .
1− x
(ii) Let (1 + x2 ) tan−1 x.

eri
dy 1
= (1 + x2 ) × + tan−1 x × 2x = 1 + 2x tan−1 x.
dx 1 + x2

(iii) Let sec−1 (x4 ).


e
gin
dy 1 4
= q × 4x3 = √ .
dx 4 4
2 x x 8−1
x x −1
 cos x 
(iv) Let y = tan−1
En

.
1 + sin
 x 
 sin π2 − x 
 
−1 
= tan     .
π
1 + cos 2 − x
arn

  
 sin 2 π4 − 2x 

= tan−1     .
π x 
1 + cos 2 4 − 2
Le

   
 2 sin π4 − 2x cos π4 − 2x 

= tan−1     .
2 π x
2 cos 4 − 2

  π x 
w.

= tan−1 tan − .
π x 4 2
y= −
4 2
ww

dy −1
= .
dx 2
!
−1 2x dy
Example 1.50. If y = tan 2
find .
1−x dx

Visit For More : www.LearnEngineering.in


Visit For More : www.LearnEngineering.in

Differential calculus 47

!
−1 2x
Solution. Given y = tan .
1 − x2
Let x = tan θ !
−1 2 tan θ
∴y = tan = tan−1 (tan 2θ) = 2θ
1 − tan2 θ
y = 2 tan−1 x.
dy 2
= .
dx 1 + x2

.in
  dy
Example 1.51. If sin−1 3x − 4x3 find .
  dx
−1 3

ng
Solution. Let y = sin 3x − 4x .
Let x = sin θ.
y = sin−1 (3 sin θ − 4 sin3 θ) = sin−1 (sin 3θ) = 3θ = 3 sin−1 x.

eri
dy 3
= √ .
dx 1 − x2

Example 1.52. If y = tan−1


 a−x 
find
dy
e
.
gin
 a − x  1 + ax dx
Solution. y = tan−1
1 + ax
Let a = tan α, x = tan θ.
 tan α − tan θ 
En

∴ y = tan−1 = tan−1 (tan(α − θ)) = α − θ = α − tan−1 (x).


1 + tan α tan θ
dy −1
= .
dx 1 + x2
arn

Logarithmic Differentiation
dy
Result. If y = loge x, find .
dx
Solution. Given y = loge x
Le

∴ x = ey .
Differentiating w.r.to y
w.

dx
= ey = x
dy
dy 1
= .
ww

dx x
dy
Note. (i) If y = b x find .
dx
Solution. Given y = b = e x log b
x

Differentiating w.r.to x

Visit For More : www.LearnEngineering.in


Visit For More : www.LearnEngineering.in

48 Engineering Mathematics - I

dy
= e x log b log b = b x log b.
dx
dy
(ii) If y = logb x find .
dx
Solution. Given y = logb x
∴ by = x.
Differentiating w.r.to x

.in
dy
by log b = 1.
dx
dy
= y1 = 1 .
dx b log b x log b ✎ ☞

ng
Worked Examples
✍ ✌

eri
Example 1.53.! Differntiate (i) log(x3 + 1) (ii) log(sin x) (iii) log(2 + sin x)
x+1 p
(iv) log √ (v) log |x| (vi) log(x).
x+2
e
gin
Solution. (i) Let y = log(x3 + 1).
dy 1 3x2
En

= 3 3x2 = 3 .
dx x + 1 x +1
(ii) Let y = log(sin x).
arn

dy 1
= cos x = cot x.
dx sin x
(iii) Let y = log10 (2 + sin x).
dy 1 cos x
Le

= cos x = .
dx (2 + sin x) log 10 (2 + sin x) log 10
!
x+1
(iv) Let y = log √ .
w.

x+2
√ 
= log (x + 1) − log x + 2 .
ww

= log (x + 1) − log (x + 2)1/2 .


1
= log (x + 1) −
log (x + 2) .
2
dy 1 1
= − .
dx (x + 1) 2(x + 2)

Visit For More : www.LearnEngineering.in


Visit For More : www.LearnEngineering.in

Differential calculus 49


log x , if x > 0



(v) Let y = log |x| =  .


log(−x) , if x < 0


 
1 1
, if x > 0  x , if x > 0
 
dy 
 

x
 
= = .
dx 


 1 (−1) , if x < 0 

  1 , if x < 0

−x x

.in
dy 1
∴ = for allx , 0.
dx x
(vi) Let y = log x = log x 1/2 .
p 

ng
dy 1 1 1
log x −1/2 =

= p .
dx 2 x 2x log x

eri
3 √
x 4 x2 + 1 dy
Example 1.54. If y = 5
, find .
(3x
√ + 2) dx
3
x 4 x2 + 1
Solution. Given y =
(3x + 2)5
e
gin
Taking log on both sides we get
  p
log y = log x3/4 + log x2 + 1 − log(3x + 2)5
3 1
En

= log x + log(x2 + 1) − 5 log(3x + 2).


4 2
Differentiating w.r.to x we get
arn

1 dy 3 1 1 1 1 3 x 15
= × + × 2 2x − 5 ×3= + 2 −
y dx 4 x 2 x + 1 3x + 2√ 4x x + 1 3x + 2
" # 3 " #
dy 3 x 15 2
x x +1 3
4 x 15
∴ =y + 2 − = + − .
dx 4x x + 1 3x + 2 (3x + 2)5 4x x2 + 1 3x + 2
Le

√ dy
x
Example 1.55. If y = x find .
√ dx
Solution. Given y = x x .
w.


log y = x log x.
Differentiating w.r.t x
ww

1 dy √ 1 1
= x + log x √
y dx x 2 x
1 1
= √ + √ log x
x 2 x

Visit For More : www.LearnEngineering.in


Visit For More : www.LearnEngineering.in

50 Engineering Mathematics - I

!
1 1
= √ 1 + log x
x 2
1 
= √ 2 + log x
2 x

dy 1 x x
=y √ = √ .
dx 2 x 2 + log x 2 x 2 + log x

.in
d
Example 1.56. Find (sin x)cos x .

[A.U. Jan. 2015]
dx
Solution. Let y = (sin x)cos x .

ng
Taking logarithms on both sides we get
log y = log (sin x)cos x = cos x log (sin x).

eri
Differentiating w.r.to x.

1 dy 1
= cos x cos x + log(sin x)(− sin x)
y dx
cos2 x
sin x
e
gin
= − sin x log(sin x)
sin
" x2 # " 2 #
dy cos x cos x cos x
=y − sin x log(sin x) = (sin x) − sin x log(sin x) .
dx sin x sin x
En

dy
Example 1.57. If y = (sin x) x , find .
dx
Solution. Let y = (sin x) x .
arn

Taking log both sides we get


log y = log (sin x) x = x log sin x.
Differentiating w.r.t. x.
Le

1 dy 1
=x cos x + log sin x.1
y dx sin x
w.

= x cot x + log sin x.


dy
= y x cot x + log sin x = (sin x) x x cot x + log sin x .
   
ww

dx

x··· dy
xx
Example 1.58. If y = x , find .
dx
···∞
xx
Solution. Let y = x x ⇒ y = xy .

Visit For More : www.LearnEngineering.in


Visit For More : www.LearnEngineering.in

Differential calculus 51

Taking log both sides we get


log y = log xy = y log x.
Differentiating w.r.t. x.
1 dy 1 dy
= y + log x .
y dx x dx
1 dy dy y
− log x = .
y dx dx x

.in
!
dy 1 y
− log x = .
dx y ! x
dy 1 − y log x y

ng
= .
dx y x
dy y2
= .
dx x(1 − y log x)

eri
Hyperbolic functions

e x − e−x
We define sinh x =
2
e
gin
e x + e−x
cosh x =
2
sinh x
tanh x =
cosh x
En

1
cosechx =
sinh x
1
sechx =
arn

cosh x
cosh x
coth x =
sinh x

Hyperbolic identities
Le

1. sinh(−x) = − sinh x.
w.

2. cosh(−x) = cosh x.

3. cosh2 x − sinh2 x = 1.
ww

4. 1 − tanh2 x = sech2 x.

5. sinh(x + y) = sinh x cosh y + cosh x sinh y.

Visit For More : www.LearnEngineering.in


Visit For More : www.LearnEngineering.in

52 Engineering Mathematics - I

6. cosh(x + y) = cosh x cosh y + sinh x sinh y.

Derivatives of hyperbolic functions.


d
1. Prove that (sinh x) = cosh x.
dx
d e x − e−x e x + e−x
! !
d 1 x −x
Proof. (sinh x) = = (e − e (−1)) = = cosh x.
dx dx 2 2 2

.in
In the same way we can establish the following.
d
2. (cosh x) = sinh x.
dx

ng
d
3. (tanh x) = sech2 x.
dx

eri
d
4. (cosechx) = −cosechx coth x.
dx
d
5. (sechx) = sechx tanh x.
dx
e
gin
d
6. (coth x) = −cosech2 x.
dx
Inverse hyperbolic functions.
En

 p 
−1 2
1. Prove that sinh x = log x + x + 1 .
Proof. Let y = sinh−1 x.
ey − e−y
.
arn

∴ x = sinh y =
2
i.e., ey − e−y = 2x.
1
ey − y − 2x = 0.
e
Le

e2y − 2xey − 1 = 0.
This is a quadratic
√ in ey .√Solving for ey we get
2x ± 4x2 + 4 x ± x2 + 1 p
w.

ey = = = (x ± x2 + 1).
2 1
We have ey > 0 always.

But x − x2 − 1 < 0.
ww


∴ ey = x − x2 + 1 is not possible.
√ √
∴ ey = x + x2 + 1 y = log(x + x2 + 1).
In a similar way we can prove the following.

Visit For More : www.LearnEngineering.in


Visit For More : www.LearnEngineering.in

Differential calculus 53

 p 
2. cosh−1 x) = log x + x2 − 1 .
!
−1 1 1+x
3. tanh x) = log .
2 1−x
Differentiation of Inverse hyperbolic functions.
d 1
1. Prove that (sinh−1 x) = √ .

.in
dx x2 + 1
Proof. y = sinh−1 x.
∴ sinh y = x.

ng
Differentiating w.r.t. x we get
dy
cosh y = 1.

eri
dx
dy 1 1 1 1
= = q = q = √ .
dx cosh y 1 + x2
cosh2 y 1 + sinh2 y

e
In a similar way we can prove the following.
gin
d 1
2. (cosh−1 x) = √ .
dx x2 − 1
d 1
3. (tanh−1 x) = .
En

dx 1 − x2
d −1
4. (cosech−1 x) = √ .
dx |x| 1 + x2
arn

d −1
5. (sech−1 x) = √ .
dx x 1 − x2
d 1
Le

6. (coth−1 x) = .
dx 1 − x2
Exercise 1 (E)
w.

1. Find the derivatives of the following functions.


ww

(i) x2 − 4xy + y2 = 4 (iv) y cos x = x2 + y2



(ii) x4 + x2 y2 + y3 = 5 (v) x + y = x4 + y4
x
x2
(iii) x+y = y2 + 1 (vi) e y = x − y

Visit For More : www.LearnEngineering.in


Visit For More : www.LearnEngineering.in

54 Engineering Mathematics - I

(ix) cos(xy) = 1 + sin y


(vii) tan−1 (x2 y) = x + xy2

(viii) sin(xy) = cos(x + y) (x) x sin y + y sin x = 1

dy
2. If (sin x)cos y = (sin y)cos x , find .

.in
dx

sin x··· dy
3. If y = (sin x)sin x , find .
dx

ng
dy
4. If x1+y + y1+x = a, where a is a constant, find .
dx

eri
5. Find the derivatives of the following using logarithmic differentiation.

(iv) xsin x
(i) (x2 + 2)2 (x4 + 4)4
e
(v) (cos x) x
gin
q
(x−1)
(ii) (x4 +1)
1
(iii) x x (vi) (tan x) x
En

6. Find the equation of the tangent line to the curve y sin 2x = x cos 2y at ( π2 , π4 ).

7. Find the equation of the tangent line to the hyperbola x2 − xy − y2 = 1 at the


arn

point (2, 1).

8. Find the equation to the tangent line to the Cardioid x2 + y2 = (2x2 + 2y2 − x)2
Le

 
at 0, 21 .

9. Find the derivative of the following functions.


w.


√ (v) cosh−1 ( x).
(i) tanh x.

ww

 
(vi) x sinh−1 x
3 − 9 + x2 .
(ii) sinh(log(x)).

(iii) ecosh 3x . (vii) coth−1 (sec x).



(iv) x coth 1 + x2 . (viii) tanh−1 (sin x).

Visit For More : www.LearnEngineering.in


Visit For More : www.LearnEngineering.in

Differential calculus 55

10. At what point of the curve y = cosh x does the tangent have slope 1.

Definition. Let c be a number in the domain D of a function f . Then f (c) is the


absolute maximum value of f on D if f (c) ≥ f (x) for all x in D. f (c) is the absolute
minimum value of f on D if f (c) ≤ x for all x in D.
Maximum and minimum values of f are called extreme values of f .

.in
Definition. The number f (c) is a local maximum value of f if f (c) ≥ f (x) when x
is near c. f (c) is the local minimum value of f if f (c) ≤ f (x) when x is near c.

ng
Increasing and decreasing test

(i) If f ′ (x) > 0, on an interval, then f is increasing on that interval.

eri
(ii) If f ′ (x) < 0, on an interval, then f is decreasing on that interval.

e
Fermat’s theorem. If f has a local maximum or minimum at c, and f ′ (c) exits,
gin
then f ′ (c) = 0.
Critical number. A critical number of a function f is a number c in the domain
of f such that either f ′ (c) = 0 or f ′ (c) does not exist.
En

Result. If f has a local maximum or minimum at c, then c is a Critical number


of f .
arn

First derivative test. Suppose that c is a critical number of a continuous


function f .
Le

(i) If f ′ changes from positive to negative at c, then f has a local maximum at c.

(ii) If f ′ changes from negative to positive at c, then f has a local minimum at c.


w.

(iii) If f ′ is positive to the left and right of c or negative to the left and right of c,
then f has no local maximum or minimum at c.
ww

Second derivative test. Suppose f ′′ is continuous near c.

(i) If f ′ (c) = 0 and f ′′ (c) > 0, then f has a local minimum at c.

Visit For More : www.LearnEngineering.in


Visit For More : www.LearnEngineering.in

56 Engineering Mathematics - I

(ii) If f ′ (c) = 0 and f ′′ (c) < 0, then f has a local maximum at c.

Concavity. If the graph of f lies above all of its tangents on an interval I, then it
is called concave upward on I. If the graph of f lies below all of its tangents on I,
it is called concave downward on I.

.in
Concavity test

(i) If f ′′ (x) > 0, for all x in I, then the graph of f is concave upward on I.

ng
(ii) If f ′′ (x) < 0, for all x in I, then the graph of f is concave downward on I.

eri
Point of inflexion. A point P on a curve y = f (x) is called a point of inflexion

e
if f is continuous there and the curve changes from concave upward to concave
gin
downward or from concave downward to concave upward at P.
Note. Points of inflection occurs at points where f ′′ (x) = 0.
✎ ☞
Worked Examples
✍ ✌
En

Example 1.59. Find the intervals on which the function f (x) = sin x + cos x is
arn

increasing or decreasing. Also find the curve is concave upwards or concave


downwards. [A.U.Jan.2015]
Solution. f (x) = sin x + cos x.
Le

f ′ (x) = cos x − sin x.


f ′′ (x) = − sin x − cos x.
w.

At critical points f ′ (x) = 0


i.e., cos x − sin x = 0.
ww

i.e., sin x = cos x.


i.e., tan x = 1.
π 5π
∴x= , .
4 4
Let us evaluate the intervals at which f decrease or increases.

Visit For More : www.LearnEngineering.in


Visit For More : www.LearnEngineering.in

Differential calculus 57

Interval f ′ (x) f
π π
0<x< + increases in (0, )
4 4
π 5π π 5π
<x< - decreases in ( , )
4 4 4 4
5π 5π
< x < 2π + increases in ( , 2π)
4 4
π
Since f ′ changes from positive to negative at x = ,
4

.in
π
f has a maximum at x = .
4
π π π 1 1 √
Maximum value of f = f ( ) = sin + cos = √ + √ = 2.

ng
4 4 4 2 2
′ 5π
Also f changes from negative to positive at x = .
4
5π 5π 5π 1 1 √

eri
Minimum value of f = f ( ) = sin + cos = − √ − √ = − 2.
4 4 4 2 2
Let us evaluate the intervals at which the curve is concave upwards or
concave downwards.
Making f ′′ = 0 we get e
gin
− sin x − cos x = 0.
i.e., sin x + cos x = 0. !
π  3π
En

cos x = − sin x = cos +x (or) cos x = cos −x


 π2  π
2
Taking cos x = cos + x implies x = + x
2 2
π
arn

i.e., = 0 which is absurd.


2 !
3π 3π
Taking cos x = cos − x implies x = −x
2 2
3π 3π
Le

i.e., 2x = ⇒x=
2 4 ! !
3π 3π
The interval to be considered are 0, , , 2π
4 4
w.

Interval f ′ (x) f

0<x< - concave downwards
4
ww


< x < 2π + concave upwards
4
3π 3π
Since f ′′ changes sign from negative to positive at x = ,x= is a
4 4
point of inflexion.

Visit For More : www.LearnEngineering.in


Visit For More : www.LearnEngineering.in

58 Engineering Mathematics - I

Example 1.60. Find the intervals on which the function f (x) = x4 − 2x2 + 3 is
increasing or decreasing. Also find the local maximum and minimum values of f
by using first derivative test. [A.U.Nov.2016]
Solution. f (x) = x4 − 2x2 + 3.
f ′ (x) = 4x3 − 4x = 4x(x2 − 1) = 4x(x − 1)(x + 1).
Critical numbers occur at points where f ′ (x) = 0

.in
i.e., 4x(x − 1)(x + 1) = 0.
x = −1, x = 0, x = 1.

ng
The critical numbers are x = −1, x = 0, x = 1.
Let us evaluate the intervals at which f decreases or increases.

eri
Interval 4x x−1 x+1 f ′ (x) f
x < −1 - - - - decreases in (−∞, −1)
−1 < x < 0 - - +
e + increases in (−1, 0)
gin
0<x<1 + - + - decreases in (0, 1)
x>1 + + + + increases in (1, ∞)

Since f ′ changes from negative to positive at x = −1,


En

f has a local minimum at x = −1.


Minimum value of f = f (−1) = 1 − 2 + 3 = 1.
arn

Also f ′ changes from positive to negative at x = 0.


∴ f has a maximum at x = 0.
Maximum value of f = f (0) = 3.
Le

Again f ′ changes from negative to positive at x = 1.


∴ f has a local minimum at x = 1.
w.

Minimum value of f = 1 − 2 + 3 = 2.

Example 1.61. Find the intervals on which the function f (x) = 3x4 − 4x3 − 12x2 + 5
ww

is increasing and decreasing. Also find the local maximum and minimum values
of f by using first derivative test. [A.U.Nov.2016]
Solution. f (x) = 3x4 − 4x3 − 12x2 + 5.

Visit For More : www.LearnEngineering.in


Visit For More : www.LearnEngineering.in

Differential calculus 59

f ′ (x) = 12x3 − 12x2 − 24x = 12x(x2 − x − 2) = 12x(x − 2)(x + 1).


Critical numbers occur at points where f ′ (x) = 0
i.e., 12x(x − 2)(x + 1) = 0.
x = −1, x = 0, x = 2.
The critical numbers are x = −1, x = 0, x = 2.

.in
Let us evaluate the intervals at which f decreases or increases.

Interval 12x x−2 x+1 f ′ (x) f

ng
x < −1 - - - - decreases in (−∞, −1)
−1 < x < 0 - - + + increases in (−1, 0)
0<x<2 + - + - decreases in (0, 2)

eri
x>2 + + + + increases in (2, ∞)

Since f ′ changes from negative to positive at x = −1,


f has a local minimum at x = −1. e
gin
Minimum value of f = f (−1) = 3 + 4 − 12 + 5 = 20.
Also f ′ changes from positive to negative at x = 0.
∴ f has a local maximum at x = 0.
En

Maximum value of f = f (0) = 5.


Again f ′ changes from negative to positive at x = 2.
arn

∴ f has a minimum at x = 2.
Minimum value of f = 3 × 16 − 4 × 8 − 12 × 4 + 5 = 48 − 32 − 48 + 5 = −27.
Le

Example 1.62. Find the local maximum and minimum values of the function
f (x) = x + 2 sin x, 0 ≤ x ≤ 2π.
w.

Solution. f (x) = x + 2 sin x.


f ′ (x) = 1 + 2 cos x.
f ′′ (x) = −2 sin x.
ww

Local maximum and minimum occur at points where f ′ (x) = 0.


i.e., 1 + 2 cos x = 0.
2 cos x = −1.

Visit For More : www.LearnEngineering.in


Visit For More : www.LearnEngineering.in

60 Engineering Mathematics - I

−1
cos x = 2 .
cos x is negative in the II and III quadrants.
2π 4π
x= 3 and x = 3 .
First derivative test
2π 2π 4π 4π
The intervals to be considered are 0 < x < , <x< , < x < 2π.
3 3 3 3

.in
Let us evaluate the intervals at which f decreases and increases.

Interval f ′ (x) = 1 + 2 cos x f


ng
0<x< + increasing
3
2π 4π
<x< - decreasing
3 3

eri
< x < 2π + increasing
3

Since f ′ changes from positive to negative at 3 ,
f has a local maximum at x =
e 2π
3 .
gin
√ √
2π 3
∴ Maximum value of f = 3 + 2 sin 2π
3 =

3 +2 2 = 2π
3 + 3.

Since f ′ changes from negative to positive at x = 3 ,

f has a minimum at x = 3 .
En

 √  √
4π − 3
∴ Minimum value of f = 3 + 2 sin 4π
3 =

3 +2 2 = 4π
3 − 3.
Second derivative test
√ √
arn

2π 3
When x = 3 , f ′′ (x) = −2 sin 2π
3 = −2 2 = − 3 < 0.

∴ f has a maximum at x = 3 .


Maximum value of f = 3 + 3.
Le

√ √
4π − 3
When x = f ′′ (x) = −2 sin 4π
3 , 3 = −2 2 = 3 > 0.

∴ f has a minimum at x = .
3
w.

4π √
∴ Minimum value of f = − 3.
3
ww

Example 1.63. Find the local maxima and minima and the points of inflexion for
the function f (x) = x4 − 4x3 .
Solution. f (x) = x4 − 4x3 .
f ′ (x) = 4x3 − 12x2 .

Visit For More : www.LearnEngineering.in


Visit For More : www.LearnEngineering.in

Differential calculus 61

f ′′ (x) = 12x2 − 24x.


Local maximum and minimum occur at points where f ′ (x) = 0
i.e., 4x3 − 12x2 = 0.
4x2 (x − 3) = 0.
x = 0, x = 3.
When x = 0, f ′′ (x) = 0.

.in
∴ f (x) does not have either mxaimum or minimum.
When x = 3, f ′′ (x) = 12 × 9 − 24 × 3 = 108 − 72 = 36 > 0

ng
∴ f has a local minimum at x = 3.
Minimum value of f = 81 − 4 × 27 = 81 − 108 = −27.

eri
At the points of inflexion, f ′′ (x) = 0
12x2 − 24x = 0 ⇒ 12x(x − 2) = 0 ⇒ x = 0, x = 2.
When x = 0, f (0) = 0.
e
gin
∴ (0, 0) is a point of inflexion.
When x = 2, f (2) = 24 − 4 × 23 = 16 − 32 = −16.
Another point of inflexion is (2, −16).
En

Example 1.64. Find the maxima and minima of the function f (x) = x2/3 (6 − x)1/3 .
Solution. f (x) = x2/3 (6 − x)1/3 .
arn

1 2
f ′ (x) = x2/3 (6 − x)−2/3 (−1) + (6 − x)1/3 x−1/3 .
3 3
2/3 1/3
" #
1 −x 2(6 − x)
Le

= + .
3 (6 − x)2/3 x1/3
" #
1 −x + 2(6 − x)
=
3 x1/3 (6 − x)2/3
w.

" #
1 −x + 12 − 2x
=
3 x1/3 (6 − x)2/3
ww

" # " #
1 12 − 3x 4−x
= = 1/3 .
3 x1/3 (6 − x)2/3 x (6 − x)2/3
Now, f ′ (x) = 0 when x = 4 and f ′ (x) does not exist when x = 0 and x = 6.
∴ The critical numbers are 0, 4, 6.

Visit For More : www.LearnEngineering.in


Visit For More : www.LearnEngineering.in

62 Engineering Mathematics - I

Let us evaluate the intervals at which f increases or decreases.

Interval 4−x x1/3 (6 − x)2/3 f ′ (x) f


−∞ < x < 0 + - + - decreases on (−∞, 0)
0<x<4 + + + + increases on (0, 4)
4<x<6 - + + - decreases on (4, 6)
- + + - decreases on (6, ∞)

.in
x>6

Since f ′ changes from negative to positive at x = 0, f has a local minimum at x = 0.


∴ Minimum value of f = f (0) = 0.

ng
Also f ′ changes from positive to negative at x = 4.
∴ f has a local maximum at x = 0.

eri
∴ Maximum value of f = f (4) = 42/3 (6 − 4)1/3 = (22 )2/3 (21/3 ) = 24/3 21/3 = 25/3 .
Again f ′ does not changes sign at x = 6.
∴ f has no maximum or minimum at x = 6.
e
gin
Example 1.65. Using first derivative test, examine for maximum and minimum
of the function f (x) = x3 − 3x + 3, x ∈ R.
En

Solution. f (x) = x3 − 3x + 3.
f ′ (x) = 3x2 − 3 = 3(x2 − 1) = 3(x − 1)(x + 1).
Critical numbers occur at points where f ′ (x) = 0.
arn

i.e., 3(x − 1)(x + 1) = 0.


i.e., x = −1, x = 1.
Le

The critical numbers are x = −1, 1.


Let us evaluate the intervals at which f decreases or increases.
w.

Interval x−1 x+1 f ′ (x) f


x < −1 - - + increases in (−∞, −1)
−1 < x < 1 - + - decreases in (−1, 1)
ww

x>1 + + - increases in (1, ∞)

Since f ′ changes from positive to negative at x = −1.


f has a local maximum at x = −1.

Visit For More : www.LearnEngineering.in


Visit For More : www.LearnEngineering.in

Differential calculus 63

∴ Maximum value of f = f (−1) = (−1)3 − 3(−1) + 3 = −1 + 3 + 3 = 5.


Also f ′ changes from negative to positive at x = 1.
∴ f has a local minimum at x = 1.
∴ Minimum value of f = f (1) = 13 − 1 × 3 + 3 = 1 − 3 + 3 = 1.

Example 1.66. Using first derivative test examine the maximum and minimum

.in
of f (x) = sin2 x, 0 < x < π.
Solution. f (x) = sin2 x.

ng
f ′ (x) = 2 sin x cos x = sin 2x.
Critical numbers occur at points where f ′ (x) = 0.

eri
i.e., sin 2x = 0.
∴ 2x = 0, or 2x = π.
∴ x = 0, or x = π2 .
The critical numbers are x = 0, x = π2 . e
gin
Let us evaluate the intervals at which f decreases or increases.

Interval sin 2x f ′ (x) f


En

π
0<x< 2 + + increases in (0, π2 )
π
x> 2 - - decreases in ( π2 , π)

Since f ′ changes from positive to negative at x = π2 .


arn

∴ f has a maximum at x = π2 .
 
∴ Maximum value of f = f ( π2 ) = sin2 π
2 = 1.
Le

Example 1.67. Find the maximum and minimum of the function


f (x) = 2x3 − 3x2 − 36x + 10.
w.

Solution. f (x) = 2x3 − 3x2 − 36x + 10.


f ′ (x) = 6x2 − 6x − 36.
f ′′ (x) = 12x − 6.
ww

Critical pints occur at f ′ (x) = 0.


i.e., 6x2 − 6x − 36 = 0.
∴ x2 − x − 6 = 0.

Visit For More : www.LearnEngineering.in


Visit For More : www.LearnEngineering.in

64 Engineering Mathematics - I

∴ (x − 3)(x + 2) = 0.
∴ x = −2, x = 3.
The critical points are at x = −2, x = 3.
At x = −2, f ′′ (x) = 12 × (−2) − 6 = −24 − 6 = −30 = −ve
∴ f is maximum at x = −2.

.in
Maximum value of
f = 2(−2)3 − 3(−2)2 − 36(−2) + 10 = −16 − 12 + 72 + 10 = 54.
At x = 3, f ′′ (x) = 12 × 3 − 6 = 36 − 6 = 30 = +ve.

ng
∴ f has minimum at x = 3.
∴ Minimum value of

eri
f = f (3) = 2 × 33 − 3 × 32 − 36 × 3 + 10 = 54 − 27 − 108 + 10 = −71.

Example 1.68. Find the maxima and minima of the function f (x) = sin x(1 + cos x),
0 ≤ x ≤ 2π. e
gin
Solution. Given f (x) = sin x(1 + cos x)
f ′ (x) = sin x(0−sin x)+(1+cos x)(cos x) = − sin2 x+cos x+cos2 x = cos x+cos 2x.
f ′′ (x) = − sin x − 2 sin 2x.
En

At critical points f ′ (x) = 0.


arn

cos x + cos 2x = 0

cos x + 2 cos2 x − 1 = 0

i.e., 2 cos2 x + cos x − 1 = 0


Le

√ √
−1 ± 1 − 4 × 2 × (−1) −1 ± 1 + 8 −1 ± 3
cos x = = =
2×2 4 4
−1 + 3 1 −1 − 3
w.

cos x = = or cos x = = −1.


4 2 4
1 π 5π
cos x = gives x = and .
ww

2 3 3
cos x = −1 gives x = π.
π √ √ √
When x = , f ′′ (x) = − sin π3 − 2 sin 2π
3 = − 23 − 2 23 = − 3 2 3 < 0.
3
π
∴ f has a maximum at x = .
3

Visit For More : www.LearnEngineering.in


Visit For More : www.LearnEngineering.in

Differential calculus 65

√ √
      3
Maximum value of f = f π π π
= sin 3 1 + cos 3 =
3 (1 + 21 ) = 3 4 3 .
5π ′′  √   √  √2 √ √
When x = , f (x) = − sin 3 − 2 sin 3 = − 2 − 2 − 23 = 23 + 2 23 = 3 2 3 > 0.
5π 10π − 3
3

∴ f has a minimum at x = .
3 √
      − 3 √ √
5π 5π
∴ Minimum value of f = f 3 = sin 3 1 + cos 5 3 = π
(1 + 21 ) = − 2 3 × 23 = −34 3 .
2

.in
When x = π, f ′′ (x) = − sin π − 2 sin 2π = 0.
∴ f has neiher maximum nor minimum at x = π.

3 3

ng
∴ Maximum value = 4√ .
−3 3
∴ Minimum value = 4 .

e eri
gin
En
arn
Le
w.
ww

Visit For More : www.LearnEngineering.in


Visit For More : www.LearnEngineering.in

.in
ng
eri
e
gin
En
arn
Le
w.
ww

Visit For More : www.LearnEngineering.in


Visit For More : www.LearnEngineering.in

2 Function of Several Variables

.in
2.1 Limits and Continuity

ng
Limit. Let f (x, y) be a function of two independent variables x and y. f (x, y) is said

eri
to have the limit ℓ as (x, y) tends to (x0 , y0 ), if corresponding to any positive number
ǫ, however small, there is a positive number η such that

e
| f (x, y) − ℓ| < ǫ where 0 < d < η,
gin
where d is the distance between (x, y) and (x0 , y0 ) given by d2 = (x − x0 )2 + (y − y0 )2 .

Note. If the limit exists, then it can be written as lim f (x, y) = ℓ.


En

x→x0
y→y0

Continuity. f (x, y) is said to be continuous at (x0 , y0 ) if, to any positive number ǫ,


arn

however small, there corresponds a positive number η such that

| f (x, y) − f (x0 , y0 )| < ǫ where 0 < d < η,


Le

where d is the distance between (x, y) and (x0 , y0 ) given above.

Result 1. The above definition is equivalent to lim f (x, y) = f (x0 , y0 ).


x→x0
w.

y→y0
Result 2. Usually the limit is the same irrespective of the path along which
the point (x, y) approaches (x0 , y0 ).
ww

( )  
i.e., lim lim f (x, y) = lim lim f (x, y) .
x→x0 y→y0 y→y0 x→x0

But this is not true always.

Visit For More : www.LearnEngineering.in


Visit For More : www.LearnEngineering.in

2 Function of Several Variables

.in
2.1 Limits and Continuity

ng
Limit. Let f (x, y) be a function of two independent variables x and y. f (x, y) is said

eri
to have the limit ℓ as (x, y) tends to (x0 , y0 ), if corresponding to any positive number
ǫ, however small, there is a positive number η such that

e
| f (x, y) − ℓ| < ǫ where 0 < d < η,
gin
where d is the distance between (x, y) and (x0 , y0 ) given by d2 = (x − x0 )2 + (y − y0 )2 .

Note. If the limit exists, then it can be written as lim f (x, y) = ℓ.


En

x→x0
y→y0

Continuity. f (x, y) is said to be continuous at (x0 , y0 ) if, to any positive number ǫ,


arn

however small, there corresponds a positive number η such that

| f (x, y) − f (x0 , y0 )| < ǫ where 0 < d < η,


Le

where d is the distance between (x, y) and (x0 , y0 ) given above.

Result 1. The above definition is equivalent to lim f (x, y) = f (x0 , y0 ).


x→x0
w.

y→y0
Result 2. Usually the limit is the same irrespective of the path along which
the point (x, y) approaches (x0 , y0 ).
ww

( )  
i.e., lim lim f (x, y) = lim lim f (x, y) .
x→x0 y→y0 y→y0 x→x0

But this is not true always.

Visit For More : www.LearnEngineering.in


Visit For More : www.LearnEngineering.in

68 Engineering Mathematics - I

Consider the following example.


x − 2y
Let f (x, y) = .
x + 2y
As (x, y) → (0, 0) along the line y = mx, we have

x − 2y
lim f (x, y) = lim
(x,y)→(0,0) (x,y)→(0,0) x + 2y
x − 2mx

.in
= lim
x→0 x + 2mx
x(1 − 2m)
= lim
x→0 x(1 + 2m)

ng
1 − 2m
= ,
1 + 2m

eri
which is (different for) lineswith different slopes.
x − 2y x
Also lim lim = lim =1
x→0 y→0 x + 2y
( ) x→0 x !
x − 2y −2y
and lim lim
y→0 x→0 x + 2y
= lim
y→0 2y
= −1.
e
gin
Hence, as (x, y) approaches (0, 0) along different paths, f (x, y) approaches different
limits. Hence, the two repeated limits are not equal and hence f (x, y) is
discontinuous at the origin.
En

Note. If a function is continuous at every point of the domain a ≤ x ≤ a′ ,


b ≤ y ≤ b′ , it is said to be continuous in that domain.
arn

✎ ☞
Worked Examples
✍ ✌
2x2 y
Le

Example 2.1. Find lim .


x→1 x2 + y2 + 1
y→2
Solution.
2x2 y 2×1×2 4 2
w.

lim = = = .
x→1 x2 + y2 + 1 1 + 4 + 1 6 3
y→2
ww

xy + 1
Example 2.2. Find lim .
x→∞
y→2
x2 + 2y2
x y + 1x y + 1x

xy + 1 2
Solution. lim 2 2
= lim  y 2
 = lim  y 2
 = = 0.
x→∞ x + 2y
y→2
x→∞ 2
y→2 x 1 + 2 x
 x→∞
y→2 x 1 + 2 x
 ∞

Visit For More : www.LearnEngineering.in


Visit For More : www.LearnEngineering.in

Function of Several Variables 69

( )
x−y  
Example 2.3. If f (x, y) = , show that lim lim f (x, y) , lim lim f (x, y) .
2x + y x→0 y→0 y→0 x→0
Solution.
( ) ( )
x−y
lim lim f (x, y) = lim lim
x→0 y→0 x→0 y→0 2x + y
 x 1
= lim = .
x→0 2x 2

.in
( )
  x−y
lim lim f (x, y) = lim lim
y→0 x→0 y→0 x→0 2x + y
!
−y

ng
= lim = −1.
y→0 y

∴ The two limits are not equal.

eri
x3 y2 + x2 y3 − 3
Example 2.4. If f (x, y) = , show that lim f (x, y) = lim f (x, y).
2 − xy x→0 y→0
y→0 x→0

Solution. lim f (x, y) = lim lim f (x, y)


!
e
gin
x→0 x→0 y→0
y→0
x3 y2 + x2 y3 − 3
!
= lim lim
x→0 y→0 2 − xy
!
−3 −3
En

= lim = .
x→0 2
 2
lim f (x, y) = lim lim f (x, y)
y→0 y→0 x→0
arn

x→0
x3 y2 + x2 y3 − 3
!
= lim lim
y→0 x→0 2 − xy
!
−3 −3
Le

= lim = .
y→0 2 2
−3
∴ lim f (x, y) = lim f (x, y) = .
x→0 y→0 2
w.

y→0 x→0

2.2 Partial Derivatives


ww

We know that, if y is a continuous function of the independent variable x, and


∆y
∆y the increment in y corresponding to an increment ∆x in x, then lim if
∆x→0 ∆x
exists, is called the differential coefficient or the derivative of y with respect to x.

Visit For More : www.LearnEngineering.in


Visit For More : www.LearnEngineering.in

70 Engineering Mathematics - I

Now let us consider a function of several independent variables.


Let u be a function of two independent variables x and y and let ∆u be the
increment in u corresponding to an increment in x, y remaining constant. If u is a
∆u
continuous function of x, the limit lim if exists is called the partial derivative
∆x→0 ∆x
∂u
of u with respect to x, and is represented by or u x .
∂x

.in
∂u u(x + ∆x, y) − u(x, y)
Hence, = lim .
∂x ∆x→0 ∆x
In the same way, if u is a continuous function of y, the increment ∆u in u,

ng
corresponding to an increment ∆y in y, x remaining constant, then the limit
u(x, y + ∆y) − u(x, y)
lim if exists is called the partial derivative of u with respect to
∆y→0 ∆y

eri
∂u
y and is represented by or uy .
∂y
It can be easily seen that the change in the value of u corresponding to a

e
change in the value of x will not in general be the same as the change in the value
gin
of u corresponding to an equal change in the value of y. For example, the area A of
a rectangular box is a function of the length x and the breadth y, being given by
the relation A = xy. When x alone changes,
En

∆A = (x + ∆x)y − xy = y∆x.
arn

When y alone changes, ∆A = x(y + ∆y) − xy = x∆y.


Though ∆x = ∆y, the values of ∆A will not be equal unless x = y. i.e., unless the
∂A ∂A
Le

box is in the form of a square. Hence, we can see that, in general , .


∂x ∂y
Likewise, if u is a continuous function of several independent variables
x, y, z, . . . , we can define the partial derivatives of u with respect to each of these
w.

independent variables. As an example, let us consider a function with three


independent variables x, y and z.
ww

∂u ∂u ∂u
Let u = f (x, y, z). The first partial derivative of u denoted by , and
∂x ∂y ∂z
treating x, y and z respectively alone as variables can be obtained. We can also
∂2 u ∂2 u ∂2 u ∂2 u ∂2 u ∂2 u
find the higher order derivatives 2 , 2 , 2 , , , , . . ..
∂x ∂y ∂z ∂x∂y ∂y∂x ∂x∂z

Visit For More : www.LearnEngineering.in


Visit For More : www.LearnEngineering.in

Function of Several Variables 71

They are evaluated as follows.

∂2 u ∂  ∂u  ∂2 u ∂  ∂u  ∂2 u ∂  ∂u 
= , = and = .
∂x2 ∂x ∂x ∂y2 ∂y ∂y ∂z2 ∂z ∂z

∂2 u ∂  ∂u 
Also, =
∂x∂y ∂x ∂y

.in
∂2 u ∂  ∂u 
and = .
∂y∂x ∂y ∂x

ng
∂2 u ∂2 u
Generally, = .
∂x∂y ∂y∂x

eri
The third and higher orders of the partial derivatives can be obtained similarly.
✎ ☞
Worked Examples
✍ ✌

e
Example 2.5. If V = πr2 h where r and h are independent variables, find
∂V
gin
and
∂r
∂V
.
∂h
Solution. Given: V = πr2 h.
En

Differentiating partially w.r.t. r,


∂V
= πh × 2r = 2πrh.
∂r
arn

Differentiating partially w.r.t. h,


∂V
= πr2 × 1 = πr2 .
∂h
Le

∂u ∂u
Example 2.6. If u = log(e x + ey ), show that + = 1.
∂x ∂y
Solution. Given: u = log(e x + ey ).
∂u ex
w.

= x .
∂x e +y ey
∂u e
= .
∂y e x + ey
ww

∂u ∂u e + eyx
+ = = 1.
∂x ∂y e x + ey

 x ∂z ∂z
Example 2.7. If z = tan−1 , prove that x + y = 0.
y ∂x ∂y

Visit For More : www.LearnEngineering.in


Visit For More : www.LearnEngineering.in

72 Engineering Mathematics - I

 x
Solution. Given: z = tan−1 .
y

∂z 1 1 y2 1 y
= = = 2 .
∂x 1 + x2 y x + y y x + y2
2 2 2
y
∂z xy
x= 2 .
∂x x + y2

.in
∂z 1  x xy2 x
= 2
− 2
= − 2 2 2
=− 2
∂y x y y (x + y ) x + y2
1+ 2
y

ng
∂z xy
y =− 2
∂y x + y2
∂z ∂z xy xy

eri
x +y = 2 2
− 2 = 0.
∂x ∂y x + y x + y2

∂u ∂u ∂u
Example 2.8. If u = (x − y)4 + (y − z)4 + (z − x)4 , find the value of + + .
Solution. u = (x − y)4 + (y − z)4 + (z − x)4 . e ∂x ∂y ∂z
gin
∂u
= 4(x − y)3 + 4(z − x)3 (−1) = 4(x − y)3 − 4(z − x)3 .
∂x
En

∂u
= 4(y − z)3 − 4(x − y)3 .
∂y
∂u
= 4(z − x)3 − 4(y − z)3 .
arn

∂z
∂u ∂u ∂u
+ + = 0.
∂x ∂y ∂z
Le

∂u ∂u ∂u 3
Example 2.9. If u = log(x3 + y3 + z3 − 3xyz), prove that + + = .
∂x ∂y ∂z x+y+z
Solution. u = log(x3 + y3 + z3 − 3xyz).
w.

∂u 3x2 − 3yz
= 3 .
∂x x + y3 + z3 − 3xyz
ww

∂u 3y2 − 3xz
= 3 .
∂y x + y3 + z3 − 3xyz
∂u 3z2 − 3yx
= 3 .
∂z x + y3 + z3 − 3xyz

Visit For More : www.LearnEngineering.in


Visit For More : www.LearnEngineering.in

Function of Several Variables 73

∂u ∂u ∂u 3x2 + 3y2 + 3z2 − 3xy − 3yz − 3zx


+ + =
∂x ∂y ∂z x3 + y3 + z3 − 3xyz

3(x2 + y2 + z2 − xy − yz − zx) 3
= 2 2 2
= .
(x + y + z)(x + y + z − xy − yz − zx) x + y + z

∂u ∂u ∂u
Example 2.10. If u = (x − y)(y − z)(z − x), show that + + = 0.

.in
∂x ∂y ∂z
Solution. Given, u = (x − y)(y − z)(z − x).
∂u
= (y − z){(x − y)(−1) + (z − x) · 1}

ng
∂x
= (y − z)(−x + y + z − x)

= (y − z)(y + z − 2x) = y2 − z2 − 2x(y − z).

eri
∂u
= (z − x){(x − y) · 1 + (y − z)(−1)}
∂y
= (z − x){x − y − y + z}
e
gin
= (z − x)(z + x − 2y)

= z2 − x2 − 2y(z − x).
∂u
= (x − y){(y − z) · 1 + (z − x) · (−1)}
En

∂z
= (x − y)(y − z − z + x)
arn

= (x − y)(x + y − 2z)

= x2 − y2 − 2z(x − y).
∂u ∂u ∂u
+ + = y2 − z2 − 2x(y − z) + z2 − x2 − 2y(z − x) + x2 − y2 − 2z(x − y)
Le

∂x ∂y ∂z
= −2{xy − xz + yz − xy + zx − yz} = −2 × 0 = 0.
w.

p ∂2 u ∂2 u
Example 2.11. If u = log x2 + y2 , show that 2 + 2 = 0.
p ∂x ∂y
2 2 1 2 2
Solution. u = log x + y = 2 log(x + y ).
ww

Differentiating partially w.r.t. x we get

∂u 1 1 x
= · 2 2
· 2x = 2 .
∂x 2 x + y x + y2

Visit For More : www.LearnEngineering.in


Visit For More : www.LearnEngineering.in

74 Engineering Mathematics - I

Again differentiating partially w.r.t. x we get

∂2 u (x2 + y2 ) · 1 − x · 2x y2 − x2
= = .
∂x2 (x2 + y2 )2 (x2 + y2 )2

Differentiating u partially w.r.t. y we get

.in
∂u 1 1 y
= · 2 2
· 2y = 2 .
x + y2

ng
∂y 2 x + y

Again differentiating partially w.r.t. y we get

eri
∂2 u (x2 + y2 ) · 1 − y · 2y x2 − y2
= = .
∂y2 (x2 + y2 )2
e (x2 + y2 )2
gin
∂2 u ∂2 u y2 − x2 + x2 − y2
+ = = 0.
∂x2 ∂y2 (x2 + y2 )2
En

∂2 u ∂2 u ∂2 u
!
2 2 2
Example 2.12. If u = log(x2 + y2 + z2 ), prove that (x + y + z ) + + = 2.
arn

∂x2 ∂y2 ∂z2


Solution. Given, u = log(x2 + y2 + z2 )
Le

∂u 2x
= 2 .
∂x x + y2 + z2
∂2 u (x2 + y2 + z2 ) · 2 − 2x · 2x
=
w.

∂x2 (x2 + y2 + z2 )2
2x + 2y2 + 2z2 − 4x2
2
=
(x2 + y2 + z2 )2
ww

2y2 + 2z2 − 2x2


= 2
(x + y2 + z2 )2
2(y2 + z2 − x2 )
= 2 .
(x + y2 + z2 )2

Visit For More : www.LearnEngineering.in


Visit For More : www.LearnEngineering.in

Function of Several Variables 75

Similarly, we have

∂2 u 2(x2 + z2 − y2 )
= 2
∂y2 (x + y2 + z2 )2
2
∂ u 2(x2 + y2 − z2 )
and 2 = 2 .
∂z (x + y2 + z2 )2
∂2 u ∂2 u ∂2 u 2[y2 + z2 − x2 + x2 + z2 − y2 + x2 + y2 − z2 ]
Now, + + =

.in
∂x2 ∂y2 ∂z2 (x2 + y2 + z2 )2
2(x2 + y2 + z2 )
= 2
(x + y2 + z2 )2

ng
2
= 2 .
x + y2 + z2
2 ∂2 u ∂2 u
!
2 ∂ u

eri
2 2
∴ (x + y + z ) + + =2
∂x2 ∂y2 ∂z2

 ∂u 2
2u 2u
 ! !2 
∂ ∂ 1 ∂u 
∂x
e
Example 2.13. If u = e xy , show that 2 + 2 = 
∂y u ∂x
+
∂y 
. [Jan 2013]
gin
Solution. u = e xy
∂u
= e xy · y = uy.
∂x
∂2 u ∂u
=y· = yuy = uy2 .
En

∂x 2 ∂x
∂u
= e xy · x = ux.
∂y
∂2 u
arn

∂u
2
= x· = x · ux = ux2 .
∂y ∂y
!2 !2
∂u ∂u
+ = u2 y2 + u2 x2
∂x ∂y
Le

= u[uy2 + ux2 ]
" 2
∂ u ∂2 u
#
=u +
w.

∂x2  ∂y2
!2 !2 
1  ∂u ∂u  ∂2 u ∂2 u
+ = + .
∂y  ∂x2 ∂y2

u  ∂x
ww

!
x y ∂u ∂u
Example 2.14. If u = y f +g find the value of x + y . [Dec 2012]
! y x ∂x ∂y
x y
Solution. u = y f +g
y x

Visit For More : www.LearnEngineering.in


Visit For More : www.LearnEngineering.in

76 Engineering Mathematics - I

!
∂u ′ x 1  y   −y 
= yf · + g′
∂x y y x x2

!
∂u x y y
x = xf′ − g′ . (1)
∂x y x x

.in
! ! !
∂u ′ x −x x ′ y
  1
= yf · 2 +f ·1+g · .
∂y y y y x x

ng
! !
∂u ′ x x y y
y = −x f + yf + g′ (2)
∂y y y x x

eri
!
∂u ∂u x
(1) + (2) ⇒ x + y = yf .
∂x ∂y y

e
gin
∂2 r ∂2 r ∂2 r 2
Example 2.15. If r2 = x2 + y2 + z2 , prove that + + = .
∂x2 ∂y2 ∂z2 r
Solution. Given: r2 = x2 + y2 + z2 .
Differentiating w.r.t. x partially we get,
En

∂r ∂r x
2r= 2x ⇒ = .
∂x ∂x r
arn

∂r
2 r − x x 2 2
∂ r
= ∂x = r − x r = r − x .
∂x2 r2 r2 r3
2
∂ r r −y2 2
Le

Similarly, 2 =
∂y r3
∂ r r − z2
2 2
and 2 = .
∂z r3
w.

∂2 r ∂2 r ∂2 r 3r2 − (x2 + y2 + z2 )
+ + =
∂x2 ∂y2 ∂z2 r3
ww

2
3r − r 2 2r2 2
= = = .
r3 r3 r

y  x ∂2 u x2 − y2
Example 2.16. If u = x2 tan−1 − y2 tan−1 , prove that = 2 .
x y ∂x∂y x + y2

Visit For More : www.LearnEngineering.in


Visit For More : www.LearnEngineering.in

Function of Several Variables 77

Solution. Differentiating u partially w.r.t. y, we get

∂u 1 1 h 2 1  −x  y i
= x2 − y + tan−1 2y
∂y y2 x x2 y2 x
1+ 1+
x2 y2
x3 xy2 −1 y
 
= + − 2y tan
x2 + y2 x2 + y2 x

.in
2 2
x(x + y ) −1 x −1 x
   
= 2 − 2y tan = x − 2y tan
x + y2 y y

ng
Differentiating this partially w.r.t. x, we get

∂2 u 11 2y2
= 1 − 2y = 1 −

eri
∂x∂y x2 y x2 + y2
1+ 2
y
x + y − 2y2
2 2 x2 − y2
= = .
x2 + y2
e x2 + y2
gin
Example 2.17. If u = xy , show that u xxy = u xyx . [Jan 2012, Jun 2008]
Solution. Given: u = ey log x
En

uy = ey log x log x = xy log x


1
u xy = yxy−1 log x + xy
x
arn

= yxy−1 log x + xy−1 = xy−1 (y log x + 1)


y
u xxy = xy−1 + (y log x + 1)(y − 1)xy−2
x
Le

= xy−2 y + xy−2 (y log x + 1)(y − 1)

= xy−2 (y − y log x − 1 + y2 log x + y)


w.

= xy−2 (2y − 1 + y log x(y − 1))

= xy−2 (y(y − 1) log x + 2y − 1) (1)


ww

u = xy

u x = yxy−1 = ye(y−1) log x

uyx = ye(y−1) log x log x + e(y−1) log x .1

Visit For More : www.LearnEngineering.in


Visit For More : www.LearnEngineering.in

78 Engineering Mathematics - I

= yxy−1 log x + xy−1 = xy−1 (y log x + 1)


y
u xyx = xy−1 + (y log x + 1)(y − 1)xy−2
x
= xy−2 y + xy−2 (y − 1)(y log x + 1)

= xy−2 (y + y2 log x + y − y log x − 1)

= xy−2 (2y − 1 + y log x(y − 1)) (2)

.in
From (1) and (2) we have u xxy = u xyx .

ng
y ∂2 u ∂2 u ∂2 u

eri
Example 2.18. If u = (x − y) f , find the value of x2 2 + 2xy + y2 2 .
x ∂x ∂x∂y ∂y
[May 2001]
y
Solution. u = (x − y) f
x
e
gin
∂u  y   −y  y
= (x − y) f ′ + f .
∂x x x2 x
∂2 u y
′′ y
   −y 
′ y
 
En

= −(x − y) · f − (x − y) f · y(−2)x−3
∂x2 x2  x x2    x
y y y −y
− 2 · f′ · 1 + f′ .
x x x x2
arn

y2 ′′  y  y ′ y
  y y y y
= (x − y) 4 f + 2 3 (x − y) f − 2 f′ − 2 f′
x x x x x x x x
2
∂ u y 2  y  2y  y   y 
x2 2 = 2 (x − y) f ′′ + (x − y) f ′ − 2y f ′ . (1)
∂x x x x x x
Le

∂u  y 1
  y 
= (x − y) f ′ · +f (−1)
∂y x x x
 y ′ y y
w.

= 1− f −f .
x x x
∂2 u  y  ′′  y  1 ′ y
  −1 !   1
′ y
= 1 − f · + f − f · .
∂y2 x x x x x x x
ww

(x − y) ′′  y  2 ′  y 
= f − f
x2 x x x
2 2 2
2∂ u y ′′ y 2y ′  y 
 
y = 2 (x − y) f − f . (2)
∂y2 x x x x

Visit For More : www.LearnEngineering.in


Visit For More : www.LearnEngineering.in

Function of Several Variables 79

∂u
Differentiating w.r.t. x partially we get
∂y
∂2 u
!
 y  ′′  y   −y  ′ y
  −1 ′ y
   −y 
= 1− f + f (−y) − f
∂x∂y x x x2 x x2 x x2
(x − y)y ′′ y   y  y  y  y 
=− 3
f + 2 f′ + 2 f′
x x x x x x
(x − y) · y ′′  y  2y ′  y 
=− f + 2f .

.in
x3 x x x
∂2 u −2y2   4y2  y 
′′ y
2xy = 2 (x − y) f + f′ . (3)
∂x∂y x x x x

ng
∂2 u ∂2 u ∂2 u
(1) + (2) + (3) ⇒ x2 2 + 2xy + y2 2 = 0.
∂x ∂x∂y ∂y
Homogeneous function. A function f (x, y) is said to be a homogeneous function

eri
in x and y of degree n if f (tx, ty) = tn f (x, y) for any positive t.
Example

e
(i) f (x, y) = x2 + y2 + 2xy is a homogeneous function of degree 3 in x and y.
gin
 
(ii) If tan−1 yx = u then tan u is a homogeneous function of degree 0.
Note. If u = f (x, y) is a homogeneous function of degree n in x and y, then u can
y
be written as u = xn F .
En

x
3
x +y 3
Example. Let u = .
x+y
arn

t3 x3 + t3 y3 t3 (x3 + y3 ) x3 + y3
u(tx, ty) = = = t2 = t2 u.
tx + ty t(x + y) x+y
u is a homogeneous function of degree 2.
Le

 y3 
x3 1 + 3
Now u =  x
y
x 1+
w.

x
 y 3
1 + x 
2
=x y
ww

1+
x
 y 3
y y 1+
= x2 F where F x
= y .
x x 1+
x

Visit For More : www.LearnEngineering.in


Visit For More : www.LearnEngineering.in

80 Engineering Mathematics - I

∴ u is a homogeneous function of degree 2 in x and y.

2.3 Euler’s Theorem

Statement. If f (x, y) is a homogeneous function of degree n in x and y having


∂f ∂f

.in
continuous partial derivatives, then x +y = nf.
∂x ∂y
Proof.
Given that f (x, y) is a homogeneous function of degree n in x and y.

ng
Hence f (x, y) can be written as

eri
y
f (x, y) = xn F .
x
∂f y  y  −y 
= nxn−1 F + xn F ′
x x2
∂x
= nxn−1 F
x
y
e
− xn−2 yF ′ .
y
gin
x x
∂f n ′ y 1
  
n−1 ′ y
 
=x F =x F .
∂y x x x
∂f ∂f y y y
En

x +y = nxn F − xn−1 yF ′ + xn−1 yF ′


∂x ∂y x x x
y
= nxn F
x
arn

∂f ∂f
x +y = n f (x, y).
∂x ∂y

Extension. If f (x1 , x2 , . . . , xn ) is a homogeneous function of degree n in


Le

∂f ∂f ∂f
x1 , x2 , . . . , xn , then x1 + x2 + · · · + xn = nf.
∂x1 ∂x2 ∂xn
Euler’s theorem on higher partial derivatives
w.

Statement. If f (x, y) is a homogeneous function of degree n in x and y then,


∂2 f ∂2 f ∂2 f
x2 2 + 2xy + y2 2 = n(n − 1) f .
∂x ∂x∂y ∂y
ww

Proof. Given, f (x, y) is a homogeneous function of degree n in x and y.


∴ By Euler’s theorem we have

∂f ∂f
x +y = n f. (1)
∂x ∂y

Visit For More : www.LearnEngineering.in


Visit For More : www.LearnEngineering.in

Function of Several Variables 81

Differentiating this partially w.r.t. x we obtain

∂2 f ∂ f ∂2 f ∂f
x + + y =n .
∂x2 ∂x ∂x∂y ∂x

∂2 f ∂2 f ∂f ∂f ∂f
=⇒ x + y =n − = (n − 1) .
∂x2 ∂x∂y ∂x ∂x ∂x

.in
Multiplying both sides by x we get

∂2 f ∂2 f ∂f
x2

ng
2
+ xy = (n − 1)x . (2)
∂x ∂x∂y ∂x

Differentiating (1) partially w.r.t. y we get

eri
∂2 f ∂2 f ∂ f ∂f
x +y 2 + =n
∂y∂x ∂y ∂y ∂y

e
gin
∂2 f ∂2 f ∂f ∂f ∂f
x +y 2 =n − = (n − 1) .
∂y∂x ∂y ∂y ∂y ∂y

∂2 f ∂2 f
Since the partial derivatives of f are continuous we have = .
En

∂x∂y ∂y∂x

∂2 f ∂2 f ∂f
∴x + y 2 = (n − 1) .
∂x∂y ∂y ∂y
arn

Multiplying both sides by y we get

∂2 f ∂2 f ∂f
Le

xy + y2 2 = (n − 1)y . (2)
∂x∂y ∂y ∂y
w.

∂2 f ∂2 f 2
2∂ f
 ∂f ∂f 
(1) + (2) =⇒ x2 + 2xy + y = (n − 1) x + y
∂x2 ∂x∂y ∂y2 ∂x ∂y
ww

= (n − 1)n f [By Euler’s theorem]

= n(n − 1) f.
✎ ☞
Worked Examples
✍ ✌

Visit For More : www.LearnEngineering.in


Visit For More : www.LearnEngineering.in

82 Engineering Mathematics - I

∂u ∂u ∂u
Example 2.19. If u = (x − y)(y − z)(z − x) prove that + + = 0 and
∂x ∂y ∂z
∂u ∂u ∂u
x +y +z = 3u.
∂x ∂y ∂z
Solution. u = (x − y)(y − z)(z − x).

∂u 
= (y − z) (x − y)(−1) + z − x

.in
∂x
= (y − z)(−x + y + z − x)

= (y − z)(y + z − 2x).

ng
∂u ∂u
Similarly,= (z − x)(z + x − 2y) and = (x − y)(x + y − 2z).
∂y ∂z

eri
∂u ∂u ∂u
+ + = y2 − z2 − 2x(y − z) + z2 − x2 − 2y(z − x) + x2 − y2 − 2z(x − y).
∂x ∂y ∂z
= −2xy + 2xz − 2yz + 2xy − 2xz + 2yz = 0

e
Also, u(tx, ty, tz) = (tx − ty)(ty − tz)(tz − tx)
gin
= t3 (x − y)(y − z)(z − x) = t3 u.
En

∴ u is a homogeneous function of degree 3 in x, y, z.


∂u ∂u ∂u
∴ By Euler’s theorem, x + y + z = 3u.
∂x ∂y ∂z
arn

 x3 + y3  ∂u ∂u
Example 2.20. If u = log , prove that x + y = 2.
x+y ∂x ∂y
 x3 + y3  x3 + y3
Le

Solution. Given: u = log ⇒ eu =


x+y x+y
w.

t3 x3 + t3 y3 t3 (x3 + y3 )
eu(tx,ty) = =
tx + ty t(x + y)
3
x +y 3
= t2 = t2 eu .
ww

x+y

∴ eu is a homogeneous function of degree 2.


Here n = 2.

Visit For More : www.LearnEngineering.in


Visit For More : www.LearnEngineering.in

Function of Several Variables 83

∴ By Euler’s theorem,
∂ u ∂
(e ) + y (eu ) = 2eu .
x
∂x ∂y
∂u ∂u
=⇒ xeu + yeu = 2eu
∂x ∂y
 ∂u ∂u 
eu x + y = 2eu .
∂x ∂y

.in
∂u ∂u
x +y = 2.
∂x ∂y
y z ∂u ∂u ∂u

ng
Example 2.21. If u = + , find the value of x + y + z .
x x ∂x ∂y ∂z
y z
Solution. Given: u = + .
x x

eri
ty tz y z
u(tx, ty, tz) = + = t0 + = t0 u.
tx tx x x
=⇒ u is a homogeneous function in x, y, z of degree 0.
By Euler’s theorem, e
gin
∂u ∂u ∂u
x
+y +z = nu = 0.u = 0.
∂x ∂y ∂z
x y z ∂u ∂u ∂u
Example 2.22. If u = + + , then show that x + y + z = 0. [Jun 2012]
En

y z x ∂x ∂y ∂z
x y z
Solution. u(x, y, z) = + + .
y z x
tx ty tz
arn

u(tx, ty, tz) = + +


ty tz tx
tx y z
= + +
t y z x
= t0 u(x, y, z).
Le

u is a homogeneous function of degree 0 in x, y, z.


∂u ∂u ∂u
w.

∴ By Euler’s theorem, x + y + z = 0.u = 0.


∂x ∂y ∂z
x2 y2 ∂u ∂u
Example 2.23. If sin u = , show that x + y = 3 tan u.
ww

x+y ∂x ∂y
x2 y2
Solution. sin u(x, y) = .
x+y
t2 x2 t2 y2 t4 (x2 y2 )
sin u(tx, ty) = = = t3 sin u.
tx + ty t(x + y)

Visit For More : www.LearnEngineering.in


Visit For More : www.LearnEngineering.in

84 Engineering Mathematics - I

sin u is a homogeneous function of degree 3 in x and y.


By Euler’s theorem,
∂ ∂
(sin u) + y (sin u) = 3 sin u.
x
∂x ∂y
∂u ∂u
x cos u + y cos u = 3 sin u
∂x ∂y

.in
 ∂u ∂u 
cos u x + y = 3 sin u
∂x ∂y
∂u ∂u 3 sin u 3 sin u
x +y = = = 3 tan u.

ng
∂x ∂y cos u cos u
x y z ∂u ∂u ∂u
Example 2.24. If u = f , , , prove that x + y + z = 0.
y z x ∂x ∂y ∂z

eri
Solution.
x y z
u(x, y, z) = f , ,

e
y z x
 tx ty tz 
gin
u(tx, ty, tz) = f , ,
ty tz tx
x y z
= f , ,
y z x
En

= u(x, y, z) = t0 u(x, y, z)

u is a homogeneous function of degree 0 in x, y, z.


∂u ∂u ∂u
arn

∴ By Euler’s theorem, x + y + z = 0.u = 0.


∂x ∂y ∂z
 x3 + y3  ∂u ∂u
Example 2.25. If u = tan−1 show that x +y = sin 2u.
x−y ∂x ∂y
Le

x3 + y3
Solution. tan u = .
x−y
tan u is a homogeneous function of degree 2 in x and y.
w.

By Euler’s theorem
∂ ∂
x (tan u) + y (tan u) = 2 tan u
ww

∂x ∂y
∂u ∂u
sec2 ux + sec2 uy = 2 tan u
∂x ∂y
∂u ∂u 2 tan u 2 sin u
x +y = = cos2 u
∂x ∂y sec2 u cos u

Visit For More : www.LearnEngineering.in


Visit For More : www.LearnEngineering.in

Function of Several Variables 85

= sin 2u.
 x+y  ∂u ∂u 1
Example 2.26. If u = sin−1 √ √ prove that x + y = tan u. [Jun 2009]
x+ y ∂x ∂y 2
 x+y  x+y
Solution. u = sin−1 √ √ ⇒ sin u = √ √ .
x+ y x+ y
sin u is a homogeneous function of degree 21 in x and y.

.in
By Euler’s theorem,
∂ ∂ 1
x (sin u) + y (sin u) = sin u.

ng
∂x ∂y 2
∂u ∂u 1
cos ux + cos uy = sin u
∂x ∂y 2

eri
∂u ∂u 1
x +y = tan u.
∂x ∂y 2
 
 x + y  ∂u ∂u −1
Example 2.27. If u = cos  √ −1 

x+ ye√ , prove that x + y


∂x ∂y
=
2
cot u. [Dec 2011]
gin

 x + y 
Solution. u = cos−1  √ √ .
 x + y
 x + y 
cos u =  √ √ .
En

x+ y
cos u is a homogeneous function of degree 21 in x and y.
∴ By Euler’s theorem
arn

∂ ∂ 1
x (cos u) + y (cos u) = · cos u.
∂x ∂y 2
∂u ∂u 1
x(− sin u) + y(− sin u) = cos u.
Le

∂x ∂y 2
!
∂u ∂ 1
− sin u x + y = cos u.
∂x ∂y 2
w.

∂u ∂ −1 cos u −1
∴ x +y = = cot u.
∂x ∂y 2 sin u 2
 x y
ww

Example 2.28. Verify Euler’s theorem for the function u = sin−1 + tan−1 .
 x y y x
Solution. Given: u = sin−1 + tan−1 .
y x
 tx   ty 
u(tx, ty) = sin−1 + tan−1
ty tx

Visit For More : www.LearnEngineering.in


Visit For More : www.LearnEngineering.in

86 Engineering Mathematics - I

  x  y 
= t0 sin−1 + tan−1
y x
= t0 u(x, y).
=⇒ u is a homogeneous function of degree 0 in x and y.
∂u ∂u
∴ By Euler’s theorem, x + y = 0.
∂x ∂y
Verification.

.in
∂u 1 1 1  −y 
= s +
∂x
x2
y y2 x2

ng
1− 2 1 +
y x2

y 1 x2 y
= p − 2 2

eri
2
y2 − x2 y x (x + y )
1 y
= p − 2 .
2
y −x 2 x + y2

x
∂u
= p
x
e − 2
xy
(1)
gin
∂x 2
y −x 2 x + y2
∂u 1  −x  1 1
= s 2
+
∂y
x2 y y2 x
1− 2 1 +
En

y x2
−xy x
= p + 2 2
y2 y2 − x2 x + y
arn

x x
=− p + 2
2
y y −x 2 x + y2
∂u x xy
y =−p + 2 (2)
Le

∂y 2
y −x 2 x + y2
∂u ∂u
(1) + (2) =⇒ = x +y = 0.
∂x ∂y
w.

Hence, Euler’s theorem is verified.


ww

1 1
Example 2.29. Verify Euler’s theorem for the function u = x 2 + y 2 (xn + yn ).
 
Solution. It is easy to verify that u is a homogeneous function of degree n + 21 in
x and y.

Visit For More : www.LearnEngineering.in


Visit For More : www.LearnEngineering.in

Function of Several Variables 87

Hence, by Euler’s theorem we have


∂u ∂u  1
x +y = n + u. (1)
∂x ∂y 2
Verification
∂u 1 1 1 1 
= x 2 + y 2 nxn−1 + (xn + yn ) x− 2

.in
∂x 2
∂u 1 1 1 1
x = n x 2 + y 2 xn + (xn + yn )x 2 .
∂x 2

ng
Since the function is symmetric in x and y we have
∂u 1 1 1 1
= n x 2 + y 2 yn + (xn + yn )y 2 .
y

eri
∂y 2
∂u ∂u 1 1 1 1 1
x +y = n x 2 + y 2 (xn + yn ) + (xn + yn ) x 2 + y 2
∂x ∂y 2
= n+
1 1
2
1

e
x 2 + y 2 (xn + yn )
gin
1
= n + u,
2
which verifies Euler’s theorem.
En

y  x
Example 2.30. If u = x2 tan−1 − y2 tan−1 , find the value of
x y
∂2 u ∂2 u 2
2∂ u
arn

x2 + 2xy + y .
∂x2 ∂x∂y ∂y2
Solution. u is a homogeneous function of degree 2 in x and y.
∂2 u ∂2 u ∂2 u
By Euler’s theorem, x2 2 + 2xy + y2 2 = 2(2 − 1)u = 2u [n = 2].
∂x ∂x∂y ∂y
Le

y y ∂2 z ∂2 z ∂2 z
Example 2.31. If z = x f + g , show that x2 2 + 2xy + y2 2 = 0.
x x ∂x ∂x∂y ∂y
y y
w.

Solution. Let u = x f and v = g .


x x
∴ z = u + v.
ww

u is a homogeneous function of degree 1 in x and y.


∴ By Euler’s theorem,
∂2 u ∂2 u 2
2∂ u
x2 + 2xy + y = n(n − 1)u = 1(1 − 1)u = 0. (1)
∂x2 ∂x∂y ∂y2

Visit For More : www.LearnEngineering.in


Visit For More : www.LearnEngineering.in

88 Engineering Mathematics - I

v is a homogeneous function of degree 0 in x and y.


By Euler’s theorem,
∂2 v ∂2 v 2
2∂ v
x2 + 2xy + y = 0(0 − 1)v = 0. (2)
∂x2 ∂x∂y ∂y2
∂2 (u + v) ∂2 (u + v) 2
2 ∂ (u + v)
(1) + (2) ⇒ x2 + 2xy + y =0
∂x2 ∂x∂y ∂y2

.in
∂2 z ∂2 z 2
2∂ z
x2 + 2xy + y = 0.
∂x2 ∂x∂y ∂y2

ng
3 3
!
−1 x + y
Example 2.32. If u = tan , prove that
x−y
∂2 u ∂2 u ∂2 u
x2 2 + 2xy + y2 2 = 2 sin u cos 3u.

eri
∂x ∂x∂y !∂y
x 3 + y3
Solution. u = tan−1 .
x−y
tan u =
x3 + y3
!
e
gin
x−y
tan u is an homogeneous function of degree 2 in x and y.
By Euler’s theorem,
∂ ∂
x (tan u) + y (tan u) = 2 tan u.
En

∂x ∂y
∂u ∂u
x · sec2 u + y · sec2 u = 2 tan u.
∂x ∂y
arn

!
2 ∂u ∂u
sec u x + y = 2 tan u.
∂x ∂y
∂u ∂u 2 tan u sin u
x +y = 2
=2· · cos2 u
∂x ∂y sec u cos u
Le

= 2 sin u cos u

∂u ∂u
w.

x +y = sin 2u. (1)


∂x ∂y
Differentiating (1) partially w.r.t. x we get
∂2 u ∂u ∂2 u
ww

∂u
x· 2 + +y = 2 cos 2u .
∂x ∂x ∂x∂y ∂x
Multiplying by x we get
∂2 u ∂u ∂2 u ∂u
x2 2
+ x + xy = 2 cos 2u · x . (2)
∂x ∂x ∂x∂y ∂x

Visit For More : www.LearnEngineering.in


Visit For More : www.LearnEngineering.in

Function of Several Variables 89

Differentiating (1) partially w.r.t. y we get


∂2 u ∂2 u ∂u ∂u
x +y 2 + = 2 cos 2u .
∂y∂x ∂y ∂y ∂y
Multiplying w.r.t. y we get
∂2 u ∂2 u ∂u ∂u
xy + y2 2 + y = 2 cos 2u · y . (3)
∂y∂x ∂y ∂y ∂y

.in
(2) + (3) ⇒
2 ∂2 u 2
!
2∂ u 2∂ u ∂u ∂u ∂u ∂u
x + 2xy +y +x +y = 2 cos 2u x + y

ng
∂x2 ∂x∂y ∂y2 ∂x ∂y ∂x ∂y
2 2 2
!
∂ u ∂ u ∂ u ∂u ∂u
x2 2 + 2xy + y2 2 = (2 cos 2u − 1) x + y
∂x ∂x∂y ∂y ∂x ∂y

eri
= (2 cos 2u − 1) sin 2u

= sin 2u{2(2 cos2 u − 1) − 1}

e = sin 2u{4 cos2 u − 3}


gin
= 2 sin u cos u(4 cos2 u − 3)

= 2 sin u(4 cos3 u − 3 cos u)


En

= 2 sin u · cos 3u.


arn

2.4 Total Derivative - Implicit functions


dy dy du
We know that if y = f (u) and u = φ(x), then = . We shall now extend this
dx du dx
Le

result to function of several variables.


Let z = f (x, y) possess continuous partial derivatives and let x, y be differentiable
w.

functions of an independent variable t. Let ∆x, ∆y and ∆z be increments in x, y and


z respectively, corresponding to an increment ∆t in t. Then the total increment in z
ww

is given by

∆z = f (x + ∆x, y + ∆y) − f (x, y)

= f (x + ∆x, y + ∆y) − f (x, y + ∆y) + f (x, y + ∆y) − f (x, y).

Visit For More : www.LearnEngineering.in


Visit For More : www.LearnEngineering.in

90 Engineering Mathematics - I

By mean value theorem, f (x + ∆x, y + ∆y) − f (x, y + ∆y) = f x (x + θ1 x, y + ∆y)∆x and


f (x, y + ∆y) − f (x, y) = fy (x, y + θ2 ∆y)∆y, 0 < θ1 , θ2 < 1.

∆z = f x (x + θ1 x, y + ∆y)∆x + fy (x, y + θ2 ∆y)∆y (1)

where θ1 and θ2 are positive proper fractions.

.in
∆z ∆x ∆y
∴ = f x (x + θ1 x, y + ∆y) + fy (x, y + θ2 ∆y) .
∆t ∆t ∆t

ng
Taking limits as ∆t → 0, we get the total derivative

dz dx dy
= f x (x, y) + fy (x, y) .
dt dt dt

eri
dz ∂z dx ∂z dy
i.e., = + . (2)
dt ∂x dt ∂y dt

e
In the same way if u = f (x1 , x2 , . . . , xn ) possesses continuous partial derivatives
gin
with respect to each of the independent variables and if x1 , x2 , . . . , xn are
differentiable functions of an independent variable t, then the total derivative of u
with respect to t is given by
En

du ∂u dx1 ∂u dx2 ∂u dxn


= + + ··· + . (3)
dt ∂x1 dt ∂x2 dt ∂xn dt
arn

Note. (2) expresses the rate of change of z w.r.t. t along the curve
x = x(t), y = y(t).
Le

Composite functions. Let z = f (u, v) and u and v are themselves functions of the
independent variables x, y so that u = φ(x, y) and v = ψ(x, y).
∂z
w.

To find , we consider y as a constant so that u and v may be supposed to be


∂x
functions of x only. Hence, by the above result we have
∂z ∂z ∂u ∂z ∂v ∂z ∂z ∂u ∂z ∂v
ww

= + . Similarly = + .
∂x ∂u ∂x ∂v ∂x ∂y ∂u ∂y ∂v ∂y
Now, in (2), if we replace t by x then y is a function of x we get

dz ∂z ∂z dy
= + .
dx ∂x ∂y dx

Visit For More : www.LearnEngineering.in


Visit For More : www.LearnEngineering.in

Function of Several Variables 91

dz ∂z dx ∂z
Similarly = + .
dy ∂x dy ∂y

Implicit Functions. Let f (x, y) = c, where c is a constant, define y as an implicit


function of x.
If u = f (x, y), then we have
du ∂ f ∂ f dy

.in
= + .
dx ∂x ∂y dx
du
But = 0, since u = f (x, y) = c

ng
dx
∂ f ∂ f dy
i.e., + =0
∂x ∂y dx

eri
∂f
dy fx
∴ = − ∂x = − .
dx ∂f fy
∂y
e
gin
dx fy
Similarly =− .
dy fx
Total differential. The total differential dz of z is defined as the principal part of
the increment ∆z which is given by
En

∂z ∂z
dz = dx + dy.
∂x ∂y
✎ ☞
arn

Worked Examples
✍ ✌
dz
Example 2.33. If z = xn ym where x = cos at, y = sin bt, find .
dt
dz ∂z dx ∂z dy
Le

Solution. We have = + .
dt ∂x dt ∂y dt
∂z ∂z
Now = nxn−1 ym , = mxn ym−1
∂x ∂y
w.

dx dy
= −a sin at, = b cos bt.
dt dt
dz
= −anxn−1 ym sin at + bmxn ym−1 cos bt
ww


dt
 an bm 
= −xn ym sin at − cos bt
x y
= − cosn at sinm bt(an tan at − bm cot bt).

Visit For More : www.LearnEngineering.in


Visit For More : www.LearnEngineering.in

92 Engineering Mathematics - I

du
Example 2.34. If u = x2 y3 , x = log t, y = et , find . [Jan 2000]
dt
du ∂u dx ∂u dy
Solution. We have = + .
dt ∂x dt ∂y dt
∂u
= y3 2x = 2xy3
∂x
∂u
= x2 3y2 = 3x2 y2

.in
∂y
dx 1 dy
= = et
dt t dt

ng
du 1
= 2xy3 + 3x2 y2 et
dt t

eri
2 log te3t
= + 3(log t)2 e2t et
t
2

e
= e3t log t + 3 log t
t

gin
e3t log t
= (2 + 3t log t).
t
du
Example 2.35. Find when u = x2 y, x = t2 , y = et .
En

dt
Solution.
∂u ∂u
= 2xy, = x2
∂x ∂y
arn

dx dy
= 2t = et
dt dt
Le

du ∂u dx ∂u dy
we have = + = 2xy.2t + x2 et
dt ∂x dt ∂y dt
= 4t2 et t + t4 et = 4t3 et + t4 et = t3 et (4 + t).
w.

1 du
Example 2.36. If u = xy + yz + zx where x = , y = et and z = e−t find . [Jun 2013]
t dt
ww

Solution.
∂u ∂u ∂u
= y + z, = x + z, = x+y
∂x ∂y ∂z
dx −1 dy dz
= 2, = et , = −e−t
dt t dt dt

Visit For More : www.LearnEngineering.in


Visit For More : www.LearnEngineering.in

Function of Several Variables 93

du ∂u dx ∂u dy ∂u dz
we have = + +
dt ∂x dt ∂y dt ∂z dt
 −1 
= (y + z) 2 + (x + z)et + (x + y)(−e−t )
t
et + e−t 1 1 
=− 2
+ (e−t + )et − + et e−t
t t t
−2 et e−t
= 2 cos ht + 1 + − −1

.in
t t t
−2 2 sin ht
= 2 cos ht +
t t
2

ng
= 2 (2t sin ht − cos ht).
t

eri
du  
Example 2.37. Find when u = sin yx , x = et , y = t2 .
dt
Solution.
∂u
= cos
 x1
, e ∂u x
= − 2 cos
 x
gin
∂x y y ∂y y y

dx dy
= et = 2t
En

dt dt
arn

du ∂u dx ∂u dy 1  x x  x
= + = cos et − cos 2t
dt ∂x dt ∂y dt y y y2 y
 x  et 2tet !  x 1 2t
!
et (t − 2)  et 
= cos 2
− 4 = cos et − = cos .
y t t y t2 t4 t3 t2
Le

du 3
Example 2.38. If u = sin−1 (x − y) where x = 3t, y = 4t3 , show that = √ .
w.

dt 1 − t2
Solution.
∂u 1 ∂u −1
ww

= p , = p
∂x 1 − (x − y)2 ∂y 1 − (x − y)2

dx dy
=3 = 12t2
dt dt

Visit For More : www.LearnEngineering.in


Visit For More : www.LearnEngineering.in

94 Engineering Mathematics - I

du ∂u dx ∂u dy
= +
dt ∂x dt ∂y dt
1 1
= p 3− p 12t2
1 − (x − y) 2 1 − (x − y) 2

3 − 12t 2 3 − 12t2
= p = p
1 − (3t − 4t3 )2 1 − (9t2 + 16t6 − 24t4 )
3 − 12t 2 3 − 12t2

.in
= √ = p
1 − 9t2 − 16t6 + 24t4 −(16t6 − 24t4 + 9t2 − 1)
3(1 − 4t2 ) 3(1 − 4t2 )
= p = p

ng
−(t2 − 1)(16t4 − 8t2 + 1) −(t2 − 1)(1 − 4t2 )2
3(1 − 4t2 ) 3
= p = √ .
(1 − 4t2 ) (1 − t2 ) 1 − t2

eri
du
Example 2.39. Find if u = cos(x2 + y2 ) and a2 x2 + b2 y2 = c2 . [Jan 2001]
dx
∂u ∂u e
gin
Solution. We have du = dx + dy
∂x ∂y

du ∂u ∂u dy dy
= + = − sin(x2 + y2 )2x − sin(x2 + y2 )2y
En

dx ∂x ∂y dx dx

Given that a2 x2 + b2 y2 = c2 .
arn

Diff. w.r.t. x we get

dy dy dy a2 x
2a2 x + 2b2 y = 0 ⇒ b2 y = −a2 x ⇒ =− 2
Le

dx dx dx b y
du  dy    −a2 x 
∴ = −2 sin(x2 + y2 ) x + y = −2 sin(x2 + y2 ) x + y 2
dx dx b y
w.

 a2 − b2  2(a2 − b2 )x
= 2 sin(x2 + y2 ) xy = sin(x2 + y2 ).
b2 y b2
ww

dz
q
Example 2.40. If z = x2 + y2 and x3 + y3 + 3axy = 5a2 , find the value of when
dx
x = y = a.
p
Solution. Given z = x2 + y2 .

Visit For More : www.LearnEngineering.in


Visit For More : www.LearnEngineering.in

Function of Several Variables 95

We have

∂z ∂z
dz =dx + dy
∂x ∂y
∂z 2x x
Now = p = p
∂x 2 x2 + y2 x2 + y2
∂z y
= p .

.in
∂y x2 + y2

Also x3 + y3 + 3axy = 5a2

ng
Diff.w.r.t. x we get
dy h dy i
3x2 + 3y2 + 3a x + y = 0
dx dx

eri
dy dy
x2 + y2 + ax + ay = 0
dx dx
dy 2
dx e
(y + ax) = −ay − x2
gin
dy −(x2 + ay)
= 2 .
dx y + ax
En

−(x2 + ay)
!
dz ∂z ∂z dy x y
∴ = + = p + p 2
dx ∂x ∂y dx x2 + y2 x2 + y2 y + ax
 dz  a a  −2a2  a a
arn

= √ + √ = √ − √ = 0.
dx x=y=a a 2 a 2 2a2 a 2 a 2

Example 2.41. If z is a function of x and y and x = eu + e−v , y = e−u − ev , show that


∂z ∂z ∂z ∂z
Le

− = x −y . [Jun 2001]
∂u ∂v ∂x ∂y
Solution. z is a composite function of u and v.
We have
w.

∂z ∂z ∂x ∂z ∂y ∂z u ∂z ∂z u ∂z −u
= + = e + (−e−u ) = e − (e ).
∂u ∂x ∂u ∂y ∂u ∂x ∂y ∂x ∂y
ww

∂z ∂z ∂x ∂z ∂y ∂z ∂z ∂z ∂z
= + = (−e−v ) + (−ev ) = − e−v − (ev ).
∂v ∂x ∂v ∂y ∂v ∂x ∂y ∂x ∂y
∂z ∂z ∂z u ∂z ∂z ∂z
− = (e + e−v ) − (e−u − ev ) = x − y .
∂u ∂v ∂x ∂y ∂x ∂y

Visit For More : www.LearnEngineering.in


Visit For More : www.LearnEngineering.in

96 Engineering Mathematics - I

Example 2.42. If z = f (x, y) where x = eu cos v, y = eu sin v, show that


∂z ∂z ∂z
y + x = e2u . [Jan 2000]
∂u ∂v ∂y
Solution. z is a composite function of u and v.
∂z ∂z ∂x ∂z ∂y ∂z u ∂z
= + = e cos v + eu sin v.
∂u ∂x ∂u ∂y ∂u ∂x ∂y
∂z ∂z ∂x ∂z ∂y ∂z ∂z
(−eu sin v) + eu cos v.

.in
= + =
∂v ∂x ∂v ∂y ∂v ∂x ∂y
∂z ∂z u ∂z u ∂z ∂z
y = y e cos v + y e sin v = eu sin veu cos v + eu sin veu sin v
∂u ∂x ∂y ∂x ∂y

ng
∂z ∂z
= e2u sin v cos v + e2u sin2 v .
∂x ∂y
∂z ∂z ∂z

eri
x = −eu sin veu cos v + eu cos veu cos v
∂v ∂x ∂y
∂z ∂z
= −e2u sin v cos v + e2u cos2 v
∂x ∂y
e
gin
∂z ∂z ∂z
Adding we get y + x = e2u .
∂u ∂v ∂y
y − x z − x ∂u ∂u ∂u
Example 2.43. If u = f , prove that x2 + y2 + z2 = 0.
xy zx ∂x ∂y ∂z
En

y − x z − x
Solution. Given: u = f ,
xy zx
y−x z−x 1 1 1 1
Let r = ,s= ,r = − , s = − .
arn

xy zx x y x z
∴ u = f (r, s).
∴ u is a function of r and s and r and s are functions of x, y and z.
Now
Le

∂u ∂u ∂r ∂u ∂s ∂u  −1  ∂u  −1 
= + = +
∂x ∂r ∂x ∂s ∂x ∂r x2 ∂s x2
w.

∂u ∂u ∂u
∴ x2 =− − . (1)
∂x ∂r ∂s
∂u ∂u ∂r ∂u ∂s ∂u  −1  ∂u
= + = + (0)
ww

∂y ∂r ∂y ∂s ∂y ∂r y2 ∂s
∂u ∂u
y2 = . (2)
∂y ∂r
∂u ∂u ∂r ∂u ∂s ∂u ∂u  1 
= + = (0) +
∂z ∂r ∂z ∂s ∂z ∂r ∂s z2

Visit For More : www.LearnEngineering.in


Visit For More : www.LearnEngineering.in

Function of Several Variables 97

∂u ∂u
z2 = . (3)
∂z ∂s

∂u ∂u ∂u
(1) + (2) + (3) =⇒ x2 + y2 + z2 = 0.
∂x ∂y ∂z

∂u ∂v
Example 2.44. If u2 + 2v2 = 1 − x2 + y2 and u2 + v2 = x2 + y2 − 2, find and .
∂x ∂x

.in
Solution. Consider u and v as functions of x and y.

u2 + 2v2 = 1 − x2 + y2 .

ng
Differentiating partially w.r.t. x

eri
∂u ∂v
2u + 4v = −2x
∂x ∂x
∂u ∂v
u + 2v
∂x ∂x
e
= −x. (1)
gin
u2 + v2 = x2 + y2 − 2

Differentiating partially w.r.t. x


En

∂u ∂v
2u + 2v = 2x
∂x ∂x
∂u ∂v
arn

u +v = x. (2)
∂x ∂x
∂v
(1) − (2) ⇒ v = −2x
∂x
∂v −2x
Le

⇒ =
∂x v
∂u (−2x)
(1) ⇒ u + 2v = −x.
∂x v
w.

∂u
u = 3x
∂x
∂u 3x
ww

⇒ = .
∂x u

Example 2.45. If z is a function of x and y and


∂2 z ∂2 z ∂2 z ∂2 u
x = u cos α − v sin α, y = u sin α + v cos α, show that + = + . [Jun 2000]
∂x2 ∂y2 ∂u2 ∂v2

Visit For More : www.LearnEngineering.in


Visit For More : www.LearnEngineering.in

98 Engineering Mathematics - I

Solution.
∂z ∂z ∂x ∂z ∂y ∂z ∂z
= + = cos α + sin α.
∂u ∂x ∂u ∂y ∂u ∂x ∂y
∂2 z ∂  ∂z  ∂x ∂  ∂z  ∂y
= +
∂u2 ∂x ∂u ∂u ∂y ∂u ∂u
∂  ∂z ∂z  ∂  ∂z ∂z 
= cos α + sin α cos α + cos α + sin α sin α

.in
∂x ∂x ∂y ∂y ∂x ∂y
2
∂ z 2
∂ z 2
∂ z ∂2 z
= 2 cos2 α + sin α cos α + cos α sin α + 2 sin2 α. (1)
∂x ∂x∂y ∂y∂x ∂y

ng
∂z ∂z ∂x ∂z ∂y ∂z ∂z
= + = (− sin α) + cos α.
∂v ∂x ∂v ∂y ∂v ∂x ∂y
∂2 z ∂  ∂z  ∂x ∂  ∂z  ∂y

eri
= +
∂v2 ∂x ∂v ∂v ∂y ∂v ∂v
∂  ∂z ∂z  ∂  ∂z ∂z 
= − sin α + cos α (− sin α) + − sin α + cos α cos α.
∂x ∂x ∂y ∂y ∂x ∂y
2
∂ z
= 2 sin2 α −
2
∂ z
sin α cos α − e2
∂ z 2
∂ z
cos α sin α + 2 cos2 α. (2)
gin
∂x ∂x∂y ∂y∂x ∂y
∂2 z ∂2 z ∂2 z 2 2 ∂2 z
(1) + (2) ⇒ + = (sin α + cos α) + (sin2 α + cos2 α)
∂u2 ∂v2 ∂x2 ∂y2
En

∂2 z ∂2 z
= + .
∂x2 ∂y2
Example 2.46. If F is u u
" 2a function # of x and y and if x = e sin v, y = e cos v, prove
arn

2
∂ F ∂ F 2 ∂ F ∂ F 2
that 2 + 2 = e−2u + 2 [Jan 2013]
∂x ∂y ∂u2 ∂v
∂F ∂F ∂x ∂F ∂y
Solution. = · + ·
∂u ∂x ∂u ∂y ∂u
Le

∂F u ∂F u
= · e sin v + · e cos v.
∂x ∂y
∂2 F
! !
∂ ∂F ∂x ∂ ∂F ∂y
w.

2
= +
∂u ∂x ∂u ∂u ∂y ∂u ∂u
" # " #
∂ ∂F u ∂F u ∂x ∂ ∂F u ∂F u ∂y
= · e sin v + · e cos v + · e sin v + · e cos v
∂x ∂x ∂y ∂u ∂y ∂x ∂y ∂u
ww

" 2
∂2 F u
" 2
∂2 F u
# #
∂ F u u ∂ F u
= · e sin v + · e cos v · e sin v + · e cos v + · e sin v · eu cos v
∂x2 ∂x∂y ∂y2 ∂y∂x
∂2 F ∂2 F 2u ∂2 F 2u ∂2 F
= 2 · e2u sin2 v + · e sin v cos v + · e sin v cos v + 2 · e2u cos2 v (1)
∂x ∂x∂y ∂y∂x ∂y

Visit For More : www.LearnEngineering.in


Visit For More : www.LearnEngineering.in

Function of Several Variables 99

∂F ∂F ∂x ∂F ∂y
= · + · .
∂v ∂x ∂v ∂y ∂v
∂F u ∂F u
= · e cos v − · e sin v.
∂x ∂y
∂2 F
! !
∂ ∂F ∂x ∂ ∂F ∂y
= +
∂v2 ∂x ∂v ∂v ∂y ∂v ∂v
" # " #
∂ ∂F u ∂F u u ∂ ∂F u ∂F u
· e sin v (−eu sin v)

.in
= · e cos v − · e sin v e cos v + · e cos v −
∂x ∂x ∂y ∂y ∂x ∂y
∂2 F ∂2 F 2u ∂2 F 2u ∂2 F
= 2 · e2u cos2 v − · e sin v cos v − · e sin v cos v + 2 · e2u sin2 v. (2)
∂x ∂x∂y ∂y∂x ∂y

ng
∂2 F ∂2 F ∂2 F 2u ∂2 F 2u
(1) + (2) ⇒ 2 + 2 = 2 e + 2 · e
∂u ∂v ∂x ∂y
" 2
∂2 F
#
2u ∂ F

eri
=e + 2
∂x2 ∂y
" 2 2
# 2 2
∂ F ∂ F ∂ F ∂ F
e−2u + 2 = 2 + 2.
∂u 2 ∂v ∂x ∂y
e
gin
Example 2.47. Transform the equation z xx + 2z xy + zyy = 0 by changing the
independent variables using u = x − y and v = x + y. [Jun 2012]
En

Solution. Consider z as a function of u and v and u and v are functions of x and y.

∂z ∂z ∂u ∂z ∂v
= · + ·
arn

∂x ∂u ∂x ∂v ∂x
∂z ∂z
= ·1+ ·1
∂u ∂v
∂z ∂z
= + .
Le

∂u ∂v!
∂2 z
!
∂ ∂z ∂u ∂ ∂z ∂v
= · + ·
∂x2 ∂u ∂x ∂x ∂v ∂x ∂x
w.

! !
∂ ∂z ∂z ∂ ∂z ∂z
= + ·1+ + ·1
∂u ∂u ∂v ∂v ∂u ∂v
∂2 z ∂2 z ∂2 z ∂2 z
ww

= 2+ + + 2
∂u ∂u∂v ∂v∂u ∂v
i.e.,z xx = zuu + zuv + zvu + zvv (1)
∂z ∂z ∂u ∂z ∂v
= · + ·
∂y ∂u ∂y ∂v ∂y

Visit For More : www.LearnEngineering.in


Visit For More : www.LearnEngineering.in

100 Engineering Mathematics - I

∂z ∂z
= (−1) + ·1
∂u ∂v
∂z ∂z
=− + .
∂u !∂v
∂2 z
!
∂ ∂z ∂u ∂ ∂z ∂v
= +
∂y2 ∂u ∂y ∂y ∂v ∂y ∂y
! !
∂ ∂z ∂z ∂ ∂z ∂z
= − + (−1) + − + ·1

.in
∂u ∂u ∂v ∂v ∂u ∂v
∂2 z ∂2 z ∂2 z ∂2 z
= 2− − + 2.
∂u ∂u∂v ∂v∂u ∂v

ng
i.e., zyy = zuu − zuv − zvu + zvv (2)
∂2 z
!
∂ ∂z
=

eri
∂x∂y ∂x ∂y
! !
∂ ∂z ∂u ∂ ∂z ∂v
= +
∂u ∂y ∂x ∂v ∂y ∂x

=
∂ ∂z ∂z
− +
!
·1+

e
− +
∂z ∂z
!
·1
gin
∂u ∂u ∂v ∂v ∂u ∂v
∂2 z ∂2 z ∂2 z ∂2 z
=− 2 + − + 2.
∂u ∂u∂v ∂v∂u ∂v
i.e., z xy = −zuu + zuv − zvu + zvv (3)
En

(1) + (2) + 2 × (3) ⇒

z xx + 2z xy + zyy = 2zuv − 2zvu + 4zvv


arn

0 = 2zuv − 2zvu + 4zvv .

∴ zuv − zvu + 2zvv = 0


Le

which is the required equation.


w.

Example 2.48. If z = f (x, y)! where x = u2 − v2 and y = 2uv, prove that


∂2 z ∂2 z 2
2
2 ∂ z ∂2 z
+ = 4(u + v ) + . [Jan 2012, Jan 2010]
∂u2 ∂v2 ∂x2 ∂y2
ww

Solution. z is a function of x and y and x and y are functions of u and v.


∴ z is a composite function of u and v.

∂z ∂z ∂x ∂z ∂y
= · + ·
∂u ∂x ∂u ∂y ∂u

Visit For More : www.LearnEngineering.in


Visit For More : www.LearnEngineering.in

Function of Several Variables 101

∂z ∂z
= · 2u + · 2v.
∂x ∂y
∂2 z
!
∂ ∂z
=
∂u2 ∂u ∂u
! !
∂ ∂z ∂x ∂ ∂z ∂y
= · + · .
∂x ∂u ∂u ∂y ∂u ∂u
" # " #
∂ ∂z ∂z ∂ ∂z ∂z

.in
= · 2u + · 2v 2u + · 2u + · 2v 2v
∂x ∂x ∂y ∂y ∂x ∂y
∂2 z ∂2 z ∂2 z ∂2 z
= 2 · 4u2 + 4uv + 4uv + 2 · 4v2 .
∂x ∂x∂y ∂y∂x ∂y

ng
∂z ∂z ∂x ∂z ∂y
= · + ·
∂v ∂x ∂v ∂y ∂v

eri
∂z ∂z
= · (−2v) + · 2u
∂x ∂y
∂z ∂z
= −2v + 2u .

∂2 z ∂ ∂z
∂x
!
∂y
e
gin
=
∂v2 ∂v ∂v
! !
∂ ∂z ∂x ∂ ∂z ∂y
= + .
∂x ∂v ∂v ∂y ∂v ∂v
En

" # " #
∂ ∂z ∂z ∂ ∂z ∂z
= −2v + 2u (−2v) + −2v · + 2u · 2u
∂x ∂x ∂y ∂y ∂x ∂y
∂2 z ∂2 z ∂2 z ∂2 z
= 4v2 2 − 4uv − 4uv + 4v2 2
arn

∂x ∂x∂y ∂y∂x ∂y
∂2 z ∂2 z 2
∂ z 2
∂ z
+ = 2 (4u2 + 4v2 ) + 2 (4u2 + 4v2 )
∂u2 ∂v2 ∂x ∂y
" 2
∂2 z
Le

#
2 2 ∂ z
= 4(u + v ) + .
∂x2 ∂y2

Example 2.49. If g(x, y) = χ(u, 2 2


! v) where u = x − y , v = 2xy, prove that
w.

2
∂ g ∂ g2 2 2
∂ χ ∂ χ
2
+ 2 = 4(x2 + y2 ) + .
∂x ∂y ∂u2 ∂v2
Solution.
ww

Similar to the previous problem.

Visit For More : www.LearnEngineering.in


Visit For More : www.LearnEngineering.in

102 Engineering Mathematics - I

du
Example 2.50. If u = x2 + y2 + z2 and x = e2t , y = e2t cos 3t, z = e2t sin 3t, find .
dt
[Dec 2011]
Solution.

du ∂u dx ∂u dy ∂u dz
= · + · + ·
dt ∂x dt ∂y dt ∂z dt

.in
h i
= 2x · 2e2t + 2y −3e2t sin 3t + 2e2t cos 3t
h i
+ 2z 3e2t cos 3t + 2e2t sin 3t

ng
h i
= 2e2t · 2e2t + 2e2t cos 3t −3e2t sin 3t + 2e2t cos 3t
h i
+ 2e2t sin 3t 3e2t cos 3t + 2e2t sin 3t

eri
= 4e4t − 6e4t cos 3t sin 3t + 4e4t cos2 3t + 6e4t sin 3t cos 3t + 4e4t sin2 3t

= 4e4t + 4e4t (cos2 3t + sin2 3t) = 4e4t + 4e4t = 8e4t .

dy e 2 2 2
gin
Example 2.51. Find if x and y are connected by the relation x 3 + y 3 = a 3 .
dx 2 2 2
Solution. Let f (x, y) = x 3 + y 3 − a 3

∂f 2 1
= x− 3 .
En

fx =
∂x 3
∂f 2 1
fy = = y− 3 .
∂y 3
arn

dy fx  x − 13  y  31
Now =− =− =− .
dx fy y x

dy y
Example 2.52. Find if f (x, y) = log(x2 + y2 ) + tan−1 .
Le

dx x
Solution.

∂f 2x 1  y 2x y 2x − y
w.

fx = = 2 + − = 2 − 2 = 2 .
∂x x + y2 y2 x 2 x +y 2 x +y 2 x + y2
1+
x2
∂f 2y 1 1 2y x 2y + x
ww

fy = = 2 + = + 2 = 2 .
∂y x + y2 y2 x x2 +y 2 x +y 2 x + y2
1+
x2
dy fx 2x − y y − 2x
Now =− =− = .
dx fy 2y + x x + 2y

Visit For More : www.LearnEngineering.in


Visit For More : www.LearnEngineering.in

Function of Several Variables 103

2.5 Jacobian and its Properties

Definition. If u and v are continuous functions of two independent variables x


and y having first order partial derivatives, then the Jacobian determinant or
the Jacobian of u and v is defined by

.in
∂(u, v)  u, v  ∂u ∂u
or J or J = ∂x ∂y
.
∂(x, y) x, y ∂v ∂v
∂x ∂y

ng
If u, v, w are continuous functions of three independent variables x, y, z having first
order partial derivatives, then the Jacobian of u, v, w w.r.t. x, y, z is defined as

eri

∂u ∂u ∂u
∂x ∂y ∂z
∂(u, v, w) ∂v ∂v ∂v .

=

e
∂(x, y, z) ∂x
∂w
∂y
∂w
∂z
∂w

gin
∂x ∂y ∂z

Properties of Jacobians
Property I. If u and v are functions of r and s where r and s are functions of x, y
En

then
∂(u, v) ∂(u, v) ∂(r, s)
= .
∂(x, y) ∂(r, s) ∂(x, y)
arn

Proof. Since u and v are functions of x and y, we have

∂u ∂u ∂r ∂u ∂s
Le

ux = = + = ur r x + u s s x .
∂x ∂r ∂x ∂s ∂x
w.

∂u ∂u ∂r ∂u ∂s
uy = = + = ur ry + u s sy .
∂y ∂r ∂y ∂s ∂y
ww

∂v ∂v ∂r ∂v ∂s
vx = = + = vr r x + v s s x .
∂x ∂r ∂x ∂s ∂x

∂v ∂v ∂r ∂v ∂s
vy = = + = vr ry + v s sy .
∂y ∂r ∂y ∂s ∂y

Visit For More : www.LearnEngineering.in


Visit For More : www.LearnEngineering.in

104 Engineering Mathematics - I


∂(u, v) ∂(r, s) ur u s r x ry
Now =
∂(r, s) ∂(x, y) vr v s s x

sy

ur u s r x s x
=
v v r s
r s y y

ur r x + u s s x ur ry + u s sy

.in
=
v r + v s v r + v s
r x s x r y s y

u x uy

ng
=
v v
x y
∂(u, v)
= .

eri
∂(x, y)

Property II. If J1 is the Jacobian of u, v with respect to x, y and J2 is the Jacobian


of x, y w.r.t. u, v then J1 J2 = 1. i.e., •
e
∂(u, v) ∂(x, y)
= 1.
gin
∂(x, y) ∂(u, v)
Proof. u is a function of x and y.
Differentiating partially with respect to u and v we get
En

∂u ∂x ∂u ∂y
1= + = u x xu + uy yu .
∂x ∂u ∂y ∂u
arn

∂u ∂x ∂u ∂y
0= + = u x xv + uy yv .
∂x ∂v ∂y ∂v
Le

v is a function of x and y.
Differentiating with respect to u and v partially we get
w.

∂v ∂x ∂v ∂y
0= + = v x xu + vy yv .
ww

∂x ∂u ∂y ∂v

∂v ∂x ∂v ∂y
1= + = v x xv + vy yv .
∂x ∂v ∂y ∂v

Visit For More : www.LearnEngineering.in


Visit For More : www.LearnEngineering.in

Function of Several Variables 105


∂(u, v) ∂(x, y) u x uy xu xv
Now =
∂(x, y) ∂(u, v) v x vy yu

yv

u x uy xu yu
=
v v x y
x y v v

u x xu + uy yv u x xv + uy yv

.in
=
v x + v y v x + v y
x u y u x v y v

1 0

ng
=
0 1

J1 J2 = 1.

eri
Property III. If the functions u, v, w of three independent variables x, y, z are not

e
independent, then the Jacobian of u, v, w with respect to x, y, z vanishes.
gin
Solution. Given: u, v and w are not independent variables.
=⇒ There exists a relation F(u, v, w) = 0.
Differentiating this w.r.t x, y and z we get
En

∂F ∂u ∂F ∂v ∂F ∂w
+ + =0
∂u ∂x ∂v ∂x ∂w ∂x
arn

∂F ∂u ∂F ∂v ∂F ∂w
+ + =0
∂u ∂y ∂v ∂y ∂w ∂y
∂F ∂u ∂F ∂v ∂F ∂w
+ + = 0.
Le

∂u ∂z ∂v ∂z ∂w ∂z
∂F ∂F ∂F
Eliminating , and from the above three equations we get
∂u ∂v ∂w
w.


∂u ∂v ∂w
∂x ∂x ∂x
∂u ∂v ∂w = 0.
ww

∂y ∂y ∂y
∂u ∂v ∂w
∂z ∂z ∂z

∂(u, v, w)
=⇒ = 0.
∂(x, y, z)

Visit For More : www.LearnEngineering.in


Visit For More : www.LearnEngineering.in

106 Engineering Mathematics - I

Functional relationship. Let u1 , u2 , . . . , un be functions of x1 , x2 , x3 , . . . , xn . Then,


a necessary and sufficient condition that u1 , u2 , . . . , un are not independent is that
∂(u1 , u2 , . . . , un )
the Jacobian = 0.
∂(x1 , x2 , . . . , xn ) ✎ ☞
Worked Examples
✍ ✌

Example 2.53. If x = r cos θ, y = r sin θ, find the Jacobian of x and y w.r.t r and θ.

.in
[Jan 2001]
Solution.

ng
∂x ∂x
x = r cos θ =⇒ = cos θ, = −r sin θ.
∂r ∂θ
∂u ∂u
y = r sin θ =⇒ = sin θ, = r cos θ.

eri
∂r ∂θ
∂(x, y) ∂x
∂r
∂x
∂θ
cos θ −r sin θ
= r cos2 θ + r sin2 θ = r.
Now, = =
∂(r, θ) ∂y ∂y sin θ r cos θ
∂r ∂θ
e
gin
Example 2.54. The transformation equations in spherical polar coordinates is
x = r sin θ cos ϕ, y = r sin θ sin ϕ, z = r cos θ. Compute the Jacobian of x, y, z w.r.t. r, θ, ϕ.
[May 2011, Jan 2009]
En

Solution.

∂x ∂x ∂x sin θ cos ϕ r cos θ cos ϕ −r sin θ sin ϕ
arn

∂r ∂θ ∂ϕ
∂(x, y, z) ∂y ∂y ∂y

= ∂r ∂θ ∂ϕ
= sin θ sin ϕ r sin ϕ cos θ r sin θ cos ϕ
∂(r, θ, ϕ)
∂z ∂z ∂z cos θ −r sin θ 0
∂r ∂θ ∂ϕ
Le


sin θ cos ϕ cos θ cos ϕ − sin ϕ

= r2 sin θ sin θ sin ϕ sin ϕ cos θ cos ϕ

w.

cos θ − sin θ 0

= r2 sin θ sin θ cos ϕ(0 + sin θ cos ϕ) − cos θ cos ϕ(0 − cos θ cos ϕ)
ww

− sin ϕ(− sin2 θ sin ϕ − cos2 θ sin ϕ)




= r2 sin θ sin2 θ cos2 ϕ + cos2 θ cos2 ϕ + sin2 θ sin2 ϕ + cos2 θ sin2 ϕ




= r2 sin θ(cos2 ϕ + sin2 ϕ) = r2 sin θ.

Visit For More : www.LearnEngineering.in


Visit For More : www.LearnEngineering.in

Function of Several Variables 107

Example 2.55. In cylindrical polar coordinates, x = ρ cos φ, y = ρ sin φ, z = z. Show


∂(x, y, z)
that = ρ.
∂(ρ, φ, z)
Solution.

∂x ∂x ∂x cos φ −ρ sin φ 0
∂ρ ∂φ ∂z
∂(x, y, z) ∂y ∂y ∂y
= ∂ρ ∂φ ∂z = sin φ ρ cos φ 0
∂(ρ, φ, z)

.in

∂z ∂z ∂z 0 0 1
∂ρ ∂φ ∂z

= 1(ρ cos2 φ + ρ sin2 φ) = ρ.

ng
∂(u, v)
Example 2.56. If u = 2xy, v = x2 − y2 , x = r cos θ, y = r sin θ, evaluate without
∂(r, θ)

eri
actual substitution. [Jan 2009]
Solution. Given that u and v are functions of x and y. x and y are functions of r
and θ.
∴ By property (1) of the Jacobians we have e
gin
∂(u, v) ∂(u, v) ∂(x, y)
= .
∂(r, θ) ∂(x, y) ∂(r, θ)

En

∂u ∂u
∂(u, v) ∂x ∂y
2y 2x
= =
∂(x, y) ∂v ∂v 2x −2y

∂x ∂y

= −4y2 − 4x2 = −4(x2 + y2 ) = −4r2 .


arn


∂x ∂x
∂(x, y) ∂r ∂θ cos θ −r sin θ

= =
∂(r, θ) ∂y ∂y sin θ r cos θ

∂r ∂θ
Le

= r cos2 θ + r sin2 θ = r
∂(u, v)
= −4(r2 )r = −4r3 .
w.

∂(r, θ)

u+v ∂(u, v)
Example 2.57. If x = uv, y = , find .
u−v ∂(x, y)
ww

∂(u, v) ∂(x, y) ∂(u, v) 1


Solution. We know that =1⇒ = .
∂(x, y) ∂(u, v) ∂(x, y) ∂(x, y)
∂(u, v)
x = uv.

Visit For More : www.LearnEngineering.in


Visit For More : www.LearnEngineering.in

108 Engineering Mathematics - I

∂x ∂x
= v, = u.
∂u ∂v
u+v
y= .
u−v
∂y u − v − (u + v) u − v − u − v −2v
= 2
= 2
= .
∂u (u − v) (u − v) (u − v)2
∂y (u − v).1 − (u + v)(−1) 2u

.in
= 2
= .
∂v (u − v) (u − v)2

ng

∂x ∂x v u
∂(x, y) ∂u ∂v
= = −2v 2u

∂(u, v) ∂y ∂y
∂u ∂v (u − v)2
(u − v)2

eri
2uv 2uv 4uv
= 2
+ 2
=
(u − v) (u − v) (u − v)2
∂(u, v) (u − v)2
∂(x, y)
=
4uv
.
e
gin
Example 2.58. If x = u(1 − v), y = uv then compute J1 and J2 and prove that
J1 J2 = 1.
Solution. We have
En

∂(x, y) ∂(u, v)
J1 = , J2 =
∂(u, v) ∂(x, y)
arn

x = u − uv, y = uv.

∂x
∂x 1 − v −u
J1 = ∂u
∂v
= = u − uv + uv = u.
∂y
∂y v u
∂u ∂v
Le

We shall express u and v interms of x and y.

x = u − uv = u − y ⇒ x + y = u.
w.

y y
y = uv ⇒ v = = .
u x+y
ww

1
we get J2 = .
u
1
Now J1 J2 = u = 1.
u

Visit For More : www.LearnEngineering.in


Visit For More : www.LearnEngineering.in

Function of Several Variables 109

x2 x3 x3 x1 x1 x2 ∂(y1 , y2 , y3 )
Example 2.59. If y1 = , y2 = , y3 = , prove that = 4.
x1 x2 x3 ∂(x1 , x2 , x3 )
[Jan 2014]
∂y1 ∂y1 ∂y1
∂x1 ∂x2 ∂x3
∂(y1 , y2 , y3 ) ∂y2 ∂y2 ∂y2
Solution. = ∂x ∂x ∂x
∂(x1 , x2 , x3 ) 1 2 3
∂y3 ∂y3 ∂y3
∂x1 ∂x2
∂x3

.in
2 −x2 x3 x3 x2
x1 x1 x1

= xx32 −x3 x1 x1

x22 x2

ng
x2 x1 −x1 x2
x3 x3 2
x3
 2
x12  x3  x1 x2 x3 x1 x2 
    
−x2 x3  x1 x2 x3 x2  x1 x3 x1 x2 x3 
=  2 2 −  − − −  +  + 2 
x12 x2 x3  x1  x2 x32

eri
x2 x3 x2 x3  x1 x2 x3 x2 x3
= −1 + 1 + 1 + 1 + 1 + 1 = 4.

Example 2.60. If u = x + y + z, uv = y + z, uvw = z, find e ∂(x, y, z)


.
gin
∂(u, v, w)
[Jan 2012, Jan 2010]
Solution. Given u = x + y + z ⇒ u = x + uv ⇒ x = u − uv = u(1 − v).
En

uv = y + z ⇒ uv = y + uvw ⇒ y = uv − uvw = uv(1 − w), uvw = z.



∂x ∂x ∂x 1 − v −u 0
∂u ∂v ∂w
∂(x, y, z) ∂y
arn


∂y ∂y
= ∂u ∂v
= v(1 − w) u(1 − w) −uv
∂w
∂(u, v, w)
∂z ∂z ∂z vw uw uv
∂u ∂v ∂w

1 − v −1 0
Le


= u(uv) v(1 − w) 1 − w −1

vw w 1
w.

= u2 v {(1 − v)(1 − w + w) + 1(v(1 − w) + vw)}

= u2 v {(1 − v) + (v − vw + vw)}
ww

= u2 v {1 − v + v} = u2 v.

∂(x, y, z)
Example 2.61. If u = xyz, v = x2 + y2 + z2 , w = x + y + z, find .
∂(u, v, w)

Visit For More : www.LearnEngineering.in


Visit For More : www.LearnEngineering.in

110 Engineering Mathematics - I

∂(x, y, z) ∂(u, v, w) ∂(x, y, z) 1


Solution. We have =1⇒ = .
∂(u, v, w) ∂(x, y, z) ∂(u, v, w) ∂(u, v, w)
∂(x, y, z)

∂u ∂u ∂u yz xz xy
∂x ∂y ∂z
∂(u, v, w) ∂v ∂v

∂v =

= ∂x ∂y ∂z 2x 2y 2z
∂(x, y, z)
∂w ∂w ∂w
1 1 1

.in
∂x ∂y ∂z

= yz(2y − 2z) − xz(2x − 2z) + xy(2x − 2y)

ng
= 2(yz(y − z) − xz(x − z) + xy(x − y))

= 2(y2 z − yz2 − x2 z + xz2 + xy(x − y))

eri
= 2(z2 (x − y) + xy(x − y) − z(x2 − y2 ))

= 2(z2 (x − y) + xy(x − y) − z(x − y)(x + y))

e
= 2(x − y)(z2 + xy − xz − yz)
gin
= 2(x − y)(z(z − x) − y(z − x))

= 2(x − y)(z − x)(z − y)


En

= −2(x − y)(y − z)(z − x).

∂(x, y, z) 1 1
Now, ⇒ = = .
∂(u, v, w) ∂(u, v, w) −2(x − y)(y − z)(z − x)
arn

∂(x, y, z)

Example 2.62. If u = x + 2y + z, v = x − 2y + 3z and w = 2xy − xz + 4yz − 2z2 , show that


Le

they are not independent. Find the relation between u, v and w.


Solution. Given: u = x + 2y + z.
w.

∂u ∂u ∂u
= 1, = 2, = 1.
∂x ∂y ∂z
ww

v = x − 2y + 3z.
∂v ∂v ∂v
= 1, = −2, = 3.
∂x ∂y ∂z

w = 2xy − xz + 4yz − 2z2 .

Visit For More : www.LearnEngineering.in


Visit For More : www.LearnEngineering.in

Function of Several Variables 111

∂w ∂w ∂w
= 2y − z, = 2x + 4z, = −x + 4y − 4z.
∂x ∂y ∂z


∂u ∂u ∂u 1 2 1
∂x ∂y ∂z
∂(u, v, w) ∂v ∂v ∂v

= ∂x = 1 −2 3

.in
∂(x, y, z) ∂y ∂z
∂w ∂w ∂w
2y − z 2x + 4z −x + 4y − 4z
∂x ∂y ∂z

= 1(−2(−x + 4y − 4z) − 3(2x + 4z)) − 2(−x + 4y − 4z − 3(2y − z))

ng
+ 1(2x + 4z + 2(2y − z))

= 2x − 8y + 8z − 6x − 12z + 2x − 8y + 8z + 12y − 6z + 2x + 4z + 4y − 2z = 0.

eri
Hence, u, v, w are not independent.
Now u + v = 2x + 4z, u − v = 4y − 2z. e
gin
(u + v)(u − v) = 2(x + 2z).2(2y − z)
En

u2 − v2 = 4(2xy − xz + 4yz − 2z2 )

u2 − v2 = 4w.
arn

2.6 Taylor’s expansion for functions of two variables


Le

We know that for a function f (x) of one single variable x, the Taylor’s expansion
w.

is

h2 ′′ h3 ′′′
f (x + h) = f (x) + h f ′ (x) +
ww

f (x) + f (x) + · · ·
2! 3!

Now let f (x, y) be a function of two independent variables x, y defined in a region


R of the xy−plane and let (a, b) be a point in R. Suppose f (x, y) has all its partial

Visit For More : www.LearnEngineering.in


Visit For More : www.LearnEngineering.in

112 Engineering Mathematics - I

derivatives in a neighbourhood of (a, b) then

 ∂ ∂
f (a + h, b + k) = f (a, b) + h + k f (a, b)
∂x ∂y
1 ∂ ∂ 2 1 ∂ ∂ 3
+ h +k f (a, b) + h +k f (a, b) + · · · +
2! ∂x ∂y 3! ∂x ∂y
 
= f (a, b) + h f x (a, b) + k fy (a, b)

.in
1 2 
+ h f xx (a, b) + 2hk f xy (a, b) + k2 fyy (a, b)
2!
1 3

ng
+ h f xxx (a, b) + 3h2 k f xxy (a, b) + 3hk2 f xyy (a, b)
3!

+ k3 fyyy (a, b) + · · · .

eri
Put x = a + h, y = b + k then h = x − a, k = y − b.
∴ The Taylor’s series can be written as

e
gin
 
f (x, y) = f (a, b) + (x − a) f x (a, b) + (y − b) fy (a, b)
1 
+ (x − a)2 f xx (a, b) + 2(x − a)(y − b) f xy (a, b) + (y − b)2 fyy (a, b)
2!
En

1
+ (x − a)3 f xxx (a, b) + 3(x − a)2 (y − b) f xxy (a, b)
3!

+ 3(x − a)(y − b)2 f xyy (a, b) + (y − b)3 fyyy (a, b) + · · · .
arn

This is known as the Taylor’s expansion of f (x, y) in the neighbourhood of (a, b) or


about the point (a, b).
Le

Put a = 0, b = 0. we get

 
f (x, y) = f (0, 0) + x f x (0, 0) + y fy (0, 0)
w.

1 2 
+ x f xx (0, 0) + 2xy f xy (0, 0) + y2 fyy (0, 0)
2!
1 3
ww


+ x f xxx (0, 0) + 3x2 y f xxy (0, 0) + 3xy2 f xyy (0, 0) + y3 fyyy (0, 0) + · · · .
3!

This is called Maclaurin’s series for f (x, y) in powers of x and y.


✎ ☞
Worked Examples
✍ ✌

Visit For More : www.LearnEngineering.in


Visit For More : www.LearnEngineering.in

Function of Several Variables 113

Example 2.63. Expand e x sin y in powers of x and y as far as the terms of third
degree. [Jun 2013]
Solution. f (x, y) = e x sin y f (0, 0) = 0

f x (x, y) = e x sin y f x (0, 0) = 0

fy (x, y) = e x cos y

.in
fy (0, 0) = 1

f xx (x, y) = e x sin y f xx (0, 0) = 0

ng
f xy (x, y) = e x cos y f xy (0, 0) = 1

fyy (x, y) = −e x sin y fyy (0, 0) = 0

eri
f xxx (x, y) = e x sin y f xxx (0, 0) = 0

f xxy (x, y) = e x cos y


e
f xxy (0, 0) = 1
gin
f xyy (x, y) = −e x sin y f xyy (0, 0) = 0

fyyy (x, y) = −e x cos y fyyy (0, 0) = −1.


En
arn

1 2 
Now f (x, y) = f (0, 0) + x f x (0, 0) + y fy (0, 0) + x f xx (0, 0) + 2xy f xy (0, 0) + y2 fyy (0, 0)
2!
Le

1 3 
+ x f xxx (0, 0) + 3x2 y f xxy (0, 0) + 3xy2 f xyy (0, 0) + y3 fyyy (0, 0) + · · ·
3!
1 
= 0 + x.0 + y.1 + x2 .0 + 2xy.1 + y2 .0
w.

2
1 3 
x .0 + 3x y.1 + 3xy2 .0 + y3 (−1) + · · ·
2
6
ww

x2 y y3
= y + xy + − + ··· .
2 6

Example 2.64. Expand e x loge (1 + y) in powers of x and y upto terms of third


degree. [Jan 2014, Dec 2011, Jan 2003]
Solution.

Visit For More : www.LearnEngineering.in


Visit For More : www.LearnEngineering.in

114 Engineering Mathematics - I

2e x
fyyy =
(1 + y)3

f (x, y) = e x loge (1 + y)
f (0, 0) = 0
f x (x, y) = e x loge (1 + y)

.in
ex f x (0, 0) = 0
fy (x, y) =
1+y fy (0, 0) = 1

ng
x
f xx = e log(1 + y)
f xx (0, 0) = 0
ex
f xy = f xy (0, 0) = 1
1+y

eri
−e x fyy (0, 0) = −1
fyy =
(1 + y)2
f xxx (0, 0) = 0
f xxx = e x log(1 + y)
e
gin
ex f xxy (0, 0) = 1
f xxy =
1+y f xyy (0, 0) = −1
−e x
f xyy = fyyy (0, 0) = 2
(1 + y)2
En

By Maclaurin’s series we have,

f (x, y) = f (0, 0) + x f x (0, 0) + y fy (0, 0)


arn

1 2 
+ x f xx (0, 0) + 2xy f xy (0, 0) + y2 fyy (0, 0)
2!
1 3
+ x f xxx (0, 0) + 3x2 y f xxy (0, 0)
3!
Le


+ 3xy2 f xyy (0, 0) + y3 fyyy (0, 0) + · · · .
1
e x loge (1 + y) = 0 + x.0 + y.1 + (x2 .0 + 2xy.1 + y2 (−1))+
w.

2
1 3
[x · 0 + 3x2 y · 1 + 3xy2 (−1) + y3 (2)] + . . . .
6
y2 x2 y xy2 y3
ww

= y + xy − + − + − ···
2 2 2 3

Example 2.65. Expand x2 y + 3y − 2 in powers of x − 1 and y + 2 upto third degree


terms. [Jun 2012]

Visit For More : www.LearnEngineering.in


Visit For More : www.LearnEngineering.in

Function of Several Variables 115

Solution. f (x, y) = x2 y + 3y − 2. f (1, −2) = 1 × (−2) + 3(−2) − 2

= −2 − 6 − 2 = −10.
f x (x, y) = 2xy.
f x (1, −2) = −4.
2
fy (x, y) = x + 3.
fy (1, −2) = 4.

.in
f xx (x, y) = 2y.
f xx (1, −2) = −4.
f xy (x, y) = 2x.
f xy (1, −2) = 2.

ng
fyy (x, y) = 0.
fyy (1, −2) = 0.
f xxx (x, y) = 0.
f xxx (1, −2) = 0.

eri
f xxy (x, y) = 2.
f xxy (1, −2) = 2.
f xyy (x, y) = 0.

fyyy (x, y) = 0. e f xyy (1, −2) = 0.


gin
fyyy (1, −2) = 0.
En

By Taylor’s theorem we have,


arn

f (x, y) = f (a, b) + (x − a) f x (a, b) + (y − b) fy (a, b)


1 h i
+ (x − a)2 f xx (a, b) + 2(x − a)(y − b) f xy (a, b) + (y − b)2 fyy (a, b)
2!
Le

1 h
+ (x − a)3 f xxx (a, b) + 3(x − a)2 (y − b) f xxy (a, b)
3!
i
+3(x − a)(y − b)2 f xyy (a, b) + (y − b)3 fyyy (a, b) + · · ·
w.

x2 y + 3y − 2 = f (1, −2) + (x − 1) f x (1, −2) + (y + 2) fy (1, −2)


1 h
ww

i
+ (x − 1)2 f xx (1, −2) + 2(x − 1)(y + 2) f xy (1, −2) + (y + 2)2 fyy (1, −2)
2!
1 h
+ (x − 1)3 f xxx (1, −2) + 3(x − 1)2 (y + 2) f xxy (1, −2)
3!
i
+3(x − 1)(y + 2)2 f xyy (1, −2) + (y + 2)3 fyyy (1, −2) + · · ·

Visit For More : www.LearnEngineering.in


Visit For More : www.LearnEngineering.in

116 Engineering Mathematics - I

1h i
= −10 − 4(x − 1) + 4(y + 2) + −4(x − 1)2 + 2(x − 1)(y + 2)
2
1h i
+ 6(x − 1)2 (y + 2) + · · ·
6
= −10 − 4(x − 1) + 4(y + 2) − 2(x − 1)2 + (x − 1)(y + 2) + (x − 1)2 (y + 2) + · · ·

Example 2.66. Find the Taylor’s series expansion of x2 y2 + 2x2 y + 3y2 in powers of

.in
(x + 2) and y − 1 upto third degree terms. [Jan 2012, Jun 2010, Jan 2010]
Solution.

ng
f (x, y) = x2 y2 + 2x2 y + 3y2 . f (−2, 1) = 4 + 8 + 3 = 15.

f x (x, y) = 2xy2 + 4xy. f x (−2, 1) = −4 − 8 = −12.

eri
fy (x, y) = 2x2 y + 2x2 + 6y. fy (−2, 1) = 8 + 8 + 6 = 22.

f xx (x, y) = 2y2 + 4y. f xx (−2, 1) = 2 + 4 = 6.


e
gin
f xy (x, y) = 4xy + 4x. f xy (−2, 1) = −8 − 8 = −16.

fyy (x, y) = 2x2 + 6. fyy (−2, 1) = 8 + 6 = 14.

f xxx (x, y) = 0. f xxx (−2, 1) = 0.


En

f xxy (x, y) = 4y + 4. f xxy (−2, 1) = 4 + 4 = 8.

f xyy (x, y) = 4x. f xyy (−2, 1) = −8.


arn

fyyy (x, y) = 0. fyyy (−2, 1) = 0.


Le

By Taylor’s theorem we have,


w.

f (x, y) = f (a, b) + (x − a) f x (a, b) + (y − b) fy (a, b)


1 h
ww

i
+ (x − a)2 f xx (a, b) + 2(x − a)(y − b) f xy (a, b) + (y − b)2 fyy (a, b)
2!
1 h
+ (x − a)3 f xxx (a, b) + 3(x − a)2 (y − b) f xxy (a, b)
3!
i
+3(x − a)(y − b)2 f xyy (a, b) + (y − b)3 fyyy (a, b) + · · ·

Visit For More : www.LearnEngineering.in


Visit For More : www.LearnEngineering.in

Function of Several Variables 117

x2 y2 + 2x2 y + 3y2 = 15 − 12(x + 2) + 22(y − 1)


1h i
+ 6(x + 2)2 − 2 × 16(x + 2)(y − 1) + 14(y − 1)2
2
1h i
+ 24(x + 2)2 (y − 1) − 24(x + 2)(y − 1)2 + · · ·
6
= 15 − 12(x + 2) + 22(y − 1) + 3(x + 2)2 − 16(x + 2)(y − 1) + 7(y − 1)2

.in
+ 4(x + 2)2 (y − 1) − 4(x + 2)(y − 1)2 + · · ·

Example 2.67. Use Taylor’s formula to expand the function defined by f (x, y) =

ng
x3 + y3 + xy2 in powers of (x − 1) and (y − 2). [May 2011]
Solution.

eri
f (x, y) = x3 + y3 + xy2 . f (1, 2) = 1 + 8 + 4 = 13.

f x (x, y) = 3x2 + y2 .
e f x (1, 2) = 3 + 4 = 7.
gin
fy (x, y) = 3y2 + 2xy. fy (1, 2) = 12 + 4 = 16.

f xx (x, y) = 6x. f xx (1, 2) = 6.

f xy (x, y) = 2y. f xy (1, 2) = 4.


En

fyy (x, y) = 6y + 2x. fyy (1, 2) = 12 + 2 = 14.

f xxx (x, y) = 6. f xxx (1, 2) = 6.


arn

f xxy (x, y) = 0. f xxy (1, 2) = 0.

f xyy (x, y) = 2. f xyy (1, 2) = 2.


Le

fyyy (x, y) = 6. fyyy (1, 2) = 6.


w.
ww

By Taylor’s theorem we have,

f (x, y) = f (a, b) + (x − a) f x (a, b) + (y − b) fy (a, b)


1 h i
+ (x − a)2 f xx (a, b) + 2(x − a)(y − b) f xy (a, b) + (y − b)2 fyy (a, b)
2!

Visit For More : www.LearnEngineering.in


Visit For More : www.LearnEngineering.in

118 Engineering Mathematics - I

1 h
+ (x − a)3 f xxx (a, b) + 3(x − a)2 (y − b) f xxy (a, b)
3!
i
+3(x − a)(y − b)2 f xyy (a, b) + (y − b)3 fyyy (a, b) + · · ·
1h i
x3 + y3 + xy2 = 13 + 7(x − 1) + 16(y − 2) + 6(x − 1)2 + 8(x − 1)(y − 2) + 14(y − 2)2
2
1h i
+ 6(x − 1)3 + 6(x − 1)(y − 2)2 + 6(y − 2)3 + · · ·
6

.in
= 13 + 7(x − 1) + 16(y − 2) + 3(x − 1)2 + 4(x − 1)(y − 2) + 7(y − 2)2

+ (x − 1)3 + (x − 1)(y − 2)2 + (y − 2)3 + · · ·

ng
Example 2.68. Expand e−x log y as a Taylor’s series in powers of x and y − 1 upto
third degree terms. [Jun 2011]

eri
Solution.

f (x, y) = e−x log y.


e
f (0, 1) = 0.
gin
f x (x, y) = −e−x log y. f x (0, 1) = 0.
e−x
fy (x, y) = . fy (0, 1) = 1.
y
En

f xx (x, y) = e−x log y. f xx (0, 1) = 0.


e−x
f xy (x, y) = − . f xy (0, 1) = −1.
arn

y
e−x
fyy (x, y) = − . fyy (0, 1) = −1.
y2
Le

f xxx (x, y) = −e−x log y. f xxx (0, 1) = 0.


e−x
f xxy (x, y) = . f xxy (0, 1) = 1.
y
w.

e−x
f xyy (x, y) = . f xyy (0, 1) = 1.
y2
ww

2e−x
fyyy (x, y) = . fyyy (0, 1) = 2.
y3

Visit For More : www.LearnEngineering.in


Visit For More : www.LearnEngineering.in

Function of Several Variables 119

By Taylor’s theorem we have,

f (x, y) = f (a, b) + (x − a) f x (a, b) + (y − b) fy (a, b)


1 h i
+ (x − a)2 f xx (a, b) + 2(x − a)(y − b) f xy (a, b) + (y − b)2 fyy (a, b)
2!
1 h
+ (x − a)3 f xxx (a, b) + 3(x − a)2 (y − b) f xxy (a, b)
3!

.in
i
+3(x − a)(y − b)2 f xyy (a, b) + (y − b)3 fyyy (a, b) + · · ·
1 1
e−x log y = y − 1 + [−2x(y − 1) − (y − 1)2 ] + [3x2 (y − 1) + 3x(y − 1)2 + 2(y − 1)3 ] + · · ·
2! 3!

ng
y
Example 2.69. Expand f (x, y) = tan−1 about (1, 1) upto the second degree terms.
x
Hence compute f (1.1, 0.9) approximately. [Jan 2005]

eri
y
Solution. Given, f (x, y) = tan−1
x
(a, b) = (1, 1) a = 1, b = 1
f (1, 1) = tan−1
1 π
=
e
gin
1 4
1  −y  x2 y −y −1
fx = 2 2
= − 2 + y2 )x2
= 2 2
f x (1, 1) =
y
1 + x2 x (x x + y 2
En

1 1 x2 x 1
fy = 2
= 2 2 )x
= 2 2
fy (1, 1) =
y x (x + y x + y 2
1 + x2
2xy 2 1
f xx = −y(−1)(x2 + y2 )−2 2x = 2 f xx (1, 1) = =
arn

(x + y ) 2 2 4 2
2 2
x + y − x2x 2
y −x 2
f xy = 2 2 2
= 2 f xy (1, 1) = 0
(x + y ) (x + y2 )2
−2xy −1
Le

fyy = x(−1)(x2 + y2 )−2 2y = 2 2 2


fyy (1, 1) =
(x + y ) 2
w.

By Taylor’s theorem we have


ww

f (x, y) = f (a, b) + (x − a) f x (a, b) + (y − b) fy (a, b)


1 
+ (x − a)2 f xx (a, b) + 2(x − a)(y − b) f xy (a, b) + (y − b)2 fyy (a, b) + · · ·
2!

Visit For More : www.LearnEngineering.in


Visit For More : www.LearnEngineering.in

120 Engineering Mathematics - I

y
tan−1 = f (1, 1) + (x − 1) f x (1, 1) + (y − 1) fy (1, 1)
x
1 
+ (x − 1)2 f xx (1, 1) + 2(x − 1)(y − 1) f xy (1, 1) + (y − 1)2 fyy (1, 1)
2!
π  −1  1 1 1
= + (x − 1) + (y − 1) + (x − 1)2 + 2(x − 1)(y − 1)0
4 2 2 2 2
2 (−1)

+ (y − 1) + ···
2

.in
π 1 11 1 
= − (x − 1 − y + 1) + (x − 1)2 − (y − 1)2 + · · ·
4 2 2 2 2
π 1 1 2 2
= − (x − y) + (x − y − 2x + 2y) + · · ·

ng
4 2 4
y π 1 1 1 1
tan−1 = − (x − 1) + (y − 1) + (x − 1)2 − (y − 1)2 + · · · .
x 4 2 2 4 4

eri
π 1 1 1 1
f (1.1, 0.9) = − (0.1) + (−0.1) + (0.1)2 − (−0.1)2 approximately
4 2 2 4 4
π
= − 0.1 = 0.685 approximately.
4
e
gin
 π
Example 2.70. Find the Taylor’s series expansion of e x sin y at the point − 1,
4
upto third degree terms. [Jan 2009]
Solution.
En

π 1
f (x, y) = e x sin y f − 1, = √
4 e 2
arn

π 1
f x (x, y) = e x sin y f x − 1, = √
4 e 2
π 1
fy (x, y) = e x cos y fy − 1, = √
4 e 2
Le

π 1
f xx = e x sin y f xx − 1, = √
4 e 2
π 1
w.

f xy = e x cos y f xy − 1, = √
4 e 2
π 1
fyx = e x cos y fyx − 1, = √
ww

4 e 2
π −1
fyy = −e x sin y fyy − 1, = √
4 e 2
π 1
f xxx = e x sin y f xxx − 1, = √
4 e 2

Visit For More : www.LearnEngineering.in


Visit For More : www.LearnEngineering.in

Function of Several Variables 121

π 1
f xxy = e x cos y f xxy − 1, = √
4 e 2
π −1
f xyy = −e x sin y f xyy − 1, = √
4 e 2
π −1
fyyx = −e x sin y fyyx − 1, = √
4 e 2
π −1
fyyy = −e x sin y

fyyy − 1, = √

.in
4 e 2

ng
eri
By Taylor’s theorem we have

e
f (x, y) = f (a, b) + (x − a) f x (a, b) + (y − b) fy (a, b)
gin
1 
+ (x − a)2 f xx (a, b) + 2(x − a)(y − b) f xy (a, b) + (y − b)2 fyy (a, b)
2!
1
+ (x − a)3 f xxx (a, b) + 3(x − a)2 (y − b) f xxy (a, b)
3!
En


+ 3(x − a)(y − b)2 f xyy (a, b) + (y − b)3 fyyy (a, b) + · · · .
 π π π π
e x sin y = f − 1, + (x + 1) f x − 1, + (y − ) fy − 1,
4 4 4 4
arn

1 2 π π π
+ (x + 1) f xx − 1, + 2(x + 1)(y − ) f xy − 1,
2! 4 4 4
π 2 π 
+ (y − ) fyy − 1,
4 4
Le

1 π π π
3
+ (x + 1) f xxx − 1, + 3(x + 1)2 (y − ) f xxy − 1,
6 4 4 4
π 2 π π 3 π 
+ 3(x + 1)(y − ) f xyy − 1, + (y − ) fyyy − 1, + ··· .
w.

4 4 4 4
1 1 1 π 1
= √ + √ (x + 1) + √ y − + √ (x + 1)2
e 2 e 2 e 2 4 2 2e
ww

1 π 1 π 1
+ √ (x + 1) y − − √ y − 2 + √ (x + 1)3
2e 4 2 2e 4 6 2e
√ √
2 π 2 π 1 π
+ (x + 1)2 y − − (x + 1) y − 2 − √ y − 3 + · · · .
e 4 e 4 6 2e 4

Visit For More : www.LearnEngineering.in


Visit For More : www.LearnEngineering.in

122 Engineering Mathematics - I

 π
Example 2.71. Expand e x cos y near the point 1, by Taylor’s series as far as
4
quadratic terms. [Jan 1996]
Solution.

π e
f (x, y) = e x cos y f 1, = √
4 2
π π e

.in
f x (x, y) = e x cos y f x 1, = e cos = √
4 4 2
π e
fy (x, y) = −e x sin y fy 1, =−√
4 2

ng
π e
f xx = e x cos y f xx 1, = √
4 2
π  −e

eri
f xy = −e x sin y f xy 1, = √
4 2
π  −e
fyy = −e x cos y fyy 1, = √
4
e 2
gin
En

By Taylor’s theorem we have

f (x, y) = f (a, b) + (x − a) f x (a, b) + (y − b) fy (a, b)


arn

1 
+ (x − a)2 f xx (a, b) + 2(x − a)(y − b) f xy (a, b) + (y − b)2 fyy (a, b)
2!
x π π π π
e cos y = f 1, + (x − 1) f x 1, + (y − ) fy 1,
4 4 4 4
Le

1 π π π
(x − 1)2 f xx 1, + 2(x − 1)(y − ) f xy 1,

+
2! 4 4 4
π 2 π 
w.

+ (y − ) fyy 1, + ···
4 4
e e π  (−e)
= √ + (x − 1) √ + y − √
2 2 4 2
ww

1 e π  (−e) π 2 (−e) 
+ (x − 1)2 √ + 2(x − 1) y − √ + y− √ + ···
2 2 4 2 4 2
!
e π 1 π 1 π
= √ 1 + (x − 1) − y − + (x − 1)2 − y − (x − 1) − y − 2 + · · · .
2 4 2 4 2 4

Visit For More : www.LearnEngineering.in


Visit For More : www.LearnEngineering.in

Function of Several Variables 123

2.7 Maxima and Minima for functions of two variables


Definition. Let f (x, y) be a continuous function defined in a closed and bounded
domain D of the xy plane and let (a, b) be an interior point of D.
(i) f (a, b) is said to be a local maximum value of f (x, y) at the point (a, b) if there
exists a neighborhood N of (a, b) such that f (x, y) < f (a, b) for all points (x, y) in

.in
N.
(ii) f (a, b) is said to be a local minimum if f (x, y) > f (a, b) for all points (x, y) in N

ng
other than (a, b).

f (x, y) f (x, y)

eri
f (a, b)
f (a, b)

eX X
gin
(x, y) (a, b) (x, y) (x, y) (a, b) (x, y)

Y Y
En

Local maximum or local minimum values are called extreme values.


Stationary point of f (x, y)
arn

A point (a, b) satisfying f x = 0 and fy = 0 is called a stationary point of f (x, y).


Necessary conditions for Maximum or minimum
If f (a, b) is an extreme value of f (x, y) at (a, b), then (a, b) is a stationary point of
Le

f (x, y) if f x and fy exist at (a, b) and f x (a, b) = 0, fy (a, b) = 0.


Sufficient conditions for extreme values of f (x, y)
w.

Let (a, b) be a stationary point of the differentiable function f (x, y).


i.e., f x (a, b) = 0, fy = (a, b) = 0.
ww

Let us define f xx (a, b) = r, f xy (a, b) = s, fyy (a, b) = t.


(i) If rt − s2 > 0 and r < 0, then f (a, b) is a maximum value.
(ii) If rt − s2 > 0 and r > 0 then f (a, b) is a minimum value.
(iii) If rt − s2 < 0, then f (a, b) is not an extreme value but (a, b) is a saddle point of

Visit For More : www.LearnEngineering.in


Visit For More : www.LearnEngineering.in

124 Engineering Mathematics - I

f (x, y).
(iv) If rt − s2 = 0, then no conclusion is possible and further investigation is
required.
Working rule to find maxima and minima of f (x, y)
∂f ∂f
step (1). Find f x = and fy = and solve for f x = 0 and fy = 0 as simultaneous
∂x ∂y

.in
equations in x and y.
Let (a, b), (a1 , b1 ), . . . be the solutions which are stationary points of f (x, y).
∂2 f ∂2 f ∂2 f
step (2). Find r = 2 , s = ,t = 2 .

ng
∂x ∂x∂y ∂y
step (3). Evaluate r, s, t at each stationary point.
At the point (a, b) if

eri
(i) rt − s2 > 0 and r < 0 then f (a, b) is a maximum value of f (x, y).
(ii) rt − s2 > 0 and r > 0 then f (a, b) is a minimum value of f (x, y).

e
(iii) rt − s2 < 0 then (a, b) is called a saddle point.
gin
(iv) rt − s2 = 0, no conclusion can be made, further investigation is required.
Critical Point
A point (a, b) is a critical point of f (x, y) if f x = 0 and fy = 0 at (a, b) or f x and fy do
En

not exist at (a, b).


Maxima or Minima occur at a critical point.
✎ ☞
arn

Worked Examples
✍ ✌

Example 2.72. Examine f (x, y) = x3 + y3 − 12x − 3y + 20 for its extreme values.


Le

[ Jun 2013, Jan 2012, May 2011, Jun 2010]


Solution. Given f (x, y) = x3 + y3 − 12x − 3y + 20.
w.

f x = 3x2 − 12 fy = 3y2 − 3

r = f xx = 6x s = f xy = 0 t = fyy = 6y
ww

For stationary points, solve f x = 0 and fy = 0.


f x = 0 ⇒ 3x2 − 12 = 0 ⇒ x2 − 4 = 0 ⇒ x2 = 4 ⇒ x = ±2.
fy = 0 ⇒ 3y2 − 3 = 0 ⇒ y2 = 1 ⇒ y = ±1.

Visit For More : www.LearnEngineering.in


Visit For More : www.LearnEngineering.in

Function of Several Variables 125

Stationary points are (2, 1), (2, −1), (−2, 1) and (−2, −1).
rt − s2 = 6x6y − 0 = 36xy.
At (2, 1), rt − s2 = 36 × 2 × 1 = 72 > 0.
At (2, 1), r = 6(2) = 12 > 0.
∴ (2, 1) is a minimum point.

.in
Minimum value is f (2, 1) = 8 + 1 − 24 − 3 + 20 = 29 − 27 = 2.
At (2, −1), rt − s2 = 36 × 2 × −1 = −72 < 0.
∴ (2, −1) is a saddle point.

ng
At (−2, 1), rt − s2 = 36 × (−2) × 1 = −72 < 0.
∴ (−2, 1) is a saddle point.

eri
At (−2, −1), rt − s2 = 36 × −2 × −1 = 72 > 0.
r = 6(−2) = −12 < 0.
∴ (−2, −1) is a maximum point.
e
gin
Maximum value f (−2, −1) = −8 − 1 + 24 + 3 + 20 = 47 − 9 = 38.

Example 2.73. Examine f (x, y) = x3 + y3 − 3axy for maximum and minimum


En

values. [Jan 1999]


Solution.
Given f (x, y) = x3 + y3 − 3axy.
arn

f x = 3x2 − 3ay

fy = 3y2 − 3ax
Le

r = f xx = 6x

s = f xy = −3a t = fyy = 6y.


w.

For stationary points, solve f x = 0 and fy = 0.


x2
f x = 0 ⇒ 3x2 − 3ay = 0 ⇒ ay = x2 ⇒ y = .
a
ww

2 x4
fy = 0 ⇒ 3y − 3ax = 0 ⇒ 2 − ax = 0.
a
 x3 
3 3
x 2 − a = 0 ⇒ x(x − a ) = 0 ⇒ x = 0 or x = a.
a
x=0⇒y=0

Visit For More : www.LearnEngineering.in


Visit For More : www.LearnEngineering.in

126 Engineering Mathematics - I

a2
x=a⇒y= a = a.
Stationary points are (0, 0) and (a, a).
At (0, 0), rt − s2 = 6x6y − 9a2 = 36xy − 9a2 = −9a2 < 0.
r = 6x = 0.
∴ No maximum or minimum at (0, 0).

.in
∴ (0, 0) is a saddle point.
At (a, a), rt − s2 = 36a2 − 9a2 = 27a2 > 0 if a , 0.
r = 6x = 6a.

ng
If a < 0, r < 0.
∴ (a, a) is a maximum point if a < 0.

eri
If a > 0, r > 0.
∴ (a, a) is a minimum point if a > 0.
Maximum value = a3 + a3 − 3a3 = −a3 if a < 0
e
gin
Minimum value = −a3 if a > 0.

Example 2.74. Discuss the maxima and minima of f (x, y) = x3 y2 (1 − x − y).


En

[Jan 2014]
Solution. Given: f (x, y) = x3 y2 (1 − x − y) = x3 y2 − x4 y2 − x3 y3 .
arn

f x = y2 [x3 (−1) + (1 − x − y)3x2 ] = x2 y2 [−x + 3 − 3x − 3y]

= x2 y2 [−4x − 3y + 3].
Le

fy = 2x3 y − 2x4 y − 3x3 y2 .

r = f xx = y2 [−12x2 − 6xy + 6x].


w.

s = f xy = 6x2 y − 8x3 y − 9x2 y2 .

t = fyy = 2x3 − 2x4 − 6x3 y.


ww

For stationary points, solve f x = 0 and fy = 0.


f x = 0 ⇒ x2 y2 [−4x − 3y + 3] = 0.
⇒ x = 0, y = 0, 4x + 3y = 3.

Visit For More : www.LearnEngineering.in


Visit For More : www.LearnEngineering.in

Function of Several Variables 127

fy = 0 ⇒ x3 y(2 − 2x − 3y) = 0.
x = 0, y = 0, 2x + 3y = 2.

Solving 4x + 3y = 3 (1)

2x + 3y = 2 (2)

.in
1
we get 2x = 1 ⇒ x = .
2
1 1
When x = , (1) ⇒ 2 + 3y = 3 ⇒ 3y = 1 ⇒ y = .
2 3

ng
1 1
∴ The stationary points are (0, 0), , .
2 3
At (0, 0), rt − s2 = 0.0 − 0 = 0.

eri
We can not say maximum or minimum. Further investigation is required.
1 1
At , ,
2 3
1 1 1 1
e 1
gin
r= − 12 × − 6 × × + 6 ×
9 4 2 3 2
1  1 1
= − 3 − 1 + 3 = (−1) − .
9 9 9
1 1 11 1 1 1 1
En

t =2 −2 −6 = − − =− .
8 16 83 4 8 4 8
 11 11 1 1 2  1 1 1 2
2
s = 6 −8 −9 = − −
43 83 49 2 3 4
arn

 1 1 2  1 2 1
= − = − = .
4 3 12 144
 1  1  1
rt − s2 = − − −
9 8 144
Le

1 1 2−1
= − = > 0 and r < 0.
72 144 144
1 1
∴ , is a maximum point.
w.

2 3
1 1 1 1 1 1 1 6 − 3 − 2 1 1
Maximum value is f , = 1− − = = = .
2 3 89 2 3 72 6 72 × 6 432
ww

Example 2.75. Find the extreme values of the function f (x, y) = x4 + y2 + x2 y.


Solution. Given: f (x, y) = x4 + y2 + x2 y.

f x = 4x3 + 2xy. r = f xx = 12x2 + 2y.

Visit For More : www.LearnEngineering.in


Visit For More : www.LearnEngineering.in

128 Engineering Mathematics - I

fy = 2y + x2 . s = f xy = 2x. t = fyy = 2.

For stationary points, solve f x = 0 and fy = 0.


f x = 0 ⇒ 4x3 + 2xy = 0 ⇒ 2x(y + 2x2 ) = 0 ⇒ x = 0, y + 2x2 = 0 ⇒ y = −2x2 .
fy = 0 ⇒ 2y + x2 = 0 ⇒ −4x2 + x2 = 0 ⇒ −3x2 = 0 ⇒ x = 0.
When x = 0, y = 0.

.in
∴ The only stationary point is (0, 0).

ng
rt − s2 = (12x2 + 2y)2 − 4x2 .

eri
At (0, 0), rt − s2 = 0.
We can not say maximum or minimum.

e
We shall investigate the nature of the function in a neighbourhood of (0, 0).
gin
We have f (0, 0) = 0. In a neighbourhood of (0, 0) on the x−axis, take the point (h, 0),
f (h, 0) = h4 > 0.
On the y−axis take the point (0, k), f (0, k) = k2 > 0.
En

On y = mx, for any m, take the point (h, mh).


arn

f (h, mh) = h4 + m2 h2 + mh3 = h2 [h2 + m2 + mh].

For the quadratic in m, m2 + mh + h2


Le

discriminant = B2 − 4AC = h2 − 4.1.h2 = −3h2 < 0 if m , 0.


∴ f (h, mh) > 0 for all m , 0.
w.

∴ In a neigbourhood of (0, 0) for all points (x, y), f (x, y) > 0.


∴ f (0, 0) is minimum and the minimum value = 0.
ww

Example 2.76. Find the maximum and minimum values of x2 − xy + y2 − 2x + y.


[ Jun 2012, Jun 2010]
Solution. f (x, y) = x2 − xy + y2 − 2x + y.

Visit For More : www.LearnEngineering.in


Visit For More : www.LearnEngineering.in

Function of Several Variables 129

f x = 2x − y − 2.

fy = −x + 2y + 1.

r = f xx = 2.

s = f xy = −1.

t = fyy = 2.

.in
For stationary points, solve f x = 0, fy = 0

(1)

ng
2x − y − 2 = 0.

−x + 2y + 1 = 0. (2)

eri
(1) ⇒ y = 2x − 2.

e
(2) ⇒ −x + 2(2x − 2) + 1 = 0
gin
−x + 4x − 4 + 1 = 0

3x − 3 = 0
En

3x = 3

x = 1.
arn

∴ y = 2 − 2 = 0.

The stationary point is (1, 0).


Le

rt − s2 = 4 + 1 = 5 > 0 and r = 2 > 0.


w.

∴ (1, 0) is a minimum point.


Minimum value of f = 1 − 2 = −1.
ww

Example 2.77. Find the extreme values of the function f (x, y) = x3 +y3 −3x−12y+20.
[Jan 2012]
Solution. f (x, y) = x3 + y3 − 3x − 12y + 20.

Visit For More : www.LearnEngineering.in


Visit For More : www.LearnEngineering.in

130 Engineering Mathematics - I

f x = 3x2 − 3.

fy = 3y2 − 12.

r = f xx = 6x.

s = f xy = 0.

t = fyy = 6y.

.in
For stationary points, solve f x = 0, fy = 0.

3x2 − 3 = 0. 3y2 − 12 = 0.

ng
3x2 = 3. 3y2 = 12.

eri
x2 = 1. y2 = 4.

x = ±1. e y = ±2.
gin
The stationary points are (1, 2), (−1, 2), (1, −2), (−1, −2).
At (1, 2), rt − s2 = 36xy = 36 × 1 × 2 = 72 > 0.
En

r = 6 > 0.
∴ (1, 2) is a minimum point.
arn

Minimum value of f = 1 + 8 − 3 − 24 + 20 = 2.
At (−1, 2), rt − s2 = 36xy = 36 × (−1) × 2 = −72 < 0.
∴ (−1, 2) is a saddle point.
Le

At (1, −2), rt − s2 = 36xy = 36 × 1 × (−2) = −72 < 0.


∴ (1, −2) is a saddle point.
w.

At (−1, −2), rt − s2 = 36xy = 72 > 0.


r = 6 × (−1) = −6 < 0.
ww

∴ (−1, −1) is a maximum point.


Maximum value of f = −1 − 8 + 3 + 24 + 20 = 38.
Maxima = 38.
Minima = 2.

Visit For More : www.LearnEngineering.in


Visit For More : www.LearnEngineering.in

Function of Several Variables 131

Example 2.78. Test for maxima and minima of the function f (x, y) = x3 y2 (6 − x − y).
[Jan 2013]
Solution. f (x, y) = x3 y2 (6 − x − y)
= 6x3 y2 − x4 y2 − x3 y3 .
f x = 18x2 y2 − 4x3 y2 − 3x2 y3 .

fy = 12x3 y − 2x4 y − 3x3 y2 .

.in
r = f xx = 36xy2 − 12x2 y2 − 6xy3 .

ng
s = f xy = 36x2 y − 8x3 y − 9x2 y2 .

t = fyy = 12x3 − 2x4 − 6x3 y.


For stationary points, f x = 0, fy = 0.

eri
f x = 0 ⇒ 18x2 y2 − 4x3 y2 − 3x2 y3 = 0

e
gin
x2 y2 (18 − 4x − 3y) = 0

i.e., 4x + 3y = 18. (1)


En

fy = 0 ⇒ 12x3 y − 2x4 y − 3x3 y2 = 0


arn

x3 y(12 − 2x − 3y) = 0

2x + 3y = 12. (2)
Le

(1) − (2) ⇒ 2x = 6
w.

x = 3.
(1) ⇒ 12 + 3y = 18
ww

3y = 6
y = 2.
The stationary point is (3, 2)
At (3, 2),

Visit For More : www.LearnEngineering.in


Visit For More : www.LearnEngineering.in

132 Engineering Mathematics - I

r = 36 × 3 × 4 − 12 × 9 × 4 − 6 × 3 × 8

= 432 − 432 − 144

= −144 < 0.

t = 12 × 9 − 2 × 81 − 6 × 27 × 2

= 108 − 162 − 324

.in
= −378.

s = 34 × 9 × 2 − 8 × 27 × 2 − 9 × 9 × 4

ng
= 612 − 432 − 324

eri
= −144.

rt − s2 = (−144)(−378) − (−144)2

= 54432 − 20736
e
gin
= 33696 > 0
Since rt − s2 > 0 and r < 0, (3, 2) is a maximum point.
∴ Maximum value of f = 27 × 4(6 − 3 − 2) = 108.
En

Example 2.79. Examine for minimum and maximum values sin x + sin y + sin(x + y).
Solution. We have f (x, y) = sin x + sin y + sin(x + y).
arn

f x = cos x + cos(x + y) fy = cos y + cos(x + y).

r = f xx = − sin x − sin(x + y)
Le

s = f xy = − sin(x + y)

t = fyy = − sin y − sin(x + y).


w.

For stationary points, solve f x = 0 and fy = 0.


ww

i.e., cos x + cos(x + y) = 0. (1)

cos y + cos(x + y) = 0. (2)

Visit For More : www.LearnEngineering.in


Visit For More : www.LearnEngineering.in

Function of Several Variables 133

(1) − (2) =⇒ cos x − cos y = 0.

i.e., cos x = cos y

=⇒x = y

Now (1) =⇒ cos x + cos 2x = 0

i.e., cos 2x = − cos x.

.in
cos 2x = cos(π − x)

=⇒2x = π − x.

ng
i.e., 3x = π
π

eri
x= .
3
π π
When x = , y = .
π π 3 3
∴ , is a stationary point.
e
gin
3 3 
π π
At ,
3 3
√ √
π 2π − 3 3 √
r = − sin − sin = − = − 3 < 0.
En

3 √3 2 √ 2√
2π 3 3 3 √
s = − sin =− ,t = − − = − 3.
3 2 2 2
arn

2 3 9
rt − s = 3 − = > 0.
4 4
π π
Since rt − s2 > 0 and r < 0, f (x, y) has a maximum value at , .
√ 3√ 3 √ √
Le

π π π π 2π 3 3 3 3 3
∴ Maximum value = f , = sin + sin + sin = + + = .
3 3 3 3 3 2 2 2 2
Example 2.80. Find the maximum and minimum values of
w.

sin x sin y sin(x + y), 0 < x, y < π. [Jan 1997]


Solution. Given f (x, y) = sin x sin y sin(x + y).
ww

f x = sin y[sin x cos(x + y) + sin(x + y) cos x] = sin y sin(2x + y).

r = f xx = 2 sin y cos(2x + y).

Visit For More : www.LearnEngineering.in


Visit For More : www.LearnEngineering.in

134 Engineering Mathematics - I

fy = sin x[sin y cos(x + y) + sin(x + y) cos y] = sin x sin(x + 2y).

s = f xy = sin x cos(x + 2y) + sin(x + 2y) cos x = sin(2x + 2y).

t = fyy = 2 sin x cos(x + 2y).

For stationary points, solve f x = 0 and fy = 0.

.in
f x = 0 ⇒ sin y sin(2x + y) = 0.
fy = 0 ⇒ sin x sin(x + 2y) = 0.
Since, x, y , 0& , π ⇒ sin x , 0, sin y , 0.

ng
∴ sin(2x + y) = 0 and sin(x + 2y) = 0.
Since, 0 < x < π, 0 < 2x < 2π

eri
0 < y < π ⇒ 0 < 2x + y < 3π.
Similarly 0 < x + 2y < 3π. Since, sin(2x + y) = 0 ⇒ 2x + y = π or 2π.
Similarly x + 2y = π or 2π.
e
gin
If 2x + y = π (1)

and x + 2y = π (2)
En

then x − y = 0 ⇒ x = y.
π
∴ (1) ⇒ 3x = π ⇒ x = .
arn

3
π
∴y= .
3 π π
∴ one stationary point is , .
3 3
Le

If 2x + y = π and x + 2y = 2π then, x − y = −π ⇒ x = y − π.
∴ 2(y − π) + y = π ⇒ 3y − 2π = π ⇒ 3y = 3π ⇒ y = π,
which is not admissible since y , π.
w.

Similarly, 2x + y = 2π and x + 2y = π is also not possible.


Now take 2x + y = 2π and x + 2y = 2π
ww

⇒ x−y=0⇒ x=y

⇒ 3x = 2π ⇒ x =
3

∴y= .
3

Visit For More : www.LearnEngineering.in


Visit For More : www.LearnEngineering.in

Function of Several Variables 135

 2π 2π 
∴ Another stationary point is , .
π π 3 3
At , .
3 3 √
π 3(−1)
r = 2 sin cos π = 2 < 0.
3 2

4π  π π 3
s = sin = sin π + = − sin = − .
3 3 3 2

.in

π 2 3(−1) √
t = 2 sin cos π = = − 3.
3 2

ng
√ √  − 3 2 3 9
rt − s2 = (− 3)(− 3) − = 3 − = > 0.
2 4 4
π π

eri
∴ , is a maximum point.
3 3 √ √
π π π π 2π 3 3  π
Maximum value = f , = sin . sin sin = sin π −
√3 √ 3 √ 3 √ 3 3 2 2 3
=
3 3 3 3 3
2 2 2
=
8
.
e
gin
 2π 2π 
At ,
3 3 √
2π 6π 3 √
r = 2 sin cos =2 .1 = 3 > 0.
3 3 2
En

2π 6π √
t = 2 sin cos = 3.
3 3 √
8π  π 3
s = sin = sin 3π − = .
arn

3 √ 3 2
√ √  3 2 3 9
rt − s2 = 3 3 − = 3 − = > 0.
2 4 4
 2π 2π 
∴ , is a minimum point.
3 3
Le

√ √ √ √
 2π 2π  2π 2π 4π 3 3 3  −3 3
Minimum value = f , = sin sin sin = − = .
3 3 3 3 3 2 2 2 8
w.

Example 2.81. In a plane triangle, find the maximum value of cos A cos B cos C.
[Jan 2000]
Solution. The angles of the ∆le ABC satisfy 0 < A, B, C < π and A + B + C = π,
ww

=⇒ C = π − (A + B). Replacing C, we get


f (A, B) = cos A cos B cos(π − (A + B)) = − cos A cos B cos(A + B), 0 < A, B < π.
fA = − cos B[cos A(− sin(A + B)) + cos(A + B)(− sin A)] = cos B sin(2A + B).

Visit For More : www.LearnEngineering.in


Visit For More : www.LearnEngineering.in

136 Engineering Mathematics - I

fB = − cos A[− sin(A + 2B)] = cos A sin(A + 2B).


r = fAA = 2 cos B cos(2A + B).
s = fAB = cos A cos(A + 2B) + sin(A + 2B)(− sin A) = cos(2A + 2B).
t = fBB = 2 cos A cos(A + 2B).
For stationary points, solve fA = 0, fB = 0.

.in
i.e., cos B sin(2A + B) = 0 and cos A sin(A + 2B) = 0.

ng
cos B = 0 or sin(2A + B) = 0 and cos A = 0 or sin(A + 2B) = 0
π
=⇒ B = or 2A + B = π or 2π
2

eri
and
π
A= or A + 2B = π or 2π.
2
e
gin
Different possibilities
π π
case (i) Let B = and A = .
2 2
⇒ A+B=π
En

=⇒ C = 0 not possible.
π
case (ii) If B = and A + 2B = π.
2
arn

⇒ A + π = π ⇒ A = 0 not possible.
π
case (iii) If B = and A + 2B = 2π.
2
⇒ A + π = 2π ⇒ A = π not possible.
π
Le

case (iv) A = , 2A + B = π.
2
⇒ π + B = π ⇒ B = 0 not possible.
π
w.

case (v) A = , 2A + B = 2π ⇒ B = π not possible.


2
case (vi) If 2A + B = π, A + 2B = π.
Subtracting A − B = 0 ⇒ A = B.
ww

π
∴ 3A = π ⇒ A = .
3
π π
=⇒ B = , C = .
3 3
case (vii) If 2A + B = π and A + 2B = 2π ⇒ A − B = −π not possible.

Visit For More : www.LearnEngineering.in


Visit For More : www.LearnEngineering.in

Function of Several Variables 137

Finally 2A + B = 2π and A + 2B = 2π ⇒ A − B = 0 ⇒ A = B.
2π 2π
3A = 2π ⇒ A = ,B = .
3 3
2π 2π π
A+B= + = 4 > π not possible.
3 3 3 π π
∴ The only stationary point is , .
π π 3 3
At , ,
3 3

.in
π 1
r = 2 cos cos π = 2 (−1) < 0.
3 2
π
t = 2 cos cos π = −1.

ng
3
 4π   π 1
s = cos = cos π + =− .
3 3 2
2
 −1 2 1 3
rt − s = (−1)(−1) − = 1 − = > 0.

eri
π π 2 4 4
∴ , is a maximum point.
3 3
π π π π π 1
∴ Maximum value of f is f , = cos cos cos = .
3 3 3
e
3 3 8
gin
Example 2.82. A flat circular plate is heated so that the temperature at any
point (x, y) is U(x, y) = x2 + 2y2 − x. Find the coldest point on the plate. [Jan 2005]
Solution. Given U = x2 + 2y2 − x.
En

U x = 2x − 1 Uy = 4y.
arn

r = U xx = 2. s = U xy = 0. t = Uyy = 4.
Le

rt − s2 = 8 > 0 and r = 2 > 0.


∴ All points are minimum points.
w.

At minimum, U x = 0 and Uy = 0.
1
U x = 0 ⇒ 2x − 1 = 0 ⇒ x = .
2
ww

Uy = 0 ⇒ 4y = 0 ⇒ y = 0.
1 
∴ The minimum point is , 0 .
2
1 
∴ The coldest point on the plate is , 0 .
2

Visit For More : www.LearnEngineering.in


Visit For More : www.LearnEngineering.in

138 Engineering Mathematics - I

2.8 Constrained Maxima and Minima - Lagrange’s Method

Lagrange’s Method
Let f (x, y, z) be the function for which the extreme values are to be found subject to
the condition
φ(x, y, z) = 0. (1)

.in
Construct the auxiliary function F(x, y, z) = f (x, y, z) + λφ(x, y, z), where λ is an
undetermined parameter independent of x, y, z which is called the Lagrange’s

ng
multiplier. Any relative extremum of f (x, y, z) subject to (1) must occur at a
stationary point of F(x, y, z).

eri
∂F ∂F ∂F ∂F
The stationary points of F are given by = 0, = 0, = 0, = 0.
∂x ∂y ∂z ∂λ
⇒ f x + λφ x = 0, fy + λφy = 0, fz + λφz = 0, φ(x, y, z) = 0.
fx fy fz
=
φ x φy φz
= = −λ and φ(x, y, z) = 0.
e
gin
Solving these equations we can find the values of x, y, z which are stationary
points of F and the values of f at these points give the maximum and minimum
values of f (x, y, z).
✎ ☞
En

Worked Examples
✍ ✌
Example 2.83. Find the maximum value of xm yn z p subject to x + y + z = a.
arn

[Jan 2009]
Solution. Let f = xm ym z p .
φ = x + y + z − a = 0. (1)
Le

We have to maximise f subject to (1).


Let F = f + λφ where λ is the Lagrange’s multiplier.
w.

F = xm yn z p + λ(x + y + z − a).
ww

F x = mxm−1 yn z p + λ, Fy = nxm yn−1 z p + λ, Fz = pxm yn z p−1 + λ.

To find the stationary points, solve F x = 0, Fy = 0, Fz = 0, φ = 0.

F x = 0 ⇒ mxm−1 yn z p = −λ.

Visit For More : www.LearnEngineering.in


Visit For More : www.LearnEngineering.in

Function of Several Variables 139

Fy = 0 ⇒ nxm yn−1 z p = −λ.

F x = 0 ⇒ pxm yn z p−1 = −λ.

From the above three equations we get

mxm−1 yn z p = nxm yn−1 z p = pxm yn z p−1 .

.in
m n p m+n+ p m+n+ p
Dividing by xm yn z p we get, = = = = .
x y z x+y+z a
ma na pa

ng
x= ,y = ,z = .
m+n+ p m+n+ p m+n+ p
!
ma na pa
The stationary point is , , .

eri
m+n+ p m+n+ p m+n+ p
am+n+p mm nn p p
∴ Max. value of f = .
(m + n + p)m+n+p

e
Example 2.84. Find the minimum value of x2 yz3 subject to 2x + y + 3z = a.
gin
[Jan 2007]
Solution. Given f = x2 yz3 .
En

φ = 2x + y + 3z − a = 0. (1)

Let F = f + λφ where λ is the Lagrange’s multiplier.


arn

F = x2 yz3 + λ(2x + y + 3z − a).

F x = 2xyz3 + 2λ, Fy = x2 z3 + λ, Fz = 3x2 yz2 + 3λ.


Le

To find the stationary points, solve F x = 0, Fy = 0, Fz = 0, φ = 0.

F x = 0 ⇒ 2xyz3 + 2λ = 0 ⇒ xyz3 = −λ.


w.

Fy = 0 ⇒ x2 z3 = −λ.
ww

Fz = 0 ⇒ 3x2 yz2 + 3λ = 0 ⇒ x2 yz2 = −λ.

Therefore
xyz3 = x2 z3 = x2 yz2

Visit For More : www.LearnEngineering.in


Visit For More : www.LearnEngineering.in

140 Engineering Mathematics - I

xyz3 = x2 z3 ⇒ y = x.

x2 z3 = x2 yz2 ⇒ y = z.

x = y = z.
a
(1) ⇒ 2x + x + 3x = a ⇒ 6x = a ⇒ x = = y = z.
6

.in
a a a
∴ The stationary point is , , .
6 6 6
 a 2 a  a 3 a6  a 6
Minimum value = = 6 = .

ng
6 6 6 6 6
Example 2.85. If u = x2 + y2 + z2 where ax + by + cz − p = 0, find the stationary value

eri
of u. [Jan 2006]
Solution. Given f = x2 + y2 + z2 .

e
φ = ax + by + cz − p = 0. (1)
gin
Let F = f + λφ where λ is the Lagrange’s multiplier.

F = x2 + y2 + z2 + λ(ax + by + cz − p).
En

F x = 2x + aλ, Fy = 2y + bλ, Fz = 2z + cλ.


arn

To find the stationary points, solve F x = 0, Fy = 0, Fz = 0, φ = 0.

−aλ −a2 λ
2x + λa = 0 ⇒ x = ⇒ ax = .
2 2
Le

Similarly
−b2 λ −c2 λ
by = , cz = .
w.

2 2
−a2 λ b2 λ c2 λ
(1) ⇒ − − =p
2 2 2
ww

a2 + b2 + c2 −2p
λ = −p ⇒ λ = 2 .
2 a + b2 + c2
ap bp cp
∴x= 2 2 2
,y = 2 2 2
,z = 2 .
a +b +c a +b +c a + b2 + c2

Visit For More : www.LearnEngineering.in


Visit For More : www.LearnEngineering.in

Function of Several Variables 141

Stationary value of u is

a2 p2 b2 p2 c2 p2
u= + +
(a2 + b2 + c2 )2 (a2 + b2 + c2 )2 (a2 + b2 + c2 )2

p2 (a2 + b2 + c2 ) p2
= = .
(a2 + b2 + c2 )2 a2 + b2 + c2

.in
Example 2.86. The temperature T at any point (x, y, z) in space is T = 400xyz2 .
Find the highest temperature on the surface of the unit sphere x2 + y2 + z2 = 1.

ng
[Jan 2005]
Solution. We have to maximize

eri
T = 400xyz2 (1)

subject to φ = x2 + y2 + z2 − 1 = 0. (2)
e
gin
Consider F = T + λφ, where λ is the Lagrange multiplier.
F = 400xyz2 + λ(x2 + y2 + z2 − 1).

F x = 400yz2 + 2λx, Fy = 400xz2 + 2λy, Fz = 400xyz + 2λz.


En

To find the stationary points, solve F x = 0, Fy = 0, Fz = 0, φ = 0.


200yz2
arn

F x = 0 ⇒ 400yz2 + 2λx = 0 ⇒ −λ = .
x
200xz 2
Fy = 0 ⇒ 400xz2 + 2λy ⇒ −λ = .
y
Le

Fz = 0 ⇒ 800xyz + 2λz = 0 ⇒ −λ = 400xy.


200yz2 200xz2
∴ = = 400xy.
x y
200yz2 200xz2
w.

Taking = we get x2 = y2 ⇒ y = ±x.


x y
200yz2 z2 √
Taking = 400xy we get = 2x ⇒ z2 = 2x2 ⇒ z = ± 2x.
x x
ww

200xz2 z2 √
Taking = 400xy we get = 2y ⇒ z2 = 2y2 ⇒ z = ± 2y.
y y
Substituting in x2 + y2 + z2 = 1 we get
1 1
x2 + x2 + 2x2 = 1 ⇒ 4x2 = 1 ⇒ x2 = ⇒ x = ±
4 2

Visit For More : www.LearnEngineering.in


Visit For More : www.LearnEngineering.in

142 Engineering Mathematics - I

1 √ 1 1
∴ y = ± ,z = ± 2 = ± √ .
2 2 2
1 1 1
The stationary points are given by x = ± , y = ± , z = ± √ .
2 2 2
To have maximum value, we must have xy positive.
 1 1 1   1 1 −1   −1 −1 1   −1 −1 −1 
∴ The points are , , √ , , , √ , , , √ , , , √ .
2 2 2 2 2 2 2 2 2 2 2 2
111 0

.in
∴ Maximum T = 400 = 50 C.
222
Example 2.87. Find the shortest and longest distance from the point (1, 2, −1) to

ng
the sphere x2 + y2 + z2 = 24 using Lagrange’s method of constrained maxima and
minima. [Jun 2011, Jan 2002]

eri
Solution. Let P(x, y, z) be any point on the sphere.
Let A be the point (1, 2, −1).

AP =
q

e
(x − 1)2 + (y − 2)2 + (z + 1)2 .
gin
Let f (x, y, z) = (x − 1)2 + (y − 2)2 + (z + 1)2 . (1)

AP is minimum or maximum if f is minimum or maximum.


En

∴ The problem is now reduced to minimise or maximise f subject to


φ(x, y, z) = x2 + y2 + z2 − 24 = 0.
arn

Consider the auxiliary function


F = f + λφ = (x − 1)2 + (y − 2)2 + (z + 1)2 + λ(x2 + y2 + z2 − 24) where λ is the
Lagrange’s multiplier.
Le

F x = 2(x − 1) + 2λx, Fy = 2(y − 2) + 2λy, Fz = 2(z + 1) + 2λz.


To find the stationary points solve
w.

F x = 0, Fy = 0, Fz = 0, φ = 0.
x−1 1
F x = 0 ⇒ 2(x − 1) + 2λx = 0 ⇒ x − 1 = −λx ⇒ −λ = =1− .
x x
y−2 2
ww

Fy = 0 ⇒ 2(y − 2) + 2λy = 0 ⇒ y − 2 = −λy ⇒ −λ = =1− .


y y
z+1 1
Fz = 0 ⇒ 2(z + 1) + 2λz = 0 ⇒ z + 1 = −λz ⇒ −λ = =1+ .
z z
1 2 1
∴1− =1− =1+ .
x y z

Visit For More : www.LearnEngineering.in


Visit For More : www.LearnEngineering.in

Function of Several Variables 143

1 2 1 2
Taking 1 − = 1 − we get = ⇒ y = 2x.
x y x y
1 1
Taking 1 − = 1 + we get z = −x.
x z
2 1 −y
Taking 1 − = 1 + we get z = = −x.
y z 2
We have x2 + y2 + z2 = 24 ⇒ x2 + 4x2 + x2 = 24 ⇒ 6x2 = 24 ⇒ x2 = 4 ⇒ x = ±2.
When x = 2, y = 4, z = −2, the first point is (2, 4, −2). Let this point be P1 .

.in
When x = −2, y = −4, z = 2, the second point is (−2, −4, 2). Let this point be P2 .
√ √ √ √ √
P1 A = 1 + 4 + 1 = 6, P2 A = 9 + 36 + 9 = 54 = 3 6

ng

∴ Shortest distance = 6

Longest distance = 3 6.

eri
Example 2.88. A rectangular box open at the top is to have a volume of 32cc.
Find the dimensions of the box which requires least amount of material for its
construction. e
[Jun 2012, Dec 2011, Jun 2010, Jan 2005]
gin
Solution. Let the dimensions of the box be Length = x, Breadth = y, height = z.
Given: Volume = 32cc.
(1)
En

=⇒ xyz = 32, x, y, z > 0.

We have to minimize the amount of material used for the construction of the box.
Let S be the surface area of the box whose top is open
arn

∴ S = xy + 2xz + 2yz (2)


Le

By Lagrange’s method
Let F = s + λφ = xy + 2yz + 2xz + λ(xyz − 32) where λ is the Lagrange’s multiplier.
F x = y + 2z + λyz, Fy = x + 2z + λxz, Fz = 2y + 2x + λxy.
w.

To find the stationary points, solve


ww

F x = 0, Fy = 0, Fz = 0, φ = 0

F x = 0 ⇒ y + 2z + λyz = 0

=⇒ y + 2z = −λyz

Visit For More : www.LearnEngineering.in


Visit For More : www.LearnEngineering.in

144 Engineering Mathematics - I

y 2z
⇒ + = −λ
yz yz
1 2
⇒ + = −λ (3)
z y
Fy = 0 ⇒ x + 2z + λxz = 0
=⇒ x + 2z = −λxz

.in
i.e., xy + 2yz = −λxyz.
xy 2yz
⇒ + = −λ

ng
xyz xyz
1 2
⇒ + = −λ (4)
z x

eri
Fz = 0 ⇒ 2y + 2x + λxy = 0
=⇒ 2x + 2y = −λxy

e
2xz + 2yz = −λxyz.
gin
2xz 2yz
⇒ + = −λ
xyz xyz
2 2
⇒ + = −λ (5)
En

y x
(3) − (4) ⇒
2 2
− =0
arn

y x
1 1
=
y x
⇒x=y (6)
Le

(3) − (5) ⇒
1 2
− =0
w.

z x
1 2
=
z x
ww

⇒ x = 2z (7)

From (6)and(7) we obtain x = y = 2z.


(1)⇒ 2z.2z.z = 32 ⇒ 4z3 = 32 ⇒ z3 = 8 ⇒ z = 2.

Visit For More : www.LearnEngineering.in


Visit For More : www.LearnEngineering.in

Function of Several Variables 145

∴ x = 4, y = 4.
The stationary point is (4, 4, 2).
The dimensions are 4cm, 4cm, 2cm.

Example 2.89. Find the dimensions of the rectangular box open at the top of
maximum capacity whose surface area is 432 sq.m. [Jun 2013]

.in
Solution. Let the dimensions of the box be x, y, z.
Given, surface area= 432.

ng
xy + 2xz + 2yz = 432 (1)

eri
Let V be the volume of the box.
We have to maximize V.

e
V = xyz (2)
gin
By lagrange’s method
F = V + λφ, where λ is the lagrange multiplier.

F = xyz + λ(xy + 2xz + 2yz − 432).


En

F x = yz + λ(y + 2z).

Fy = xz + λ(x + 2z).
arn

Fz = xy + λ(2x + 2y).

Fλ = xy + 2xz + 2yz − 432.


Le

For stationary points, F x = 0, Fy = 0, Fz = 0, Fλ = 0.

F x = 0 ⇒ yz + λ(y + 2z) = 0
w.

⇒ xyz + λ(xy + 2xz) = 0. (1)


ww

Fy = 0 ⇒ xz + λ(x + 2z) = 0

⇒ xyz + λ(xy + 2yz) = 0. (2)

Fz = 0 ⇒ xy + λ(2x + 2y) = 0

Visit For More : www.LearnEngineering.in


Visit For More : www.LearnEngineering.in

146 Engineering Mathematics - I

xyz + λ(2xz + 2yz) = 0. (3)

(1) + (2) + (3) ⇒


3xyz + λ(2xy + 4xz + 4yz) = 0

3xy + 2λ(xy + 2xz + 2yz) = 0

3xyz + 2λ × 432 = 0

.in
3xyz = −864λ
3xyz xyz

ng
λ=− =−
864 288
Substituting the value of λ in F x = 0 we get

eri
xyz
yz − (y + 2z) = 0
288

x
1−
288 e
(y + 2z) = 0
gin
x
1= (y + 2z)
288
En

xy + 2xz = 288 (4)

xyz
Fy = 0 ⇒ xz − (x + 2z) = 0
288
arn

y
1− (x + 2z) = 0
288
Le

y y
1− (x + 2z) = 0, 1 = (x + 2z)
288 288

xy + 2yz = 288 (5)


w.

xyz
Fz = 0 ⇒ xy − (2x + 2y) = 0
288
ww

z z
1− (2x + 2y) = 0, 1 = (2x + 2y)
288 288

2xz + 2yz = 288 (6)

Visit For More : www.LearnEngineering.in


Visit For More : www.LearnEngineering.in

Function of Several Variables 147

(4) − (5) ⇒ 2z(x − y) = 0, z , 0

∴ x−y=0

⇒ x = y.

(5) − (6) ⇒ xy − 2xz = 0

x(y − 2z) = 0, x , 0

.in
∴ y − 2z = 0

y = 2z.

ng
∴ x = y = 2z

eri
(4) ⇒ 2z × 2z + 2 · 2z · z = 288

4z2 + 4z2 = 288

e
z2 = 36.
gin
z = ±6.

z = −6 is not possible

∴ z = 6.
En

∴ y = 2z = 12

x = 2z = 12
arn

The dimensions of the box to have maximum capacity are


Le

Length = 12m

Breadth = 12m
w.

Height = 6m.
ww

Visit For More : www.LearnEngineering.in


Visit For More : www.LearnEngineering.in

.in
ng
eri
e
gin
En
arn
Le
w.
ww

Visit For More : www.LearnEngineering.in


Visit For More : www.LearnEngineering.in

3 Integral Calculus

.in
ng
3.1 The definite integral

Integration as the limit of a sum

y = f (x)
e eri
gin
n rectangles
f (xk )
·········
En

∆x
x1 = a x2 x3 xn+1 = b
arn

Consider the graph of the positive function y = f (x). Let us find the area
under the curve y = f (x),between the x−axis and the ordinates x = a and x = b.
Le

b−a
Divide this area into n rectangles of equal width ∆x = . Let the x− co ordinates
n
at the left hand side of the rectangles be x1 = a, x2 , x3 , . . . xn+1 = b. Consider a typical
w.

rectangle, the kth one with height f (xk ). The area of this rectangle is f (xk )∆x. The
sum of all the areas of the n rectangles is
ww

f (x1 )∆x + f (x2 )∆x + · · · + f (xn )∆x.


n
P
In summation convention form, this can be written as f (xk )∆x. This
k=1
gives an estimate of the area under the curve but it is not exact. To improve the

Visit For More : www.LearnEngineering.in


Visit For More : www.LearnEngineering.in

150 Engineering Mathematics - I

estimate we must take a large number of very thin rectangles. This can be
achieved by allowing n → ∞ and making ∆x → 0.
n
P
∴ Area under the curve = lim f (xk )∆x.
n=∞ k=1
The lower and upper limits on the sum correspond to the first and the
last rectangle where x = a and x = b respectively. Hence the above limit can be
b

.in
P
written as lim f (x)∆x. Since the number of rectangles increase without bound,
∆x→0 x=a
we drop the subsript k from xk and write f (x) which is the value of f at a typical
value of x. If this can actually be found, it is called the definite integral of f (x)

ng
Rb
from x = a and x = b and it is written as f (x)dx.
a
Rb

eri
n
P b
P
∴ f (x)dx = lim f (xk )∆x = lim f (x)∆x.
n=∞ k=1 ∆x→0 x=a
a
Note. If we approximate each strip by a rectangle that has the same base as the

e
strip and whose height is the same as the right edge of the strip; (i.e., xk is taken
gin
at the right end points of the kth rectangle and f (xk ) as the height ) then also the
above result will be achieved.
Definition of a definite integral. If f is a function defined for a ≤ x ≤ b, we
En

b−a
divide the interval [a, b] into n subintervals of equal width ∆x = . We let
n
x0 (= a), x1 , x2 , · · · xn (= b) be the end points of these subintervals and if we choose
arn

x1∗ , x2∗ , · · · xn∗ as any sample ponits in these subintervals, so that xi∗ lies in the ith
subinterval [xi−1 , xi ]. Then the definite integral of f from a to b is defined as
Rb n
f (xi∗ )∆x, provided that the limit exists and gives the same value
P
f (x)dx = lim
Le

n=∞ i=1
a
for all possible choices of sample points. If it does exist, we say that f is
integrable on [a, b].
w.

Rb n
ww

f (xi∗ )∆x can also be written as, for ∈> 0, there is an


P
Result (1). f (x)dx = lim
n=∞ i=1
a
Rb n
f (xi∗ )∆x <∈ for every integer n > N and for
P
integer N such that f (x)dx −
a i=1
every choice of xi∗ in [xi−1 , xi ].

Visit For More : www.LearnEngineering.in


Visit For More : www.LearnEngineering.in

Integral Calculus 151

n
f (xi∗ )∆x is called the Reimann sum.
P
Result (2). The sum
i=1

Theorem 1. If f is continuos on [a, b], (or) if f has only a finite number of jump
Rb
discontinuities, then f is integrable on [a, b].[i.e., f (x)dx exists].
a
Rb n
P
Theorem 2.If f is integrable on [, b], then f (x)dx = lim f (xi )∆x where ∆x =

.in
n=∞ i=1
a
b−a
and xi = a + i∆x.
n ✎ ☞

ng
Worked Examples
✍ ✌

R1

eri
Example 3.1. Using the area property evaluate x2 dx. Show that the sum of the
0
1
areas of the upper approximating rectangles approaches .
3

e
Solution. Consider the function y = f (x) = x2 . Divide the area under the curve
1−0 1
gin
y = x2 between x = 0 and x = 1 into 4 rectangles of equal width ∆x = = .
4 4
1
The width of each rectangle is .
" # " 4# " # " #
1 1 1 1 3 3
En

The subintervals are 0, , , , , and , 1 .


4 4 2 2 4 4
The height of the rectangles is approximated to the value of the
1 1 3
ordinates at x = 0, , & .
arn

4 2 4
! ! !
1 1 3
∴ Area of the required portion = f (0)∆x + f ∆x + f ∆x + f ∆x.
4 2 4
Le

!2 !2 !2
2 1 1 1 1 1 3 1
=0 . + . + . + .
4 4 4 2 4 4 4
" #
1 1 1 9
w.

= 0+ + +
4 16 4 16
" #
1 1+4+9
=
ww

4 16
1 14 7
= × = .
4 16 32

If the height of the rectangles is approximated to the value of the

Visit For More : www.LearnEngineering.in


Visit For More : www.LearnEngineering.in

152 Engineering Mathematics - I

ordinates at the right edge of each rectangle then,


! ! !
1 1 3
Area = f ∆x + f ∆x + f ∆x + f (1)∆x.
4 2 4
!2 !2 !2
1 1 1 1 3 1 1
= . + . + . + 12 ·
4 4 2 4 4 4 4
" # " #
1 1 1 9 1 1 + 4 + 9 + 16 1 30 15
= + + +1 = = × = .

.in
4 16 4 16 4 16 4 16 32
Now, let us divide the area " into # "n rectangles
# " each of their
# with width
1−0 1 1 1 2 n−1 n

ng
∆x = = . The subintervals are 0, , , , · · · , , =1 .
n n n n n n n
The heights of the rectangle are the values of the function f (x) = x2 at
1 2 3 n

eri
the points , , · · · .
n n n n
!2 !2 !2  n 2 1
1 1 2 1 3 1
∴ Area = . + . + . + ··· + .
n n n n
e n n n n
gin
1 12 22 32 n2
" #
= + + + · · · +
n n2 n2 n2 n2
1 h i
= 3 12 + 22 + 32 + · · · + n2
n "
En

#
1 n(n + 1)(2n + 1) (n + 1)(2n + 1)
= 3 =
n 6 6n2
(n + 1)(2n + 1)
lim (Area) = lim
arn

n=∞ n=∞ 6n2


n (1 + 1n )(2 + 1n ) 2 1
2
= lim = = .
n=∞ 6n2 6 3
Le

Properties of definite integrals.


The following properties can be easily proved using Riemann sums.
w.

Ra Rb
(i) f (x)dx = − f (x)dx.
b a
ww

Ra
(ii) f (x)dx = 0.
a

Rb
(iii) cdx = c(b − a),where c is any constant.
a

Visit For More : www.LearnEngineering.in


Visit For More : www.LearnEngineering.in

Integral Calculus 153

Rb Rb Rb
(iv) [ f (x) + g(x)]dx = f (x)dx + g(x)dx.
a a a

Rb Rb
(v) c f (x)dx = c f (x)dx,where c is any constant.
a a

Rb Rb Rb
(vi) [ f (x) − g(x)]dx = f (x)dx − g(x)dx.

.in
a a a

Rb Rc Rb
(vii) If a < c < b then f (x)dx = f (x)dx + f (x)dx.

ng
a a c

Comparison properties of the integral.

eri
Rb
(i) If f (x) ≥ 0 for a ≤ x ≤ b, then f (x)dx ≥ 0.
a

(ii) If f (x) ≥ g(x) for a ≤ x ≤ b, then


Rb
e
f (x)dx ≥
Rb
g(x)dx.
gin
a a

Rb
(iii) If m ≤ f (x) ≤ M for a ≤ x ≤ b, then m(b − a) ≤ f (x)dx ≤ M(b − a).
a
En

Proof. Given m ≤ f (x) ≤ M


Integrating between a and b we get
Rb Rb Rb
arn

mdx ≤ f (x)dx ≤ Mdx


a a a
Rb Rb Rb
m dx ≤ f (x)dx ≤ M dx
a a a
Le

Rb
m(b − a) ≤ f (x)dx ≤ M(b − a).
a

The first fundamental theorem of calculus


w.

Statement. If f is continuous on [a, b], then the function g defined by


Rx
ww

g(x) = f (t)dt, a ≤ x ≤ b
a

is continuous on [a, b] and is differentiable on (a, b) and g′ (x) = f (x).


Proof.Choose x and x + b ∈ (a, b).

Visit For More : www.LearnEngineering.in


Visit For More : www.LearnEngineering.in

154 Engineering Mathematics - I

y
Zx+h Zx
Now, g(x + h) − g(x) = f (t)dt − f (t)dt
a a
Zx Zx+h Zx
= f (t)dt + f (t)dt − f (t)dt m M

a x a
Zx+h

.in
= f (t)dt. 0 x u v x+h x
x

ng
g(x + h) − g(x) 1 x+h
R
Hence, for h , 0, we have = f (t)dt. (1)
h h x
Let us assume that h > 0. Since f is continuous on [x, x + h], by the

eri
extreme value theorem, there exists numbers u and v in [x, x + h] such that
f (u) = m and f (v) = M, where m and M are the absolute minimum and maximum

e
values of f on [x, x + h]. By the comparison property of the definite integral we
gin
have
x+h
R
mh ≤ f (t)dt ≤ Mh.
x
x+h
En

R
f (u)h ≤ f (t)dt ≤ f (v)h.
x
Since h > 0, dividing throughout by h we get
x+h
arn

R
f (u) ≤ h1 f (t)dt ≤ f (v). (2)
x
(2) can be proved in a similar way for h < 0.
Now, allowing h → 0 and since u and v lie in [x, x + h], we have u → x
Le

and v → x. Since f is continuous at x,we have


w.

∴ lim f (u) = lim f (u) = f (x)


h→0 u→x

and lim f (v) = lim f (v) = f (x).


h→0 v→x
ww

By (2) we have
Zx+h
1
lim f (u) ≤ lim f (t)dt ≤ lim f (v)
h→0 h→0 h h→0
x

Visit For More : www.LearnEngineering.in


Visit For More : www.LearnEngineering.in

Integral Calculus 155

g(x + h) − g(x)
f (x) ≤ lim ≤ f (x) [by (1)]
h→0 h
i.e., f (x) ≤ g′ (x) ≤ f (x).

By Squeeze Theorem we get

g′ (x) = f (x). (3)

.in
Since every differentiable function is continuous,
we obtain g is also continuous on (a, b).

ng
If x = a or x = b, from (3) we get the one sided limits at a and b.
This shows that g is continuous on [a, b].
Rx √

eri
Example 3.2. Find the derivative of the function g(x) = 1 + t2 dt.
√ 0
Solution. Since f (t) = 1 + t2 is continuous on [0, x], by part 1 of the Fundamental
theorem of calculus we get g′ (x) = 1 + x2
e√
gin
 4 
d Rx 
Example 3.3. Find  sec tdt.
dx 1
Solution. Let u = x 4
En

 x4   u 
Z  Z
d  d 


∴  sec tdt =  sec tdt
dx   dx  
arn

1 1
 u 
Z
d   du

=  sec tdt · [By Chain rule]
du dx
1
Le

= sec u · 4x3 [by FTC 1]

= sec x4 · 4x3 .
w.

Second Fundamental theorem of calculus


Rb
Statement.If f is continuous on [a, b], then f (x)dx = F(b) − F(a) where F is any
ww

a
antiderivative of f . [i.e, a function such that F ′ = f ].
Rx
Proof. Let g(x) = f (t)dt.
a
By the First Fundamental theorem of calculus we have g′ (x) = f (x). i.e,

Visit For More : www.LearnEngineering.in


Visit For More : www.LearnEngineering.in

156 Engineering Mathematics - I

g is an antiderivative of f .
If F is any other antiderivative of f on [a, b], we know that F and g differ
by a constant.
∴ F(x) = g(x) + c for a < x < b.
Since both F and g are continuous on [a, b] the above relation holds when x = a and

.in
x = b.
i.e lim F(x) = F(a) & lim F(x) = F(b).
x→a+ x→b−
Also lim g(x) = g(a) & lim g(x) = g(b).

ng
x→a+ x→b−
Hence we obtain F(x) = g(x) + c for all x in [a, b] (1)
Ra
When x = a, we have g(a) = f (t)dt = 0.

eri
a
Using (1) for x = b and x = a we get

F(b) − F(a) = [g(b) + c] − [g(a) + c]


e
gin
= g(b) − g(a)
Zb
= g(b) = f (t)dt.
a
En

Result. The First and Second Fundamental Theorem of Calculus are combined
together is called the Fundamental Theorem of Calculus.
arn

Statement of Fundamental Theorem of Calculus.


Suppose f is continuous on [a, b]
Rx
(i) If g(x) = f (t)dt, then g′ (x) = f (x).
Le

a
Rb
(ii) f (x)dx = F(b) − F(a), where F is any antiderivative of f . (i.e., F ′ = f .)
a
w.

Note (1). The Fundamental theorem of calculus says that differentiation and
integration are inverse processors.
Note (2). By part(ii) of the Fundamental theorem of calculus we have
ww

Rb
f (x)dx = F(b) − F(a) where F ′ = f .
a
Rb
i.e F ′ (x)dx = F(b) − F(a).
a

Visit For More : www.LearnEngineering.in


Visit For More : www.LearnEngineering.in

Integral Calculus 157

The above result is sometimes called as the Net Change Theorem.


Net Change Theorem
Statement. The integral of a rate of change is the net change.
Rb
i.e F ′ (x)dx = F(b) − F(a).

a ☞
Worked Examples
✍ ✌

.in
R3
Example 3.4. Evaluate (x2 − 2x)dx by using Reimann sum by taking right end
0
points as the sample points. Hence verify it by using fundamental theorem of

ng
calculus.
Solution. Let f (x) = x2 − 2x.

eri
Divide the interval [0, 3] into n sub intevals with equal width
3−0 3
∆x = = .
n n "
3
# "
3 6
# "
The intervals are 0, , , , , , · · ·
6 9
# "
e3n − 3 3n
,
#
.
gin
n n n n n n n
The sample points xi∗ are the right end points of the sub intervals.
3 6 9 3n
∴ xi∗ = , , , · · · .
n n n n
En

The corresponding f (xi∗ ) = xi∗ 2 − 2xi∗ are calculated and tabulated as


follows.
arn

3 6 9 3n
xi∗ n n n ··· n
9 6 36 12 81 18 9n2 6n
f (xi∗ ) n2
− n n2
− n n2
− n ··· n2
− n
Le

By Reimann’s sum we have,


Z3 n
X
(x2 − 2x)dx = lim f (xi∗ )∆x
w.

n=∞
0 i=1

9n2 6n 3
" ! ! ! ! #
9 6 3 36 12 3 81 18 3
= lim − + − + − + · · · + −
n=∞ n2 n n n2 n n n2 n n n2 n n
ww

" 3 2
! 3 2
! 3 2
!
3 3 3 3 3 3
= lim 3
· 1 − 2 · 2 + 3 · 22 − 2 · 4 + 3 · 32 − 2 · 6
n=∞ n n n n n n
3 32
!#
3 2
+ · · · + 3 · n − 2 · 2n
n n

Visit For More : www.LearnEngineering.in


Visit For More : www.LearnEngineering.in

158 Engineering Mathematics - I

33  2 2 2 2
 32
= lim 1 + 2 + 3 + · · · + n − lim (2 + 4 + 6 + · · · + 2n)
n=∞ n3 n=∞ n2
27 h n(n + 1)(2n + 1) i 9 h n(n + 1) i
= lim 3 − lim 2 · 2
n=∞ n 6 n=∞ n 2
3 1 1
h n (1 + n )(2 + n ) i h n (1 + 1n ) i 27
2
27
= lim − 9 lim = × 2 − 9 = 0.
6 n=∞ n3 n=∞ n2 6

.in
By the Fundamental Theorem of Calculus we have
Z3 !3
2 x3 x2 27
(x − 2x)dx = −2· = − 9 = 9 − 9 = 0.

ng
3 2 0 3
0

eri
R1 √
Example 3.5. Evaluate the integral 1 − x2 dx interms of areas.
0
R1 √ √
Solution.
0
e
1 − x2 dx represents the area under the curve y = 1 − x2 between
gin
the x−axis and the lines x = 0 and x = 1.
y
p
Now, y = 1 − x2
En

y2 = 1 − x2
0 1x
2 2
x + y = 1.
arn

This is a circle with centre at the origin and radius = 1 unit. Since we
need the area between x = 0 and x = 1, the required area is the area of the portion
1 π
Le

of the circle that lies in the first quadrant = · π × 12 = .


4 4
R1 √ π
∴ 1 − x2 dx = .
4
w.

R3
ww

Example 3.6. Evaluate the integral (x − 1)dx interms of areas.


0
R3
Solution. (x − 1)dx represents the area under the curve y = (x − 1) between the
0
x−axis and the lines x = 0 and x = 3.

Visit For More : www.LearnEngineering.in


Visit For More : www.LearnEngineering.in

3 Integral Calculus

.in
ng
3.1 The definite integral

Integration as the limit of a sum

y = f (x)
e eri
gin
n rectangles
f (xk )
·········
En

∆x
x1 = a x2 x3 xn+1 = b
arn

Consider the graph of the positive function y = f (x). Let us find the area
under the curve y = f (x),between the x−axis and the ordinates x = a and x = b.
Le

b−a
Divide this area into n rectangles of equal width ∆x = . Let the x− co ordinates
n
at the left hand side of the rectangles be x1 = a, x2 , x3 , . . . xn+1 = b. Consider a typical
w.

rectangle, the kth one with height f (xk ). The area of this rectangle is f (xk )∆x. The
sum of all the areas of the n rectangles is
ww

f (x1 )∆x + f (x2 )∆x + · · · + f (xn )∆x.


n
P
In summation convention form, this can be written as f (xk )∆x. This
k=1
gives an estimate of the area under the curve but it is not exact. To improve the

Visit For More : www.LearnEngineering.in


Visit For More : www.LearnEngineering.in

150 Engineering Mathematics - I

estimate we must take a large number of very thin rectangles. This can be
achieved by allowing n → ∞ and making ∆x → 0.
n
P
∴ Area under the curve = lim f (xk )∆x.
n=∞ k=1
The lower and upper limits on the sum correspond to the first and the
last rectangle where x = a and x = b respectively. Hence the above limit can be
b

.in
P
written as lim f (x)∆x. Since the number of rectangles increase without bound,
∆x→0 x=a
we drop the subsript k from xk and write f (x) which is the value of f at a typical
value of x. If this can actually be found, it is called the definite integral of f (x)

ng
Rb
from x = a and x = b and it is written as f (x)dx.
a
Rb

eri
n
P b
P
∴ f (x)dx = lim f (xk )∆x = lim f (x)∆x.
n=∞ k=1 ∆x→0 x=a
a
Note. If we approximate each strip by a rectangle that has the same base as the

e
strip and whose height is the same as the right edge of the strip; (i.e., xk is taken
gin
at the right end points of the kth rectangle and f (xk ) as the height ) then also the
above result will be achieved.
Definition of a definite integral. If f is a function defined for a ≤ x ≤ b, we
En

b−a
divide the interval [a, b] into n subintervals of equal width ∆x = . We let
n
x0 (= a), x1 , x2 , · · · xn (= b) be the end points of these subintervals and if we choose
arn

x1∗ , x2∗ , · · · xn∗ as any sample ponits in these subintervals, so that xi∗ lies in the ith
subinterval [xi−1 , xi ]. Then the definite integral of f from a to b is defined as
Rb n
f (xi∗ )∆x, provided that the limit exists and gives the same value
P
f (x)dx = lim
Le

n=∞ i=1
a
for all possible choices of sample points. If it does exist, we say that f is
integrable on [a, b].
w.

Rb n
ww

f (xi∗ )∆x can also be written as, for ∈> 0, there is an


P
Result (1). f (x)dx = lim
n=∞ i=1
a
Rb n
f (xi∗ )∆x <∈ for every integer n > N and for
P
integer N such that f (x)dx −
a i=1
every choice of xi∗ in [xi−1 , xi ].

Visit For More : www.LearnEngineering.in


Visit For More : www.LearnEngineering.in

Integral Calculus 151

n
f (xi∗ )∆x is called the Reimann sum.
P
Result (2). The sum
i=1

Theorem 1. If f is continuos on [a, b], (or) if f has only a finite number of jump
Rb
discontinuities, then f is integrable on [a, b].[i.e., f (x)dx exists].
a
Rb n
P
Theorem 2.If f is integrable on [, b], then f (x)dx = lim f (xi )∆x where ∆x =

.in
n=∞ i=1
a
b−a
and xi = a + i∆x.
n ✎ ☞

ng
Worked Examples
✍ ✌

R1

eri
Example 3.1. Using the area property evaluate x2 dx. Show that the sum of the
0
1
areas of the upper approximating rectangles approaches .
3

e
Solution. Consider the function y = f (x) = x2 . Divide the area under the curve
1−0 1
gin
y = x2 between x = 0 and x = 1 into 4 rectangles of equal width ∆x = = .
4 4
1
The width of each rectangle is .
" # " 4# " # " #
1 1 1 1 3 3
En

The subintervals are 0, , , , , and , 1 .


4 4 2 2 4 4
The height of the rectangles is approximated to the value of the
1 1 3
ordinates at x = 0, , & .
arn

4 2 4
! ! !
1 1 3
∴ Area of the required portion = f (0)∆x + f ∆x + f ∆x + f ∆x.
4 2 4
Le

!2 !2 !2
2 1 1 1 1 1 3 1
=0 . + . + . + .
4 4 4 2 4 4 4
" #
1 1 1 9
w.

= 0+ + +
4 16 4 16
" #
1 1+4+9
=
ww

4 16
1 14 7
= × = .
4 16 32

If the height of the rectangles is approximated to the value of the

Visit For More : www.LearnEngineering.in


Visit For More : www.LearnEngineering.in

152 Engineering Mathematics - I

ordinates at the right edge of each rectangle then,


! ! !
1 1 3
Area = f ∆x + f ∆x + f ∆x + f (1)∆x.
4 2 4
!2 !2 !2
1 1 1 1 3 1 1
= . + . + . + 12 ·
4 4 2 4 4 4 4
" # " #
1 1 1 9 1 1 + 4 + 9 + 16 1 30 15
= + + +1 = = × = .

.in
4 16 4 16 4 16 4 16 32
Now, let us divide the area " into # "n rectangles
# " each of their
# with width
1−0 1 1 1 2 n−1 n

ng
∆x = = . The subintervals are 0, , , , · · · , , =1 .
n n n n n n n
The heights of the rectangle are the values of the function f (x) = x2 at
1 2 3 n

eri
the points , , · · · .
n n n n
!2 !2 !2  n 2 1
1 1 2 1 3 1
∴ Area = . + . + . + ··· + .
n n n n
e n n n n
gin
1 12 22 32 n2
" #
= + + + · · · +
n n2 n2 n2 n2
1 h i
= 3 12 + 22 + 32 + · · · + n2
n "
En

#
1 n(n + 1)(2n + 1) (n + 1)(2n + 1)
= 3 =
n 6 6n2
(n + 1)(2n + 1)
lim (Area) = lim
arn

n=∞ n=∞ 6n2


n (1 + 1n )(2 + 1n ) 2 1
2
= lim = = .
n=∞ 6n2 6 3
Le

Properties of definite integrals.


The following properties can be easily proved using Riemann sums.
w.

Ra Rb
(i) f (x)dx = − f (x)dx.
b a
ww

Ra
(ii) f (x)dx = 0.
a

Rb
(iii) cdx = c(b − a),where c is any constant.
a

Visit For More : www.LearnEngineering.in


Visit For More : www.LearnEngineering.in

Integral Calculus 153

Rb Rb Rb
(iv) [ f (x) + g(x)]dx = f (x)dx + g(x)dx.
a a a

Rb Rb
(v) c f (x)dx = c f (x)dx,where c is any constant.
a a

Rb Rb Rb
(vi) [ f (x) − g(x)]dx = f (x)dx − g(x)dx.

.in
a a a

Rb Rc Rb
(vii) If a < c < b then f (x)dx = f (x)dx + f (x)dx.

ng
a a c

Comparison properties of the integral.

eri
Rb
(i) If f (x) ≥ 0 for a ≤ x ≤ b, then f (x)dx ≥ 0.
a

(ii) If f (x) ≥ g(x) for a ≤ x ≤ b, then


Rb
e
f (x)dx ≥
Rb
g(x)dx.
gin
a a

Rb
(iii) If m ≤ f (x) ≤ M for a ≤ x ≤ b, then m(b − a) ≤ f (x)dx ≤ M(b − a).
a
En

Proof. Given m ≤ f (x) ≤ M


Integrating between a and b we get
Rb Rb Rb
arn

mdx ≤ f (x)dx ≤ Mdx


a a a
Rb Rb Rb
m dx ≤ f (x)dx ≤ M dx
a a a
Le

Rb
m(b − a) ≤ f (x)dx ≤ M(b − a).
a

The first fundamental theorem of calculus


w.

Statement. If f is continuous on [a, b], then the function g defined by


Rx
ww

g(x) = f (t)dt, a ≤ x ≤ b
a

is continuous on [a, b] and is differentiable on (a, b) and g′ (x) = f (x).


Proof.Choose x and x + b ∈ (a, b).

Visit For More : www.LearnEngineering.in


Visit For More : www.LearnEngineering.in

154 Engineering Mathematics - I

y
Zx+h Zx
Now, g(x + h) − g(x) = f (t)dt − f (t)dt
a a
Zx Zx+h Zx
= f (t)dt + f (t)dt − f (t)dt m M

a x a
Zx+h

.in
= f (t)dt. 0 x u v x+h x
x

ng
g(x + h) − g(x) 1 x+h
R
Hence, for h , 0, we have = f (t)dt. (1)
h h x
Let us assume that h > 0. Since f is continuous on [x, x + h], by the

eri
extreme value theorem, there exists numbers u and v in [x, x + h] such that
f (u) = m and f (v) = M, where m and M are the absolute minimum and maximum

e
values of f on [x, x + h]. By the comparison property of the definite integral we
gin
have
x+h
R
mh ≤ f (t)dt ≤ Mh.
x
x+h
En

R
f (u)h ≤ f (t)dt ≤ f (v)h.
x
Since h > 0, dividing throughout by h we get
x+h
arn

R
f (u) ≤ h1 f (t)dt ≤ f (v). (2)
x
(2) can be proved in a similar way for h < 0.
Now, allowing h → 0 and since u and v lie in [x, x + h], we have u → x
Le

and v → x. Since f is continuous at x,we have


w.

∴ lim f (u) = lim f (u) = f (x)


h→0 u→x

and lim f (v) = lim f (v) = f (x).


h→0 v→x
ww

By (2) we have
Zx+h
1
lim f (u) ≤ lim f (t)dt ≤ lim f (v)
h→0 h→0 h h→0
x

Visit For More : www.LearnEngineering.in


Visit For More : www.LearnEngineering.in

Integral Calculus 155

g(x + h) − g(x)
f (x) ≤ lim ≤ f (x) [by (1)]
h→0 h
i.e., f (x) ≤ g′ (x) ≤ f (x).

By Squeeze Theorem we get

g′ (x) = f (x). (3)

.in
Since every differentiable function is continuous,
we obtain g is also continuous on (a, b).

ng
If x = a or x = b, from (3) we get the one sided limits at a and b.
This shows that g is continuous on [a, b].
Rx √

eri
Example 3.2. Find the derivative of the function g(x) = 1 + t2 dt.
√ 0
Solution. Since f (t) = 1 + t2 is continuous on [0, x], by part 1 of the Fundamental
theorem of calculus we get g′ (x) = 1 + x2
e√
gin
 4 
d Rx 
Example 3.3. Find  sec tdt.
dx 1
Solution. Let u = x 4
En

 x4   u 
Z  Z
d  d 


∴  sec tdt =  sec tdt
dx   dx  
arn

1 1
 u 
Z
d   du

=  sec tdt · [By Chain rule]
du dx
1
Le

= sec u · 4x3 [by FTC 1]

= sec x4 · 4x3 .
w.

Second Fundamental theorem of calculus


Rb
Statement.If f is continuous on [a, b], then f (x)dx = F(b) − F(a) where F is any
ww

a
antiderivative of f . [i.e, a function such that F ′ = f ].
Rx
Proof. Let g(x) = f (t)dt.
a
By the First Fundamental theorem of calculus we have g′ (x) = f (x). i.e,

Visit For More : www.LearnEngineering.in


Visit For More : www.LearnEngineering.in

156 Engineering Mathematics - I

g is an antiderivative of f .
If F is any other antiderivative of f on [a, b], we know that F and g differ
by a constant.
∴ F(x) = g(x) + c for a < x < b.
Since both F and g are continuous on [a, b] the above relation holds when x = a and

.in
x = b.
i.e lim F(x) = F(a) & lim F(x) = F(b).
x→a+ x→b−
Also lim g(x) = g(a) & lim g(x) = g(b).

ng
x→a+ x→b−
Hence we obtain F(x) = g(x) + c for all x in [a, b] (1)
Ra
When x = a, we have g(a) = f (t)dt = 0.

eri
a
Using (1) for x = b and x = a we get

F(b) − F(a) = [g(b) + c] − [g(a) + c]


e
gin
= g(b) − g(a)
Zb
= g(b) = f (t)dt.
a
En

Result. The First and Second Fundamental Theorem of Calculus are combined
together is called the Fundamental Theorem of Calculus.
arn

Statement of Fundamental Theorem of Calculus.


Suppose f is continuous on [a, b]
Rx
(i) If g(x) = f (t)dt, then g′ (x) = f (x).
Le

a
Rb
(ii) f (x)dx = F(b) − F(a), where F is any antiderivative of f . (i.e., F ′ = f .)
a
w.

Note (1). The Fundamental theorem of calculus says that differentiation and
integration are inverse processors.
Note (2). By part(ii) of the Fundamental theorem of calculus we have
ww

Rb
f (x)dx = F(b) − F(a) where F ′ = f .
a
Rb
i.e F ′ (x)dx = F(b) − F(a).
a

Visit For More : www.LearnEngineering.in


Visit For More : www.LearnEngineering.in

Integral Calculus 157

The above result is sometimes called as the Net Change Theorem.


Net Change Theorem
Statement. The integral of a rate of change is the net change.
Rb
i.e F ′ (x)dx = F(b) − F(a).

a ☞
Worked Examples
✍ ✌

.in
R3
Example 3.4. Evaluate (x2 − 2x)dx by using Reimann sum by taking right end
0
points as the sample points. Hence verify it by using fundamental theorem of

ng
calculus.
Solution. Let f (x) = x2 − 2x.

eri
Divide the interval [0, 3] into n sub intevals with equal width
3−0 3
∆x = = .
n n "
3
# "
3 6
# "
The intervals are 0, , , , , , · · ·
6 9
# "
e3n − 3 3n
,
#
.
gin
n n n n n n n
The sample points xi∗ are the right end points of the sub intervals.
3 6 9 3n
∴ xi∗ = , , , · · · .
n n n n
En

The corresponding f (xi∗ ) = xi∗ 2 − 2xi∗ are calculated and tabulated as


follows.
arn

3 6 9 3n
xi∗ n n n ··· n
9 6 36 12 81 18 9n2 6n
f (xi∗ ) n2
− n n2
− n n2
− n ··· n2
− n
Le

By Reimann’s sum we have,


Z3 n
X
(x2 − 2x)dx = lim f (xi∗ )∆x
w.

n=∞
0 i=1

9n2 6n 3
" ! ! ! ! #
9 6 3 36 12 3 81 18 3
= lim − + − + − + · · · + −
n=∞ n2 n n n2 n n n2 n n n2 n n
ww

" 3 2
! 3 2
! 3 2
!
3 3 3 3 3 3
= lim 3
· 1 − 2 · 2 + 3 · 22 − 2 · 4 + 3 · 32 − 2 · 6
n=∞ n n n n n n
3 32
!#
3 2
+ · · · + 3 · n − 2 · 2n
n n

Visit For More : www.LearnEngineering.in


Visit For More : www.LearnEngineering.in

158 Engineering Mathematics - I

33  2 2 2 2
 32
= lim 1 + 2 + 3 + · · · + n − lim (2 + 4 + 6 + · · · + 2n)
n=∞ n3 n=∞ n2
27 h n(n + 1)(2n + 1) i 9 h n(n + 1) i
= lim 3 − lim 2 · 2
n=∞ n 6 n=∞ n 2
3 1 1
h n (1 + n )(2 + n ) i h n (1 + 1n ) i 27
2
27
= lim − 9 lim = × 2 − 9 = 0.
6 n=∞ n3 n=∞ n2 6

.in
By the Fundamental Theorem of Calculus we have
Z3 !3
2 x3 x2 27
(x − 2x)dx = −2· = − 9 = 9 − 9 = 0.

ng
3 2 0 3
0

eri
R1 √
Example 3.5. Evaluate the integral 1 − x2 dx interms of areas.
0
R1 √ √
Solution.
0
e
1 − x2 dx represents the area under the curve y = 1 − x2 between
gin
the x−axis and the lines x = 0 and x = 1.
y
p
Now, y = 1 − x2
En

y2 = 1 − x2
0 1x
2 2
x + y = 1.
arn

This is a circle with centre at the origin and radius = 1 unit. Since we
need the area between x = 0 and x = 1, the required area is the area of the portion
1 π
Le

of the circle that lies in the first quadrant = · π × 12 = .


4 4
R1 √ π
∴ 1 − x2 dx = .
4
w.

R3
ww

Example 3.6. Evaluate the integral (x − 1)dx interms of areas.


0
R3
Solution. (x − 1)dx represents the area under the curve y = (x − 1) between the
0
x−axis and the lines x = 0 and x = 3.

Visit For More : www.LearnEngineering.in


Visit For More : www.LearnEngineering.in

Integral Calculus 159

y
The required area = A1 − A2 .

A1 is a triangle with base = 2units A1


x=3

and height = 2units. A2


x
  x=0
Put x = 3 in y = x − 1.

.in
1
∴ Area of A1 = × 2 × 2 = 2.
2

ng
A2 is a triangle with base = 1 unit and height = 1unit.
1 1
∴ Area of A2 = × 1 × 1 = .
2 2
R3

eri
1 3
(x − 1)dx = 2 − = .
0 2 2

Example 3.7. Prove that


Rb
xdx =
b2 − a2
eby interpretting interms of areas.
gin
a 2
[A.U Nov 2016]
Rb
Solution. xdx represents the area under the curve y = x between the x−axis
a
En

and the lines x = a and x = b.


Required area = Area of ∆OCD− Area of ∆OAB
1 1
= × OD × CD − × OB × AB
arn

2 2 y
1 1 C
= b·b− a·a A b
2 2
b2 a2 b2 − a2 a
= − =
Le

2 2 2 b−a
Zb 0 a B Db x
b2 − a2
∴ xdx = .
2
w.

a
n
(xi3 + xi sin xi )∆x as an integral on the interval [0, π].
P
Example 3.8. Express lim
n→∞ i=1
ww

Solution. According to Riemann sum,the limit


n Rπ
lim (xi3 + xi sin xi )∆x = (x3 + x sin x)dx.
P
n→∞ i=1
0

Example 3.9. Evaluate the Riemann sum for f (x) = x3 − 6x, taking the sample

Visit For More : www.LearnEngineering.in


Visit For More : www.LearnEngineering.in

160 Engineering Mathematics - I

R3
points to be the right end points and a = 0, b = 3, n = 6. Also Evaluate (x3 − 6x)dx.
0
3−0 1
Solution. Divide the interval [0, 3] into 6 subintervals with equal width = .
h i h i h i h i h i h i 6 2
Hence, the intervals are 0, 21 , 21 , 1 , 1, 23 , 23 , 2 , 2, 52 , 25 , 3
1 3 5
The right end points are , 1, , 2, , 3.
2 2 2

.in
The value of f (x) at these points are as follows.

x(k) x3 −6x f (xk ) f (xk )∆x


1 1
- 23 - 23

ng
2 8 -3 8 16
1 1 −6 −5 - 52
3 27
−9 - 45 - 45

eri
2 8 8 16
2 8 -12 -4 -2
5 125 5 5
−15
2
3
8
27 −18 e 8
9
16
9
gin
2

n
X
Reimann Sum = f (xk )∆xk
k=1
En

23 5 45 5 9
=− − − −2+ +
16 2 16 16 2
−23 − 40 − 45 − 32 + 5 + 72
=
arn

16
−140 + 77 −63
= = .
16 16
R3
Le

Let us evaluate (x3 − 6x)dx.


0
Divide the interval [0, 3] into n subintervals with equal width
3−0 3
w.

∆x = = .
n n h i h i h i h i
Hence the intervals are 0, 3n , n3 , n6 , 6n , n9 , · · · 3n−3 3n
n , n
The sample points xi∗ are the right end points of the sub intervals
ww

∴ xi∗ = n3 , 6n , n9 , · · · 3n
n.
The corresponding f (xi∗ ) = xi3 − 6xi are calculated and tabulated as
follows.

Visit For More : www.LearnEngineering.in


Visit For More : www.LearnEngineering.in

Integral Calculus 161

3 6 9 3n
xi∗ n n n ··· n
33 3 63 6 93 9 33 n3 3n
f (xi∗ ) n3
−6· n n3
−6· n n3
−6· n ··· n3
−6· n

By Reimann sum
Z3 n
X
(x3 − 6x)dx = lim f (xi∗ )∆x
n=∞
0 i=1

.in
33 63 33 n3
" ! ! ! #
3 3 6 3 3n 3
= lim −6· + 3 −6· + ··· + −6·
n=∞ n3 n n n n n n3 n n
" 4 2
#
3 3

ng

3 3 3 3
= lim 4 1 + 2 + 3 + · · · + n − 6 · 2 (1 + 2 + 3 + · · · + n)
n=∞ n n
 34 n(n + 1) 2
 !
2
!
3 n(n + 1) 
= lim  4 −6· 2

eri

n=∞ n 2 n 2
" 4 2
3 n (n + 1)2 32 n(n + 1)
#
= lim 4 −6· 2
n=∞ n 4 n 2

e 1
!2 !
1 
gin
4 2

 4 n 1 + n 1 + 
 3 n 32 n 
= lim  4 · −6· 2 
n=∞  n
 4 n 2 

En

 !2 !
 81 1 1 
= lim  · 1 + − 27 · 1 + 
n=∞ 4 n n
81 81 − 108 27
= − 27 = =− .
arn

4 4 4
Indefinite Integrals.
The Fundamental theorem of
calculus gives the relation between
Le

R
antiderivatives and integrals. The notation f (x)dx is traditionally used for an
Rb
antiderivative of f and is called an indefinite integral. Thus f (x)dx = F(x)
w.

a
means F ′ (x) = f (x).
d R
[F(x)] = f (x) then f (x)dx = F(x)
dx
ww

Z
d
In other words if [F(x)] = f (x), then f (x)dx = F(x).
dx
d
Also we have [F(x) + c] = f (x) then
R dx
f (x)dx = F(x) + c,where c is called the constant of integration.

Visit For More : www.LearnEngineering.in


Visit For More : www.LearnEngineering.in

162 Engineering Mathematics - I

With this notation, we can derive the basic results in integration, using the
basic results in differentiation.
Basic results.
d n
1. We have (x ) = nxn−1 .
dx
d xn+1
!
(or) = xn .

.in
dx n + 1
R xn+1
∴ xn dx = + c.
n+1

ng
d 1 R 1
2. (log x) = ⇒ dx = log x + c.
dx x x

eri
d x R
3. (e ) = e x ⇒ e x dx = e x + c.
dx
R bx
b x dx =
4.
log b
+ c.
e
gin
R
5. sin xdx = − cos x + c.
R
6. cos xdx = sin x + c.
En

R
7. sec2 xdx = tan x + c.
arn

R
8. cosec2 xdx = − cot x + c.
R
9. sec x tan xdx = sec x + c.
Le

R
10. cosecx cot xdx = −cosecx + c.
R 1
11. √ dx = sin−1 x + c.
w.

1 − x2
R 1
12. dx = tan−1 x + c.
1 + x2
ww

R 1
13. √ dx = sec−1 x + c.
x x2 −1
R
14. sinh xdx = cos hx + c.

Visit For More : www.LearnEngineering.in


Visit For More : www.LearnEngineering.in

Integral Calculus 163

R
15. cosh xdx = sin hx + c.
R 1 √
16. √ dx = cos h−1 x + c or log(x + x2 − 1) + c.
x2 − 1
R 1 √
17. √ dx = sin h−1 x + c or log(x + x2 + 1) + c.
+1 x2
Basic properties of indefinite integrals.

.in
If f (x) and g(x) are functions of x and k is a constant, then the following
properties are true.

ng
R R R
(i) [ f (x) + g(x)]dx = f (x)dx + g(x)dx + c.
R R R
(ii) [ f (x) − g(x)]dx = f (x)dx − g(x)dx + c.

eri
R R
(iii) k f (x)dx = k f (x)dx + c.
✎ ☞
Worked Examples

e ✌
gin
Z
Example 3.10. Evaluate (10x4 − 2 sec2 x)dx.
Z Z Z
Solution. (10x − 2 sec x)dx = 10 x dx − 2 sec2 xdx + c
4 2 4
En

Z2 x5
= 10√· −12!tan x + c = 2x5 − 2 tan x + c.
Example 3.11. Evaluate 2 5
x − 3 x + 2 dx.
x
1
arn

Z2 ! Z2 Z2 Z2
2 √1 2 1
Solution. x − 3 x + 2 dx = x dx − 3 x 2 dx + x−2 dx
x
1 1 1 1
Le

 3 2
3 2
!2
x−1
!
x  x 2 
= − 3  3  +
 
3 1 2
−1 1
1
!2
w.

1 3 3 2h 3 i 1
= [2 − 1 ] − 3 × 2 − 1 −
2
3 3 x 1
1 √ 1
= [8 − 1] − 2[2 2 − 1] − [ − 1]
ww

3 2
7 √ 1
= −4 2+2+
3 2
21 + 12 + 3 √ 36 √ √
= −4 2= − 4 2 = 6 − 4 2.
6 6

Visit For More : www.LearnEngineering.in


Visit For More : www.LearnEngineering.in

164 Engineering Mathematics - I

Z3
Example 3.12. Evaluate (x3 − 6x)dx.
0
Z3 Z3 Z3 !3 !3
3 3 x4 x2
Solution. (x − 6x)dx = x dx − 6 xdx = −6
4 0 2 0
0 0 0
1
= [34 − 0] − 3[32 − 0]

.in
4
1 81 81 − 108 27
= × 81 − 27 = − 27 = =− .
4 4 4 4

ng
Z2 !
3 3
Example 3.13. Evaluate 2x − 6x + 2 dx.
x +1

eri
0
Z2 ! Z2 Z2 Z2
3 3 3 1
Solution. 2x − 6x + 2 dx = 2 x dx − 6 xdx + 3 2
dx
x +1 x +1
0
e0
!2
0
!2
0
gin
x4 x2  2
=2· −6 + 3 tan−1 x
4 0 2 0
0

1  
= [24 − 0] − 3[22 − 0] + 3 tan−1 2 − tan−1 0
2
En

= 8 − 12 + 3 tan−1 2 = 3 tan−1 2 − 4.

Zπ/6
arn

2 x
 
Example 3.14. Evaluate cos dx.
2
0
Le

Zπ/6 Zπ/6
2 x 1
 
Solution. cos dx = (1 + cos x)dx
2 2
0 0
w.

 π/6
Zπ/6

Z 
1  
=  dx + cos xdx
2 
ww

0 0
1  π/6
  π/6 
= x dx + sin x
2 0 0
1 π  π  1 "π 1# π 1
= − 0 + sin − sin 0 = + = + .
2 6 6 2 6 2 12 4

Visit For More : www.LearnEngineering.in


Visit For More : www.LearnEngineering.in

Integral Calculus 165

Z9 √
2x2 + x2 x − 1
Example 3.15. Evaluate dx.
x2
1
Z9 √ Z9 √
2x2 x2 x 2x2 x2 x
!
+ −1 1
Solution. dx = + − 2 dx
x2 x2 x2 x
1 1
Z9 

.in
1

= 2 + x 2 − x−2 dx
1
Z9 Z9 Z9

ng
1
=2 dx + x dx −
2 x−2 dx
1 1 1
 9  x 23 9 !9
 

eri
x−1
= 2 x +  3  −

 

1
2 1
−1 1
!
2 3   1
3
e
= 2[9 − 1] + 9 2 − 1 +
9
−1
gin
2 8
= 16 + × 26 −
3 9
52 8 144 + 156 − 8 292
= 16 + − = = .
3 9 9 9
En

Z
1
Example 3.16. Evaluate 2
dx
sin 2x
Z x cos2
sin x + cos2 x
Z
1
arn

Solution. dx = dx
sin2 x cos2 x sin2 x cos2 x
sin2 x cos2 x
Z Z
= 2
dx + dx
Z sin x cos Z
2x sin2 x cos2 x
Le

= sec2 xdx + cosec2 xdx

= tan x − cot x + c.
w.

3.2 The Substitution Rule


ww

R
Integrals of functions of the form f (ax + b)dx.
This type of integrals can be evaluated by making a substitution ax + b = t.
R
For example consider (2x + 3)2 dx.

Visit For More : www.LearnEngineering.in


Visit For More : www.LearnEngineering.in

166 Engineering Mathematics - I

Let t = 2x + 3
dt = 2dx
dt
∴ dx =
2Z
1 t3
Z Z
2 2 dt 1 1
Now, (2x + 3) dx = t · = t2 dt = · + c = · (2x + 3)3 + c.
2 2 2 3 6
Standard results.

.in
Z
1. Evaluate (ax + b)n dx.

ng
Solution. Let ax + b = t.
dt
adx = dt ⇒ dx = .

eri
a
dt 1 tn+1 (ax + b)n+1
Z Z
(ax + b)n dx = tn = · +c= + c.
a a n+1 a(n + 1)

e
In a similar way, the following results can be easily derived.
gin
Z
1 log(ax + b)
2. dx = + c.
ax + b a
eax+b
Z
eax+b dx =
En

3. + c.
a
Z
cos(ax + b)
4. sin(ax + b)dx = − + c.
a
arn

Z
sin(ax + b)
5. cos(ax + b)dx = + c.
a
Z
tan(ax + b)
Le

6. sec2 (ax + b)dx = + c.


a
Z
cosec(ax + b)
7. cosec2 (ax + b)dx = − + c.
w.

a
Z
sec(ax + b)
8. sec(ax + b) tan(ax + b)dx = + c.
ww

a
Z
cos(ax + b)
9. cosec(ax + b) cot(ax + b)dx = − + c.
a
✎ ☞
Worked Examples
✍ ✌

Visit For More : www.LearnEngineering.in


Visit For More : www.LearnEngineering.in

Integral Calculus 167

Z
Example 3.17. Evaluate sin2 3xdx.
Z Z
2 1
Solution. sin 3xdx = (1 − cos 6x)dx
2
"Z Z #
1
= dx − cos 6xdx
2
" #
1 sin 6x
= x− +c

.in
2 6
x sin 6x
= − + c.
2 12

ng
Z
Example 3.18. Evaluate cos mx cos nxdx.

eri
Solution. case(i) when m , n.
Z Z
1
cos mx cos nxdx = (cos(m + n)x + cos(m − n)x) dx
2
e !
gin
Z Z
1
= cos(m + n)xdx + cos(m − n)x dx
2
!
1 sin(m + n)x sin(m − n)x
= + + c.
2 m+n m−n
En

case(ii) If m = n
Z Z
cos mx cos nxdx = cos mx cos mxdx
arn

Z
= cos2 mxdx
Z
1
= (cos 2mx + 1)dx
Le

2
"Z Z #
1
= cos 2mxdx + dx
2
w.

" #
1 sin 2mx
= +x +c
2 2m
sin 2mx x
ww

= + + c.
4m 2

Z
Example 3.19. Evaluate cos3 2xdx.

Visit For More : www.LearnEngineering.in


Visit For More : www.LearnEngineering.in

168 Engineering Mathematics - I

Z Z !
3 1 3
Solution. cos 2xdx = cos 6x + cos 2x dx
4 4
Z Z cos 3θ = 4 cos3 θ − 3 cos θ
1 3
= cos 6xdx + cos 2xdx 1 3
4 4 cos3 θ = cos 3θ + cos θ
1 sin 6x 3 sin 2x 4 4
= · + · +c 3 1 3
4 6 4 2 cos 2x = cos 6x + cos 2x.
sin 6x sin 2x 4 4
= +3 + c.

.in
24 8
Z
Example 3.20. Evaluate sin4 xdx.

ng
Z Z  2
Solution. sin4 xdx = sin2 x dx
Z !2
1 − cos 2x

eri
= dx.
2 1 + cos 2θ
Z cos2 θ =
1 2 2
= (1 + cos 2x − 2 cos 2x)dx 1 + cos 4x
4 cos2 2x = .
=
1
Z
1+ e
1 + cos 4x
!
− 2 cos 2x dx
2
gin
4 2
Z
1 1 1
= (1 + + cos 4x − 2 cos 2x)dx
4 2 2
En

Z !
1 3 1
= + cos 4x − 2 cos 2x dx
4 2 2
" Z Z Z #
1 3 1
= dx + cos 4xdx − 2 cos 2xdx
arn

4 2 2
" #
1 3 1 sin 4x sin 2x 3 sin 4x sin 2x
= + −2 +c= x+ − + c.
4 2 2 4 2 8 32 4
Le

Z
Example 3.21. Evaluate sin2 4xdx.
Z Z !
2 1 − cos 8x
Solution. sin 4xdx = dx.
w.

2
2 sin2 x = 1 − cos 2x
Z !
1 1
= − cos 8x dx
2 2
ww

"Z Z # 2 sin2 4x = 1 − cos 8x


1
= dx − cos 8xdx 1 − cos 8x
2 sin2 4x =
" # 2
1 sin 8x x sin 8x
= x− +c= − + c.
2 8 2 16

Visit For More : www.LearnEngineering.in


Visit For More : www.LearnEngineering.in

Integral Calculus 169

x2
Z
Example 3.22. Evaluate dx.
(ax + b)3
t−b
Solution. Let ax + b = t ⇒ ax = t − b ⇒ x =
a
dt
adx = dt ⇒ dx = .
a
Z t − b 2
x2
Z
a dt

.in
dx =
(ax + b)3 t 3 a
2
Z t2 + b − 2b t

ng
1 a2 a dt
=
a t3
1 b2 1 2b 1
Z !
1
= + · − dt

eri
a t a2 t3 a t2
b2
"Z Z Z #
1 1 2b
= dt + 2 t−3 dt − t−2 dt
a t a a

=
1
"
b
log t + 2 ·
2
e
t −2

2b
×
t−1
#
+c
gin
a a (−2) a (−1)
b2 1 2b 1
" #
1
= log t − 2 · 2 + +c
a 2a t a t
b2
" #
En

1 1 2b 1
= log(ax + b) − 2 · + +c
a a (ax + b)2 a (ax + b)
1 b2 1 2b 1
= log(ax + b) − 3 · + +c
arn

a 2a (ax + b)2 a2 (ax + b)

R
3.3 Evaluation of integrals of the form f (g(x))g′ (x)dx.
Le

= g(x) du = g′ (x)dx.
Method. Let u Z Z
Now, f (g(x))g′ (x)dx = f (u)du.
w.

This can be evaluated by earlier methods.


✎ ☞
Worked Examples
✍ ✌
ww

Z p
Example 3.23. Evaluate 2x 1 + x2 dx.
Solution. Let 1 + x2 = u.
2xdx = du.

Visit For More : www.LearnEngineering.in


Visit For More : www.LearnEngineering.in

170 Engineering Mathematics - I

3

Z Z Z
p 1 u2 2 3
∴ 2
2x 1 + x dx = udu = u 2 du = 3 + c = (1 + x2 ) 2 + c.
2
3

Z p
Example 3.24. Evaluate x2 1 − 4x3 dx.
Solution. Let u = 1 − 4x3 .

.in
du = −4 × 3x2 dx.
du
x2 dx = −

ng
12 !
√ −du
Z p Z
2 3
∴ x 1 − 4x dx = u
12

eri
Z
1 1
=− u 2 du
12
3
1 u2
=− +c
12 32
e
gin
1 2 3 1 3
=− (1 − 4x3 ) 2 + c = − (1 − 4x2 ) 2 + c.
12 3 18
En

Z
1
Example 3.25. Evaluate x )(1 + e−x )
dx.
Z (1 + e Z
1 1
arn

Solution. x −x
dx = dx
(1 + e ) (1 + e ) 1
(1 + e x )(1 + x )
e
ex
Z
= dx
(1 + e x )(1 + e x )
Le

ex
Z
= x )2
dx 1 + ex = t
Z (1 + e
dt
e x dx = dt
w.

= 2
t
t−1
Z
1 1
= t−2 dt = +c=− +c=− + c.
−1 t 1 + ex
ww

Z
Example 3.26. Evaluate x3 cos(x4 + 2)dx.

Visit For More : www.LearnEngineering.in


Visit For More : www.LearnEngineering.in

Integral Calculus 171

Solution. Let x4 + 2 = t.

4x3 dx = dt
dt
x3 dx =
Z Z4
dt
∴ x3 cos(x4 + 2)dx = cos t
4

.in
sin(x4 + 2)
Z
1 1
= cos tdt = sin t + c = + c.
4 4 4

ng
eri
Z
x
Example 3.27. Evaluate √ dx.
1 − 4x2

e
gin
Solution. Let 1 − 4x2 = t.
En

−8xdx = dt
dt
xdx = −
arn

Z Z8 !
x 1 dt
∴ √ dx = √ −
1 − 4x2 t 8
Z
1 1
t− 2 dt
Le

=−
8
1
1 t2 1 √ 1p
=− 1 +c=− ×2 t+c=− 1 − 4x2 + c.
8 2 8 4
w.
ww

x3
Z
Example 3.28. Evaluate √ dx. [A.U. Dec. 2015]
4 + x2

Visit For More : www.LearnEngineering.in


Visit For More : www.LearnEngineering.in

172 Engineering Mathematics - I

Solution. Let 4 + x2 = t.

2xdx = dt
dt
xdx =
2
x3 x2
Z Z
∴ √ dx = √ xdx
4 + x2 4 + x2

.in
Z
t − 4 dt
= √
t 2
Z 
1 1 1

t 2 − 4t− 2 dt

ng
=
2
 3 1 
1  t 2 t 2 
=  3 − 4 1  + c
2 2

eri
2
"  3 #
1 2 p
= 4 + x2 2 − 8 4 + x2 + c.
2 3
Z p
e
1 + x2 x5 dx.
gin
Example 3.29. Evaluate

Solution. Let 1 + x2 = t.

2xdx = dt
En

dt
xdx =
2
arn

Z p Z p
5
∴ 2
1 + x x dx = 1 + x2 x4 · xdx
Z p
= 1 + x2 (x2 )2 · xdx
Le


Z
dt
= t(t − 1)2 ·
2
√ 2
Z
1
w.

= t(t + 1 − 2t)dt
2
Z 
1 5 1 3

= t 2 + t 2 − 2t 2 dt
2
ww

Z Z Z !
1 5 1 3
= t 2 dt + t 2 dt − 2 t 2 dt
2
 7 3 5 
1  t 2 t 2 t 2 
=  7 + 3 − 2 5  + c.
2 2 2 2

Visit For More : www.LearnEngineering.in


Visit For More : www.LearnEngineering.in

Integral Calculus 173

" #
1 2 2 27 2 2 23 4 2 25
= (1 + x ) + (1 + x ) − (1 + x ) + c
2 7 3 5
1 7 1 3 2 5
= (1 + x2 ) 2 + (1 + x2 ) 2 − (1 + x2 ) 2 + c.
7 3 5

ex
Z

.in
Example 3.30. Evaluate x dx.
e −1
2
x
Solution. Let e 2 = t.

ng
xdx
e2 · = dt
2
x x
ex
Z Z
e2 e2
∴ dx = dx

eri
x x
e2 − 1 e2 − 1
Z x
e2 x
= x · e 2 dx
Z e2 − 1
=
1+t
2dt
e
gin
t
Z !
1
=2 + 1 dt
t
"Z Z #
1
En

=2 dt + dt + c
t
= 2[log t + t] + c
arn

 x x 
= 2[log e 2 − 1) + e 2 − 1 ] + c.
Le

Z
1
Example 3.31. Evaluate dx.
Z Z 1 + tan x
1 1
Solution. dx = dx
w.

1 + tan x sin x
1+
Z cos x Z
cos x 1 2 cos x
= dx = dx
ww

sin x + cos x 2 sin x + cos x


Z
1 sin x + cos x + cos x − sin x
= dx
2 sin x + cos x
Z !
1 cos x − sin x
= 1+ dx
2 sin x + cos x

Visit For More : www.LearnEngineering.in


Visit For More : www.LearnEngineering.in

174 Engineering Mathematics - I

Z Z
1 1 cos x − sin x
= dx + dx
2 Z 2 sin x + cos x
1 1 dt
= x+ t = sin x + cos x
2 2 t
1 1 dt = (cos x − sin x)dx
= x + log(t) + c
2 2
x 1
= + log(sin x + cos x)) + c.
2 2

.in
Z
tan x
Example 3.32. Evaluate dx.
sec x + cos x
sin x

ng
Z Z
tan x cos x
Solution. dx = dx
sec x + cos x 1
+ cos x
Z cos x

eri
sin x
= 2
dx
Z 1 + cos x
−dt
= t = cos x
1 + t2
e
gin
= − tan−1 (t) + c dt = − sin xdx

= − tan−1 (cos x) + c −dt = sin xdx

Z4 √
En

Example 3.33. Evaluate 2x + 1dx.


0
Solution. Let 2x + 1 = t.
arn

dt
2dx = dt ⇒ dx = .
2
When x = 0, t = 1.
Le

When x = 4, t = 9.
Z4 √ Z9
√ dt
∴ 2x + 1dx = t
w.

2
0 1
Z9
1 1
ww

= t 2 dt
2
1
 3 9
1  t 2  1 2 3 1 1 26
=  3  = × [9 2 − 1] = [27 − 1] = × 26 = .
2 2 2 3 3 3 3
1

Visit For More : www.LearnEngineering.in


Visit For More : www.LearnEngineering.in

Integral Calculus 175

Z2
1
Example 3.34. Evaluate dx.
(3 − 5x)2
1
Solution. Let 3 − 5x = t.
dt
−5dx = dt ⇒ dx = − .
5
When x = 0, t = −2.

.in
When x = 2, t = −7.
Z2 Z−7 !
1 1 dt
∴ = −

ng
(3 − 5x)2 t2 5
1 −2
Z−7
1
=− t−2 dt

eri
5
−2
!−7
1 t−1
=−
5 −1 −2
e
gin
!−7 ! " #
1 1 1 −1 1 1 −2 + 7 1 5 1
= = + = = · = .
5 t −2 5 7 2 5 14 5 14 14

Ze
log x
En

Example 3.35. Evaluate dx.


x
1
Solution. Let t = log x.
arn

1
dt = dx.
x
When x = 1, t = log 1 = 0.
When x = e, t = log e = 1.
Le

Ze Z1 !1
log x t2 1 1
∴ dx = tdt = = −0= .
x 2 0 2 2
w.

1 0

Integrals of symmetric functions


ww

Theorem. Suppose f is continuous on [−a, a]


Ra Ra
(i) If f is even, then f (x)dx = 2 f (x)dx.
−a 0
Ra
(ii) If f is odd, then f (x)dx = 0.
−a

Visit For More : www.LearnEngineering.in


Visit For More : www.LearnEngineering.in

176 Engineering Mathematics - I

Za Z0 Za
Proof. We have f (x)dx = f (x)dx + f (x)dx
−a −a 0
Z−a Za
=− f (x)dx + f (x)dx.
0 0
In the first integral let us make the substitution

.in
t = −x.

dt = −dx.

ng
When x = 0, t = 0.

When x = −a, t = a.

eri
Za Za Za
∴ f (x)dx = − f (−t)(−dt) + f (x)dx.
−a 0 0

=
Za
f (−t)dt +
Za
ef (x)dx.
gin
0 0
Za Za
= f (−x)dx + f (x)dx.
En

0 0
(i) If f is even, then f (−x) = f (x).
Za Za Za Za
∴ f (x)dx = f (x)dx + f (x)dx. = 2 f (x)dx.
arn

−a 0 0 0
(ii) If f is odd then f (−x) = − f (x)
Za Za Za
∴ f (x)dx = − f (x)dx + f (x)dx = 0.
Le

−a 0 0

Z2
w.

Example 3.36. Evaluate (x6 + 1)dx.


−2
ww

Solution. Let f (x) = x6 + 1.

f (−x) = (−x)6 + 1 = x6 + 1.

∴ f(x) is even.

Visit For More : www.LearnEngineering.in


Visit For More : www.LearnEngineering.in

Integral Calculus 177

By the properties of definite integrals


Z2 Z2
∴ f (x)dx = 2 f (x)dx
−2 0
Z2
=2 (x6 + 1)dx

.in
0
 2
Z2 

Z
= 2  x6 dx + dx
 

ng
 
0 0
 x7 2
 !  " 7 #

2  2
= 2   + (x)0  = 2 −0+2−0
7 0 7

eri
" # " #
128 128 + 14 2 × 142 284
=2 +2 =2 = = .
7 7 7 7

Example 3.37. Evaluate


Z1
tan x
dx.e
gin
1 + x2 + x4
−1
tan x
Solution. Let f (x) = .
1 + x2 + x4
tan(−x)
En

f (−x) =
1 + (−x)2 + (−x)4
− tan x
= = − f (x).
1 + x2 + x4
arn

∴ f (x) is odd.
By the properties of definite integrals
Z1
f (x)dx = 0
Le

−1
Z1
tan x
i.e, dx = 0.
w.

1 + x2 + x4
−1
ww

3.4 Integration of rational functions by Partial Fractions.

Techniques of resolving a given fraction into partial fractions.


p(x)
A rational function is generally of the form where p(x) and q(x) are
q(x)

Visit For More : www.LearnEngineering.in


Visit For More : www.LearnEngineering.in

178 Engineering Mathematics - I

algebraic expressions.
If the degree of p(x) is less than the degree of q(x) then the fraction
p(x)
is called a proper fraction.
q(x)
p(x)
If the degree of p(x) ≥ degree of q(x), then the given fraction is
q(x)
improper.

.in
p(x)
While resolving a fraction into partial fractions, the following
q(x)
technique can be adopted.

ng
p(x)
Case (i) Let be proper and if q(x) is expressed as linear factors.
q(x)
1+x
For example, if the given fraction is , then we can express

eri
(x − 1)(x + 2)(x − 3)
the fraction in the following way
1+x A B C
= + + .

e
(x − 1)(x + 2)(x − 3) x − 1 x + 2 x − 3
gin
p(x)
Case (ii) Let be proper and q(x) contains repeated linear factors.
q(x)
x2 + 2x − 3
Consider the following example .This must be written as
(x − 1)3 (2x + 3)
En

x2 + 2x − 3 A B C D
3
= + 2
+ 3
+ .
(x − 1) (2x + 3) x − 1 (x − 1) (x − 1) 2x +3
p(x)
arn

Case (iii) Let be proper and q(x) contains nonfactorisable second degree
q(x)
factors.
x2 + 3x + 2
Consider the following example . This can be resolved
(x − 1)2 (x2 + 4)(x2 + 9)
Le

into partial fractions as follows.


x2 + 3x + 2 A B Cx + D Ex + F
2 2 2
= + 2
+ 2 + 2 .
(x − 1) (x + 4)(x + 9) x − 1 (x − 1) x +4 x +9
w.

Methodology for improper fractions.


Case (i) If degree of p(x) = degree of q(x).
ww

x2 + x + 1
Example 1. Consider .
(x − 1)(x − 2)
This must be resolved as
x2 + x + 1 B C
= A+ + .
(x − 1)(x − 2) x−1 x−2

Visit For More : www.LearnEngineering.in


Visit For More : www.LearnEngineering.in

Integral Calculus 179

"
p(x)
= A constant + the usual methods adopted for proper fractions. ]
q(x)
x3 + 2x + 4 B Cx + D
Example 2. 2
= A+ + 2 .
(x − 1)(x + 4) x − 1 x +9
Case (ii) If the degree of p(x) is one more than that of q(x), then
p(x)
= A First degree expression + usual method adopted in proper functions.
q(x)
x4 + x + 4 C Dx + E

.in
Example 1. 2
= Ax + B + + 2 .
(x − 1)(x + 2) x−1 x +2
x3 + 2x + 3 C D
Example 2. 2
= Ax + B + + .
(x + 2) x + 2 (x + 2)2

ng
Case (iii) If the degree of p(x) is 2 more than that of q(x), then
p(x)
= A second degree expression + usual method adopted in proper functions.
q(x)

eri
x7 + 2x + 4 D E F Gx + H
Example 1. 2 2
= Ax2 + Bx + C + + + 2
+ 2 .
(x + 2)(x − 1) (x + 1) x + 2 x − 1 (x − 1) x +1
This procedure can be extended for any improper fraction depending on the nature
of the fraction
p(x)
q(x)
.
e
gin
Standard results.
R 1
1. Evaluate dx.
x2
+ a2
En

Solution. Let x = a tan θ.

dx = a sec2 θdθ.
arn

Z Z
1 1
∴ dx = · a sec2 θdθ
x2 + a2 Z a2 tan2 θ + a2
1
= · a sec2 θdθ
a2 (tan2 θ + 1)
Le

Z
1 1
= 2
· sec2 dθ
a
Z sec θ
1
= dθ
w.

a
1 1 −1  x 
= θ + c = tan + c.
a a a
ww

R 1
2. Evaluate dx.
x − a2
2

1 1 A B
Solution. Let = = +
x2 −a2 (x − a)(x + a) x − a x + a

Visit For More : www.LearnEngineering.in


Visit For More : www.LearnEngineering.in

180 Engineering Mathematics - I

A(x + a) + B(x − a)
=
(x − a)(x + a)
∴ 1 = A(x + a) + B(x − a).

Put x = a.
1
1 = A · 2a ⇒ A = .
2a

.in
Put x = −a.
1
1 = B · (−2a) ⇒ B = − .
2a "
1 −1

ng
#
1 2a 2a 1 1 1
∴ 2 = + = −
x − a2 x − a x + a 2a x − a x + a
Z Z !
1 1 1 1

eri
∴ dx = − dx
x2 − a2 2a x − a x + a
Z Z !
1 1 1
= dx − dx
2a x−a x+a
=
1  
e
log(x − a) − log(x + a) + c =
1
log
 x − a
+ c.
gin
2a 2a x+a
R 1
3. Evaluate dx.
a2
− x2
1 1 A B A(a + x) + B(a − x)
En

Solution. Let 2 2
= = + =
a −x (a − x)(a + x) a − x a + x (a − x)(a + x)
∴ 1 = A(a + x) + B(a − x).
Put x = a.
arn

1
1 = A · 2a ⇒ A = .
2a
Put x = −a.
1
Le

1 = B · (2a) ⇒ B = .
2a
1 1 " #
1 2a 2a 1 1 1
w.

∴ 2 = + = +
a − x2 a − x a + x 2a a − x a + x
Z Z !
1 1 1 1
∴ dx = + dx
a2 − x2 2a a − x a + x
ww

Z Z !
1 1 1
= dx + dx
2a a−x a+x
" #
1 log(a − x)
= + log(a + x) + c
2a −1

Visit For More : www.LearnEngineering.in


Visit For More : www.LearnEngineering.in

Integral Calculus 181

1  
log(a + x) − log(a − x) + c
=
2a
1 a + x
= log + c.
2a a−x
Z
1
Type 1. Evaluation of integrals of the form 2
dx
ax + bx + c
Method. Express the denominator to any one of the forms x2 − a2 , x2 + a2 or a2 − x2

.in
and then apply the correct formula.
✎ ☞
Worked Examples
✍ ✌

ng
Z
1
Example 3.38. Evaluate 2
dx.
Z xZ + 8x − 7
1 1
Solution. dx = dx

eri
2
x + 8x − 7 2
(x + 4) − 7 − 16
Z
1
= dx
(x + 4)2 − 23
=
Z

e
1
√ dx
gin
(x + 4) − ( 23)2
2
 √ 
1  x + 4 − 23 
= √ log  √  + c.
2 23 x + 4 + 23
Z
1
En

Example 3.39. Evaluate 2


dx.
Z Z1 + x − x
1 1
Solution. 2
dx = dx
1+x−x 1 − (x2 − x)
arn

Z
1
=   dx
1 − (x − 21 )2 − 41
Z
1
= dx
Le

1 − (x − 21 )2 + 14
Z
1
= 5 1 2
dx
4 − (x − 2 )
w.

Z
1
= √ dx
5 2
( 2 ) − (x − 21 )2
 √5
ww


1
1 
2 + x − 2 
= √ log  √  + c
2 25 2
5
− x + 1
2
√ 
1  5 + 2x − 1 
= √ log  √  + c.
5 5 − 2x + 1

Visit For More : www.LearnEngineering.in


Visit For More : www.LearnEngineering.in

182 Engineering Mathematics - I

Z
1
Example 3.40. Evaluate 2
dx.
Z 2x
Z −x+5
1 1
Solution. dx = ! dx
2x2 − x + 5 x 5
2 x2 − +
2 2
Z
1 1
= 2
dx
2 1
!
5 1
x− + −

.in
4 2 16
Z
1 1
= !2 dx
2 1 40 − 1

ng
x− +
4 16
Z
1 1
= !2 dx
2

eri
1 39
x− +
4 16
Z
1 1
=  √ 2 dx
2
x−
1
!
e2
 39 
+ 
gin

4 4
 
1 1  x − 1  2 4x − 1
!
−1  4 −1
= √ tan  √  + c = √ tan √ +c
2 39 39 39 39
4 4
En

Type 2. Evaluation of integrals using partial fractions.


arn

Z
x+5
Example 3.41. Evaluate dx.
x2 + x − 2
Le

x+5 x+5 A B
Solution. Let 2 = = + .
x +x−2 (x + 2)(x − 1) x + 2 x − 1
A(x − 1) + B(x + 2)
=
w.

(x + 2)(x − 1)
∴ x + 5 = A(x − 1) + B(x + 2).
ww

Put x = 1.
6 = 3B ⇒ B = 2.
Put x = −2.
3 = −3A ⇒ A = −1.

Visit For More : www.LearnEngineering.in


Visit For More : www.LearnEngineering.in

Integral Calculus 183

x+5 1 2
∴ =− +
Z x2
+ x − 2 Zx + 2 x − 1 !
x+5 1 2
∴ dx = − + dx
x2 + x − 2 x+2 x−1
Z Z
1 1
=− dx + 2 dx = − log(x + 2) + 2 log(x − 1) + c.
x+2 x−1

.in
x2 + 2x − 1
Z
Example 3.42. Evaluate dx.
2x3 + 3x2 − 2x
x2 + 2x − 1 x2 + 2x − 1 x2 + 2x − 1

ng
Solution. = = .
2x3 + 3x2 − 2x x(2x2 + 3x − 2) x(x + 2)(2x − 1)
x2 + 2x − 1 A B C
Let = + +
x(x + 2)(2x − 1) x x + 2 2x − 1

eri
A(x + 2)(2x − 1) + Bx(2x − 1) + Cx(x + 2)
=
x(x + 2)(2x − 1)

e
∴ x2 + 2x − 1 = A(x + 2)(2x − 1) + Bx(2x − 1) + Cx(x + 2).
Put x = 0.
gin
1
−2A = −1 ⇒ A = .
2
Put x = −2.
−1
En

B(−2)(−5) = 4 − 4 − 1 ⇒ 10B = −1 ⇒ B = .
10
1
Put x = .
2
1 5 1 5C 1 1
arn

C· · = +1−1⇒ = ⇒C= .
2 2 4 4 4 5
1 1 1
x2 + 2x − 1
∴ 3 = 2 − 10 + 5 .
2x + 3x2 − 2x x x + 2 2x − 1
Le

Z 2
x + 2x − 1 1
Z
1 1
Z
1 1
Z
1
∴ dx = dx − dx + dx
2x3 + 3x2 − 2x 2 x 10 x+2 5 2x − 1
1 1 1 log(2x − 1)
w.

= log x − log(x + 2) + · +c
2 10 5 2
1 1 1
= log x − log(x + 2) + · log(2x − 1) + c.
2 10 10
ww

sec2 x
Z
Example 3.43. Evalutate dx. [A.U. Dec. 2015]
tan2 x + 3 tan x + 2
Solution. Let tan x = t.

Visit For More : www.LearnEngineering.in


Visit For More : www.LearnEngineering.in

184 Engineering Mathematics - I

sec2 xdx = dt.

sec2 x
Z Z
1
∴ 2
dx = 2
dt
tan x + 3 tan x + 2 Z t + 3t + 2
1
= dt

.in
(t + 1)(t + 2)
Z
(t + 2) − (t + 1)
= dt
(t + 1)(t + 2)

ng
Z !
1 1
= − dt
(t + 1) t + 2
Z Z
1 1
= dt − dt + c

eri
(t + 1) t+2
= log(t + 1) − log(t + 2) + c
!
= log
e t+1
t+2
+c
gin
!
tan x + 1
= log + c.
tan x + 2
En

Z
3x + 1
Example 3.44. Evaluate dx.
arn

(x − 1)2 (x + 3)
3x + 1 A B C
Solution. Let 2
= + 2
+
(x − 1) (x + 3) x − 1 (x − 1) x+3
A(x − 1)(x + 3) + B(x + 3) + C(x − 1)2
Le

=
(x − 1)2 (x + 3)
∴ 3x + 1 = A(x − 1)(x + 3) + B(x + 3) + C(x − 1)2 .
w.

Put x = 1.
4B = 4 ⇒ B = 1.
Put x = −3.
ww

−1
16C = −8 ⇒ C = .
2
Equating the coefficients of x2 on both sides we get,
1
A + C = 0 ⇒ A = −C = .
2

Visit For More : www.LearnEngineering.in


Visit For More : www.LearnEngineering.in

Integral Calculus 185

1 1
3x + 1 1
∴ = 2 + − 2
(x − 1)2 (x + 3) x − 1 (x − 1)2 x + 3
Z Z Z Z
3x + 1 1 1 1 1 1
∴ dx = dx + dx − dx
(x − 1)2 (x + 3) 2 x−1 (x − 1)2 2 x+3
Z
1 1
= log(x − 1) + (x − 1)−2 dx − · log(x + 3) + c
2 2

.in
1  (x − 1)−1
= log(x − 1) − log(x + 3) + +c
2 ! −1
1 x−1 1
= log − + c.

ng
2 x+3 x−1

2x2 − x + 4

eri
Z
Example 3.45. Evaluate dx.
x3 + 4x
2
2x − x + 4 2x − x + 4 A Bx + C A(x2 + 4) + (Bx + C)x
2
Solution. Let 3 = = + 2 = .
x(x2 + 4) x(x2 + 4)
x + 4x x x +4
e
∴ 2x2 − x + 4 = A(x2 + 4) + (Bx + C)x.
gin
Put x = 0.
4A = 4 ⇒ A = 1.
En

Equating the coefficients of x2 on both sides we get,


A + B = 2 ⇒ 1 + B = 2 ⇒ B = 1.
Equating the coefficients of x on both sides we get,
arn

C = −1.
2x2 − x + 4 1 x−1 1 x 1
∴ 3
= + 2 = + 2 − 2 .
x + 4x x x + 4Z x x + 4 Z x + 4
2x2 − x + 4 In the second integral
Z Z
1 x 1
Le

∴ 3
= dx + 2
dx − 2
dx
x + 4x x Z x +Z4 x +4 let x2 + 4 = t
1 dt 1
= log x + − dx
t 2 x + 22
2 2xdx = dt
w.

1 1  x 
dt
= log x + log t − tan−1 +c xdx = .
2 2 2 2
1 1 −1  x 
ww

2
= log x + log(x + 4) − tan + c.
2 2 2

Z
2
Example 3.46. Evaluate dx.
(1 − x)(1 + x2 )

Visit For More : www.LearnEngineering.in


Visit For More : www.LearnEngineering.in

186 Engineering Mathematics - I

2 A Bx + C
Solution. Let 2
= +
(1 − x)(1 + x ) 1 − x 1 + x2
A(1 + x2 ) + (Bx + C)(1 − x)
= .
(1 − x)(1 + x2 )
∴ 2 = A(1 + x2 ) + (Bx + C)(1 − x).
Put x = 1.

.in
2A = 2 ⇒ A = 1.
Equating the coefficients of x2 on both sides we get,
A − B = 0 ⇒ A = B = 1.

ng
Equating the coefficients of x on both sides we get,
B−C = 0

eri
B = C ⇒ C = 1.
2 1 x+1 1 x 1
∴ = + = + +
(1Z− x)(1 + x2 ) 1 − x Z1 + x2 1 −Zx 1 + x2 Z 1 + x2

2
dx =
1
dx + e x
dx +
1
dx
gin
2
(1 − x)(1 + x ) 1−x 2 2 In the second integral
Z x +1 x +1
log(1 − x) 1 dt
= + + tan−1 x + c let 1 + x2 = t
−1 t 2
1 2xdx = dt
= − log(1 − x) + log t + tan−1 x + c
En

2 dt
1 xdx = .
= − log(1 − x) + log(1 + x2 ) + tan−1 x + c. 2
2
arn

Z
10
Example 3.47. Evaluate using partial fractions dx.
(x − 1)(x2 + 9)
Le

[A.U. Dec. 2015]


10 A Bx + C A(x2 + 9) + (Bx + C)(x − 1)
Solution. Let = + 2 = .
(x − 1)(x2 + 9) x − 1 x +9 (x − 1)(x2 + 9)
w.

∴ 10 = A(x2 + 9) + (Bx + C)(x − 1).


Put x = 1.
ww

10A = 10 ⇒ A = 1.
Equating the coefficients of x2 on both sides we get,
A + B = 0 ⇒ B = −A = −1.
Equating the constants on both sides we get,

Visit For More : www.LearnEngineering.in


Visit For More : www.LearnEngineering.in

Integral Calculus 187

9A − C = 10 ⇒ C = 9A − 10 = 9 − 10 = −1.

10 1 −x − 1
∴ 2
= + 2
(x − 1)(x + 9) x − 1 x + 9
1 x 1
= − 2 − 2 .
Zx − 1 x + 9Z x + 9

.in
Z Z
10 1 x 1
∴ 2
dx = dx − 2
dx − 2
dx
(x − 1)(x + 9) x−1 x +9 x +9
Z
1 2x 1 −1  x 
= log(x − 1) − dx − tan +c

ng
2 x2 + 9 3 3
1 1  x
= log(x − 1) − log(x2 + 9) − tan−1 +c
2 3 3

eri
p 1  x 
= log(x − 1) − log( x2 + 9) − tan−1 +c
! 3 3
(x − 1) 1  x
= log √ − tan−1 + c.
e
x2 + 9 3 3
gin
4x2 − 3x + 2
Z
Example 3.48. Evaluate dx.
En

4x2 − 4x + 3
4x2 − 3x + 2
Solution. The integrand 2 is an improper fraction with the degrees
4x − 4x + 3
of the Nr and Dr same.
arn

∴By the method of partial fractions we have


4x2 − 3x + 2 Bx + C
2
= A+ 2 [∵4x3 − 4x + 3 is a nonfactorizable
4x − 4x + 3 4x − 4x + 3
A(4x2 − 4x + 3) + Bx + C
Le

= second degree factor]


4x2 − 4x + 3
∴ 4x2 − 3x + 2 = A(4x2 − 4x + 3) + Bx + C.
w.

Equating the coefficients of x2 .


4A = 4 ⇒ A = 1.
ww

Equating the coefficients of x


−4A + B = −3 ⇒ −4 + B = −3 ⇒ B = 4 − 3 = 1.
Equating the constants
3A + C = 2 ⇒ 3 + C = 2 ⇒ C = 2 − 3 = −1.

Visit For More : www.LearnEngineering.in


Visit For More : www.LearnEngineering.in

188 Engineering Mathematics - I

4x2 − 3x + 2 x−1
∴ 2
=1+ 2
4x − 4x + 3 4x − 4x + 3
x−1
=1+
4(x2 − x + 3/4)
x−1 x−1
=1+   =1+  
1 2 3 1
4 (x − 2 ) + 4 − 4 4 (x − 21 )2 + 21

.in
Z  
4x2 − 3x + 2
Z
 x−1 
∴ dx = 1 +    dx
4x2 − 4x + 3 4 (x − 21 )2 + 21
Z Z
1 x−1

ng
= dx + dx
4 (x − 12 )2 + 21
x − 12 − 21
Z
1

eri
= x+ dx
4 (x − 12 )2 + 21
x − 12
Z Z
1 1 1
= x+ 1 1
dx − dx.
1
(x − 2 )2 + 12
e4
1
Z
(x − 2 )2 + 2
1 dt 1
Z
8
1
gin
= x+ 1
− dx
4 t+ 2 2 8 (x − 2 )2 + ( √1 )2
1
2

In the second integral


En

!2
1
Put x − =t
2
arn

!
1
2 x − dx = dt
2
!
1 dt
x − dx = .
2 2
Le

 
1 1 1 1  x − 1 
2
= x+ log(t + ) − 1 tan−1  1  + c
8 2 8 √ √
w.

2 2
! √
2 −1 2x − 1 √
!
1 1 2 1
= x+ log (x − ) + − tan × 2 +c
8 2 2 8 2
ww

! !
1 2 1 1 1 −1 2x − 1
= x+ log x + − x + − √ tan √ +c
8 4 2 4 2 2
4x2 − 4x + 3
! !
1 1 2x − 1
= x+ log − √ tan−1 √ + c.
8 4 4 2 2

Visit For More : www.LearnEngineering.in


Visit For More : www.LearnEngineering.in

Integral Calculus 189

x4 − 2x2 + 4x + 1
Z
Example 3.49. Evaluate dx. [A.U Nov 2016].
x3 − x2 − x + 1
Solution. The integrand is an improper fraction in which the degree of the Nr is
1 more than that of the Dr. Hence, by the method of partial fractions we have
x4 − 2x2 + 4x + 1 x4 − 2x2 + 4x + 1 x4 − 2x2 + 4x + 1
= =
x3 − x2 − x + 1 x2 (x − 1) − (x − 1) (x − 1)(x2 − 1)
x4 − 2x2 + 4x + 1 x4 − 2x2 + 4x + 1

.in
= = .
(x − 1)(x − 1)(x + 1) (x − 1)2 (x + 1)
x4 − 2x2 + 4x + 1 C D E
Let 2
= Ax + B + + 2
+
(x − 1) (x + 1) x − 1 (x − 1) x+1

ng
(Ax + B)(x − 1)2 (x + 1) + C(x − 1)(x + 1) + D(x + 1) + E(x − 1)2
=
(x − 1)2 (x + 1)

eri
∴ x4 − 2x2 + 4x + 1 = (Ax + B)(x − 1)2 (x + 1) + C(x − 1)(x + 1) + D(x + 1) + E(x − 1)2 .
Put x = 1 : 2D = 1 − 2 + 4 + 1 = 4 ⇒ D = 2.
Put x = −1 : 4E = 1 − 2 + 4 + 1 = 4 ⇒ 4E = −4 ⇒ E = −1.

e
Equating the coefficients of x4 on both sides we get,
gin
A = 1.
Put x = 0 : B − C + D + E = 1
B − C + 2 − 1 = 1.
En

B − C = 0. (1)
Put x = 2 : (2A + B) · 3 + 3C + 3D + E = 16 − 8 + 8 + 1
arn

6A + 3B + 3C + 3D + E = 17.
6 + 3B + 3C + 6 − 1 = 17
3B + 3C = 17 − 11 = 6
Le

B + C = 2. (2)
(1) + (2) ⇒ 2B = 2 ⇒ B = 1.
w.

(1) ⇒ B = C = 1.
ww

Visit For More : www.LearnEngineering.in


Visit For More : www.LearnEngineering.in

190 Engineering Mathematics - I

x4 − 2x2 + 4x + 1 1 2 1
∴ 3 2
= x+1+ + 2

x −x −x+1 x − 1 (x − 1) x+1
Z 4
x − 2x2 + 4x + 1
Z Z Z Z Z
1 1 1
∴ 3 2
dx = xdx + dx + dx + 2 2
dx − dx
x −x −x+1 x−1 (x − 1) x+1
x2
Z
= + x + log(x − 1) + 2 (x − 1)−2 dx − log(x + 1) + c
2
x2 (x − 1)−1

.in
= + x + log(x − 1) + 2 − log(x + 1) + c
2 −1
x2
!
x−1 2
= + x + log − + c.
2 x+1 x+1

ng
x3
Z
Example 3.50. Evaluate dx.
(x − 1)(x − 2)

eri
Solution. The integrand is an improper fraction in which the degree of the Nr is
1 more than that of the Dr.

x3 C e
∴ By the method of partial fractions we have
D
gin
= Ax + B + +
(x − 1)(x − 2) x−1 x−2
(Ax + B)(x − 1)(x − 2) + C(x − 2) + D(x − 1)
=
(x − 1)(x − 2)
En

∴ x3 = (Ax + B)(x − 1)(x − 2) + C(x − 2) + D(x − 1).


Put x = 1 : −C = 1 ⇒ C = −1.
Put x = 2 : D = 8.
arn

Equating the coefficients of x3 on both sides we get,


A = 1.
Put x = 0 : 2B − 2C − D = 0
Le

2B + 2 − 8 = 0.
2B − 6 = 0
w.

2B = 6 ⇒ B = 3.
x3 1 8
∴ = x+3− +
(x − 1)(x − 2) x−1 x−2
ww

Z
x 3 Z Z Z
1
Z
1
∴ dx = xdx + 3 dx − dx + 8 dx
(x − 1)(x − 2) x−1 x−2
x2
= + 3x − log(x − 1) + 8 log(x − 2) + c.
2

Visit For More : www.LearnEngineering.in


Visit For More : www.LearnEngineering.in

Integral Calculus 191

x3 + x
Z
Example 3.51. Evaluate dx.
x−1
Solution. The integrand is an improper fraction in which the degree of the Nr is
2 more than that of the Dr.
∴ By the method of partial fractions we have
x3 + x D (Ax2 + Bx + C)(x − 1) + D
= Ax2 + Bx + C + = .
x−1 x−1 x−1

.in
∴ x3 + x = (Ax2 + Bx + C)(x − 1) + D.
Put x = 1 : D = 2.

ng
Equating the coefficients of x3 on both sides we get,
A = 1.

eri
Equating the coefficients of x2 on both sides we get,
−A + B = 0.
B = A = 1.
Put x = 0 : −C + D = 0 e
gin
C = D = 2.

x3 + x 2
En

∴ = x2 + x + 2 + .
x−1 x−1
Z 3 Z Z Z Z
x +x 1
∴ dx = x2 dx + xdx + 2 dx + 2 dx
x−1 x−1
arn

x3 x2
= + + 2x + 2 log(x − 1) + c.
3 2
Le

3.5 Integration of irrational functions


w.

Standard Results
ww

Z
1
1. Evaluate √ dx.
a2 − x2
Solution. Let x = a sin θ ⇒ dx = a cos θdθ.

Visit For More : www.LearnEngineering.in


Visit For More : www.LearnEngineering.in

192 Engineering Mathematics - I

Z Z
1 1
∴ √ dx = p · a cos θdθ
a2 − x2 a 2 − a2 sin2 θ
Z
1
= q · a cos θdθ
2 2
a (1 − sin θ)
Z
1
= √ · a cos θdθ
a 2 cos2 θ

.in
Z Z
1
= · a cos θdθ = dθ + c = θ + c
a cos θ
Z
1  x
√ dx = sin−1 + c.

ng
a2 − x2 a

eri
Z
1
2. Evaluate √dx.
+ x2 a2
Solution. Let x = a sin hθ
e
gin
dx = a cos hθdθ.
En

Z Z
1 1
∴ √ dx = √ · a cos hθ dθ
2
a +x 2 a + a2 sin h2 θ
2
arn

Z
1
= p · a cos hθ dθ
a2 (1 + sin h2 θ)
Z
1
= √ · a cos hθ dθ
a 2 cos h2 θ
Le

Z Z
1
= · a cos hθ dθ = dθ + c = θ + c
a cos hθ
Z
1 −1 x
  p
w.

√ dx = sin h + c = log(x + x2 + a2 ) + c.
a2 − x2 a
ww

Z
1
3. Evaluate √ dx.
x2 − a2

Visit For More : www.LearnEngineering.in


Visit For More : www.LearnEngineering.in

Integral Calculus 193

Solution. Let x = a cos hθ

dx = a sin hθdθ.
Z Z
1 1
∴ √ dx = √ · a sin hθ dθ
2
x −a 2 a 2 cos h 2 θ − a2
Z
1
= p · a sin hθ dθ
a2 (cos h2 θ − 1)

.in
Z
1
= √ · a sin hθ dθ
Z a2 sin h2 θ
1

ng
= · a sin hθ dθ
a sin hθ
Z
= dθ + c = θ + c

eri
Z
1 −1 x
  p
√ dx = cos h + c = log(x + x2 − a2 ) + c.
a2 − x2 a

4. Evaluate
Z p
a2 − x2 dx. e
gin
Solution. Let x = a sin θ

dx = a cos θdθ.
En

Z p Z p
∴ 2 2
a − x dx = a2 − a2 sin2 θ · a cos θ dθ
arn

Z q
= a2 (1 − sin2 θ) · a cos θ dθ
Z p
= a2 cos2 θ · a cos θ dθ
Le

Z
= a2 cos2 θ dθ
Z
1 + cos 2θ
= a2
w.


2
a2
Z
= (1 + cos θ)dθ
2
ww

a2
" Z Z #
= dθ + cos 2θdθ
2
a2
" #
sin 2θ
= θ+ +c
2 2

Visit For More : www.LearnEngineering.in


Visit For More : www.LearnEngineering.in

194 Engineering Mathematics - I

a2
"  x #
−1 1
= sin + · 2 sin θ cos θ + c
2 a 2
a2  x  a2 x p
= sin−1 + · · cos2 θ + c
2 a 2 a
a2 −1 x
  ax p
= sin + 1 − sin2 θ + c
2 a 2
a2   ax r x2
−1 x

.in
= sin + 1− 2 +c
2 a 2 a
a2  x  ax √a2 − x2
= sin−1 + +c
2 a 2 a

ng
a2
Z p  x x p
a2 − x2 dx = sin−1 + a2 − x2 + c.
2 a 2

eri
Z p
5. Evaluate a2 + x2 dx.
Solution. Let x = a sin hθ
dx = a cos hθdθ.
e
gin
Z p Z p
∴ a2 + x2 dx = a2 + a2 sin h2 θ · a cos hθ dθ
Z p
= a2 (1 + sin h2 θ) · a cos hθ dθ
En

Z p
= a2 cos h2 θ · a cos hθ dθ
arn

Z
= a2 cos h2 θ dθ
Z
1 + cos h2θ
= a2 dθ
2
Le

a2
Z
= (1 + cos h2θ)dθ
2
a2
" Z Z #
w.

= dθ + cos h2θdθ
2
a2
" #
sin h2θ
= θ+ +c
ww

2 2
a2
" #
−1 x
  1
= sin h + · 2 sin hθ cos hθ + c
2 a 2
a 2  x  a2 x p
= sin h−1 + · · cos h2 θ + c
2 a 2 a

Visit For More : www.LearnEngineering.in


Visit For More : www.LearnEngineering.in

Integral Calculus 195

a2  x ax p
= sin h−1 + 1 + sin h2 θ + c
2 a 2
a2   ax r x2
−1 x
= sin h + 1+ 2 +c
2 a 2 a
a2  x  ax √a2 + x2
= sin h−1 + +c
2 a 2 a
a2  x x p
sin h−1

.in
= + a2 + x2 + c.
2 a 2
a2
Z p p xp 2
a2 + x2 dx = log(x + x2 + a2 ) + a + x2 + c.
2 2

ng
Z p
6. Evaluate x2 − a2 dx.

eri
Solution. Let x = a cos hθ
dx = a sin hθdθ.


Z p
2 2
x − a dx =
Z p
e
a2 cos h2 θ − a2 · a sin hθ dθ
gin
Z p
= a2 (sin h2 θ − 1) · a sin hθ dθ
Z p
a2 sin h2 θ · a sin hθ dθ
En

=
Z
2
=a sin h2 θ dθ
arn

Z
2 cos h2θ − 1
=a dθ
2
a2
Z
= (cos h2θ − 1)dθ
2
Le

a2
"Z Z #
= cos h2θdθ − dθ
2
a2 sin h2θ
" #
w.

= −θ +c
2 2
a2   x 
= sin hθ cos hθ − cos h−1 +c
ww

2 a
a2  p 2
x −1 x
 
= sin h θ · − cos h +c
2 a a
ax p 2 a2 −1 x
 
= cosh θ − 1 − cosh c
2 2 a

Visit For More : www.LearnEngineering.in


Visit For More : www.LearnEngineering.in

196 Engineering Mathematics - I

r
ax x2 a2 −1 x
 
= 2
− 1 − cosh c
2 a 2 a

ax x2 − a2 a2  x
= − cosh−1 c
2 a 2 a
xp 2 a2  x
= x − a2 − cosh−1 c.
2 2 a
a2
Z p
xp 2 p

.in
x2 − a2 dx = x − a2 − log(x + x2 − a2 ) + c.
2 2
Z
1

ng
3.5.1 Type I. Evaluation of integrals of the form √ dx
ax2 + bx + c
Evaluation procedure. By the method of completing the square technique,

eri
√ √ √ √
express ax2 + bx + c in any one of the forms a2 − x2 , x2 − a2 or a2 + x2 and
apply the correct formula.
✎ ☞
✍ e
Worked Examples

gin
Z
1
Example 3.52. Evaluate √ dx.
x2 + 16x + 100
En

Z Z
1 1
Solution. √ dx = p dx
2
x + 16x + 100 2
(x + 8) + 100 − 64
Z
1
= dx
arn

p
(x + 8)2 + 36
Z
1
= p dx
(x + 8)2 + 62
Le

 p 
= log (x + 8) + (x + 8)2 + 62 + c
 p 
= log (x + 8) + x2 + 16x + 100 + c.
w.

Z
1
Example 3.53. Evaluate √ dx.
9 + 8x − x2
ww

Z Z
1 1
Solution. √ dx = p dx
9 + 8x − x 2 9 − (x2 − 8x)
Z
1
= p dx
9 − (x − 4)2 − 16


Visit For More : www.LearnEngineering.in


Visit For More : www.LearnEngineering.in

Integral Calculus 197

Z
1
= p dx
9 − (x − 4)2 + 16
Z
1
= p dx
25 − (x − 4)2
Z
1
= p dx
52 − (x − 4)2
!
−1 x − 4

.in
= sin + c.
5
Z
1

ng
Example 3.54. Evaluate √ dx.
2
9x + 24x
Z Z
1 1

eri
Solution. √ dx = q   dx
9x2 + 24x 9 x2 + 24x 9
Z
1 1
=
3
q
2
x + 3 e
8x
dx
gin
Z
1 1
= q dx
3 4 2

16
x+ 3 − 9
En

Z
1 1
=
3
q 2  2 dx
x + 43 − 34
 s 
!2
arn

!
1  4 4 16 
= log  x + + x+ −  + c
3 3 3 9
 ! r 
1  3x + 4 8 
= log   + x2 + x + c.
Le

3 3 3

Z
ℓx + m
w.

3.5.2 Type II. Evaluation of integrals of the form √ dx


ax2 + bx + c
d  2 
ww

Evaluation procedure. Assume express ℓx + m = A · ax + bx + c + B. Find


dx
the values of A and B and then substitute for ℓx + m in the integral which will be
evaluated easily.
✎ ☞
Worked Examples
✍ ✌

Visit For More : www.LearnEngineering.in


Visit For More : www.LearnEngineering.in

198 Engineering Mathematics - I

Z
3x + 1
Example 3.55. Evaluate √ dx.
2x2 + x + 3
d
Solution. Let 3x + 1 = A (2x2 + x + 3) + B
dx
= A(4x + 1) + B.
Equating the coefficients of x on both sides we get,
3
4A = 3 ⇒ A = .

.in
4
Equating the constants
3
A+B=1⇒ +B=1
4

ng
3 1
B=1− = .
4 4

eri
3 1
∴ 3x + 1 = (4x + 1) + .
4 4
Z
3x + 1
3
Z (4x + 1) + 1
4 4 dx. e
gin
∴ 2
dx = √
2x + x + 3 2
Z 2x + x + 3 Z
3 4x + 1 1 1
= √ dx + √ dx.
4 2x 2+x+3 4 2x 2+x+3
En

Z Z
3 −1 1 1
= (2x2 + x + 3) 2 d(2x2 + x + 3) + s ! dx
4 4 x 3
2 x2 + +
2 2
arn

1
3 (2x2 + x + 3) 2
Z
1 1
= 1
+ √ s dx
4 4 2 !2
2 1 3 1
x+ + −
4 2 16
Le

Z
3 p
2
1 1
= × 2 2x + x + 3 + √ s dx
4 4 2 !2
1 24 − 1
x+ +
w.

4 16
Z
3 p 1 1
= 2x2 + x + 3 + √ dx
!2  √ 2
s
2 4 2
ww

1  23 
 
x+ + 
4 4

 s 
!2
3p 2 1  1 1 23 
= 2x + x + 3 + √ log  x + + x+ +  + c
2 4 2 4 4 16

Visit For More : www.LearnEngineering.in


Visit For More : www.LearnEngineering.in

Integral Calculus 199

 r 
3p 2 1  4x + 1 2
x 3 
= 2x + x + 3 + √ log  + x + +  + c.
2 4 2 4 2 2
Z
1
Example 3.56. Evaluate √ dx by using trignometric substitution. Hence
a2 − x2
6x + 5
use it in evaluating dx. [A.U Nov 2016]
6 + x − 2x2
Solution. For the first part, refer Result(1). Let us evaluate the second part.

.in
d
Let 6x + 5 = A (6 + x − 2x2 ) + B
dx

ng
= A(1 − 4x) + B.
Equating the coefficients of x on both sides we get
6 3
−4A = 6 ⇒ A = − = − .

eri
4 2
Equating the constants,we get
3
A+B=5⇒− +B=1
2
B=5+ = .
3 13
e
gin
2 2
3 13
∴ 6x + 5 = − (1 − 4x) + .
2 2
Z −3 13
(1 − 4x) +
Z
6x + 5
En

2 2
∴ √ dx = √ dx.
6 + x − 2x 2 6 + x − 2x 2
Z Z
3 1 − 4x 13 1
=− √ dx + √ dx.
2 2
arn

6 + x − 2x 2 6 + x −Z2x2
Z
3 −1 13 1
=− (6 + x − 2x2 ) 2 d(6 + x − 2x2 ) + q dx
2 2 2(3 + 2x − x2 )
1
Le

3 (6 + x − 2x2 ) 2
Z
13 1
=− 1
+ √ q dx
2 2 2 2 3 − (x2 − 2x )
Z
13 1
w.

p
= −3 6 + x − 2x2 + √ q o dx
2 2
n
1 2 1
3 − (x − 4 ) − 16
Z
13 1
ww

p
= −3 6 + x − 2x2 + √ q dx
2 2 3 − (x − 14 )2 + 16 1
Z
p
2
13 1
= −3 6 + x − 2x + √ q dx
2 2 ( 74 )2 − (x − 14 )2

Visit For More : www.LearnEngineering.in


Visit For More : www.LearnEngineering.in

200 Engineering Mathematics - I

 x − 14 
 
p
2
13 −1 
= −3 6 + x − 2x + √ sin  7  + c
2 2 4
!
p 13 −1 4x − 1
= −3 6 + x − 2x2 + √ sin + c.
2 2 7
Z
4x − 3
Example 3.57. Evaluate √ dx.
x2 + 2x − 1

.in
d
Solution. Let 4x − 3 = A (x2 + 2x − 1) + B
dx
= A(2x + 2) + B.

ng
Equating the coefficients of x on both sides we get,
2A = 4 ⇒ A = 2.

eri
Equating the constants
2A + B = −3 ⇒ 4 + B = −3 ⇒ B = −3 − 4 = −7.
∴ 4x − 3 = 2(2x + 2) − 7.
Z Z
e
gin
4x − 3 2(2x + 2) − 7
∴ √ dx = √ dx.
2
x + 2x − 1 2
Z x + 2x − 1 Z
2x + 2 1
=2 √ dx − 7 √ dx.
x 2 + 2x − 1 x 2 + 2x − 1
En

Z Z
−1 1
= 2 (x2 + 2x − 1) 2 d(x2 + 2x − 1) − 7 p dx
(x + 1)2 − 1 − 1
1
(x2 + 2x − 1) 2
Z
1
arn

=2 1
−7 p dx
2 (x + 1)2 − 2
Z
p
2
1
= 4 x + 2x − 1 − 7 q  √ 2 dx
2
(x + 1) − 2
Le

 r 
p   √ 2 
2 2
= 4 x + 2x − 1 − 7 log  x + 1 + (x + 1) − 2  + c

w.

p  p 
= 4 x2 + 2x − 1 − 7 log x + 1 + x2 + 2x − 1 + c.
Z
x
ww

Example 3.58. Evaluate √ dx.


3 − 2x − x2
d
Solution. Let x = A (3 − 2x − x2 ) + B
dx
= A(−2 − 2x) + B.

Visit For More : www.LearnEngineering.in


Visit For More : www.LearnEngineering.in

Integral Calculus 201

Equating the coefficients of x on both sides we get,


1
−2A = 1 ⇒ A = − .
2
Equating the constants
−2A + B = 0 ⇒ 1 + B = 0 ⇒ B = −1.

.in
∴ x = − (−2 − 2x) − 1.
2
Z Z − 1 (−2 − 2x) − 1
x 2
∴ √ dx = √ dx.

ng
3 − 2x − x 2 2
Z 3 − 2x − x Z
1 −2 − 2x 1
=− √ dx − √ dx.
2 3 − 2x − x 2 3 − 2x − x 2

eri
Z Z
1 −1 1
=− (3 − 2x − x2 ) 2 d(3 − 2x − x2 ) − p dx
2 3 − (x2 + 2x)
1 Z
=−

1 3 − 2x − x
2 2
− e 1
dx
gin
1
p
2 2 3 − (x + 1)2 − 1

Z
p
2
1
= − 3 − 2x − x − dx
3 − (x + 1)2 + 1
Z
1
En

p
= − 3 − 2x − x2 − p dx
4 − (x + 1)2
Z
p
2
1
= − 3 − 2x − x − p dx
arn

22 − (x + 1)2
!
−1 x + 1
p
2
= − 3 − 2x − x − sin + c.
2
Le

Z √
3.5.3 Type III. Evaluation of integrals of the form ax2 + bx + c dx
w.

Evaluation procedure. By completing the square form, express ax2 + bx + c as


any of x2 − a2 , a2 + x2 , a2 − x2 and apply the appropriate formula.
ww

✎ ☞
Worked Examples
✍ ✌

R √
Example 3.59. Evaluate x2 − 4x + 6 dx.

Visit For More : www.LearnEngineering.in


Visit For More : www.LearnEngineering.in

202 Engineering Mathematics - I

Z p Z p
Solution. x2 − 4x + 6 dx = (x − 2)2 + 6 − 4 dx
Z p
= (x − 2)2 + 2 dx
Z q √
= (x − 2)2 + ( 2)2 dx

!q
x−2
= (x − 2)2 + ( 2)2

.in
2

( 2)2 √
q !
+ 2 2
log (x − 2) + (x − 2) + ( 2) + c
2

ng
!
x−2 p 2  p
2

= x − 4x + 6 + log (x − 2) + x − 4x + 6 + c.
2

eri
3.5.4 Type IV.Integration by trigonometric substitution.

e
Certain integrals can be easily evaluated by trigonometric substitution.
gin
There are three cases.


(i) Any integral involving quantities of the form a2 − x2 can be evaluated by
En

the substitution x = a sin θ.


arn

(ii) Integrals involving functions of the form a2 + x2 can be evaluated by the


substitution x = a tan θ or x = a sin hθ.


Le

(iii) Integrals involving functions of the form x2 − a2 can be evaluated by the


substitution x = a sec θ or x = a cos hθ.
w.

✎ ☞
Worked Examples
✍ ✌
ww

Z √
9 − x2
Example 3.60. Evaluate dx.
√ x2 √
Solution. Since 9 − x2 is of the form a2 − x2 , we can evaluate this integral by
the substitution.

Visit For More : www.LearnEngineering.in


Visit For More : www.LearnEngineering.in

Integral Calculus 203

x = 3 sin θ. ⇒ dx = 3 cos θdθ.


Z √ Z p
9 − x2 9 − 9 sin2 θ
∴ dx = 3 cos θdθ
x2 9 sin2
θ
q
Z 9(1 − sin2 θ)
= 3 cos θdθ
√ 9 sin2 θ
cos2 θ

.in
Z
= 2
cos θdθ
Z sin θ
cos θ
= cos θdθ

ng
sin2 θ
cos2 θ
Z
= 2

Z sin θ

eri
= cot2 θdθ
Z
= (cosec2 θ − 1)dθ
Z
e Z
gin
= cosec2 θdθ − dθ
cos θ
= − cot θ − θ + c = − −θ+c
√ sin θ
En

cos2 θ −1 x
 
=− − sin +c
sin θ 3
p
1 − sin2 θ −1 x
 
=− − sin +c
arn

sin θ 3
r
x2
1−
−1 x
 
9
=− x − sin +c
3
Le

s3
3 9 − x2  x
=− − sin−1 +c
w.

x 9 3
1p  x
=− 9 − x2 − sin−1 + c.
x 3
ww

R 1
Example 3.61. Evaluate √ dx.
x2 x2 + 4 √
Solution. Since the integral involves one factor of the form x2 + 4,we can
evaluate this integral by the substitution

Visit For More : www.LearnEngineering.in


Visit For More : www.LearnEngineering.in

204 Engineering Mathematics - I

x = 2 tan θ ⇒ dx = 2 sec2 θdθ.


Z Z
1 1
∴ √ dx = √ · 2 sec2 θdθ
x2 x2 +4 4 tan2 θ 2
4 tan θ + 4
sec2 θ
Z
1
=
2
p  dθ
tan2 θ 4 tan2 θ + 1
sec2 θ
Z
1

.in
= √ dθ
4 tan2 θ sec2 θ
sec2 θ
Z
1
= dθ

ng
4
Z tan2 θ sec θ
1 sec θ
= dθ
4 tan2 θ

eri
cos2 θ
Z
1 1
= · dθ
4 cos θ sin2 θ
Z
1 cos θ 1
= · dθ
4
1
Z
e
sin θ sin θ
gin
= cosecθ · cot θ dθ
4
1
= − cosecθ + c
4
En

1p
=− cosec2 θ + c
4
1p
=− 1 + cot2 θ + c
4r
arn

1 4 x
=− 1+ 2 +c tan θ = , cot θ = 2/x
4 x 2
s √
1 x2 + 4 x2 + 4
Le

=− + c = − + c.
4 x2 4x

R x
Example 3.62. Evaluate dx. √
w.

+4 x2 √
Solution. Since the integrand involves one factor of the form x2 + 4, we can
evaluate this integral by the substitution
ww

x = 2 tan θ ⇒ dx = 2 sec2 θdθ.


Z Z
x 2 tan θ
∴ √ dx = √ · 2 sec2 θdθ
x2 + 4 4 tan2 θ + 4

Visit For More : www.LearnEngineering.in


Visit For More : www.LearnEngineering.in

Integral Calculus 205

tan θ sec2 θ
Z
=4 p dθ
4(tan2 θ + 1)
tan θ sec2 θ
Z
=4 √ dθ
2 sec2 θ
tan θ sec2 θ
Z
=2 dθ
sec θ
Z

.in
= 2 tan θ · sec θ dθ

= 2 sec θ + c

ng
p
= 2 sec2 θ + c
p
= 2 1 + tan2 θ + c

eri
r
x2
=2 1+ +c
4

=2
e
4 + x2
2
p
+ c = 4 + x2 + c.
gin
Note. This, integral can be evaluated easily by the substitution 4 + x2 = t.

x3 + 1
Z
En

Example 3.63. Evaluate √ dx.


1 − x2 √
Solution. Since the integrand contains one factor of the form a2 − x2 , let us
make the substitution
arn

x = sin θ [a = 1]
dx = cos θdθ
Le

x3 + 1 sin3 θ + 1
Z Z
√ dx = p cos θdθ
1 − x2 1 − sin2 θ
sin3 θ + 1
Z
w.

= √ · cos θdθ
cos2 θ
sin3 θ + 1
Z
= · cos θdθ
ww

cos θ
Z
= (sin3 θ + 1) dθ
Z Z
= sin3 θ dθ + dθ

Visit For More : www.LearnEngineering.in


Visit For More : www.LearnEngineering.in

206 Engineering Mathematics - I

Z !
3 1
= sin θ − sin 3θ dθ + θ
4 4
Z Z
3 1
= sin θdθ − sin 3θdθ + θ
4 4
!
3 1 − cos 3θ
= (− cos θ) − +θ+c
4 4 3
3p 2 1
=− cos θ + (4 cos3 θ − 3 cos θ) + sin−1 x + c

.in
4 12
3p 1 3 1p 2
=− 1 − sin2 θ + (cos2 θ) 2 − cos θ + sin−1 x + c
4 3 4

ng
3p 1 3 1p
=− 1 − x2 + (1 − sin2 θ) 2 − 1 − sin2 θ + sin−1 x + c
4 3 4
3p 1 3 1p
=− 1 − x2 + (1 − x2 ) 2 − 1 − x2 + sin x +c

eri
4 3 4
1 3
p
= (1 − x2 ) 2 − 1 − x2 + sin−1 x + c.
3

Example 3.64. Evaluate


Z

1
dx. e
gin
x3 x2 − 9 √
Solution. Since the integrand contains one factor of the form a2 − x2 , we can
make the substitution
En

x = 3 sec θ [∴ a = 3]
dx = 3 sec θ · tan θdθ
arn

Z Z
1 1
√ dx = √ · 3 sec θ tan θdθ
x3 x2 −9 Z33 sec3 θ · 9 sec2θ−9
1 1
= p sec θ tan θdθ
9
Le

sec3 θ · 9(sec2 θ − 1)
Z
1 1
= √ sec θ tan θdθ
9 3 2
Z sec θ × 3 tan θ
w.

1 1
= 3
sec θ tan θdθ
27 sec θ tan θ
Z
1 1
= dθ
ww

27 sec 2θ
Z
1
= cos2 θdθ
27
Z
1 1 + cos 2θ
= dθ
27 2

Visit For More : www.LearnEngineering.in


Visit For More : www.LearnEngineering.in

Integral Calculus 207

"Z Z #
1
= dθ + cos 2θdθ
54
" #
1 sin 2θ
= θ+ +c
54 2
1 1
= θ+ sin θ cos θ + c
54 54
1  x 1 p 1
= sec−1 + sin2 θ · +c

.in
54 3 54 sec θ
1  x  1 p 3
= sec−1 + 1 − cos2 θ · + c
54 3 54 x

ng
r
1  x  1 9 3
= sec−1 + 1− 2 · +c
54 3 54 x x

1  x  1 x −9 2
sec−1

eri
= + + c.
54 3 18 x2
Z
1
Example 3.65. Evaluate 3
(a2 + x2 ) 2
dx.
√ e
gin
Solution. Since the integrand contains a factor of the form a2 + x2 , we can make
the substitution
x = a tan θ ⇒ dx = a sec2 θdθ.
En

Z Z
1 1
∴ 3
dx = 3
· a sec2 θdθ
(a2 + x2 ) 2 (a2 + a2 tan2 θ) 2
arn

Z
1
= 3
· a sec2 θdθ
(a2 (1 + tan2 θ)) 2
Z
1
= · a sec2 θdθ
Le

3
(a2 sec2 θ) 2
Z
1
= · a sec2 θdθ
a3 sec3 θ
w.

Z
1 1
= 2

a Z sec θ
1
ww

= cos θ dθ
a2
1
= sin θ + c
a2
1 tan θ
= +c
a2 sec θ

Visit For More : www.LearnEngineering.in


Visit For More : www.LearnEngineering.in

208 Engineering Mathematics - I

1 x 1
= 2
· √ +c
a a sec2 θ
x 1
= 3 √ +c
a 1 + tan2 θ
x 1 x a x
= 3 q +c= 3 √ +c= √ + c.
a 2 a a2 + x2 a2 a2 + x2
1 + ax2

.in
3.6 Integration by parts

ng
Every differentiation rule has a corresponding integration rule. The rule
that correspond to the product for differentiation is called the integration by parts.

eri
From differential calculus we have
d dv du
(uv) = u + v
dx dx dx
Integrating both sides with respect to x
e
gin
Z Z Z
d dv du
(uv)dx = u dx + v dx
dx dx dx
Z Z
uv = udv + vdu
En

Z Z
udv = uv − vdu.

Priority rules for choosing u.


arn

1. Inverse functions.
Le

2. logarithmic functions.

3. Polynomials in x.
w.

4. Any other function other than the above three.


ww

Visit For More : www.LearnEngineering.in


Visit For More : www.LearnEngineering.in

Integral Calculus 209

✎ ☞
Worked Examples
✍ ✌
Z R1
Example 3.66. Evaluate tan−1 xdx and hence deduce the value of tan−1 xdx
0
[A.U.Nov 2016]
Solution. Let u = tan−1 x Z dv = Z
dx
1

.in
du = dx dv = dx ⇒ v = x.
1 + x2
Applying
Z integration
Z by parts
tan−1 dx = udv

ng
Z
= uv − vdu

eri
Z
−1 1
= (tan x) · x − x· dx In the second integral
Z 1 + x2
1 dt
= x · tan−1 x − put 1 + x2 = t
1
t 2
e 2xdx = dt
gin
= x · tan−1 x − log t + c
2 dt
−1 1 2 xdx = .
= x · tan x − log(1 + x ) + c. 2
2
En

Z1  1 1 h i1
Now, tan−1 x dx = x tan−1 x − log(1 + x2 )
0 2 0
0
arn

1
= tan−1 1 − 0 − (log 2 − log 1)
2
π 1
= − log 2.
4 2
Le

Z
Example 3.67. Evaluate sin−1 xdx.
w.

Solution. Let u = sin−1 x dv = dx


Z Z
1
du = √ dx dv = dx ⇒ v = x.
1 − x2
ww

Applying
Z integration
Z by parts
−1
sin x dx = u dv
Z
= uv − v du

Visit For More : www.LearnEngineering.in


Visit For More : www.LearnEngineering.in

210 Engineering Mathematics - I

Z
1
= (x sin−1 x) − x· √ dx
Z 1 −!x2 In the second integral
1 −dt
= x · sin−1 x − √ put 1 − x2 = t
2
Z t
1 1
= x · sin−1 x + (t)− 2 + c − 2xdx = dt
2
 1 dt
−1 1  t 2  xdx = − .
= x · sin x +  1  + c 2

.in
2 2
Z p
∴ sin−1 x dx = (x sin−1 x) + 1 − x2 + c.

ng
Z
Example 3.68. Evaluate x sin xdx.

eri
Solution. Let u = x dv = sin xdx
Z Z
du = dx dv = sin xdx ⇒ v = − cos x.
Applying
Z integration
Z by parts
Z
e
gin
x sin xdx = udv = uv − vdu
Z
= −x · cos x − − cos xdx = −x cos x + sin x + c.
En

Z
Example 3.69. Evaluate xe x dx.

Solution. Let u = x dv = e x dx
arn

Z Z
du = dx dv = e x dx ⇒ v = e x .
Applying
Z integration
Z byZparts
x
Le

xe dx = udv = uv − vdu
Z
= x · e − e x dx
x
w.

= xe x − e x + c = e x (x − 1) + c.
Z
ww

Example 3.70. Evaluate log xdx.

Solution. Let u = log x dv = dx


Z Z
1
du = dx dv = dx ⇒ v = x.
x

Visit For More : www.LearnEngineering.in


Visit For More : www.LearnEngineering.in

Integral Calculus 211

Applying
Z integration
Z by parts
Z
log xdx = udv = uv − vdu
Z
1
= x · log x − x · dx
x
Z
= x log x − dx + c = x log x − x + c = x(log x − 1) + c.

.in
Z
log x 2 dx.

Example 3.71. Evaluate [A.U. Dec. 2015]

Solution. Let u = log x 2



dv = dx

ng
Z Z
1
du = 2(log x) dx dv = dx ⇒ v = x.
x
Applying integration by parts

eri
Z Z Z
2
log x dx = udv = uv − vdu
Z
2 1
= log x · x −
e
x · 2 log x dx
x
gin
Z
2
= x log x − log xdx

= x log x 2 − 2x(log x − 1) + c. [From the provious problem]



En

Z
Example 3.72. Evaluate xn log xdx.

Solution. Let u = log x dv = xn dx


arn

xn+1
Z Z
1
du = dx dv = xn dx ⇒ v = .
x n+1
Applying integration by parts
Le

Z Z Z
n
x log xdx = udv = uv − vdu

xn+1
Z n+1
x 1
= · log x − · dx
w.

n+1 n+1 x
xn+1
Z
1
= log x − xn dx + c
n+1 n+1
ww

xn+1 1 xn+1
= log x − · +c
n+1 n + 1 !n + 1
xn+1 1
= log x − + c.
n+1 n+1

Visit For More : www.LearnEngineering.in


Visit For More : www.LearnEngineering.in

212 Engineering Mathematics - I

Z
Example 3.73. Evaluate x2 e x dx.

Solution. Let u = x2 dv = e x dx
Z Z
du = 2xdx dv = e x dx ⇒ v = e x .
Applying
Z integration
Z by Zparts
2 x
x e dx = udv = uv − vdu

.in
Z
= x2 · e x − e x · 2x dx

ng
Z
= x2 e x − 2 xe x dx

= x2 e x − 2 xe x − e x + c
 
[Refer Example 3.79]

eri
= x2 e x − 2xe x + 2e x + c = e x (x2 − 2x + 2) + c
Z
Example 3.74. Evaluate x2 tan−1 xdx.
e
gin
Solution. Let u = tan−1 x dv = x2 dx
x3
Z Z
1 2
du = dx dv = x dx ⇒ v = .
1 + x2 3
En

Z Applying
Z integration
Z by parts
x2 tan−1 xdx = udv = uv − vdu

x3
Z 3
x 1
arn

−1
= tan x − · dx
3 3 1 + x2
x3 x3
Z
1
= tan−1 x − dx Let 1 + x2 = t
3 3 1 + x2
x3 x2
Z
1
Le

= tan−1 x − · xdx 2xdx = dt


3 3 1 + x2
dt
x3
Z
−1 1 t − 1 dt xdx = .
= tan x − · 2
w.

3 3 t 2
x3
Z !
−1 1 1
= tan x − 1 − dt
3 6 t
ww

x 3 1
Z
1
Z
1
−1
= tan x − dt + dt
3 6 6 t
x3 1 1 x3 1 + x2 1
= tan−1 x − · t + · log t + c = tan−1 x − + log(1 + x2 ) + c.
3 6 6 3 6 6

Visit For More : www.LearnEngineering.in


Visit For More : www.LearnEngineering.in

Integral Calculus 213

+ x + x2
Z
−1 x1
Example 3.75. Evaluate etan dx. [A.U. Dec. 2015]
1 + x2
Solution. Let u = tan−1 x or x = tan udx
1
du = dx dx = sec2 u du.
1 + x2
Applying integration by parts

.in
+ x + x2 1 + x2
Z Z !
tan−1 x 1 tan−1 x x
e dx = e + dx
1 + x2 1 + x2 1 + x2
Z
−1
 x 
= etan x 1 + dx

ng
Z 1Z+ x2
−1 −1
 x 
= etan x dx + etan x 2
dx
1 + x

eri
Z Z
= eu sec2 u du + eu tan u du
Z Z
eu sec2 u du + tan u d eu

=
Z
e Z
gin
= eu sec2 u du + eu tan u − eu sec2 u du
−1
= eu tan u + c = xetan x
+ c.
En

3.7 Trigonometric Integrals


arn

Theorem 1.Prove the reduction formula


R 1 n−1R
sinn xdx = − cos x sinn−1 x + sinn−2 xdx and deduce that
π
n π
n π
Le

R2 n n − 1 R2 n−2 R2 n
sin dx = sin xdx. Find also the general value of sin xdx. Hence
0 n 0 0
π π
R2 R2
w.

evaluate sin xdx and sin8 xdx.


7
[A.U.Dec 2016]
0 Z 0 Z
n
Proof. Write sin xdx = sinn−1 x sin xdx.
ww

Choose u = sinn−1 x dv = sin xdx


Z Z
du = (n − 1) sinn−2 x · cos xdx dv = sin xdx ⇒ v = − cos x.
Applying integration by parts

Visit For More : www.LearnEngineering.in


Visit For More : www.LearnEngineering.in

214 Engineering Mathematics - I

Z Z Z
sinn xdx = udv = uv − vdu
Z
= − sinn−1 x · cos x − − cos x · (n − 1) sinn−2 x cos xdx
Z
= − sinn−1 x · cos x + (n − 1) sinn−2 x cos2 xdx
Z
= − sinn−1 x · cos x + (n − 1) sinn−2 x(1 − sin2 x)dx

.in
Z Z
= − sinn−1 x · cos x + (n − 1) sinn−2 xdx − (n − 1) sinn xdx

ng
Z Z Z
sinn xdx + (n − 1) sinn xdx = − sinn−1 x cos x + (n − 1) sinn−2 xdx
Z Z
sinn xdx = − sinn−1 x cos x + (n − 1)
sinn−2 xdx

eri
n
Z Z
−1 n−1 (n − 1)
sinn xdx = sin x cos x + sinn−2 xdx
n n
π
Taking limits from 0 to everywhere we get e
gin
2
π π
Z2 ! π2 Z2
−1 n−1 n−1
sinn xdx = sin x cos x + sinn−2 xdx.
n 0 n
En

0 0
π π
Z 2 Z2
n−1 n−1
=0+ sinn−2 xdx = sinn−2 xdx.
n n
arn

0 0

Applying the same procedure again for the integral on the RHS we get
π π
Z2 Z2
n−3
Le

sinn−2 xdx = sinn−4 xdx


n−2
0 0
π π
2 Z2
w.

Z
n−1 n−3
∴ sinn xdx = · sinn−4 xdx
n n−2
0 0
ww

Repeating this process n times we get


π
Z 2
n−1 n−3 n−5
sinn xdx = · · · · · · · · (ultimate integral).
n n−2 n−4
0

Visit For More : www.LearnEngineering.in


Visit For More : www.LearnEngineering.in

Integral Calculus 215

π π
R2 R2 π
If n is even, the ultimate integral is sinn−n xdx = dx = [x]02 = π2 .
0 0
π
2
R  π
If n is odd, the ultimate integral is sin xdx = − cos x 2 = −[cos π2 − cos 0]
0
0
= −[0 − 1] = 1.
π
R2
∴ The general value of sinn xdx is

.in
π 0 n − 1 n − 3 n − 5 2
Z2 

 · · , · · · · 1, if n is odd
sinn xdx =  n n−2 n−4 3


n−1 n−3 n−5 1 π

ng

· · ,··· · , if n is even.



0 n n−2 n−4 2 2
π
Z2

eri
7−1 7−3 7−5 6 4 2 16
Now, sin7 xdx = · · ·1= · · ·1= .
7 7−2 7−4 7 5 3 35
0
π

Also,
Z2
sin8 xdx =
7 5 3 1 π 35π
· · · · = . e
gin
8 6 4 2 2 256
0

Theorem 2.Prove the reduction formula π


En

R 1 n−1R R2
n n−1
cos xdx = cos x sin x + n−2
cos xdx. Hence reduce the value of cosn dx.
n n 0
Z Z
n n−1
Proof. Write cos xdx = cos x cos xdx.
arn

Choose u = cosn−1 x dv = cos xdx


Z Z
du = (n − 1) cosn−2 x · (− sin x)dx dv = cos xdx ⇒ v = sin x.
Le

Applying
Z integration
Z by parts
Z
n
cos xdx = udv = uv − vdu
w.

Z
= cos x · sin x − sin x · (n − 1) cosn−2 x(− sin x)dx
n−1

Z
= cosn−1 x · sin x + (n − 1) cosn−2 x sin2 xdx
ww

Z
= cosn−1 x · sin x + (n − 1) cosn−2 x(1 − cos2 x)dx
Z Z
= cosn−1 x · sin x + (n − 1) cosn−2 xdx − (n − 1) cosn xdx

Visit For More : www.LearnEngineering.in


Visit For More : www.LearnEngineering.in

216 Engineering Mathematics - I

Z Z Z
cosn xdx + (n − 1) cosn xdx = cosn−1 x sin x + (n − 1) cosn−2 xdx
Z Z
n cosn xdx = cosn−1 x sin x + (n − 1)
cosn−2 xdx
Z Z
1 (n − 1)
cosn xdx = cosn−1 x sin x + cosn−2 xdx.
R n R n
If n is odd, the ultimate integral is cosn−(n−1) xdx = cos xdx = sin x.

.in
R R
If n is even, the ultimate integral is cosn−n xdx = dx = x.
π
Now,applying limit from 0 to we obtain
π
2 π π

ng
Z2 ! π2 Z2 Z2
1 n − 1 n − 1
cosn xdx = · cosn−1 x sin x + cosn−2 xdx = cosn−2 xdx.
n 0 n n
0 0 0

eri
By repeatedly applying this process we obtain
π
R2 n−1 n−3 n−5
cosn xdx = · · · · · · · · (ultimate integral)
0 n n−2 n−4
 π π
If n is even, the ultimate integral is x 2 = − 0 = .
e2
π
2
gin
0
π
R2  π
If n is odd, the ultimate integral is cos xdx = sin x 2 = 1.
0
π  n − 1 0n − 3 n − 5 2
Z2 
· · , · · · · 1, if n is odd
En



cosn xdx =  n n−2 n−4 3


∴  n−1 n−3 n−5 1 π
· · , · · · · , if n is even.



0 n n−2 n−4 2 2
Result. By looking at the above two theorems, we obtain the following result
arn

If n is odd (say n = 2k + 1),then


π π
R2 2k+1 R2 2.4.6....2k
sin xdx = cos2k+1 xdx = .
0 0 3.5.7...(2k + 1)
Le

If n is even (say n = 2k),then


π π
R2 2k R2 1.3.5....(2k − 1) π
sin xdx = cos2k xdx = · .
0 0 2.4.6.....(2k) 2
w.

For example.
π π π
R2 7 R2 R2
sin xdx = cos dx = cos2.3+1 xdx
7 [n = 3]
ww

0 0 0
2.4.6 16
= .
=
π π
3.5.7
π
35
R2 8 R2 R2
sin xdx = cos8 dx = cos2.4 xdx [n = 4]
0 0 0

Visit For More : www.LearnEngineering.in


Visit For More : www.LearnEngineering.in

Integral Calculus 217

1.3.5.7 π 35π
= × = .
2.4.6.8 Z 2 256
Evaluation of integrals of the form sinm x · cosn xdx
Evaluation procedure

(i) If the power of cosine is odd (say n = 2k + 1)then write


Z Z

.in
m n
sin x cos xdx = sinm x · cos2k+1 xdx
Z
= sinm x · cos2k x cos xdx

ng
Z
= sinm x · (cos2 x)k cos xdx
Z

eri
= sinm x · (1 − sin2 x)k cos xdx

Now make the substitution t = sin x, and hence the integral is evaluated
easily.
e
gin
(ii) If
Z the power of sineZis odd (say n = 2k + 1)then write
sinm x cosn xdx = sin2k+1 x · cosn x cos xdx
En

Z
= sin2k x · sin x cosn xdx
Z
= (1 − cos2 x)k · cosn x sin xdx.
arn

Now make the substitution t = cos x, and hence the integral is evaluated
easily.
Le

(iii) If the powers of both sine and cosine are even, then use the relations
(1 − cos 2x)
sin2 x =
2
w.

(1 + cos 2x)
cos2 x =
2
so that the resulting integrals can be evaluated easily.
ww

(iv) If the powers of both sine and cosine are odd, then apply either case(i) or
case(ii).
✎ ☞
Worked Examples
✍ ✌

Visit For More : www.LearnEngineering.in


Visit For More : www.LearnEngineering.in

218 Engineering Mathematics - I

Z
Example 3.76. Evaluate sin5 x cos2 xdx.
Solution. Here the power of sine is odd.
R R
∴ We write sin5 x cos2 xdx = sin4 x · sin x cos2 xdx
R
= Z (sin2 x)2 · cos2 x sin xdx
= (1 − cos2 x)2 · cos2 x sin xdx
put cos x = t

.in
Z
= (1 − t2 )2 · t2 (−dt) − sin xdx = dt
Z
= − (1 + t4 − 2t2 )t2 dt sin xdx = −dt.

ng
Z
= − (t2 + t6 − 2t4 )dt

eri
" 3
t7 t5 cos3 x cos7 x 2
#
t
=− + −2· +c=− − + cos5 x + c.
3 7 5 3 7 5

Z
e
gin
Example 3.77. Evaluate sin6 x cos3 xdx.
Solution. Here the power of cosine is odd.
R R
∴ we write sin6 x cos3 xdx = sin6 x · cos2 x · cos xdx
En

R
= Z sin6 x(1 − sin2 x) · cos xdx
= t6 (1 − t2 )dt put sin x = t
arn

Z
= (t6 − t8 )dt cos xdx = −dt.
t7 t9 sin7 x sin9 x
= − +c= − + c.
7 9 7 9
Le

Z
Example 3.78. Evaluate sin9 x cos5 xdx.
w.

Solution. Here, the powers of both sine and cosine are odd.
R R
∴ we write sin9 x cos5 xdx = sin9 x · cos4 x · cos xdx
ww

R
= Z sin9 x(1 − sin2 x)2 · cos xdx
= t9 (1 − t2 )2 dt put sin x = t
Z
= t9 (1 + t4 − 2t2 )dt cos xdx = dt.

Visit For More : www.LearnEngineering.in


Visit For More : www.LearnEngineering.in

Integral Calculus 219

Z
= (t9 + t13 − 2t11 )dt

t10 t14 t12 sin10 x sin14 x sin12 x


= + −2 +c= + − + c.
10 14 12 10 14 6
Z
Evaluation of integrals of the form tanm x · secn xdx
Evaluation procedure.

.in
(i) If
Z the power of secant
Z is even (say n = 2k) then write the given integral as

ng
m n
tan x sec xdx = tanm x · sec2k xdx
Z
= tanm x · sec2k−2 x sec2 xdx

eri
Z
= tanm x · sec2(k−1) x sec2 xdx
Z
=
e
tanm x · (sec2 x)k−1 sec2 xdx put tan x = t
gin
Z
= tanm x · (1 + tan2 x)k−1 sec2 xdx sec2 xdx = dt.
Z
= tm (1 + t2 )k−1 dt, which can be evaluated easily.
En

(ii) If
Z the power of tangent
Z is odd (say m = 2k + 1)then write the given integral as
tanm x secn xdx = tan2k+1 x · secn xdx
arn

Z
= tan2k x · tan x secn−1 x sec xdx
Z
= (tan2 x)k · secn−1 x sec x tan xdx put sec x = t
Le

Z
= (sec2 x − 1)k secn−1 x sec x tan xdx sec x tan xdx = dt.
Z
w.

= (t2 − 1)k · tn−1 dt, which can be evaluated easily.

✎ ☞
ww

Worked Examples
✍ ✌
Z
Example 3.79. Evaluate tan6 x sec4 xdx.
Solution. Here, the power of secant is even. Hence we write the given integral as

Visit For More : www.LearnEngineering.in


Visit For More : www.LearnEngineering.in

220 Engineering Mathematics - I

Z Z
tan6 x sec4 xdx = tan6 x sec2 x sec2 xdx
Z put tan x = t
= tan6 x(1 + tan2 x) sec2 xdx
Z sec2 xdx = dt.
= t6 (1 + t2 )dt
t7 t9
+ +c =
7 9

.in
7
tan x tan9 x
= + + c.
7 9

ng
Z
Example 3.80. Evaluate tan5 x sec7 xdx.
Solution. Here, the power of tangent is odd. Hence we write the given integral

eri
as Z Z
5 7
tan x sec xdx = tan4 x sec6 x sec x tan xdx
Z put sec x = t
=
e
(sec2 x − 1)2 sec6 x sec x tan xdx
gin
Z sec x tan xdx = dt.
2 2 6
= (t − 1) t dt
Z
= (t4 + 1 − 2t2 )t6 dt
En

Z
= (t10 + t6 − 2t8 )dt

t11 t7 t9 sec11 x sec7 x 2


− sec9 x + c.
arn

= + −2 +c= +
11 7 9 11 7 9
Some standard results.
R
Le

1. Find tan xdxZ Z


sin x
Solution. tan xdx = dx cos x = t
Z cos x
1
w.

= (−dt) − sin xdx = dt


t
= − log t + c sin xdx = −dt.
ww

= − log(cos x) + c (or)
!
−1 1
= log(cos x) + c = log + c = log(sec x) + c.
cos x

Visit For More : www.LearnEngineering.in


Visit For More : www.LearnEngineering.in

Integral Calculus 221

R
2. Find sec xdxZ Z
sec x + tan x
Solution. sec xdx = sec x · dx
sec x + tan x
Z
1 sec x + tan x = t
= (dt)
t (sec x tan x + sec2 x)dx = dt
= log t + c
sec x(sec x + tan x)dx = dt.

.in
= log(sec x + tan x) + c.
R
3. Find cosecxdx
Z Z
cosecx + cot x

ng
Solution. cosecxdx = cosecx · dx
cosecx + cot x
Z
1 cosecx + cot x = t
= (−dt)
t (−cosecx cot x − cosec2 x)dx = dt

eri
= − log t + c
−cosecx(cosecx + cot x)dx = dt.
= − log(cosecx + cot x) + c
e
gin
Z
4. Find cot x dx.
Z Z Z
cos x d(sin x)
Solution. cot x dx = dx = = log(sin x) + c.
sin x sin x
En

✎ ☞
Worked Examples
✍ ✌
Z
tan3 xdx.
arn

Example 3.81. Evaluate


Z Z
3
Solution. tan xdx = tan2 x tan xdx
Z
(sec2 x − 1) tan xdx
Le

=
Z Z
2
= sec x tan xdx − tan xdx
w.

Z
= tan xd(tan x) − log sec x

tan2 x
ww

− log(sec x) + c.
=
2
Z
Example 3.82. Evaluate sec3 xdx.
R R
Solution. sec3 xdx = sec x · sec2 xdx

Visit For More : www.LearnEngineering.in


Visit For More : www.LearnEngineering.in

222 Engineering Mathematics - I

Let us apply integration by parts


Choose, u = sec x dv = sec2 xdx
Z Z
du = sec x tan xdx dv = sec2 xdx ⇒ v = tan x
Z Z Z
3
sec xdx = udv = uv − vdu
Z

.in
= sec x · tan x − tan x · sec x · tan xdx
Z
= sec x tan x − sec x tan2 xdx

ng
Z
= sec x tan x − sec x(sec2 x − 1)dx
Z Z Z
3 3

eri
sec xdx = sec x tan x − sec xdx + sec xdx
Z Z
sec3 xdx + sec3 xdx = sec x tan x + log(sec x + tan x) + c
Z
e
2 sec3 xdx = sec x tan x + log(sec x + tan x) + c
gin
Z
1
sec3 xdx = sec x tan x + log(sec x + tan x) + c.

2
En

3.8 Improper integrals

Definition of an improper integral Type-I


arn

Rt
(i) If f (x)dx exists for every number t ≥ a, then
a
R∞ Rt
Le

f (x)dx = lim f (x)dx


t→∞
a a
provided this limit exists (as a finite number).
w.

Rb
(ii) If f (x)dx exists for every number t ≤ b, then
t
Rb Rb
ww

f (x)dx = lim f (x)dx


t→−∞
−∞ t
provided this limit exists (as a finite number).
R∞ Rb
The improper integrals f (x)dx and f (x)dx are called convergent if the
a −∞

Visit For More : www.LearnEngineering.in


Visit For More : www.LearnEngineering.in

Integral Calculus 223

corresponding limit exists and divergent if the limit does not exist.
R∞ Ra
(iii) If both f (x)dx and f (x)dx are convergent then we define
a −∞
R∞ Ra R∞
f (x)dx = f (x)dx + f (x)dx.
−∞ −∞ a
Here a is any real number.

.in
✎ ☞
Worked Examples
✍ ✌

ng
R∞
Example 3.83. Discuss the convergence of e−x dx.
0

eri
Z∞ Zt
−x
Solution. e dx = lim e−x dx
t→∞
0 0

= lim
e−x
!t
e
= − lim e − e

−t 0
 1
!
= − lim t − 1 = −(−1) = 1
gin
t→∞ −1 t→∞ t→∞ e
0
Z∞
∴ e−x dx is convergent.
En

R∞ 1
Example 3.84. Determine whether the integral 2
dx is convergent or divergent.
1 x
arn

Z∞ Zt
1 1
Solution. dx = lim dx
x2 t→∞ x2
1 1
Zt
Le

= lim x−2 dx
t→∞
1
!t !t
x−1
!
1 1
w.

= lim = − lim = − lim − 1 = −(−1) = 1.


t→∞ −1 t→∞ x t→∞ t
1 1
Z∞
1
ww

∴ dx is convergent.
x2
1

R∞ 1
Example 3.85. Determine the convergence of dx.
1 x

Visit For More : www.LearnEngineering.in


Visit For More : www.LearnEngineering.in

224 Engineering Mathematics - I

Z∞ Zt
1 1
Solution. dx = lim dx
x t→∞ x
1 1

= lim (log x)t1 = lim (log t − log 1) = lim log t = ∞


t→∞ t→∞ t→∞
i.e the limit does not exist, as a finite number.
R∞ 1
∴ dx is divergent.
1 x

.in
R∞

ng
1
Example 3.86. Discuss the convergence of dx.
0 a2 + x2
Z∞ Zt
1 1

eri
Solution. dx = lim dx
a2 + x2 t→∞ a2 + x2
0 0
1  −1  x t
= lim tan
t→∞ a
1  −1 t
a 0

e
gin
= lim tan − tan−1 0
t→∞ a a
1 −1
= (tan ∞ − 0)
a
1 π  π
En

= −0 = which is a finite number.


a 2 2a
R∞ 1
∴ 2 2
dx is convergent.
0 a +x
arn

R∞ dx
Example 3.87. Determine whether the integral √ convergent or divergent
x
Le

1
Z∞ Zt  1 t
dx − 21
 x 2  √
Solution. √ = lim x dx = lim  1  = 2 lim ( t − 1) = 2 · ∞ = ∞.
x t→∞ t→∞
2 0
t→∞
w.

1 1
Here the limit does not exist as a finite number.
R∞ 1
∴ √ dx is divergent.
x
ww

R∞
Example 3.88. Discuss the convergence of the integral log xdx.
1

Visit For More : www.LearnEngineering.in


Visit For More : www.LearnEngineering.in

Integral Calculus 225

Z∞ Zt
Solution. log x dx = lim log xdx
t→∞
1 1
Zt
 
1
 
= lim  log x · x t0 −

x · dx [using integration by parts]
 
t→∞  x 
1
h i
= 2 lim t log t − (x)t1 = 2 lim t log t − t + 1 = ∞
 

.in
t→∞ t→∞
R∞
∴ log xdx is divergent.
1

ng
Z∞
log x
Example 3.89. Determine whether the integral dx is convergent or
x

eri
1
divergent. Evaluate if the integral is convergent. [A.U. Dec. 2015]
Z∞ Zt Zt
log x log x
Solution.
x
dx = lim
t→∞ x
e
dx = lim
t→∞
log xd(log x)
gin
1 1 1
!t
(log x)2
= lim
t→∞ 2 1
1  
= lim (log t)2 − (log 1)2
En

2 t→∞
1
= lim (log t)2 = ∞, [which is not a finite number.]
2 t→∞
Z∞
arn

log x
∴ dx is divergent.
x
1

R∞ 1
Le

Example 3.90. For what values of p is the integral p


dx convergent?
1 x
Z∞ Zt
1
Solution. dx = lim x−p dx
w.

xp t→∞
1 1
!t
x−p+1
= lim
ww

t→∞ −p + 1
1
!
1 1−p 1 1
= lim (t − 1) = · −1 .
1 − p t→∞ 1 − p t p−1
1
If p > 1, then p − 1 > 0 and hence t p−1 → ∞ as t → ∞ and hence → 0.
t p−1

Visit For More : www.LearnEngineering.in


Visit For More : www.LearnEngineering.in

226 Engineering Mathematics - I

R∞ 1 1 1
∴ p
dx = (0 − 1) = , which is a finite number.
1 x 1− p p−1
R∞ 1
∴ p
dx is convergent if p > 1.
1 x
If p < 1, then p − 1 < 0 or 1 − p > 0.
1
Hence p−1 = t1−p → ∞ as t → ∞.
t
R∞ 1

.in
1
∴ p
dx = (∞ − 1) = ∞.
1 x 1 − p
R∞ 1
∴ dx diverges, if p < 1.

ng
p
1 x
R∞ 1
Also, when p = 1, dx = ∞ [Example 3.96]
1 x

eri
R∞ 1
∴ p
dx is divergent if p = 1.
1 x
R∞ 1
Hence,
1 x
p
e
dx is convergent if p > 1 and divergent if p ≤ 1.
gin
Z0
Example 3.91. Discuss the convergence of e x dx
En

−∞
R0 R0
Solution. e x dx = lim e x dx
t→−∞
−∞ t
arn

= lim [e x ]0t = lim [e0 − et ] = 1 − 0 = 1, which is a finite number


t→−∞ t→−∞
∴ The given integral is convergent.
Le

R0 1
Example 3.92. Evaluate dx.
−∞ (1 − 3x)2
Z0 Z0
1
w.

Solution. 2
dx = lim (1 − 3x)−2 dx
(1 − 3x) t→−∞
−∞ t
#0 #0
(1 − 3x)−1
" "
1 1 1 1
ww

= lim = lim = lim [1 − 0] = .


t→−∞ (−1)(−3) t
t→−∞ 3 1 − 3x
t 3 t→−∞ 3

R0
Example 3.93. Evaluate xe x dx.
−∞

Visit For More : www.LearnEngineering.in


Visit For More : www.LearnEngineering.in

Integral Calculus 227

Z0 Z0
x
Solution. xe dx = lim xd(e x )
t→−∞
−∞ t
Z0
 
 
x 0 x 
= lim  x · e t − e dx

t→−∞  
t
h i
= lim 0 − et − (e x )0t

.in
t→−∞
h i h i
= lim −et − (1 − et ) = lim −et − 1 + et = lim (−1) = −1.
t→−∞ t→−∞ t→−∞

ng
Z∞
1
Example 3.94. Evaluate dx.

eri
4 + x2
−∞
Z∞ Z0 Z∞
1 1 1
Solution. dx = dx + dx
4 + x2 4 + x2 4 + x2
−∞ −∞
e
0
gin
Z0 Zt
1 1
= lim 2
dx + lim dx
t→−∞ 4+x t→−∞ 4 + x2
t 0
!0 !t
1 −1 x 1 −1 x
En

= lim tan + lim tan


t→−∞ 2 2 t t→∞ 2 2 0
1  t 0 1
  t t
= lim tan−1 (0) − tan−1 + lim tan−1 − tan−1 (0)
2 t→−∞ 2 t 2 t→∞ 2 0
arn

1  −1  1 
= tan 0 − tan−1 (−∞) + tan−1 (∞) − tan−1 0
2 2
1  π  1  π  1 π π π
= 0− − + −0 = + = .
2 2 2 2 2 2 2 2
Le

Z∞
Example 3.95. If a > 0, discuss the convergency or divergency of
w.

sin xdx.
a
Z∞ Zt h it
ww

Solution. sin xdx = lim sin xdx = lim − cos x = − lim [cos t − cos a].
t→∞ t→∞ a t→∞
a a
We know that cost is bounded and it lies between −1 and 1.
R∞
Hence the integral sin xdx oscillates finitely[It neither converges nor diverges].
a

Visit For More : www.LearnEngineering.in


Visit For More : www.LearnEngineering.in

228 Engineering Mathematics - I

Z0
Example 3.96. Discuss the convergence of x sin xdx.
−∞
Z0 Z0
Solution. x sin xdx = lim xd(− cos x)
t→−∞
−∞ t
Z0
 
 

.in
0
= lim [−x cos x]t − − cos xdx
 
t→−∞  
t

= lim [0 − t cos t + (sin x)0t ]

ng
t→−∞

= lim (−t cos t + 0 − sin t)


t→−∞
R0

eri
Since cos x and sin x oscillates between −1 and 1, x sin xdx oscillates finitely.
−∞

Definition of an improper integral type II


e
gin
(i) If f is continuous on [a, b) and is discontinuous at b, then
Zb Zt
f (x)dx = lim− f (x)dx
En

t→b
a a
b−∈
Z
= lim f (x)dx
arn

∈→0
a
if the limit exists (as a finite number).
Le

(ii) If f is continuous on (a, b] and is discontinuous at a, then


Zb Zb
f (x)dx = lim+ f (x)dx
t→a
w.

a t
Zb
= lim f (x)dx
∈→0
ww

a+∈
if the limit exists (as a finite number).
Rb
The improper integral f (x)dx is called convergent if the corresponding limit
a
exists and divergent if the limit does not exist.

Visit For More : www.LearnEngineering.in


Visit For More : www.LearnEngineering.in

Integral Calculus 229

Rc Rb
(iii) If f hss a discontinuity at c where a < c < b, and both f (x)dx and f (x)dx
a c
are convergent, then we define
Rb Rc Rb
f (x)dx = f (x)dx + f (x)dx.
a a c
✎ ☞
Worked Examples
✍ ✌

.in
Z1
1
Example 3.97. Evaluate √ dx if possible.
x

ng
0
Solution. The integrand has an infinite discontinuity at the lower limit 0.
Z1 Z1  1 1
1 − 12  x 2  √
√ dx = lim x dx = lim  1  = 2 lim (1 − ∈) = 2.

eri

x ∈→0 ∈→0
2 ∈
∈→0
0 0+∈

Z3
dx
Example 3.98. Determine whether the integral
e √ is convergent or
x
gin
0
divergent. Evaluate if it is convergent. [A.U. Dec. 2015]
Solution. The integrand has an infinite discontinuity at the lower limit 0.
Z3 Z3  1 3
√ √

En

1 1  x 2 
∴ √ dx = lim x− 2 dx = lim  1  = 2 lim ( 3 − ∈) = 2 3,
x ∈→0 ∈→0
2 ∈
∈→0
0 0+∈

[which is a finite number.]


arn


Hence, the given integral is convergent and its values is 2 3.
Z5
1
Example 3.99. Find the value of the improper integral √ dx if possible.
Le

x−2
2
Solution. The integrand has an infinite discontinuity at the lower limit 2.
w.

Z5 Z5 5
 (x − 2) 21 

√ √
1 − 21

∴ √ dx = lim (x − 2) dx = lim  1
 = 2 lim ( 3 − ∈) = 2 3.
x−2 ∈→0 ∈→0 ∈→0
2 2+∈ 2 2+∈
ww

Z1
1
Example 3.100. Evaluate √ dx if possible.
1 − x2
0
Solution. The integrand has an infinite discontinuity at the upper limit 1.

Visit For More : www.LearnEngineering.in


Visit For More : www.LearnEngineering.in

230 Engineering Mathematics - I

Z1 Z1−∈
1 1
∴ √ dx = lim √ dx
1− x2 ∈→0 1 − x2
0 0

= lim (sin−1 x)1−∈


0
∈→0
  π
= lim sin−1 (1− ∈) − sin−1 0 = sin−1 1 − 0 = .
∈→0 2

.in
π
Z2
Example 3.101. Discuss the convergence of sec xdx if possible.
0

ng
Solution. The integrand has an infinite discontinuity at the upper limit π2 .
π π
Z2 Z2 −∈
∴ sec xdx = lim sec xdx

eri
∈→0
0 0
π
−∈
= lim (log(sec x + tan x))02
∈→0
 π π
e
= lim log(sec( − ∈) + tan( − ∈)) − log(sec 0 + tan 0)

gin
∈→0 2 2
π π
= log(sec( ) + tan( )) − log 1 = log ∞ = ∞.
π
2 2
R2
∴ sec xdx diverges.
En

0
Z3
1
Example 3.102. Evaluate the improper integral √ dx if possible.
arn

3−x
2
[A.U.Nov 2016]
Solution. The integrand has an infinite discontinuity at the upper limit 3.
Le

Z3 Z3−∈ 3−∈
 (3 − x) 21 

1 − 21
∴ √ dx = lim (3 − x) dx = lim   
3−x ∈→0 ∈→0 − 12 2
2 2

w.

p 
= lim (−2) 3 − (3− ∈) − 3 − 2
∈→0
√ 
= −2 lim 3 − 3+ ∈ − 1 = −2(0 − 1) = 2.
ww

∈→0

Z1
1
Example 3.103. Evaluate 2
dx if possible.
x3
−1
Solution. The integrand has an infinite discontinuity at x = 0,which lies between

Visit For More : www.LearnEngineering.in


Visit For More : www.LearnEngineering.in

Integral Calculus 231

−1 and 1.
Z1 Z0 Z1
1 1 1
∴ 2
dx = 2
dx + 2
dx
x 3 x 3 x3
−1 −1 0
Z0−∈ Z1
2 2
= lim x− 3 dx + lim x− 3 dx
∈→0 ∈→0
−1 0+∈

.in
 1 −∈  1 1
 x 3   x 3 
= lim  1  + lim  1 
∈→0 ∈→0
3 −1 1  3 ∈ 

ng
1

= 3 lim (− ∈) 3 + 1 + 3 lim 1 − (∈) 3 = 3 × 1 + 3 × 1 = 6.
∈→0 ∈→0

Z3

eri
1
Example 3.104. Evaluate the improper integral dx if possible.
x−1
0
Solution. The integrand has an infinite discontinuity at x = 1 which lies between
0 and 3.
e
gin
Z3 Z1 Z3
1 1 1
∴ dx = dx + dx
x−1 x−1 x−1
0 0 1
En

Z1−∈ Z3
1 1
= lim dx + lim dx
∈→0 x−1 ∈→0 x−1
0 1+∈
arn

1−∈
+ lim log |x − 1| 31+∈
  
= lim log |x − 1| 0
∈→0 ∈→0

= lim log | ∈ | − log | − 1| + log 2 − log | ∈ |
∈→0
Le

= log 0 + log 2 − log 0

= −∞ + log 2 + ∞ which is an indeterminate


w.

R3 1
∴ dx diverges.
0 x−1
ww

Z1
1
Example 3.105. Evaluate dx if possible.
x
−1
Solution. The integrand has an infinite discontinuity at x = 0 which lies between

Visit For More : www.LearnEngineering.in


Visit For More : www.LearnEngineering.in

232 Engineering Mathematics - I

−1 and 1.
Z1 Z0 Z1
1 1 1
∴ dx = dx + dx
x x x
−1 −1 0
Z0−∈ Z1
1 1
= lim dx + lim dx
∈→0 x ∈→0 x
−1 0+∈

.in
h i−∈ h i1
= lim log |x| + lim log |x|
∈→0 −1 ∈→0 −∈
 
= lim log |− ∈ | − log | − 1| + lim log 1 − log | ∈ |

ng
∈→0 ∈→0

= lim log | ∈ | − log(∈)
∈→0
!

eri
|∈|
= lim log = lim log 1 = 0, which is a finite number
∈→0 |∈| ∈→0
R1 1
∴ dx converges.
−1 x
e
gin
A comparison test for improper integrals.
Comparison theorem. Suppose that f and g are continuous functions with
f (x) ≥ g(x) ≥ 0 for x ≥ a.
En

R∞ R∞
(i) If f (x)dx is convergent, then g(x)dx is convergent.
a a
arn

R∞ R∞
(ii) If g(x)dx is divergent, then f (x)dx is divergent.
a a
✎ ☞
Worked Examples
Le

✍ ✌
Z∞
2
Example 3.106. Show that e−x dx is convergent.
w.

0
R∞ R1 R∞
Solution. e−x2 dx = e −x2 dx +
2
e−x dx
ww

0 0 1
R1
e −x2 dx =a definite value and hence it is convergent.
0
R∞ 2
Let us consider the integral e−x dx.
1

Visit For More : www.LearnEngineering.in


Visit For More : www.LearnEngineering.in

Integral Calculus 233

For all x ≥ 1,we have x2 ≥ x.


⇒ −x2 ≤ −x
2 2
⇒ e−x ≤ e−x ⇒ e−x ≥ e−x .
Z∞ Zt
−x t
Now e dx = lim e−x dx = lim −e−x 1
t→∞ t→∞
1 1
!
1 1

.in
 
−t −1
= − lim e − e =− 0− = , which is finite.
t→∞ e e
R∞
∴ e−x dx converges.

ng
1
R∞ 2
By comparison theorem e−x dx converges.
0

eri
R∞
⇒ e−x2 dx is convergent.
0

Z∞
Example 3.107. Prove that the integral
e 1 + e−x
dx is divergent.
gin
x
1
1 + e−x 1 e−x 1
Solution. We have for all x ≥ 1, = + >
x x x x
R∞ 1
We know from Example 3.101, dx diverges [∴ p = 1]
En

1 x
R∞ 1 + e−x
∴ By comparison theorem, dx is divergent.
1 x
arn

Example 3.108. Determine the convergency or divergency of the integral


Z∞
1
3
dx, using comparison test.
x +1
Le

1
1 1 1 1
Solution. For all x ≥ 1, we have, < 3 ⇒ 3 > 3 .
+1 x x3x x +1

R 1
w.

We have from Example 3.101, 3


dx converges [p > 1].
1 x
R∞ 1
∴ By comparison theorem 3
dx is convergent.
1 x +1
ww

Example 3.109. Determine the convergency or divergency of the integral


Z∞
cos2 x
dx.
x2
2

Visit For More : www.LearnEngineering.in


Visit For More : www.LearnEngineering.in

234 Engineering Mathematics - I

Solution. We know that for all x ≥ 2 cos2 x ≤ 1.


cos2 x 1
⇒ 2
≤ 2. [∴ x2 > 0]
x x
1 cos2 x
or 2 ≥ .
x x2
R∞ 1
By Example 3.101, 2
dx converges [p > 1].
2 x
R∞ cos2 x

.in
∴ By comparison theorem dx also converges.
2 x2
Z∞

ng
1
Example 3.110. Discuss the convergency or divergency of the integral dx.
x + ex
2
1 1
= e−x .

eri
Solution. For all x > 2, we have <
x + ex ex
1
⇒ e−x > .
x + ex
Z∞ Zt
−x
Now e dx = lim
t
e−x dx = lim −e−x 2
e
gin
t→∞ t→∞
2 2
!

−t −2
 1 1
= − lim e − e =− 0− 2 = 2 which is finite.
t→∞ e e
R∞
En

∴ e−x dx is convergent.
2
R∞ 1
∴ By comparison theorem, dx is convergent.
x + ex
arn

2
Le
w.
ww

Visit For More : www.LearnEngineering.in


Visit For More : www.LearnEngineering.in

4 Multiple Integrals

.in
ng
4.1 Evaluation of double integrals

Definition. Let f (x, y) be a single

eri
valued and bounded function of two
Y
independent variables x and y which R

is defined in a closed region R of


e
gin
the xy plane. Divide the region
R into rectangles by drawing lines
parallel to the coordinate axes. Let
En

Ai be the ith rectangle. Let (xi , yi )


X
be any point inside the ith rectangle O
arn

and ∆Ai be its area. Let there be


n rectangles A1 , A2 , . . . , An which lie
completely inside R.
Le

Let us consider the sum


w.

n
X
f (x1 , y1 )∆A1 + f (x2 , y2 )∆A2 + · · · + f (xn , yn )∆An = f (xi , yi )∆Ai . (1)
i=1
ww

As n → ∞, the number of rectangles increase indefinitely such that the largest


linear dimension of the rectangle which is the diagonal of ∆Ai tends to zero. The
limit of the sum (1) if exists is defined to be the double integral of f (x, y) over the

Visit For More : www.LearnEngineering.in


Visit For More : www.LearnEngineering.in

236 Engineering Mathematics - I

"
region R and it is denoted by f (x, y)dA.
R
n
X "
i.e., lim f (xi , yi )∆Ai = f (x, y)dA.
n→∞
∆Ai →0 i=1 R

.in
If A is a typical rectangle whose dimensions parallel to the coordinate axes are dx
and dy then dA = dxdy. Then the above double integral can be written as

ng
" " "
f (x, y)dA = f (x, y)dxdy = f (x, y)dydx.
R R R

eri
In cartesian coordinates the following cases are to be taken into account while
evaluating the double integrals.

Case (i) Consider the case where e Y R


gin
the limits are constants. Let the
region R be a rectangle given by
En

R = {(x, y) : a ≤ x ≤ b, c ≤ y ≤ d}, where


a, b, c, d are constants, then, the double X
O
integral is given by
arn

Zb Zd Zd
   b 
Z Z  Z 

f (x, y)dxdy =  f (x, y)dy dx or =
 
 f (x, y)dx  dy.

R  
a c c a
Le

i.e., if the limits are constants, the order of integration, is immaterial, provided
proper limits are taken.
w.

Case (ii) Consider the case when Y


the limits of x are constants and
ww

the limits of y are functions of x. h(x)


In this case the region R is given
by R = {(x, y) : a ≤ x ≤ b, g(x) ≤ y ≤ h(x)}, g(x)

where a, b are constants. Now the X


x=a x=b
double integral becomes

Visit For More : www.LearnEngineering.in


Visit For More : www.LearnEngineering.in

Multiple Integrals 237

" Zb  Zh(x)


 

 
f (x, y)dxdy =  f (x, y)dy dx.
 
R a g(x)
i.e., if the limits of the innermost integral are functions of x, then the order of
evaluation of the integral is, first with respect to y and finally with respect to x.

.in
Case (iii) Consider the case when

ng
Y
the limits of y are constants and
y=d
the limits of x are functions of

eri
y. Now the region R is given
g(y) h(y)
by R = {(x, y) : g(y) ≤ x ≤ h(y), c ≤ y ≤ d},
where c, d are constants. The double
e y=c
gin
integral now takes the form X

" Zd  Zh(y)


 

 
f (x, y)dxdy =  f (x, y)dx dy.
En

 
R c g(y)
i.e., if the limits of the innermost integral are functions of y, the order of
integration is first with respect to x and finally with respect to y.
arn

✎ ☞
Worked Examples
✍ ✌
"
Le

Example 4.1. Evaluate dxdy over the region R bounded by x = 0, x = 2, y = 0,


R
y = 2. [Jan 1996]
w.

" Z2 Z2
Solution. dxdy = dxdy.
ww

R 0 0
Since the limits are constants, the order of integration is immaterial.
" Z2 Z2 Z2
∴ dxdy = (x)20 dy = 2dy = 2 dy = 2[y]20 = 4.
R 0 0 0

Visit For More : www.LearnEngineering.in


Visit For More : www.LearnEngineering.in

238 Engineering Mathematics - I

Z1 Z2
Example 4.2. Evaluate x(x + y)dydx. [Jun 1996]
0 1

Z1 Z2 Z1 Z2
 
 
Solution. x(x + y)dydx =  x(x + y)dy dx
 

0 1 0 1

.in
Z1  !2  Z1
y2 
!!
 2 2 2 1
=  x (y)1 + x  dx = x +x 2− dx
2 1 2
0 0

ng
Z1 3 1
!1
3 x2
! !
2 3x x 1 3 13
= x + dx = + = + = .
2 3 0 2 2 0 3 4 12

eri
0

Note. In some cases, the integrand can be expressed as the product of two

e
quantities which are independent. In that case the evaluation of the double
gin
integral is equivalent to the product of the integrals involving the independent
integrands as the following examples suggest.
En

Za Zb
dxdy
Example 4.3. Evaluate .
xy
1 1
arn

Za Zb Za Zb
dx dy dx dy
Solution. = = (log x)a1 (log y)b1 = log a log b.
x y x y
1 1 1 1
Le

Z1 Z1
dxdy
Example 4.4. Evaluate √ .
w.

p
1 − x2 1 − y2
0 0

Z1 Z1 Z1 Z1
ww

dxdy dx dy
Solution. √ p = √ p
1 − x2 1 − y2 1 − x2 1 − y2
0 0 0 0
π π π2
= (sin−1 x)10 (sin−1 y)10 = = .
22 4

Visit For More : www.LearnEngineering.in


Visit For More : www.LearnEngineering.in

Multiple Integrals 239

Z2 Z4 Z4 Z2
y
Example 4.5. Prove that (xy + e )dydx = (xy + ey )dxdy
1 3 3 1

Z2 Z4 Z2 !y=4
y y2
Solution. (xy + e )dydx = x + ey dx
2 y=3
1 3 1
Z2 

.in
x 
= (16 − 9) + (e4 − e3 ) dx
2
1

ng
Z2 Z2
7
= xdx + (e4 − e3 )dx
2
1 1

eri
!2
7 x2
+ (e4 − e3 )(x)21
=
2 2 1
7 21
= (4 − 1) + (e4 − e3 )(2 − 1) =
4
e4
+ e4 − e3 . (1)
gin
Z4 Z2 Z4 ! x=2
y x2 y
(xy + e )dxdy = y +e x dy
2 x=1
3 1 3
Z4
En

!
3 y
= y + e dy
2
3
!4
3 y2
arn

y
= +e
22 3
3 21
= (16 − 9) + e4 − e3 = + e4 − e3 . (2)
4 4
Le

From (1) and (2) we have the required result.



Z1 Z1+x2
dxdy
w.

Example 4.6. Evaluate .


1 + x2 + y2
0 0
Solution. Since the limits of the innermost integral are functions of x, the order
ww

of evaluation must be, first w.r.t. y and finally w.r.t. x.


√ √ 
Z1 Z1+x2 Z1  Z1+x2 
dxdy  dy 
2 2
= 2 2  dx
1+x +y 1 + x + y 


0 0 0 0

Visit For More : www.LearnEngineering.in


Visit For More : www.LearnEngineering.in

240 Engineering Mathematics - I

Z1 " # √1+x2
1 y
= √ tan−1 √ dx
1+x 2 1 + x2 0
0
Z1 Z1
1 π 1
= √ (tan−1 1)dx = √ dx
1+x 2 4 1 + x2
0 0
π p
1 π √
= log(x + 1 + x2 )0 = (log(1 + 2)).

.in
4 4
√2 2
Za Za −y q

ng
Example 4.7. Evaluate a2 − x2 − y2 dxdy.
0 0
Solution. Since the limits of the innermost integral are functions of x, the order

eri
of evaluation must be first w.r.t. x and finally w.r.t. y.
√ √
2 2 2 2
Za Za −y q Za Za −y q
a2 − x2 − y2 dxdy =
e (a2 − y2 ) − x2 dxdy
gin
0 0 0 0
Za  q  x= √a2 −y2
 x a2 − y2 x 
=  a2 − y2 − x2 + sin−1 p  dy
2 2 a2 − y2 x=0
En

0
Za
a2 − y2 −1
= sin (1)dy
2
arn

0
Za
π
= (a2 − y2 )dy
4
0
Le

!a
π 2 y3
= a y−
4 3 0
π 3 a3 π 2a3 πa3
!
w.

= a − = = .
4 3 4 3 6

Za Za2 +x2
ww

dxdy
Example 4.8. Evaluate .
x2 + y2 + a2
0 0
Solution. Since the limits of the innermost integral are functions of x, the order
of integration must be first w.r.t. y and finally w.r.t. x.

Visit For More : www.LearnEngineering.in


Visit For More : www.LearnEngineering.in

Multiple Integrals 241

√ √
Za Za2 +x2 Za Za2 +x2
dxdy dy
∴ = dx.
x2 + y2 + a2 x2 + y2 + a2
0 0 x=0y=0

Za Za2 +x2
dy

.in
= √ 2 dx.
x=0 y=0 x2 + a2 + y2
Za ! √a2 +x2
1 y

ng
= √ tan−1 √ dx.
x2 + a2 x2 + a2 0
x=0
Za
1

eri
= √ (tan−1 1 − tan−1 0)dx
x2 + a2
x=0
Za Za
π π 1 1
= √
2
x +a 2 4
dx = ·
4

x + a2
2
dx
e
gin
x=0 x=0
π  p a
= log x + x2 + a2
4  0
π p  
= log a + 2a2 − log a
En

4
πh  √  i
= log a + 2a − log a
4  √ 
a 1+ 2 √ 
arn

π π 
= log = log 1 + 2 .
4 a 4
"
Example 4.9. Evaluate xydxdy over the positive quadrant of the circle
Le

R
x2 + y2 = a2 . [Jan 2014, Jun 1996]
Solution. The region of integration is bounded by the coordinate axes y = 0, x = 0
w.

and the circle x2 + y2 = a2 . Y


Since the innermost integration is w.r.t.
B(0, a)
x, divide the region into strips parallel
ww

P Q
to the x-axis. Along a typical strip, the

q is constant but x varies


y−coordinate x=0
from 0 to a2 − y2 . Finally y varies from X
(0, 0) y=0 (a, 0)
0 to a.

Visit For More : www.LearnEngineering.in


Visit For More : www.LearnEngineering.in

242 Engineering Mathematics - I

√2 2 √
" Za Za −y Za " 2 # a2 −y2
x
∴ xydxdy = xydxdy = y dy
2 0
R y=0 x=0 y=0
Za #a
1 2 y2 y4 1 a4 a4 a4 a4
" " # " #
1 1

.in
2 2
= y(a − y )dy = a − = − = 1− = .
2 2 2 4 0 2 2 4 4 2 8
y=0

ng
Example 4.10. Evaluate xydxdy over the region in the positive quadrant
x y
bounded by + = 1.
a b

eri
Solution. The region of integration is bounded by the coordinate axes y = 0, x = 0
x y
and the line + = 1.
a b
Y
Since the innermost integration (0, b)
e B
gin
is w.r.t. x, divide the region into strips x y
x=0 a + b =1
parallel to the x-axis. Along a typical P Q

strip PQ, x varies from 0 to a 1 − by . y


 
En

(0, 0) y=0
X
A
varies from 0 to b. a(1− y ) (a, 0)
" Zb Z b
∴ xydxdy = xydxdy
arn

R y=0 x=0
Zb !a(1− by )
x2
= y· dy
2
Le

0
y=0
Zb "  #
1 y 2
= y a2 1 − − 0 dy
2 b
w.

y=0
Zb
a2 y2 2y
!
= y 1+ 2 − dy
ww

2 b b
y=0
Zb
a2 y3 2y2
!
= y+ 2 − dy
2 b b
y=0

Visit For More : www.LearnEngineering.in


Visit For More : www.LearnEngineering.in

Multiple Integrals 243

#b
a2 y2 y4 2y3
"
= + −
2 2 4b2 3b 0
2
" 2 4 2b3
#
a b b
= + 2−
2 2 4b 3b
2
" 2 2 2b2
#
a b b
= + −
2 2 4 3

.in
2
a 6b + 3b − 8b2
" 2 2
#
=
2 12
2 2 a b2
2

ng
a b
= =
2 12 24
"
Example 4.11. Evaluate xydxdy where R is the region bounded by the

eri
R
parabola y2 = x, the x−axis and the line x + y = 2 lying on the first quadrant.
Solution. Solving y2 = x and x + y = 2 we get the points of intersection.
e
y2 + y − 2 = 0 ⇒ (y + 2)(y − 1) = 0 ⇒ y = 1, −2. When y = 1, x = 1.
gin
Y
Divide the region into strips parallel to
En

the x−axis . Along a typical strip, x (0, 2)


y2 = x
varies from y2 to 2−y and finally y varies (1, 1)

from 0 to 1.
arn

P Q
(2, 1)
X
x=2−y
" Z1 Z2−y Z1 !2−y
x2
Le

xydxdy = xydxdy = y dy
2 y2
R y=0 x=y2 y=0

Z1 Z1 
w.

y  1 
= (2 − y)2 − y4 dy = y 4 + y2 − 4y − y4 dy
2 2
y=0 y=0
ww

Z1 !1
1 
3 2 5
 1 y2 y4 y3 y6
= 4y + y − 4y − y dy = 4 + −4 −
2 2 2 4 3 6 0
0
! " #
1 4 1 4 1 1 24 + 3 − 16 − 2 1 9 3
= + − − = = × = .
2 2 4 3 6 2 12 2 12 8

Visit For More : www.LearnEngineering.in


Visit For More : www.LearnEngineering.in

244 Engineering Mathematics - I

"
Example 4.12. Evaluate xydxdy where A is the region bounded by x = 2a and
A
the curve x2 = 4ay. [Jan 2006]
Solution.

.in
Let us evaluate this double integral first
w.r.t. y and then w.r.t. x. Divide the
region into strips parallel to the y−axis.

ng
PQ is one such strip. x2 = 4ay Q x = 2a
x2
Along this strip, y varies form 0 to

eri
4a
and finally x varies from 0 to 2a. X
P

" Z2a Z4a


x2
e
gin
xydxdy = xydydx
A x=0 y=0
x2
Z2a " 2 # 4a
En

y
= x dx
2 0
x=0
Z2a Z2a 5
x4
!
arn

x
= x 2
dx = dx
32a 32a2
x=0 x=0
#2a
x6 a4
"
1 1 6
= = (64a − 0) = .
Le

32a2 6 0 32a2 × 6 3
"
Example 4.13. Evaluate xy(x + y)dxdy over the area between y = x2 and y = x.
w.

R
Solution. Solving the two equations we get the points of intersection.
y = x2 , y = x.
ww

=⇒ x2 = x =⇒ x2 − x = 0 =⇒ x(x − 1) = 0 =⇒ x = 0 & x = 1.
When x = 0, y = 0, when x = 1, y = 1.
∴ The points of intersection are (0, 0) and (1, 1).

Visit For More : www.LearnEngineering.in


Visit For More : www.LearnEngineering.in

Multiple Integrals 245

Y
Let us evaluate this double
integral, first w.r.t. y and then w.r.t. x.
y=x (1, 1)
Divide this region into strips parallel to
Q
y = x2
the y−axis. PQ is one such strip. Along
P

.in
this strip, y varies from x2 to x and x X
(0, 0)
varies from 0 to 1.
" Z1 Zx

ng
xy(x + y)dxdy = (x2 y + xy2 )dydx
R x=0 y=x2

Z1 !x !x !

eri
y3 2 y2
= x +x dx
3 x2 2 x2
x=0
Z1
=
x 3 6 x2 2
e4
!
(x − x ) + (x − x ) dx
gin
3 2
0
Z1
x4 x6 x4 x7
!
= − + − dx.
2 2 3 3
En

0
!1
x5 x7 x5 x8 1 1 1 1 3
= − + − = − + − = .
10 14 15 24 0 10 14 15 24 56
arn

"
Example 4.14. Evaluate ydxdy over the region R bounded by y = x and
R
y = 4x − x2 .
Le

Solution. The equation of the parabola is


y = −(x2 − 4x) = −((x − 2)2 − 4) = −(x − 2)2 + 4
w.

⇒ y − 4 = −(x − 2)2
vertex is (2, 4) and it is open downwards.
ww

Solving the two equations we get


x = 4x − x2 ⇒ x2 + x − 4x = 0 ⇒ x2 − 3x = 0 ⇒ x(x − 3) = 0 ⇒ x = 0, x = 3.
When x = 0, y = 0.
When x = 3, y = 12 − 9 = 3.

Visit For More : www.LearnEngineering.in


Visit For More : www.LearnEngineering.in

246 Engineering Mathematics - I

The points of intersection are (0, 0), (3, 3).


Y
Let us evaluate the double integral first
(2, 4)
w.r.t. y and then w.r.t. x. Divide the Q
(3, 3)
region into strips parallel to the y−axis.
y = 4x − x2

.in
y=x
PQ is one such strip. Along this strip, y
P
varies from x to 4x − x2 . Finally x varies X
(0, 0)
from 0 to 3.

ng
" Z 2
Zx=3 4x−x Z3 2 !4x−x2
y
ydxdy = ydydx = dx

eri
2 x
R x=0 y=x x=0
Z3 Z3 
1 
2 2 1 2
 
16x2 + x4 − 8x3 − x2 dx
=
2
(4x − x ) − x dx =
2
e
gin
x=0 0
!3
x3 x5 x4
35
# "
1 1 3 4
= 15 + −8 = 5×3 + −2×3
2 3 5 4 0 2 5
! !
1 9 27 25 + 9 − 30 27 4 54
En

= 27 5 + − 6 = = = .
2 5 2 5 2 5 5

4.2 Change of order of integration


arn

Za2 y=g
Z 2 (x)
Consider the integral f (x, y)dydx.
Le

x=a1 y=g1 (x)

To evaluate this integral, since the innermost integral has functions of x as


w.

limits, first we integrate w.r.t. y and then we integrate w.r.t x. Sometimes, it is


very difficult to evaluate in this order. By changing the order of integration we
ww

can easily evaluate. In this example the original order of integration is first w.r.t.
y and then w.r.t. x. By the change of order of integration we need to integrate
first w.r.t. x and then w.r.t. y. For this, divide the region into strips parallel to the
x-axis. Consider one such strip. The limits for x will be functions of y which are the

Visit For More : www.LearnEngineering.in


Visit For More : www.LearnEngineering.in

Multiple Integrals 247

values of x corresponding to the curves in which the ends of the strip lies. Finally
the values of y will be constants according to the movement of this strip along the
given region. If the first integration is with respect to y then divide the region into
strips parallel to the y−axis and then find the corresponding limits for y and x and
then evaluate.

.in
✎ ☞
Worked Examples
✍ ✌

ng
2
Z1 Z x

Example 4.15. Change the order of integration in f (x, y)dydx. [Jan 2014]
0 0 √

eri
Solution. The region of integration is given by y = 0, y = 2 x, x = 0, x = 1.
i.e., y = 0, y2 = 4x, x = 0, x = 1.

e
gin
Y
By changing the order of
(1, 2)
integration, the first integration is w.r.t. 4 x
y =
2
x=0
x. Hence, divide the region into strips
En

P Q
x=1
parallel to the x−axis. PQ is one such
y2 y=0 X
strip. Along this strip, x varies from
4
arn

to 1. Finally y varies from 0 to 2.



2
Z1 Z x Z2 Z1
∴ f (x, y)dydx = f (x, y)dxdy.
Le

0 0 y=0 2
x= y4

Za Za
w.

x
Example 4.16. Change the order of integration in dxdy and hence
x2 + y2
0 y
ww

evaluate. [Jan 2014, Jun 2013, Jun 2011]


Solution. The region of integration is given by x = y, x = a, y = 0, y = a.
i.e., the region of integration is bounded by the lines x = 0, x = a, y = 0, y = x
By changing the order of integration, the first integration is w.r.t y. The double

Visit For More : www.LearnEngineering.in


Visit For More : www.LearnEngineering.in

248 Engineering Mathematics - I

integral takes the form


Za Za "
x x
dxdy = dydx.
x + y2
2 x2 + y2
0 y R

Divide the shaded region into strips parallel to the y-axis. PQ is one such strip.

.in
Along this strip, y varies from 0 to x and finally x varies from 0 to a.

ng
Y
y=a
Za Za Za Zx (a, a)
x x

eri
∴ dxdy = dydx Q
x + y2
2 x + y2
2
0 y x=0 y=0
x=0 x=a

y
Za

=
!x

x
1 −1 y
= x
x
tan
x 0
dx
e X
gin
x=0 (0, 0) P (a, 0)
Za
π π πa
= dx = (x)a0 = .
4 4 4
x=0
En

Z∞ Z∞
e−y
Example 4.17. Evaluate dydx by changing the order of integration.
y
0 x
arn

[May 2011, Jan 1999]


Solution. The region of integration is y = x, y = ∞, x = 0, x = ∞.
Here, the first integration is w.r.t y and then we integrate w.r.t x.
Le

Y
By changing the order of integration,
w.

the innermost integration must be


w.r.t.x.
ww

x=0
So, we divide the region of integration
y=x
into strips parallel to the x−axis.
X
Along a typical strip, x varies from 0 to (0, 0) y=0

y and finally y varies from 0 to ∞.

Visit For More : www.LearnEngineering.in


Visit For More : www.LearnEngineering.in

Multiple Integrals 249

Z∞  Zy −y 
 y 
Z∞ Z∞ Z∞ −y
 
e−y  e Z
e
 
dydx = dx dy =  dx dy
 
∴ 
y y  y
0 x y=0 x=0 y=0 0
Z∞ −y Z∞ !∞
e y e−y
= (x)0 dy = −y
e dy = = −[e−∞ − e0 ] = −[0 − 1] = 1
y −1 0

.in
x=0 0

Z3 Z4−y
Example 4.18. Evaluate (x + y)dxdy by changing the order of integration.

ng
0 1
[Jan 2003]
p

eri
Solution. The region of integration is bounded between x = 1, x = 4 − y and
y = 0, y = 3. i.e.,x = 1, x2 = 4 − y and y = 0, y = 3.
x = 1, x2 = −(y − 4) and y = 0, y = 3.

e
gin
The region is bounded between x = 1
Y
and the parabola x2 = −(y−4) with vertex
(0, 4)
En

(0, 4) and y = 0 and y = 3. By changing


the order of integration the innermost y=3 (1, 3)

integration is w.r.t.y. Hence, divide this


arn

x=1 x2 = 4−y
region into strips parallel to the y−axis.
X
Along a typical strip y varies from 0 to (0, 0) (1, 0) (2, 0)
y=0
4 − x2 and finally x varies from 1 to 2.
Le


Z3 Z4−y 4−x2
Z2 Z Z2 !4−x2
y2
(x + y)dxdy = (x + y)dydx = xy + dx
w.

2 0
0 1 x=1 y=0 x=1
Z2
(4 − x2 )2
!
2
ww

= x(4 − x ) + dx
2
x=1
Z2
16 + x4 − 8x2
!
3
= 4x − x + dx
2
1

Visit For More : www.LearnEngineering.in


Visit For More : www.LearnEngineering.in

250 Engineering Mathematics - I

!2
x2 x4 1 x5 x3
= 4 − + 8x + −4
2 4 2 5 3 1
1 1 4
= 2(4 − 1) − (16 − 1) + 8(2 − 1) + (32 − 1) − (8 − 1)
4 10 3
15 31 28 840 − 225 + 186 − 560 241
=6− +8+ − = = .

.in
4 10 3 60 60

b a2 −x2
Za Za
Example 4.19. Change the order of integration in x2 dydx and hence

ng
0 0
evaluate it. [Jan 2012]

b a2 −x2

eri
Solution. The region of integration is given by y = 0, y = a , x = 0, x = a.
b2
i.e., y = 0, y2 = 2 (a2 − x2 )
a
y2 x2
b2
= 1 −
a2 e
gin
x2 y2
+ = 1.
a2 b2
After changing the order of integration,
Y
En

the first integration is w.r.t. x and finally


x=0
w.r.t. y. Divide the region into strips (0, b)
Q x=a
parallel to the x− axis. PQ is one such P
arn

y=0
X
strip.
q Along this strip x varies from 0 to
2
a 1 − by2 . Finally y varies from 0 to b.
r
Le

√ 2
b a2 −x2 a 1− y2
Za Za Zb Z b

∴ x2 dydx = x2 dxdy y = b sin θ


w.

0 0 y=0 x=0
r dy = b cos θdθ
2
Zb 3 a
! 1− y2
x b when y = 0, b sin θ = 0
= dy
ww

3 0
y=0 ⇒θ=0
Zb when y = b, b sin θ = b
1 a3 2
= (b − y2 )3/2 dy π
3 b3 ⇒ sin θ = 1 ∴ θ =
0 2
Zb
a3
= 3 (b2 − y2 )3/2 dy
3b
0

Visit For More : www.LearnEngineering.in


Visit For More : www.LearnEngineering.in

Multiple Integrals 251

π
Z2
a3
= 3 (b2 − b2 sin2 θ)3/2 b cos θdθ
3b
0
π
Z2
a3
= 3 b3 (1 − sin2 θ)3/2 b cos θdθ

.in
3b
0
π
Z2
a3 b

ng
= (cos2 θ)3/2 cos θdθ
3
0
π
Z2

eri
a3 b
= cos3 θ · cos θdθ
3
0
π

=
a3 b
Z2
cos4 θdθ e
gin
3
0
a3 b 3 1 π πa3 b
= · · · = .
3 4 2 2 16

En

Za a+ Z a2 −y2

Example 4.20. Change the order of integration in xydxdy and then


0 a−

arn

a2 −y2
evaluate it. [Jan 2000]
Solution. The region
q of integration
q is between
x = a − a2 − y2 , x = a + a2 − y2 , y = 0, y = a.
Le

i.e.,(x − a)2 = a2 − y2 , (x − a)2 = a2 − y2 , y = 0, y = a.


(x − a)2 + y2 = a2 , (x − a)2 + y2 = a2 , y = 0, y = a.
w.

∴ The region of integration is the circle Y


y=a
ww

with (a, 0) as centre and radius a and


between y = 0 and y = a.
y=0 X
When we change the order of (a, 0)

integration, the inner most integration


must be w.r.t y.

Visit For More : www.LearnEngineering.in


Visit For More : www.LearnEngineering.in

252 Engineering Mathematics - I

Hence, divide the region into strips parallel to y−axis. Along a typical strip, y
p
varies from 0 to a2 − (x − a)2 and finally x varies from 0 to 2a.
√2 2 √
2 2
Za a+ Z a −y Z2a aZ−(x−a)
xydxdy = xydydx
0
√ 2 2 x=0 y=0
a− a −y

.in
Z2a a2 −(x−a)2
y2
!
= x dx
2 0
x=0

ng
Z2a Z2a
1 2 1 2
= x(a − (x − a) )dx = (a2 x − x(x2 − 2ax + a2 ))dx
2 2

eri
0 0
Z2a  !2a !2a 
1 2 3 2 2 1  x3 x4 
= (a x − x + 2ax − a x)dx = 2a − 
2 2 3 0 4 0 
0
1 2a 3 16a4
" #
a4 16
" #
e
1 4 2a4
gin
= 8a − = − 4 = a4 = .
2 3 4 2 3 2 3 3

Example 4.21. Change the order of integration and hence evaluate


En

Za Za
y2
p dydx.
√ y4 − a2 x2
0 ax

arn

Solution. The region of integration is given by y = ax, y = a, x = 0 and x = a.


i.e., y2 = ax, y = a, x = 0 and x = a. By changing the order of integration, the first
integration is w.r.t. x.
Le

Y
Hence, we divide the region into strips
y=a
w.

parallel to the x-axis. Along a typical


y2 x=0 y=0
strip x varies from 0 to and finally y y2 = ax
a
X
ww

varies from 0 to a.
y2
Za Za Za Za
y2 y2
p dydx = p dxdy
√ y4 − a2 x2 y4 − a2 x2
0 ax y=0 x=0

Visit For More : www.LearnEngineering.in


Visit For More : www.LearnEngineering.in

Multiple Integrals 253

y2
Za Za
1 y2
= r  dxdy
a y2 2
y=0 x=0
a − x2

Za    ya2
1  2 −1  x 
= y sin  2  dy

.in
a y
y=0 a x=0
Za
1
= y2 (sin−1 1)dy

ng
a
0
!a
π y3
=

eri
2a 3 0
π a3πa2
= = .
2a 3 6

e
Note. While changing the order of integration, we may come across situations
gin
where the region has to be divided into several parts and evaluation has to be
done accordingly. The following examples illustrate this aspect.
Z1 Z2−y
En

Example 4.22. Change the order of integration and hence evaluate xydxdy.
0 0
[Dec 2011]
arn

Solution. The region of integration is given by x = 0, x = 2 − y, y = 0, y = 1.


x+y=2
Le

Y
The region of integration is
OACE. After changing the order of B(0, 2)
w.

x+y=2
integration, the first integration is w.r.t.
y and final integration is w.r.t. x. E Q C(1, 1)
y=1
ww

Q′
Divide the region into two regions x=0 x=1
A(2, 0)
ODCE and DAC. X
O(0, 0) P D(1, 0) P′
Z1 Z2−y " " y=0
∴ xydxdy = xydydx+ xydydx.
0 0 ODCE DAC

Visit For More : www.LearnEngineering.in


Visit For More : www.LearnEngineering.in

254 Engineering Mathematics - I

Consider the region ODCE


Divide the region into strips parallel to the y−axis. PQ is one such strip.
Along this strip, y varies from 0 to 1 and finally x varies from 0 to 1.
" Z1 Z1
∴ xydydx = xydydx.
ODCE x=0 y=0

.in
Z1 !1
y2
= x· dx
2 0
x=0

ng
Z1
1
= x(1 − 0)dx
2
0

eri
Z1
1
= xdx
2

1 x2
!1
0

e
gin
= ·
2 2 0
1
= (1 − 0)
4
En

1
= .
4
Consider the region DAC
Divide this region into strips parallel to y−axis. P′ Q′ is one such strip.
arn

Along this strip, y varies from 0 to 2 − x and finally x varies from 1 to 2.


" Z2 Z2−x
∴ xydydx = xydydx.
Le

DAC x=1 y=0


Z2 !2−x
y2
= x· dx.
w.

2 0
x=1
Z2
1
x(2 − x)2 dx.
ww

=
2
1
Z2
1
= x(4 + x2 − 4x)dx.
2
0

Visit For More : www.LearnEngineering.in


Visit For More : www.LearnEngineering.in

Multiple Integrals 255

Z2
1
= (4x + x3 − 4x2 )dx.
2
0
 !2 !2 !2 
1  x2 x4 x3 
= 4 · + −4· 
2 2 1 4 1 3 1
!
1 1 4

.in
= 2[4 − 1] + [16 − 1] − (8 − 1)
2 4 3
!
1 15 28
= 6+ −
2 4 3

ng
!
1 72 + 45 − 112
=
2 12

eri
1 5
= (5) =
24 24
Z1 Z2−y
1 5 6 + 5 11
∴ xydxdy = +
4 24
=
24e = .
24
gin
0 0

Z1 Z2−y
Example 4.23. Change the order of integration in xydxdy and hence
En

0 y
evaluate. [Jan 2001]
Solution. The region of integration is x = y, x = 2 − y, y = 0, y = 1.
arn

i.e.,x = y, x + y = 2, y = 0, y = 1. Solving we get 2x = 2 ⇒ x = 1.


B is (1, 1), A is (2, 0).

Y
Le

The region of integration is OAB.


2
=

After the change of order of integration (0, 2)


y
x+

y
w.

the innermost integration is w.r.t.y and


x

B(1, 1)
the next integration is w.r.t.x. x=0
Q
Q′
ww

Divide this region into two regions OCB x=1


A(2, 0)
and CBA. X
O(0, 0) P C(1, 0) P′
Z1 Z2−y " " y=0
xydxdy = xydydx + xydydx.
0 y OCB CBA

Visit For More : www.LearnEngineering.in


Visit For More : www.LearnEngineering.in

256 Engineering Mathematics - I

Consider the region OCB


Divide this region into strips parallel to the y−axis.
One such strip is PQ.
Along this strip y varies from 0 to x.
Finally x varies from 0 to 1.

.in
" Z1 Zx Z1 !x
y2
xydydx = xydydx = x dx
2 0

ng
OCB x=0 y=0 x=0
Z1 Z1
1 2 1
= x(x − 0)dx = x3 dx
2 2

eri
0 0
!1
1 x4
=
2 4 0
1
= . e
gin
8

Consider the region CBA


Divide this region into strips parallel to the y−axis.
En

One such strip is P′ Q′ . Along this strip y varies from 0 to 2 − x and finally x varies
from 1 to 2.
arn

" Z2 Z2−x Z2 !2−x


y2
xydydx = xydydx = x dx
2 0
CBA x=1 y=0 x=1
Le

Z2 Z2
1 1
= x(2 − x)2 dx = x(4 + x2 − 4x)dx
2 2
x=1 x=1
w.

Z2  !2 !2 !2 
1 3 1  x2
2 x4 x3 
= (4x + x − 4x )dx = 4 + −4 
2 2 2 1 4 1 3 1
ww

x=1
" # " #
1 1 4 1 15 28
= 2(4 − 1) + (16 − 1) − (8 − 1) = 6+ −
2 4 3 2 4 3
! !
1 72 + 45 − 112 1 5 5
= = = .
2 12 2 12 24

Visit For More : www.LearnEngineering.in


Visit For More : www.LearnEngineering.in

Multiple Integrals 257

Z1 Z2−y
1 5 3+5 8 1
∴ xydxdy = + = = = .
8 24 24 24 3
0 y

√a √
Z 2 Za2 −x2
Example 4.24. Evaluate y2 dydx by changing the order of integration.

.in
0 x
p a
Solution. The region of integration is bounded by y = x, y = a2 − x2 , x = 0, x = √ .
2
a
i.e.,y = x, x2 + y2 = a2 , x = 0, x = √ .

ng
2 Y
The required region is OAB. Draw AC B(0, a)
P′ Q′

eri
 
perpendicular to OB. C A √a , √a
2 2

Divide this region into two parts OAC P Q


X
and CAB. After changing the order of O x= √a

integration, the innermost integration e x=0


2
gin
must be w.r.t.x.
√a √
Z 2 Za2 −x2 " "
2 2
y2 dxdy.
En

∴ y dydx = y dxdy+
0 x OAC CAB

Consider the region OAC


arn

Divide this region into strips parallel to the x−axis.


Let PQ be one such strip.
a
Le

Along this strip x varies from 0 to y and finally y varies from 0 to √ .


√a √a 2
" Z 2 Zy Z2
y
y2 dxdy = y2 dxdy = y2 (x)0 dy
w.

OAC y=0 0 y=0


√a
2 ! √a
y4 1 a4 a4
Z
2
ww

3
= y dy = = = .
4 0 4 4 16
0

Consider the region CAB


Divide this region into strips parallel to the x−axis.

Visit For More : www.LearnEngineering.in


Visit For More : www.LearnEngineering.in

258 Engineering Mathematics - I

p
One such strip is P′ Q′ . Along this strip x varies from 0 to a2 − y2 and finally y
varies from √a to a.
2

"
2 2
Za Za −y Za √2 2
a −y
y2 dxdy = 2
y dxdy = 2
y (x)0 dy
CAB y= √a x=0 y= √a
2 2

.in
Za q
= y2 a2 − y2 dy
√a

ng
2
" #
a 1 π
y = a sin θ, dy = a cos θdθ, when y = √ ⇒ sin θ = √ , θ =
2 2 4

eri
 π
When y = a ⇒ a = a sin θ ⇒ sin θ = 1 ⇒ θ =
2
π π
Z2 Z2
= a2 sin2 θa cos θa cos θdθ =
e
a4 sin2 θ cos2 θdθ
gin
π π
4 4
π π
2 2
a4 a4
Z Z
2 2
= 4 sin θ cos θdθ = (2 sin θ cos θ)2 dθ
4 4
En

π π
4 4
π π
Z2 Z2
a4 a4 1 − cos 4θ
= (sin2 2θ)dθ = dθ
4 4 2
arn

π π
4 4
 π π 
4
Z2 Z2 
a  
=  dθ − cos 4θdθ
8 
Le


π π
4 4
 !π 
a4  π2 sin 4θ 2 
= (θ) π − 
8  4 4 π
w.

a4 π π 1
!
= − − (sin 2π − sin π)
8 2 4 4
ww

a4 π πa4
= = .
8 4 32

a4 πa4 a4
∴ Value of the integral= + = [π + 2].
16 32 32

Visit For More : www.LearnEngineering.in


Visit For More : www.LearnEngineering.in

Multiple Integrals 259


Z1 Z2−x2
x
Example 4.25. Evaluate p dydx by changing the order of
x2 + y2
0 x
integration.
p
Solution. The region of integration is y = x, y = 2 − x2 , x = 0, x = 1.
i.e., y = x, x2 + y2 = 2, x = 0, x = 1. Y

.in x
After changing the order of √

y=
B(0, 2)
P′ Q′
integration the inner most integration C A(1, 1)

ng
is w.r.t.x and the next integration is P Q

y=0
X
w.r.t. y. The given region is OAB. Draw O x=1
x=0

eri
AC perpendicular to OB.
Divide this region into two parts OAC
and CAB.

Z1 Z2−x2 " e "
gin
x x x
∴ p dydx = p dxdy+ p dxdy
x2 + y2 x2 + y2 x2 + y2
0 x OAC CAB
En

Consider the region OAC


Divide this region into strips parallel to the x−axis.
Let PQ be one such strip. Along this strip x varies from 0 to y and finally y varies
arn

from 0 to 1.
" Z1 Zy Z1 Zy
x x 1 2x
dxdy = dxdy = dxdy
Le

p p p
x2 + y2 2
x +y2 2 x2 + y2
OAC y=0 x=0 y=0 x=0
Z1 Zy Z1  2 1 y
1 2 2 − 21 2 2 1  (x + y2 ) 2 
= (x + y ) d(x + y )dy =  dy
w.

1

2 2 2
y=0 0 y=0 0

Z1   Z1 √
ww

2 21
= (2y ) − y dy = ( 2y − y)dy
y=0 y=0

√ y2 1
!1
y2 2 1 1 √
!
= 2 − = − = ( 2 − 1).
2 0 2 0 2 2 2

Visit For More : www.LearnEngineering.in


Visit For More : www.LearnEngineering.in

260 Engineering Mathematics - I

Consider the region CAB


Divide this region into strips parallel to the x−axis.
p
One such strip is P′ Q′ . Along this strip x varies from 0 to 2 − y2 and finally y

varies from 1 to 2.
√ √ √ √ 2
"
2
Z 2 Z2−y Z 2 2 2−y

.in
1 
x x 1 2
 (x + y ) 
2 
p dxdy = p dxdy =  1
 dy
x2 + y2 x2 + y2 2 2
CAB y=1 x=0 y=1 0

ng

Z2 √ 2
y2
!
1
= (2 − y)dy =
2 2y −
2 1
y=1

eri
√ √ 1 √ √ 1
2( 2 − 1) − (2 − 1) = 2( 2 − 1) −
=
√ 2 2
2−1 √ √ 1
Value of the integral =
√ 2
+ 2( 2 − 1) −
√ √ 2
e
gin
2 − 1 + 2 2( 2 − 1) − 1
=
√ 2 √ √
2−1+4−2 2−1 2− 2
= = .
2 2
En

Za 2a−x
Z
Example 4.26. Change the order of integration in the integral xydydx and
arn

0 x2
a
evaluate it. [Jan 2013]
x2
Solution. The region of integration is given by y = , y = 2a − x, x = 0, x = a.
a
x2 = ay, x + y = 2a, x = 0, x = a.
Le

Y
After changing the order of
w.

B(0, 2a) x2 = ay
integration, the first integration is w.r.t.
Q′
P′
y and the final integration is w.r.t. x.
ww

C A(a, a) 2a
y=
OAB is the required region. Divide this P Q x+
X
region into OAC and CAB. y=0 O x=a
x=0
Za 2a−x
Z " "
∴ xydydx = xydxdy+ xydxdy.
0 x2 OAC CAB
a

Visit For More : www.LearnEngineering.in


Visit For More : www.LearnEngineering.in

Multiple Integrals 261

Consider the region OAC


Divide this region into strips parallel to the x−axis. PQ is one such strip.

Along this strip, x varies from 0 to ay and finally y varies from 0 to a.

" Za Z ay
∴ xydxdy = xydxdy
OAC y=0 x=0

.in
Za ! √ay
x2
= y dy
2 0
y=0

ng
Za
1
= y · (ay − 0)dy
2
0

eri
Za !a
a 2 a y3 a 3 a4
= y dy = = ·a = .
2 2 3 0 6 6
0
Consider the region CAB
e
gin
Divide this region into strips parallel to the x−axis. P′ Q′ is one such strip.

" 2a−y
Z2a Z
En

∴ xydxdy = xydxdy
CAB y=a x=0
Z2a
arn

!2a−y
x2
= y dy
2 0
y=a
Z2a
1
Le

= y(2a − y)2 dy
2
a
Z2a
1
w.

= y(4a2 + y2 − 4ay)dy
2
a
Z2a
ww

1
= (4a2 y + y3 − 4ay2 )dy
2
a
 !2a !2a !2a 
1  2 y2 y4 y3 
= 4a + − 4a · 
2 2 a 4 a 3 a 

Visit For More : www.LearnEngineering.in


Visit For More : www.LearnEngineering.in

262 Engineering Mathematics - I

!
1 2 2 2 1 4 4 4a 3 3
= 2a (4a − a ) + [16a − a ] − [8a − a ]
2 4 3
4 4
!
1 15a 28a
= 6a4 + −
2 4 3
1 72a + 45a − 112a4
4 4 a4 5a4
!
= = [72 + 45 − 112] = .
2 12 24 24
Za 2a−x

.in
a4 5a4 4a4 + 5a4 9a4 3a4
Z
∴ xydydx = + = = = .
6 24 24 24 8
0 x2
a

ng
Z1 Z2−x
Example 4.27. Change the order of integration in the integral xydydx and

eri
0 x2
evaluate it.
Solution. In the above Example, put a = 1.
Z1 Z2−x
3 e
gin
∴ xydydx = .
8
0 x2

4.3 Double integral in polar coordinates


En

Usually the double integral in polar coordinates (r, θ) is of the form


arn

Zθ2 Zf2 (θ)


f (r, θ)drdθ.
θ=θ1 r= f1 (θ)
Le

Result. Reduction
π
formula
π
Z2 Z2
w.

cosn θdθ = sinn θdθ


0 0
n−1n−3 2
· · · 1 if n is odd
ww

=
n n−2 3
n−1n−3 1π
= ··· if n is even.
n n−2 22
✎ ☞
Worked Examples
✍ ✌

Visit For More : www.LearnEngineering.in


Visit For More : www.LearnEngineering.in

Multiple Integrals 263

Zπ Za
Example 4.28. Evaluate rdrdθ. [Jan 2014]
0 0

Zπ Za Zπ !a Zπ
r2 a2 a2 π πa2
Solution. rdrdθ = dθ = dθ = [θ] = .
2 0 2 2 0 2
0 0 0 0

.in
Zπ Z
sin θ

Example 4.29. Evaluate rdrdθ. [Jan 2014]


0 0

ng
Zπ Z
sin θ Zπ 2 !sin θ
r
Solution. rdrdθ = dθ

eri
2 0
0 0 0

1 11π π
= sin2 θdθ = = .
2
0
e222 8
gin
Zπ Z
cos θ

Example 4.30. Evaluate rdrdθ.


0 0
En

Zπ Z
cos θ Zπ 2 !cos θ
r
Solution. rdrdθ = dθ
2 0
arn

0 0 0

1
= cos2 θdθ
2
0
Le

π
Z2
1 1π π
= 2 cos2 θ = = .
2 22 4
0
w.

Z2 Zπ
Example 4.31. Evaluate r sin2 θdrdθ. [Jan 2013]
ww

0 0

Z2 Zπ
 

r  sin2 θdθ dr

Solution. I =

 
0 0

Visit For More : www.LearnEngineering.in


Visit For More : www.LearnEngineering.in

264 Engineering Mathematics - I

 π 
Z2  Z2 
 2 
= r 2 sin θdθ dr
 
0 0
Z2 !2
1 π π r2 π
=2 r· · dr = · = · 4 = 4.
2 2 2 2 0 4
0

.in
π
Z2 2Zcos θ
Example 4.32. Evaluate r2 drdθ.
−π

ng
2
0

π π π
Z2 !2 cos θ Z2 Z2
r3 1 8 16 2 32
8 cos3 θdθ = 2 cos3 θdθ =

eri
Solution. I = dθ = = .
3 0 3 3 3 3 9
−π −π 0
2 2

Z2 aZcos θ p
r a2 − r2 drdθ.
π

e
gin
Example 4.33. Evaluate
0 0

π π
Z2 aZcos θ p Z2 aZcos θ
−1
En

1
Solution. 2 2
r a − r drdθ = (a2 − r2 ) 2 (−2rdr)dθ
2
0 0 0 0
π
Z  2 2 3 a cos θ
−1  (a − r2 ) 2 
arn

=  3
 dθ
2 2
0 0
π
Z  2
−1 2 2 2 2 3
2 3

Le

= (a − a cos θ) − (a ) 2 2 dθ
2 3
0
π
Z 2
−1
w.

= (a3 sin3 θ − a3 )dθ


3
0
 π π 
ww

 Z2 Z2 
−1  3
sin3 θdθ − a3 dθ

= a
3  
0 0
−a3 2 −a3 2 π
! !
π
= 1− = −
3 3 2 3 3 2

Visit For More : www.LearnEngineering.in


Visit For More : www.LearnEngineering.in

Multiple Integrals 265

a3 π 2 a3 3π − 4 a3
! !
= − = = (3π − 4).
3 2 3 3 6 18
"
Example 4.34. Evaluate r3 drdθ where A is the area between the circles
A
r = 2 sin θ and r = 4 sin θ. [Jan 2006]

.in
Solution. The region of integration is the shaded portion.

ng
The first integration is w.r.t. r.
Q
Consider the radius vector PQ. In the

eri
region r varies from 2 sin θ to 4 sin θ. θ x = 4 sin θ
P
varies from 0 to π. 4 x = 2 sin θ
2
" Zπ 4Zsin θ Zπ !4 sin θ θ
3
r drdθ = 3
r drdθ =
r4
e dθ O
r
gin
4 2 sin θ
A θ=0 r=2 sin θ θ=0

1
= (44 sin4 θ − 24 sin4 θ)dθ
4
En

θ=0

1
= (256 sin4 θ − 16 sin4 θ)dθ
4
arn

θ=0
Zπ Zπ
1 4
= 240 sin θdθ = 60 sin4 θdθ
4
θ=0 θ=0
Le

π
Z 2

= 60 × 2 sin4 θdθ
θ=0
w.

3 1 π 45π
= 120 = .
422 2
"
ww

Example 4.35. Evaluate r3 drdθ over the area bounded between the circles
R
r = 2 cos θ and r = 4 cos θ.
Solution.

Visit For More : www.LearnEngineering.in


Visit For More : www.LearnEngineering.in

266 Engineering Mathematics - I

θ
We first integrate w.r.t. r. Consider the π
θ= 2 P
radial strip OP. Along the strip r varies
−π
from 2 cos θ to 4 cos θ. θ varies from to
2
π r
. O
2

.in
π π
" Z2 4Zcos θ Z2 4 !4 cos θ
3 3 r
r drdθ = r drdθ = dθ
4 2 cos θ θ = − π2

ng
R −π 2 cos θ −π
2 2
π π
Z2 Z2
1 1
= (44 cos4 θ − 24 cos4 θ)dθ = (256 cos4 θ − 16 cos4 θ)dθ

eri
4 4
−π −π
2 2
π π
Z 2 Z2
1 3 1 π 45π
=
4
240 cos4 θdθ = 60(2)
e cos4 θdθ = 120
422
=
2
.
gin
−π 0
2

4.4 Change of Variables in double integral


En

The evaluation of the double integral can be made easy in several


occasions by changing the variables.
arn

Case"(i) Change of variables from (x, y) to u and v.


Let f (x, y)dxdy be the given double integral.
R
Le

Let x = g(u, v) and y = h(u, v). By this transformation the elementary area dxdy is
∂(x, y)
transformed to dxdy = |J|dudv where J = is the Jacobian of the
∂(u, v)
transformation.
w.

" "
f (x, y)dxdy = F(u, v)|J|dudv.
R R
ww

Case (ii) Change of variables from Cartesian to polar coordinates.


Let x = r cos θ, y = r sin θ.
∂(x, y)
Then dxdy = |J|drdθ, where J = .
∂(u, v)

Visit For More : www.LearnEngineering.in


Visit For More : www.LearnEngineering.in

Multiple Integrals 267


∂x ∂x cos θ −r sin θ
∂r ∂θ = r cos2 θ + r sin2 θ = r.
Now, J = ∂y ∂y
=
sin θ r cos θ
∂r ∂θ
∴ dxdy = rdrdθ. " "
f (x, y)dxdy = F(r, θ)rdrdθ.
R R
✎ ☞

.in
Worked Examples
✍ ✌
Example

4.36. By changing into polar coordinates, evaluate the integral
Z 2

ng
Z2a 2ax−x
(x2 + y2 )dydx. [Jan 1999]
0 0

eri
p
Solution. The limits for y are y = 0, y = 2ax − x2 ⇒ y2 = 2ax − x2
x2 + y2 − 2ax = 0 ⇒ (x − a)2 − a2 + y2 = 0 ⇒ (x − a)2 + y2 = a2 .

e
gin
The limits for x are x = 0, 2a.
The region of integration is the upper Y
En

semi circle, with centre (a, 0) and radius


a. In polar coordinates r = 2a cos θ
arn

x = r cos θ, y = r sin θ, dxdy = rdrdθ. x= 0 x = 2a


2 2 2
x + y − 2ax = 0 ⇒ r − 2ar cos θ = 0
O (a, 0) y=0
X
⇒ r(r − 2a cos θ) = 0 ⇒ r = 0, r = 2a cos θ.
Le

The limits for r are r = 0, 2a cos θ.


π
θ varies from 0 to .
w.

2
π π
Z2 2aZcos θ Z2 4 !2a cos θ
2 r
∴I= r rdrdθ = dθ
ww

4 0
θ=0 0 0
π
2
3 1 π 3πa4
Z
1
= 24 a4 cos4 θdθ = 4a4 = .
4 422 4
0

Visit For More : www.LearnEngineering.in


Visit For More : www.LearnEngineering.in

268 Engineering Mathematics - I


Z2 Z2x−x2
Example 4.37. Transform the integral (x2 + y2 )dydx into polar coordinates
0 0
and hence evaluate it. [May 2011]
Solution. In the previous problem, put a = 1.

.in

Z2 Z2x−x2
x
Example 4.38. Evaluate p dydx by changing into polar
x2 + y2

ng
0 0
coordinates. √
[Jan 2001]
Z2 Z2x−x2
x
Solution. Let I = dydx.

eri
p
x2 + y2
0 0 p
The limits for y are y = 0, y = 2x − x2 ⇒ y2 = 2x − x2 . x varies from 0 to 2.
x2 + y2 − 2x = 0 ⇒ (x − 1)2 − 1 + y2 = 0
e
gin
⇒ (x − 1)2 + y2 = 1.

1
En

Y
=
It is a circle with centre (1, 0) and radius 2
+y
1) 2

θ
= 1. The region of integration is the os

2c
(x

r=

upper semi circle. To change into polar


arn

coordinates we have x= 0 x=2

x = r cos θ, y = r sin θ and dxdy = rdrdθ. X


O (1, 0) (2, 0)
Le

y=0
2 2
Now, x + y − 2x = 0
w.

r2 − 2r cos θ = 0 ⇒ r(r − 2 cos θ) = 0


ww

r = 0, r = 2 cos θ.

∴ Limits for r are r = 0, 2 cos θ.

Visit For More : www.LearnEngineering.in


Visit For More : www.LearnEngineering.in

Multiple Integrals 269

π
θ varies from 0 to .
2
π π
Z2 2Zcos θ Z2 !2 cos θ
r cos θ r2
I= √ rdrdθ = cos θ dθ
r2 2 0
0 0 0
π π
2 Z2

.in
Z
1
= cos θ4 cos2 θdθ = 2 cos3 θdθ
2
0 0
2 4

ng
= 2 .1 = .
3 3
Za Za
x2

eri
Example 4.39. Evaluate p dxdy by changing to polar coordinates.
x2 + y2
0 y
Solution. The limits for x are x = y, x = a. Limits for y are y = 0, y = a.
The shaded portion is the region of integration.
e
gin
Changing into polar coordinates we have x = r cos θ, y = r sin θ, dxdy = rdrdθ.
a
When x = a, r cos θ = a ⇒ r = .
cos θ
a
∴ r varies from 0 to
En

cos θ
π
θ varies from 0 to .
4
π a π
Z4 Zcos θ 2 Z4 ! a
arn

2
r cos θ r3 cos θ
2 Y
I= rdrdθ = cos θ dθ
r 3 0 y=a
θ=0 0 0
π π
4 Z4 r = cosa θ
a3
Z
1 1
x
Le

y=

2
= cos θ 3 dθ = sec θdθ x= 0
3 cos θ 3 x=a
0 0
a3  π a3  √ 
O X
w.

= log(sec θ + tan θ) 04 = log( 2 + 1) . y=0


3 3

Example 4.40. Transform the integral into polar coordinates and hence evaluate
ww


Za Za2 −x2 q
x2 + y2 dydx. [Jun 2012]
0 0
Solution.

Visit For More : www.LearnEngineering.in


Visit For More : www.LearnEngineering.in

270 Engineering Mathematics - I


The limits for y are y = 0, y = a2 − x2
y2 = a2 − x2 Y

2
π/
x2 + y2 = a2 . 2

=
2 =a

θ
x2 + y
The limits for x are x = 0, x = a.

.in
P
x=0
By changing into polar coordinates we

0
θ=
O X
have x = r cos θ, y = r sin θ, dxdy = rdrdθ. y= 0

ng
In the region of integration, r varies x=a
π
from 0 to a and θ varies from 0 to .
2

eri
√ π
Za Za2 −x2 q Z2 Za p
x2 + y2 dydx = r2 rdrdθ
0 0 θ=0 r=0

Z2 Za
π
e π π
gin
Z2 !a Z2
r3 a3 a3 π a3 π πa3
= r2 drdθ = dθ = dθ = · [θ]02 = = .
3 0 3 3 3 2 6
0 0 0 0

Za Za2 −x2
En

dxdy
Example 4.41. Transform the integral p into polar
√ a2 − x2 − y2
0 ax−x2
coordinates and then evaluate it. [Jan 2012]
arn

Solution.
Y
The limits for y are
Le

p p
y = ax − x2 y= a2 − x2 P

y2 = ax − x2 y2 = a2 − x2
w.

O 
a
 X
2,0
2 2 2 2 2
x + y − ax = 0 x +y =a ,

x2 + y2 = a2 is a circle with centre (0, 0),


ww

radius= a. x2 + y2 − ax = 0 is a circle with


a  a
centre , 0 and radius .
2 2
The limits for x are x = 0, x = a.

Visit For More : www.LearnEngineering.in


Visit For More : www.LearnEngineering.in

Multiple Integrals 271

Consider the curve x2 + y2 − ax = 0


By changing into polar coordinates we have,

r2 − a · r cos θ = 0

r(r − a cos θ) = 0

.in
r , 0 ⇒ r = a cos θ.

Consider the equation x2 + y2 = a2 . We have r = a.

ng
In the region of integration we have r varies from a cos θ to a and θ varies
π
from 0 to .

eri
2 √ π
Za Za2 −x2 Z2 Za
dxdy rdrdθ
∴ p = √
a2 − x2 − y2 a2 − r2

0 ax−x2
e
θ=0 a cos θ
gin
π
Z2 Z0
−tdt
= dθ
π t
Z2 θ=0 r=a sin θ
=− (t)0a sin θ dθ
t2 = a2 − r2
En

Z2
π
2tdt = −2rdr.
=− (0 − a sin θ)dθ rdr = −tdt
arn

0
π when r = a cos θ,
Z2
=a sin θdθ t2 = a2 − a2 cos2 θ
Le

0
π
= a2 (1 − cos2 θ)
2
= a (− cos θ)0
= a2 sin2 θ.
π
w.

 
= −a cos − cos 0 = −a(0 − 1) = a.
2 t = a sin θ.

when r = a. t = 0.
ww

" q
Example 4.42. Evaluate a2 − x2 − y2 dxdy where R is the semicircle
R
x2 + y2 = ax in the first quadrant by changing to polar coordinates.

Visit For More : www.LearnEngineering.in


Visit For More : www.LearnEngineering.in

272 Engineering Mathematics - I

 a 2 a2  a 2 a2
Solution. x2 + y2 − ax = 0 ⇒ x − + y2 − =0⇒ x− + y2 = .
2 4 2 4
By changing into polar coordinates we get x = r cos θ, y = r sin θ, dxdy = rdrdθ.
r2 − ar cos θ = 0 ⇒ r(r − a cos θ) = 0 ⇒ r = 0, r = a cos θ.
Limits for r are 0, a cos θ.
π
Limits for θ are 0, .

.in
2
π
" q Z2 aZcos θ p

ng
Now I = a2 − x2 − y2 dxdy = a2 − r2 rdrdθ
R 0 r=0
2 2 2
Let t = a − r ⇒ 2tdt = −2rdr ⇒ tdt = −rdr

eri
when r = 0, t = a and when r = a cos θ, t = a sin θ
π π π
Z2 aZsin θ Z2 3 !a sin θ Z2
I= −ttdtdθ = −
t
dθ =
−1
e (a3 sin3 θ − a3 )dθ
gin
3 a 3
0 a 0 0
3 2 3 4 − 3π a3
! ! !
−1 3 2 3 π −a π −a
= a .1 − a = .1 − = = (3π − 4).
3 3 2 3 3 2 3 6 18
En

R∞ R∞ 2 +y2 ) R∞ 2
Example 4.43. Evaluate e−(x dxdy and hence evaluate e−x dx.
0 0 0
[Jan 2014, Jun 2011, Jan 2010, Jan 2004]
arn

Z∞ Z∞
2 +y2 )
Solution. Let I = e−(x dxdy.
0 0
Y
Le

Since x varies from 0 to ∞ and y varies


from 0 to ∞, the region of integration is P
x=0
w.

the entire first quadrant. r

Changing to polar coordinates we get


θ
X
ww

x = r cos θ, y = r sin θ and dxdy = rdrdθ. O


y=0
Here, r varies from 0 to ∞ and θ varies
π
from 0 to .
2

Visit For More : www.LearnEngineering.in


Visit For More : www.LearnEngineering.in

Multiple Integrals 273

π
Z∞ Z∞ Z2 Z∞
2 +y2 ) 2 dt
∴I= e−(x dxdy = e−r rdrdθ Let r2 = t, 2rdr = dt ⇒ rdr =
2
0 0 0 0
π
Z Z∞2
dt
= e−t dθ When r = 0, t = 0, when r = ∞, t = ∞
2

.in
0 0
π
Z2 !∞
1 e−t
= dθ

ng
2 −1 0
0
π
Z2
1 1 π π

eri
=− (0 − 1)dθ = (θ)02 = .
2 2 4
0
Z∞
2
To find e−x dx.
e
gin
0
Z∞ Z∞
2 +y2 ) π
We have, e−(x dxdy =
4
0 0
En

Z∞ Z∞
2 2 π
e−x e−y dxdy =
4
0 0
arn

Z∞ Z∞
−x2 2 π
e dx e−y dy =
4
0 0
 ∞ 2
Le

Z
π

−x 2
 e dx =
 
4
0
Z∞ √
w.

−x2 π
e dx = .
2
0
ww

" s
1 − x2 − y2
Example 4.44. Evaluate dxdy over the positive quadrant of the
1 + x2 + y2
circle x2 + y2 = 1.
Solution.

Visit For More : www.LearnEngineering.in


Visit For More : www.LearnEngineering.in

274 Engineering Mathematics - I

By changing into polar coordinates we


get x = r cos θ, y = r sin θ and dxdy = rdrdθ. Y

2
π/
Now x2 + y2 = 1

θ=
=⇒ r2 = 1 P
x=0

.in
i.e., r = 1. For the region of integration,

1
r=

0
θ=
we have r varies from 0 to 1 and θ varies θ
X
π O y=0
from 0 to .

ng
2

" s

eri
1 − x2 − y2
Let I = dxdy
1 + x2 + y2
π
Z2 Z1 r
=
1 − r2
rdrdθ e
gin
1 + r2
0 0
π
Z Z12
r(1 − r2 )
= √ drdθ
1 − r4
En

0 0
π
Z Z1
 2

r r3 
 
=  √ − √  drdθ
arn

 1 − r4 1 − r4 
0 0
π 
Z2  Z1 Z1

 1 2r 1 −4r 3 
= √ dr + √ dr dθ
 
2 1 − r4 4 1 − r4 
Le

0 0 0
π
Z  Z1 Z1
 2

 1 dt 1 4
d(1 − r ) 

=  √ dr + √  dθ [t = r2 ]
2 4

w.

1 − t2 1 − r4 
0 0 0
π
Z  2 1 1
 
 1 −1 1 1  (1 − r4 ) 2  
=  (sin t)0 +    dθ
ww

2 4 1
0 2 0
π
Z 2 !
1π 1
= + (0 − 2) dθ
22 4
0

Visit For More : www.LearnEngineering.in


Visit For More : www.LearnEngineering.in

Multiple Integrals 275

π
Z2 !
π 1
= − dθ
4 2
0
!
π 1 π
= − (θ)02
4 2
!
π 1 π

.in
= −
4 2 2
π2 π
= − .
8 4

ng
"
x2 y2
Example 4.45. By transforming into polar coordinates evaluate dxdy
x2 + y2

eri
over the annular region between the circles x2 + y2 = a2 and x2 + y2 = b2 , (b > a)
[Jan 2013]
Y
Solution. By changing into polar
coordinates we get x = r cos θ, y = r sin θ e b
gin
x=0
and dxdy = rdrdθ, x2 + y2 = r2 . a
In the region of integration we O X
y=0
En

have r varies from a to b and θ varies


from 0 to 2π.
arn

" Z2π Zb
x2 y2 r2 cos2 θ · r2 sin2 θ
dxdy = · rdrdθ
Le

x2 + y2 r2
θ=0 r=a
Z2π Zb
= r3 cos2 θ sin2 θdrdθ
w.

θ=0 r=a
Z2π !b
2 2r4
ww

= cos θ sin θ · dθ.


4 r=a
0
Z2π
1
= cos2 θ sin2 θ(b4 − a4 )dθ
4
0

Visit For More : www.LearnEngineering.in


Visit For More : www.LearnEngineering.in

276 Engineering Mathematics - I

Z2π
b4 − a4
= (sin θ cos θ)2 dθ
4
0
Z2π !2
b4 − a4 sin 2θ
= dθ
4 2
0
Z2π
b4 − a4

.in
= sin2 2θdθ.
16
0
Z2π

ng
b4 − a4 1 − cos 4θ
= dθ
16 2
0

eri
 2π
Z2π

4 4
b − a 
Z 
=  dθ − cos 4θdθ
32  
0 0

=
b4 − a4  2π

[θ]0 − e
" #2π 
sin 4θ  b4 − a4
 = (2π) =
(b4 − a4 )π
gin
32 4 0 32 16

4.5 Area enclosed by plane curves


En

"
Result. Area of a region R is given by dxdy.
R
arn

✎ ☞
Worked Examples
✍ ✌

Example 4.46. Find the area of the circle of radius a by double integration.
Le

[Jan 2006]
Solution.
The circle is x2 + y2 = a2 . and finally x varies from 0 to a.
w.

By symmetry Area = 4× area in the first



quadrant. Za Za2 −x2 Za √
2 2
dydx = 4 (y)0 a −x dx
ww

Divide the region in the first quadrant ∴ Area = 4


x=0 y=0 x=0
into strips parallel to the y-axis. Along a
√ Za !a
p xp 2 a2 −1  x 
typical strip, y varies from 0 to a2 − x2 =4 a2 − x2 dx =4 a − x2 + sin
2 2 a 0
0

Visit For More : www.LearnEngineering.in


Visit For More : www.LearnEngineering.in

Multiple Integrals 277

a2 −1 a2 π
!
=4 sin (1) = 4 = πa2 .
2 2 2

.in
(0, a) √
y = a2 − x 2

x=0

ng
O X
y= 0 (a, 0)

eri
x2 +y2 = a2

x2 y2
Example 4.47. Find the area of the ellipse + = 1 using double integration.
e a2 b2
[Jun 2013]
gin
Solution.

Y
En

Consider the area in the first quadrant.



(0, b) y = ab a2 − x 2
Divide this area into strips parallel to
x=0
the y-axis. Along a typical strip, y varies
arn

bp 2 y=0
X
from 0 to a − x2 and finally x varies (a, 0)
a
from 0 to a.
Le

By symmetry, Area of the ellipse = 4× Area in the first quadrant.



b 2 −x2
Za a Za Za Za
w.


b
a2 −x2 bp 2
Area = 4 dydx = 4 a
(y)0 dx = 4 a − x2 dx
a
x=0 y=0 x=0 0
ww

Za p !a
4b 2 2
4b x p 2 2
a2 −1  x 
= a − x dx = a −x + sin
a a 2 2 a 0
0
4b a2 π
= sin−1 (1) = 2ab = πab.
a 2 2

Visit For More : www.LearnEngineering.in


Visit For More : www.LearnEngineering.in

278 Engineering Mathematics - I

Example 4.48. Using double integral, find the area bounded by the parabola
y2 = 4ax and x2 = 4ay [Jun 2013, May 2011, Jan 2010]
Solution.

Y
To find the points of intersection, solve

.in
x2 = 4ay

the two equations.


A(4a, 4a)
2
y = 4ax (1)

ng
x2 = 4ay (2)
y=0
X
O

eri
x42−y264a3 x = 0
(2)2 ⇒ x4 = 16a
3
= x(x 64a3 ) = 0
16a2 −· 4ax x=0
y2 = 4ax
x=
= 0, x33
64a x− 64a = 0
3

e
gin
x3 = 64a3 ⇒ x = 4a.

When x = 0, y = 0. One point of intersection is O(0, 0).


En

When x = 4a, 4ay = 16a2


16a2
y= = 4a.
4a
∴ The other point of intersection is A(4a, 4a).
arn

The shaded portion is the required region. Divide the region into strips
y2 p
parallel to the x−axis. Along one such strip, x varies form to 4ay and finally y
4a
Le

varies form 0 to 4a. √


Z4a Z4ay
∴ Required area = dxdy
w.

y=0 2
x= y4a

Z4a √
4ay
= [x] y2 dy
ww

4a
y=0
Z4a " p
y2
#
= 4ay − dy
4a
0

Visit For More : www.LearnEngineering.in


Visit For More : www.LearnEngineering.in

Multiple Integrals 279

Z4a Z4a
√ 1/2 1
=2 a· y dy − y2 dy
4a
0 0
 4a !4a
√  y3/2  1 y3
= 2 a ·  3  −
  ·
2 0
4a 3 0
√ 2 1

.in
= 2 a × · (4a)3/2 − 64a3
3 12a

4 a √ 16a2
= 8a a −
3 3

ng
32a2 16a 2 16a2
= − = Sq.units
3 3 3

eri
Example 4.49. Find the area common to y2 = 4x and x2 = 4y using double
integration. [Dec 2011]
Solution. Put a = 1 in the previous example.
e
gin
Example 4.50. Find by double integration, the area between the parabolas
3y2 = 25x and 5x2 = 9y. [Jun 2012]
Solution. To find the points of intersection, solve the two equations.
En

25
y2 = x (1)
3
9 Y
arn

x2 = y (2)
5 x2 = 95 y

2 4 81 2
(2) ⇒ x = y A(3, 5)
25
81 × ✚✚
25
Le

✚ 4=

25x x
3
x4 = 27x O(0, 0)
X
w.

x4 − 27x = 0

x(x3 − 27) = 0 y2 = 25
3 x
ww

x = 0, x3 − 27 = 0

x3 = 27

x = 3.

Visit For More : www.LearnEngineering.in


Visit For More : www.LearnEngineering.in

280 Engineering Mathematics - I

when x = 0, 9y = 0 ⇒ y = 0.
One point of intersection is O(0, 0).
where x = 3, 9y = 5 × 9
y = 5.
∴ The other point of intersection is (3, 5).

.in
The shaded portion is the region of integration. Divide the region √ into strips
3y2 3 y
parallel to the x−axis. Along one such strip, x varies from to √ and finally y
25 5

ng
varies from 0 to 5. √
3 y

Z5 Z 5 Z5 √
3 y

5
∴ Area = dxdy = (x) dy

eri
3y2
25
0 3y2 0
25

Z5
3y2
=
3 √
√ y−
25
!
dy
e
gin
5
0
 5 !5
3  y3/2  3 y3 2 √ 1
= √  3  − · = √ [5 5] − · 53 = 10 − 5 = 5 Sq.units.
5 2 0 25 3 0 5 25
En

Example 4.51. Find the smaller of the areas bounded by the ellipse 4x2 + 9y2 = 36
and the straight line 2x + 3y = 6. [Jan 2012]
arn

Solution. Solving the given two equations, we get the points of intersection.

Y
Le

, 2)
2 2 B(0
4x + 9y = 36 (1) 4x2 +9y2 = 36

2 x+
2x + 3y = 6 (2) 3y=
w.

O
X
From (2), 3y = 6 − 2x A(3
, 0)
6 − 2x
ww

i.e., y = .
3
Substituting in (1) we get

9(6 − 2x)2
4x2 + = 36
9

Visit For More : www.LearnEngineering.in


Visit For More : www.LearnEngineering.in

Multiple Integrals 281

4x2 + 36 + 4x2 − 24x = 36

8x2 − 24x = 0

x2 − 3x = 0

x(x − 3) = 0 ⇒ x = 0 or x = 3.

.in
6
When x = 0, (2) ⇒ y = = 2.
3
One point of intersection is (0, 2)

ng
6−6
When x = 3, y = = 0.
3
The other point of intersection is (3, 0).

eri
The shaded portion is the smaller area. Divide the regionrinto strips parallel to
6 − 2x 4 2
the y−axis. Along one such strip, y varies from to 4 − x and finally x
3 9
varies from 3 to 0.
e
gin
q
4− 94 x2
Z0 Z0 q
  4− 94 x2
Z
∴ Area = dxdy = y 6−2x dx
3
En

x=3 y= 6−2x 3
3

Z0  r !
 4 2 6 − 2x 
=  4 − x −  dx
9 3
arn

3
Z0 r Z0 Z0
4 2
= (9 − x2 )dx − 2dx + xdx
9 3
Le

3 3 3
#0 " #0
2 x2
"
2 xp 2
9 −1  x  0
= 9 − x + sin − 2[x]3 +
3 2 2 3 3 3 2 3
w.

" #
2 9π 1 −3π 3π
= 0− − 2(0 − 3) + [0 − 9] = +6−3=3−
3 22 3 2 2
 π  3(2 − π)
Sq.units.
ww

=3 1− =
2 2

Example 4.52. Find the smaller of the areas bounded by y = 2 − x and x2 + y2 = 4


using double integral.

Visit For More : www.LearnEngineering.in


Visit For More : www.LearnEngineering.in

282 Engineering Mathematics - I

Solution. Solving the given two equations, we get the points of intersection.

We have x2 + y2 = 4 ⇒ x2 + (2 − x)2 = 4 ⇒ x2 + 4 + x2 − 4x = 4 ⇒ 2x2 − 4x = 0

⇒ x2 − 2x = 0 ⇒ x(x − 2) = 0 ⇒ x = 0, x = 2.

.in
Y
When x = 0, y = 2, when x = 2, y = 0.
(0, 2)
The points of intersection are (0, 2) and

ng
y = 2− x
x=0
(2, 0). The shaded portion is the required
region. Divide this region into strips O X

eri
y= 0 (2, 0)
parallel to the y-axis. Along one such

typical strip, y varies from 2−x to 4 − x2
and finally x varies from 0 to 2.
e
gin

" Z2 Z4−x2 Z2 √ Z2 p
4−x2
Now area = dxdy. = dydx = (y)2−x dx = ( 4 − x2 − (2 − x))dx
R x=0 y=2−x x=0 x=0
En

 x !2 2 2
!
xp 4 x
= 4 − x2 + sin−1 − 2(x)20 +
2 2 2 0 2 0
4 π
arn

= 2 sin−1 (1) − 2(2 − 0) + = 2 − 4 + 2 = π − 2.


2 2

Example 4.53. Find the area bounded by the parabola y2 = 4 − x and y2 = 4 − 4x as


Le

a double integral and evaluate it. [Jan 2001]


Solution. The parabola y2 = 4 − x = −(x − 4). Its vertex is at (4, 0).
Consider the parabola y2 = 4 − 4x = −4(x − 1) . Its vertex is at (1, 0).
w.

Both the parabolas are open to the left. Let us solve the two equations to find the
points of intersection, y2 = 4 − x and y2 = 4 − 4x.
ww

∴ 4 − x = 4 − 4x ⇒ 4x − x = 0 ⇒ 3x = 0 ⇒ x = 0 ⇒ y2 = 4 ⇒ y = ±2.

Visit For More : www.LearnEngineering.in


Visit For More : www.LearnEngineering.in

Multiple Integrals 283

Y
The points of intersection are
(2, 0), (−2, 0). The shaded portion is the
required region. Divide this region into (1, 0) (4, 0)
O X
strips parallel to the x-axis. Along one
y2

.in
typical strip, x varies from 1 − to 4−y2 .
4
Finally y varies from −2 to 2.

ng
" x=4−y2
Z2 Z2 Z2
y2
Z !!
4−y2 2
Now area = dxdy. = dydx = (x) 2 dy = 4−y − 1− dy
1− y4 4
R y=−2 2 y=−2 y=−2

eri
x=1− y4

Z2 !2 !2
y2 y3 1 y3
!
2 2 2
= 4−y −1+ dy = 4(y)−2 − − (y)−2 +
4 3 −2 4 3 −2
y=−2
e
gin
1 1 16 16
= 4(2 + 2) − (8 + 8) − (2 + 2) + (8 + 8) = 16 − −4+
3 12 3 12
16 4 12
= 16 − 4 − + = 12 − = 12 − 4 = 8.
3 3 3
En

Example 4.54. Find the area of the region bounded by y = x − 2 and y2 = 2x + 4.


Solution. The parabola is y2 = 2x + 4 = 2(x + 2).
arn

The vertex is (−2, 0).


To find the points of intersection, let us solve the two equations.
The curves are y = x − 2, y2 = 2x + 4.
Le

(x − 2)2 = 2x + 4.

x2 − 4x + 4 − 2x − 4 = 0
w.

x2 − 6x = 0

x(x − 6) = 0 ⇒ x = 0, x = 6.
ww

When x = 0, y = −2.
When x = 6, y = 4.

Visit For More : www.LearnEngineering.in


Visit For More : www.LearnEngineering.in

284 Engineering Mathematics - I

Y
The points of intersection are (0, −2) and
(6, 4). The shaded portion is the required
(6, 4)
region. Divide the region into strips
parallel to the x-axis. Along one such

.in
X
y2 − 4 (−2, 0)
O
strip, x varies from to y + 2 and
2 (0, −2)
finally y varies from −2 to 4.

ng
" Z4 Zy+2 Z4

eri
y+2
Now, Area = dxdy = dxdy = (x) 2 dy
x= y 2−4
R y=−2 2 y=−2
x= y 2−4

=
Z4
y+2−
y2
!!
− 2 dy =
Z4
y+2−
y2
!
e
+ 2 dy =
Z4
y+4−
y2
!
dy
gin
2 2 2
−2 −2 −2
!4
y2 y3 1 1
= + 4y − = (16 − 4) + 4(4 + 2) − (64 + 8) = 6 + 24 − 12 = 18.
2 6 −2 2 6
En

Area as double integral in polar coordinates


!
Result. Area in polar coordinates is given by rdrdθ.
R
arn

✎ ☞
Worked Examples
✍ ✌
Example 4.55. Find the area of the cardioid r = a(1 − cos θ).
Le

Solution.
w.

Since the curve is symmetric about the θ

initial line, Area = 2× area above the P


ww

θ=π
initial line. r
O θ=0
Along this region, r varies from 0 to
a(1 − cos θ) and θ varies from 0 to π. r = a(1−cos θ)

Visit For More : www.LearnEngineering.in


Visit For More : www.LearnEngineering.in

Multiple Integrals 285

Zπ Z θ)
a(1−cos Zπ !a(1−cos θ) Zπ
r2
Area = 2 rdrdθ = 2 dθ = a2 (1 − cos θ)2 dθ
2 0
θ=0 r=0 0 0
Zπ Zπ
1 + cos 2θ
= a2 (1 − cos2 θ − 2 cos θ)dθ = a2 (1 + − 2 cos θ)dθ
2
0 0

.in
Zπ !π !
3 cos 2θ 3 1 sin 2θ
=a 2
( + − 2 cos θ)dθ = a2 (θ)π0 + − 2(sin θ)π0
2 2 2 2 2 0
0

ng
3πa2
!

= a2 = .
2 2

eri
Example 4.56. Find the area of one loop of the lemniscate of Bernoulli
r2 = a2 cos 2θ.
Solution. The curve is symmetric about both the axes. One loop lies to the right
of y-axis and symmetric about the x-axis. e
gin
∴ Area = 2× area of the portion which lies above the axis.
√ π
Along the region r varies√from 0 to a cos 2θ and θ varies from 0 to .
π 4
Z4 a Zcos 2θ
En

∴ Area = 2 rdrdθ
θ


4

θ=0 r=0
π/

s
co
θ=

π
4 !a √cos 2θ a2
arn

=
P
r2
r2
Z
=2 dθ
2 0
θ=0 r
π
O
Z 4
Le

= a2 cos 2θdθ θ = −π/4


0
! π4
sin 2θ a2  π  a2
w.

2
=a = sin − 0 = .
2 0 2 2 2
ww

Example 4.57. Find the area between r = 2 cos θ andπ r = 4 cos θ. π


Z2 Z2
Solution.π r=4 cos θ 1
Z2 Z
π
Z2 !4 cos θ = (16 cos2 θ − 4 cos2 θ)dθ = 6 cos2 θdθ
r 2 2
−π −π
Area = rdrdθ = dθ 2 2
2 2 cos θ
θ= −π
2
r=2 cos θ θ= −π
2

Visit For More : www.LearnEngineering.in


Visit For More : www.LearnEngineering.in

286 Engineering Mathematics - I

π
θ
Z2 π
θ= P
2 1π 2
=6×2 cos θdθ = 12 × = 3π.
22
0
O
r

.in
θ = − π2

ng
Example 4.58. Find the area inside the circle r = a sin θ but lying outside the
cardioid r = a(1 − cos θ). [Jan 2009]

eri
Solution. The equation of the given curves are r = a sin θ, r = a(1 − cos θ). Solving,
we get a sin θ = a(1 − cos θ) ⇒ sin θ = 1 − cos θ.
⇒ sin θ + cos θ = 1 ⇒ (sin θ + cos θ)2 = 1.
sin2 θ + cos2 θ + 2 sin θ cos θ = 1 ⇒ 1 + sin 2θ = 1 e
gin
π
⇒ sin 2θ = 0 ⇒ 2θ = 0 or 2θ = π ⇒ θ = 0, θ = .
2
θ
The shaded portion is the required
En

θ = π/2
region. Consider the radius vector OP.
Q
The limits for r and θ as OP traverses
arn

the required region is as follows. r P r = a sin θ


θ=π
varies from a(1 − cos θ) to a sin θ and θ O θ=0
r
π
varies from 0 to .
Le

2 r = a(1−cos θ)
π π
Z 2 Z sin θ
r=a Z 2
2 a sin θ
!
r
Area = rdrdθ = dθ
2
w.

a(1−cos θ)
θ=0 r=a(1−cos θ) θ=0

π π
Z2  Z2 
a2
ww

1 2 2 2 2
 
= a sin θ − a (1 − cos θ) dθ = sin2 θ − 1 − cos2 θ + 2 cos θ dθ
2 2
0 0
2 2 a2
!
a 1π π 1π a  π 
= − − + 2.1 = 2− = (4 − π).
2 22 2 22 2 2 4

Visit For More : www.LearnEngineering.in


Visit For More : www.LearnEngineering.in

Multiple Integrals 287

Example 4.59. Find the area which is inside the circle r = 3a cos θ and outside the
cardioid r = a(1 + cos θ). [Jan 2013]
Solution. The shaded portion is the required area. Solving the two equations we
get the points of intersection, r = 3a cos θ and r = a(1 + cos θ).

3
θ

π/
=

.in
θ
∴ 3a cos θ =a(1 + cos θ)
r

ng
3 cos θ =1 + cos θ (3a/2,0) (3a, 0)

2 cos θ =1 π π
Z3 3aZcos θ Z3 " #3a cos θ
1 π π r2

eri
Required area ⇒ θabove
cos θ==2 × area = , − θ .== 2 rdrdθ. = 2 dθ
2 3 3 2 r=a(1+cos θ)
θ=0 r=a(1+cos θ) θ=0
π
3

=2
Z
e
[9a2 cos2 θ − a2 (1 + cos θ)2 ]dθ
gin
0
π
Z3
=2 (9a2 cos2 θ − a2 − a2 cos2 θ − 2a2 cos θ)dθ
En

0
π
Z3
= 2a2 (9 cos2 θ − 1 − cos2 θ − 2 cos θ)dθ
arn

0
π
Z3
= 2a2 (8 cos2 θ − 2 cos θ − 1)dθ
Le

0
π
Z3 !
2 8(1 + cos 2θ)
= 2a − 2 cos θ − 1 dθ
2
w.

0
π
Z3
= 2a2 (4 + 4 cos 2θ − 2 cos θ − 1)dθ
ww

0
π
Z3
= 2a2 (3 + 4 cos 2θ − 2 cos θ)dθ
0

Visit For More : www.LearnEngineering.in


Visit For More : www.LearnEngineering.in

288 Engineering Mathematics - I

 π π π 
 Z3 Z3 Z3 
2 
= 2a 3 dθ + 4 cos 2θdθ − 2 cos θdθ

 
0 0 0
 ! π3 
2
 π
3
sin 2θ π
3

= 2a 3(θ)0 + 4 − 2(sin θ)0 
2 0
π !  π !
2
 2π
= 2a 3 − 0 + 2 sin − sin 0 − 2 sin − sin 0

.in
3 3 3
 √ √ 
3 3 
= 2a2 π + 2 ·  = 2πa2 Sq.units.

−2·
2 2

ng
4.6 Triple integral in Cartesian coordinates

eri
Let the function f (x, y, z) be defined in a specific region V in space for all x, y and
z with proper limits. The triple integral of f (x, y, z) w.r.t. x, y and z is defined as
Zz1 Zy1 Zx1
e
gin
f (x, y, z)dxdydz.
z0 y0 x0
The evaluation of the triple integral is as follows.
(i) If all the limits are constants, then the integration can be performed in any
En

order with proper limits.


g2 (z) fZ
Zb Z 2 (y,z)

(ii) Consider f (x, y, z)dxdydz.


arn

a y=g1 (z) x= f1 (y,z)


First we integrate with respect to x treating y and z as constants. Next we integrate
the resulting function of y and z w.r.t.y treating z as constant. Finally integrate the
Le

resulting function of z w.r.tz.


✎ ☞
Worked Examples
✍ ✌
w.

Z1 Z2 Z2
Example 4.60. Evaluate x2 yzdxdydz.
ww

0 0 1
Solution.
Z1 Z2 Z2 Z1 Z2 !2
2 x3
x yzdxdydz = yz dydz
3 1
0 0 1 0 0

Visit For More : www.LearnEngineering.in


Visit For More : www.LearnEngineering.in

Multiple Integrals 289

Z1 Z2
1
= yz(8 − 1)dydz
3
0 0
Z1 !2 Z1
7 y2 7
= z dz = z4dz
3 2 0 6
0 0
!1
28 z2

.in
=
6 2 0
28 1 7
= = .
6 2 3

ng
Za Zb Zc

eri
Example 4.61. Find (x2 + y2 + z2 )dxdydz. [Jun 2009]
0 0 0
Solution.

e
gin
Za Zb Zc Za Zb !c
2 2 2 x3
(x + y + z )dxdydz = + y2 x + z2 x dydz
3 0
0 0 0 0 0
Za Zb
En

c3
!
2 2
= + cy + cz dydz
3
0 0
Za  3 !b 
arn

 c b y3 2 b

=  3 (y)0 + c 3 + cz (y)0  dz
0
0
Za
c3 b cb3
!
2
Le

= + + cz b dz
3 3
0
!a
bc3 a cb3 a z3
= (z)0 + (z)0 + bc
w.

3 3 3 0
abc3 ab3 c a3 bc abc 2
= + + = (a + b2 + c2 ).
3 3 3 3
ww

log 2Zx Z
Z x+y

Example 4.62. Evaluate e x+y+z dxdydz. [Jan 2009]


0 0 0

Visit For More : www.LearnEngineering.in


Visit For More : www.LearnEngineering.in

290 Engineering Mathematics - I

Solution.
log 2Zx Z
Z x+y log 2Zx Z
Z x+y log 2Zx
Z
 x+y 
 Z 
e x+y+z dxdydz = e x ey ez dxdydz = e x ey  ez dz dxdy
 
 
0 0 0 0 0 0 0 0 0
Z 2Zx
log Z 2Zx
log
x+y
= e x ey (ez )0 dxdy = e x ey (e x+y − 1)dxdy

.in
0 0 0 0
Z 2Zx
log

= (e2x+2y − e x ey )dxdy

ng
0 0
Z 2 Zx
log Zx
 
 
= e2x e2y dy − x y 
e e dy dx
 

eri
 
0 0 0
log 2 !x
e2y
Z !
= e2x x y x
− e (e )0 dx
0
e
2 0
gin
Z 2
log
e2x 1
! !
2x x x
= e − − e (e − 1) dx
2 2
0
En

log 2
e4x e2x
Z !
2x x
= − − e + e dx
2 2
0
arn

Z 2
log
e4x 3e2x
!
x
= − + e dx
2 2
0
!log 2 !log 2
1 e4x 3 e2x
Le

log 2
= − + (e x )0
2 4 0 2 2 0
1 3
= [16 − 1] − (4 − 1) + (2 − 1)
w.

8 4
15 9
= − +1
8 4
15 − 18 + 8 5
ww

= = .
8 8
Z1 Z1−xZx+y
Example 4.63. Evaluate ez dzdydx. [Jan 1996]
0 0 0

Visit For More : www.LearnEngineering.in


Visit For More : www.LearnEngineering.in

Multiple Integrals 291

Solution.
Z1 Z1−xZx+y Z1 Z1−x Z1 Z1−x
z z  x+y
e x+y − 1 dydx

e dzdydx = e 0 dydx =
0 0 0 0 0 0 0
1
Z Z1−x Z1  Z1−x Z1−x 
 
e x ey − 1 dydx =
 x
ey −
 
= e dy dx
 

.in
0 0 0 0 0
Z1  
1−x
= e x ey 0 − (y)1−x
0 dx

ng
0
Z1   Z1
x 1−x
= e (e − 1) − (1 − x) dx = (e − e x − 1 + x)dx

eri
0 0
!1
x2 1
= e(x)10 − (e x )10 − (x)10 + = e − (e − 1) − 1 +
2 2
1 1
=e−e+1−1+ = . e 0
gin
2 2

Z3 Z1 Z xy
Example 4.64. Evaluate xydzdydx. [Jan 2001]
En

1 1 0
x
Solution.

Z3 Z1 Z xy Z3 Z1 √ Z3 Z1 Z3 Z1

arn

xy 3 3
xydzdydx = xy(z)0 dydx = xy xydydx = x 2 y 2 dydx
1 1 0 1 1 1 1 1 1
x x x x

Z3  5 1
 y 2  Z3  ! 25 
3 2 3 
 1
Le


= x  5  dx =
2 x 2 1 −  dx
2 1
5 x 
1 x 1
Z3  5 3 ! 
2 1 2  x 2 
w.

3

= x − dx =  5  − (log x)31 
2
5 x 5 2 1
1
2 2 √
! !
2 2 5
ww

= (3 2 − 1) − log 3 = (9 3 − 1) − log 3 .
5 5 5 5
2
Z1 Z1−x (x+y)
Z
Example 4.65. Evaluate xdzdydx. [Jan 1996]
0 0 0

Visit For More : www.LearnEngineering.in


Visit For More : www.LearnEngineering.in

292 Engineering Mathematics - I

Solution.
2
Z1 Z1−x (x+y)
Z Z1 Z1−x Z1 Z1−x
(x+y)2
xdzdydx = x(z)0 dydx = x(x + y)2 dydx
0 0 0 0 0 0 0
Z1 3 1−x
! Z1
(x + y) 1
= x dx = x((x + 1 − x)3 − x3 )dx

.in
3 0 3
0 0
Z1 Z1  !1 !1 
1 1 3 4 1  x2 x5 
= x(1 − x )dx = (x − x )dx =  −

ng

3 3 3 2 0 5 0
0 0
! !
1 1 1 1 5−2 1
= − = = .

eri
3 2 5 3 10 10

log 2Zx x+log


Z Z y
e x+y+z dzdydx.
Example 4.66. Evaluate
e [Jun 2013]
gin
0 0 0
Solution.

log 2Zx x+log


Z Z y log 2Zx
Z
 x+y+z  x+log y
e x+y+z dzdydx = e dydx
En

0
0 0 0 0 0
Z 2Zx
log
 
= e x+y+x+log y − e x+y dydx.
arn

0 0
Z 2Zx
log
 
= e2x ey+log y − e x · ey dydx
Le

0 0
Z 2Zx
log
 
= e2x · ey · elog y − e x ey dydx
w.

0 0
Z 2Zx
log
 
e2x yey − e x ey dydx
ww

=
0 0
log 2
 Zx Zx

Z 
 2x y x y 
= e ye dy − e e dy  dx


0 0 0

Visit For More : www.LearnEngineering.in


Visit For More : www.LearnEngineering.in

Multiple Integrals 293

log 2
 Zx

Z 
 2x y x y x
= e yd(e ) − e (e )0  dx


0 0
log 2
Z 

Zx
 

   
 2x  y  x
 y   − e x (e x − 1)

= 
e 

 ye 0 − e dy  
 dx

   

0 0
Z 2
log

.in
 x  
= e2x xe x − ey 0 − e2x + e x dx
0

ng
log 2
Z
= {e2x (xe x − e x + 1) − e2x + e x }dx
0

eri
Z 2
log
 
= xe3x − e3x + e2x − e2x + e x dx
0
log 2
Z
e
gin
 
= xe3x − e3x + e x dx
0
Z 2
log log 2
Z log 2
Z
3x 3x
= xe dx − e dx + e x dx.
En

0 0 0
log 2 !log 2
e3x e3x
Z !
log 2
= xd − + ex 0
arn

3 3 0
0
!log 2 log 2
xe3x e3x
Z
1 
= − dx − e3 log 2 − 1 + (elog 2 − 1)
Le

3 0 3 3
0
3x
!log 2
8 e
1
= log 2 · − − [8 − 1] + (2 − 1)
3 3 9
w.

0
!
8 8 1 7
= log 2 − − − +1
3 9 9 3
ww

!
8 7 7
= log 2 − + −1
3 9 3
!
8 7 + 21 − 9
= log 2 −
3 9

Visit For More : www.LearnEngineering.in


Visit For More : www.LearnEngineering.in

294 Engineering Mathematics - I

8 19
= log 2 − .
3 9
$
dxdydz
Example 4.67. Evaluate p for all positive values of x, y, z for
1 − x2 − y2 − z2
which the integral is real. [Jan 2013, Jan 2012]
Solution. The integral is real for all values of x2 + y2 + z2 < 1.

.in
√ √
$ 1−x 2 2
Z1 Z1−x2 Z −y
dxdydz 1
∴ p = p dzdydx

ng
1 − x2 − y2 − z2 1 − x2 − y2 − z2
0 y=0 z=0
 √1−x2 −y2

Z1 Z1−x2  −1 z 

eri
= sin p  dydx
1 − x2 − y2 0
0 0

Z Z1−x2
1

=
e (sin−1 1 − sin−1 0)dydx
gin
0 0

Z1 Z1−x2
π
= dydx
2
En

0 0
Z1 √
π 2
= [y]0 1−x dx
2
arn

0
Z1 p
π
= 1 − x2 dx
2
0
Le

" #1
π xp 2
1 −1
= 1 − x + sin x
2 2 2 0
" #
π 1 −1
w.

= sin 1 − 0
2 2
π 1 π π2
= · · = .
ww

2 2 2 8
√ √ 2 −x2 −y2
Za Za2 −x2 aZ
1
Example 4.68. p dzdydx. [Dec 2011]
a2 − x2 − y2 − z2
0 0 0

Visit For More : www.LearnEngineering.in


Visit For More : www.LearnEngineering.in

Multiple Integrals 295

Solution.
√2 2 2
 √a2 −x2 −y2
√ √
Za Za2 −x2 aZ−x −y Za Za2 −x2
1  −1 z 
p dzdydx = sin p  dydx
a2 − x2 − y2 − z2 a2 − x2 − y2 0

0 0 0 0 0

Za Za2 −x2  
= sin−1 1 − sin−1 0 dydx

.in
0 0

Za Za2 −x2
π
= dydx.

ng
2
0 0
Za √
π 2 −x2
[y]0 a

eri
= dx
2
0
Za p
π
a2 − x2 dx
e
=
2
gin
0
" 2 #a
π a −1 x xp 2 2
= sin + a −x
2 2 a 2 0
π a2 −1
" #
En

= sin 1 − 0
2 2
πa2 π π2 a2
= · = .
4 2 8
$
arn

dxdydz
Example 4.69. Evaluate , where V is the region bounded by
(x + y + z + 1)3
V
x = 0, y = 0, z = 0, and x + y + z = 1. [Jan 2014, Dec 2011]
Le

Solution. The limits for x, y and z are as follows.

z : 0 to 1 − x − y
w.

y : 0 to 1 − x

x : 0 to 1
ww

$ Z1 Z1−x 1−x−y
Z
dzdydz
∴ = (1 + x + y + z)−3 dzdydx
(1 + x + y + z)3
V 0 0 0

Visit For More : www.LearnEngineering.in


Visit For More : www.LearnEngineering.in

296 Engineering Mathematics - I

Z1 Z1−x" #1−x−y
(1 + x + y + z)−2
= dydx
−2 0
0 0
Z1 Z1−x
1 
=− 2−2 − (1 + x + y)−2 dydx
2
0 0
Z1 Z1−x

.in
!
1 1 −2
=− − (1 + x + y) dydx
2 4
0 0
Z1 

ng
#1−x 
(1 + x + y)−1
"
1  1 1−x 
=−  [y]0 −  dx
2 4 −1 0
0

eri
Z1 !
1 1
=− (1 − x) + 2−1 − (1 + x)−1 dx
2 4
0

1  1 1 1 x2
= −  (x)0 − ·
!1
1 1
e 1


+ [x]0 − (log(1 + x))0 
gin
2 4 4 2 0 2
!
1 1 1 1
=− − + − log 2
2 4 8 2
!
1 2−1+4
En

=− − log 2
2 8
!
1 5 1 5
=− − log 2 = log 2 − .
2 8 2 16
arn

$
Example 4.70. Find the value of xyzdxdydz through the positive spherical
octant for which x2 + y2 + z2 ≤ a2 . [Jan 2014, Jun 2010]
Le

Solution. The limits for x, y, z are as follows.


q
z : 0 to a2 − x2 − y2
w.

p
y : 0 to a2 − x2

x : 0 to a.
ww

√ √
$
2 −x2 −y2
aZ
Za Za2 −x2
∴ xyzdxdydz = xyzdzdydx.
x=0 y=0 z=0

Visit For More : www.LearnEngineering.in


Visit For More : www.LearnEngineering.in

Multiple Integrals 297

√ √
Za Za2 −x2 a2 −x2 −y2
z2
" #
= xy · dydx.
2 0
x=0 y=0

Za Za2 −x2
1
= xy · (a2 − x2 − y2 )dydx
2
0 0

.in
Za Za2 −x2
1  
= a2 xy − x3 y − xy3 dydx
2
0 0

ng
Za " 2 2 4
# √a2 −x2
1 y y y
= a2 x · − x3 − x dx
2 2 2 4 y=0
0

eri
Za
a2 x 2 x3 2
!
1 2 2 x 2 2 2
= (a − x ) − (a − x ) − (a − x ) dx
2 2 2 4
0

1
Za
e
a4 x a2 x3 a2 x3 x5 x 4
!
gin
4 2 2
= − − + − (a + x − 2a x ) dx
2 2 2 2 2 4
0
Za
a4 x5 a4 x5 a2 3
!
1 2 3
= x−a x + − x− + x dx
En

2 2 2 4 4 2
0
!a 4 a
!a !a !a !a !
1 a4 x2 1 x6 a4 x2 1 x6 a2 x4
!
2 x
= −a + − − +
2 2 2 0 4 0 2 6 0 4 2 0 4 6 0 2 4 0
arn

1  a6✁ a6✁ a6 a6✁ a6 a6✁


 
=  ✁ − ✁ + − ✁ − + ✁ 
2 ✁4 ✁4 12 ✁8 24 ✁8
!
1 6 1 1
Le

= ×a −
2 12 24
a6 2 − 1
!
=
2 24
w.

a 6
= .
48
ww

Visit For More : www.LearnEngineering.in


Visit For More : www.LearnEngineering.in

298 Engineering Mathematics - I

4.7 Volume as triple integral

Result. If V is the volume bounded by a definite$ region D in space, then the


volume is calculated by the triple integral V = dxdydz with proper limits.
D

.in
x2 y2 z2
Example 4.71. Find the volume of the ellipsoid + + = 1 using triple
a2 b2 c2

ng
integral.
Solution. We know that the ellipsoid is symmetric about the
$ coordinate planes.

eri
∴ Volume of the ellipsoid = 8×Volume in the first octant = 8 dxdydz.
V
x2 y2 z2
The ellipsoid is given by 2 + 2 + 2 = 1
a b c r
e x2 y2
gin
In the first octant, z varies from 0 to c 1− −
r a2 b2
x2
y varies from 0 to b 1 − 2
a
and x varies from 0 to a.
En

r
2 2
1− x2 − y2
q
2
a b 1− x2 c a b
arn

Z Z a Z
∴ Volume = 8 dzdydx
x=0 y=0 z=0
q
2
b 1− x
Le

r
Za Z a2 c
2
1− x2 − y2
2
a b
=8 (z)z=0 dydx
x=0 y=0
w.

q
2
b 1− x
Za Z a2 r
x2 y2
= 8c 1− − dydx
a2 b2
ww

x=0 y=0
q
2
b (1− x )
Za Z a2 s !
1 x2
= 8c b2 1 − 2 − y2 dydx
b a
x=0 y=0

Visit For More : www.LearnEngineering.in


Visit For More : www.LearnEngineering.in

Multiple Integrals 299

q
2
 b 1− x2
Za  2 x2
s a
 b (1 − ) 2
! 
8c a2 −1 y y x 
=  sin q  +2 b2 1 − 2 − y2  dx
b 2 x 2 a
b2 1 −
 
x=0 a2 0
Za b2 1 − 
x2
8c a2
= sin−1 1dx
b 2
0

.in
Za
x2
!
π
= 4bc 1 − 2 dx
2 a
0

ng
!a
x3
= 2πbc x − 2
3a 0
 a  4πabc

eri
= 2πbc a − = .
3 3

x2 y2 z2
Note. The volume of the portion of the ellipsoid 2 + 2 + 2 = 1 which lies in
1
a b c
1 4πabc πabc e
gin
the first octant is × Volume of the ellipsoid = × = .
8 8 3 6

Example 4.72. Find the volume of the sphere x2 +y2 +z2 = a2 using triple integrals.
En

[Jan 2006]
Solution. We know that the sphere is symmetric about the coordinate planes.
arn

∴ Volume of the sphere = 8× Volume of the portion of the sphere


that lies in the first octant.
The sphere is given by x2 + y2 + z2 = a2 .
Le

p
In the first octant z varies from 0 to a2 − x2 − y2 .

y varies from 0 to a2 − x2 .
w.

and x varies from 0 to a.


$
Volume of the sphere = 8 × dzdydx.
ww

V
√ √ 2 −x2 −y2
Za Za2 −x2 aZ

=8 dzdydx
x=0 y=0 z=0

Visit For More : www.LearnEngineering.in


Visit For More : www.LearnEngineering.in

300 Engineering Mathematics - I


Za Za2 −x2 √2 2 2
a −x −y
=8 (z)0 dydx
x=0y=0

Za Za2 −x2 q
=8 a2 − x2 − y2 dydx
x=0 y=0

.in
Za ! a2 −x2
a2 − x2 y y
q
=8 sin−1 √ + 2 2
a −x −y2 dx
2 a2 − x2 2 0
x=0

ng
Za
a2 − x2 −1
=8 sin (1)dx
2
x=0

eri
Za
π
=4 (a2 − x2 )dx
2
x=0

= 2π a x −
x3
2
!a
e
gin
3 0
3 4πa3
!
3 a
= 2π a − = .
3 3
En

Note
arn

The volume of the portion of the sphere that lies in the first octant is
1 1 4πa3 πa3
× Volume of the sphere = × = .
8 8 3 6
Le

Example 4.73. Find the volume of the tetrahedron bounded by the plane
w.

x y z
+ + = 1 and the coordinate planes. [Jun 2006]
a b c
Solution. Let D be the region in space bounded by the tetrahedron.
ww

x y z
The region of integration is given by + + = 1, x = 0, y = 0, z = 0.
a b c  x y
Along the region of integration z varies from 0 to c 1 − − .
a b
 x
y varies from 0 to b 1 − .
a

Visit For More : www.LearnEngineering.in


Visit For More : www.LearnEngineering.in

Multiple Integrals 301

and x varies from 0 to a.


x x y
Za b(Z1− a ) c(1−
Za−b)
∴ Volume = dzdydx.
x=0 y=0 z=0
1− ax
Za b(Z )
c(1− x − y )
= (z)0 a b dydx.

.in
x=0 y=0
x
Za b(Z1− a ) 
x y

ng
= c 1 − − dydx.
a b
x=0 y=0
Za !b(1− ax )

eri
xy y2
=c y− − dx.
a 2b 0
x=0
Za
=c
n 
b 1−
x x 
a e
− b 1−
a
x 1 2 
a
− b 1−
2b
x 2 o
a
dx
gin
x=0
Za 
bc x 2
= 1− dx
2 a
En

x=0
 a
 1 − x 3 

bc 
=   a  
2  3 −1 
a
arn

0
−abc
= [0 − 1]
6
abc
= .
6
Le

Example 4.74. Find the volume of the portion of the cylinder x2 +y2 = 1 intercepted
w.

between the plane x = 0 and the paraboloid x2 + y2 = 4 − z. [Jun 2012]


Solution. The surfaces are x2 + y2 = 1, x = 0 and x2 + y2 = 4 − z.
ww

For the required volume integral


x varies from 0 to 1

y varies from 0 to 1 − x2
z varies from 0 to 4 − x2 − y2 .

Visit For More : www.LearnEngineering.in


Visit For More : www.LearnEngineering.in

302 Engineering Mathematics - I

√ 2 −y2
Z1 Z1−x2 4−x
Z
Volume = 4 dzdydx
x=0 y=0 z=0

2
Z1 Z1−x
4−x2 −y2
= 4 [z]0 dydx

.in
x=0 y=0

Z1 Z1−x2
=4 (4 − x2 − y2 )dydx

ng
x=0 y=0
Z1  √ √ 3
! √1−x2 
2 2 y 
4(y)0 1−x − x2 (y)0 1−x −

=4  dx.

eri
3 0 
x=0

x = sin θ.
Z1
=4 2
p
2
(4 − x ) 1 − x −
(1 − x2 )3/2
!
dx.
e dx = cos θdθ.
gin
3
0
When x = 0, sin θ = 0 ⇒ θ = 0.
Zπ/2
(cos2 θ)3/2
!
π
=4 (4 − sin2 θ) cos θ − cos θdθ When x = 1 sin θ = 1 ⇒ θ = .
3 2
En

0
Zπ/2
cos3 θ
!
2
=4 4 cos θ − sin θ cos θ − cos θdθ
3
arn

Zπ/2
cos4 θ
!
2 2 2
=4 4 cos θ − sin θ cos θ − dθ
3
Le

0
Zπ/2
cos4 θ
!
=4 4 cos2 θ − sin2 θ(1 − sin2 θ) − dθ
w.

3
0
Zπ/2
cos4 θ
!
2 2 4
ww

=4 4 cos θ − sin θ + sin θ − dθ


3
0
 π/2
Zπ/2 Zπ/2 Zπ/2

 Z 
1
= 4 4 cos2 θdθ − sin2 θdθ + sin4 θdθ − cos4 θdθ
 
 3 
0 0 0 0

Visit For More : www.LearnEngineering.in


Visit For More : www.LearnEngineering.in

Multiple Integrals 303

" #
1 π 1 π 3 1 π 1 3 1 π
=4 4· · − · + · · − · · ·
2 2 2 2 4 2 2 3 4 2 2
" #
1 π 3 1
=4· · 4−1+ −
2 2 4 4
" #
3 1
7=π 3+ −
4 4

.in
" #
1
=π 3+
2

ng
= Cubic units.
2

Example 4.75. Find the volume bounded by the cylinder x2 +y2 = 4 and the planes

eri
y + z = 4 and z = 0. [May 2011]
Solution.

e
gin
The given surface is x2 + y2 = 4, z = 0 and
y + z = 4.
In the region, x varies from −2 to 2,
En

√ √
y varies from − 4 − x2 to 4 − x2 , z
varies from 0 to 4 − y.
arn


Z2 Z4−x2 Z4−y
∴ Volume = dzdydx

x=−2 y=− 4−x2 z=0
Le


Z2 Z4−x2
4−y
= [z]0 dydx.

w.

x=−2 y=− 4−x2



Z2 Z4−x2
= (4 − y)dydx.
ww


x=−2 y=− 4−x2

Z2  √ " 2 # √4−x2 


2 y 
4(y) √4−x 2 −
  dx.
= √
− 4−x 2 − 4−x2 
x=−2

Visit For More : www.LearnEngineering.in


Visit For More : www.LearnEngineering.in

304 Engineering Mathematics - I

Z2 "  p p  1 #
2 2 2 2
= 4 4 − x + 4 − x − 4 − x − (4 − x ) dx.
2
−2
Z2 p
= 8 4 − x2 dx.
−2

.in
Z2 p
=8×2 4 − x2 dx.
0

ng
" p #2
x 2
4 −1  x 
= 16 4 − x + sin
2 2 2 0

eri
h i
= 16 2 sin−1 1
π
= 16 × 2 ×
2
= 16π Cubic units.
e
gin
Example 4.76. Find the volume of the solid bounded by the cylinders x2 + y2 = a2
and x2 + z2 = a2 .
En

Solution. The surfaces are the cylinders x2 + y2 = a2 and x2 + z2 = a2 .


arn

In the given region,


x varies from −a to a,
√ √
Le

y varies from − a2 − x2 to a2 − x2 and


√ √
z varies from − a2 − x2 to a2 − x2 .
w.

√ √
Za Za2 −x2 Za2 −x2
Volume = dzdydx.
ww

√ √
x=−a y=− a2 −x2 z=− a2 −x2

Za Za2 −x2 √
2 2
= [z] √a −x dydx.
− a −x2
2

x=−a y=− a2 −x2

Visit For More : www.LearnEngineering.in


Visit For More : www.LearnEngineering.in

Multiple Integrals 305


Za Za2 −x2  p p 
= a2 − x2 + a2 − x2 dydx.
x=−a y=− √a2 −x2

Za Za2 −x2 p
= 2 a2 − x2 dydx
x=−a y=− √a2 −x2

.in
Za p √
2 2
=2 a2 − x2 (y) √a −x
2 2
dx
− a −x
x=−a

ng
Za p p p 
=2 2
a −x 2 2 2 2 2
a − x + a − x dx
x=−a

eri
Za p p
=2 a2 − x2 · 2 a2 − x2 dx.
x=−a
Za "
x3
!a #
e
gin
2 2 2 a
=4 (a − x )dx. = 4 a · (x)−a −
3 −a
x=−a
2a3 4a3 16a3
" # " #
2 1 3 3 3
= 4 a (a + a) − (a + a ) = 4 2a − =4× = Cubic units.
3 3 3 3
En

Example 4.77. Find the volume of the region bounded by the surfaces y2 = 4ax
and x2 = 4ay and the plane z = 0 and z = 3.
arn

Solution. Let us find the common points of the region bounded by

y2 = 4ax (1)
Le

and x2 = 4ay. (2)


x2
(2) ⇒ y = .
w.

4a
Substituting in (1) we get
ww

x4
= 4ax.
16a2
x4 = 64a3 x.

x4 − 64a3 x = 0

Visit For More : www.LearnEngineering.in


Visit For More : www.LearnEngineering.in

306 Engineering Mathematics - I

x(x3 − 64a3 ) = 0

x = 0, x3 = 64a3

x = 4a.

For the given region,


x varies from 0 to 4a,

.in
x2 √
y varies from to 4ax,
4a
z varies from 0 to 3.

ng

Z4a Z4axZ3
∴ Volume = dzdydx

eri
x=0 y= x2 z=0
4a
√ √
Z4a Z4ax Z4a Z4ax
3
= [z]0 dydx. = 3dydx.
x=0 y= x2 e x=0 y= x2
gin
4a 4a

Z4a √
=3 [y] x24ax dx.
4a
x=0
En

Z4a √ x2
!
=3 4ax − dx.
4a
0
arn

 √ x3/2 4a 1
 " # " 3 #4a 
x 
= 3 2 a − · 
3/2 0 4a 3 0 
!
√ 2 3/2 1 3
Le

= 3 2 a (4a) − · 64a
3 12a

√ 16a2
!
4 a
=3 · 4a 4a −
3 3
w.

16a2
!
16a √ √
=3 a·2 a−
3 3
ww

2 2 16a2
!
32a 16a
=3 − =3× = 16a2 Cubic units.
3 3 3
Example 4.78. Find the volume of the solid bounded by the surfaces y2 = x and
the planes x + y + z = 2 and z = 0.

Visit For More : www.LearnEngineering.in


Visit For More : www.LearnEngineering.in

Multiple Integrals 307

Solution. The given region is bounded by


y2 = x (1)

x+y+z=2 (2)

z = 0. (3)

Whenz = 0, (2) ⇒ x + y = 2

.in
x = 2 − y.

Substituting in (1) we get

ng
y2 = 2 − y

y2 + y − 2 = 0

eri
(y − 1)(y + 2) = 0

∴ y = −2, 1.

For the given region we have, e


gin
y varies from −2 to 1,
x varies from y2 to 2 − y and
En

z varies from 0 to 2 − x − y.
Z1 Z2−y 2−x−y
Z
∴ Volume = dzdxdy
arn

y=−2 x=y2 z=0

Z1 Z2−y
2−x−y
= [z]0 dxdy
Le

y=−2 x=y2

Z1 Z2−y
= (2 − x − y)dxdy
w.

y=−2 x=y2

Z1  " 2 #2−y 
 2−y x 2−y 

= 2 · [x]y2 − − y[x]y2  dy
ww

2 y2
y=−2
Z1 !
2 1 2 4 2
= 2{2 − y − y } − {(2 − y) − y } − y{2 − y − y } dy.
2
−2

Visit For More : www.LearnEngineering.in


Visit For More : www.LearnEngineering.in

308 Engineering Mathematics - I

Z1 !
1
= 4 − 2y − 2y2 − [4 + y2 − 4y − y4 ] − [2y − y2 − y3 ] dy
2
−2
Z1
y2 y4
!
2 2 3
= 4 − 2y − 2y − 2 − + 2y + − 2y + y + y dy
2 2
−2
Z1
3y2 y4

.in
!
3
= 2 − 2y − +y + dy
2 2
−2
#1 " #1 " 4 #1 " #1
y2 3 y3 1 y5
"

ng
y
= 2[y]1−2 −2 − + +
2−2 2 3 −2 4 −2 2 5 −2
1 1 1
= 2(1 + 2) − (1 − 4) − (1 + 8) + (1 − 16) + (1 + 32)

eri
2 4 10
9 15 33
=6+3− − +
2 4 10
9 15 33 180 − 90 − 75 + 66 81
=9− −
2 4
+
10
=
e
20
=
20
Cubic units.
gin
En
arn
Le
w.
ww

Visit For More : www.LearnEngineering.in


Visit For More : www.LearnEngineering.in

5 Differential Equations

.in
ng
5.1 Introduction

A differential equation is an equation involving one dependent variable and its

eri
derivatives with respect to one or more independent variables.
Ordinary Differential Equation

e
An ordinary differential equation is an equation in which there is only one
gin
independent variable and so the derivatives involved in it are ordinary
derivatives.
Partial differential equation
En

A partial differential equation is an equation in which there are two or more


independent variables and partial differential coefficients with respect to any one
arn

of them.
Examples
∂2 u ∂2 u
(i) 2 = a2 2 .
Le

∂x ∂t
2 ∂2 u 2
2∂ u 2∂ u
(ii) x + 2xy + y = 0.
∂x2 ∂x∂y ∂y2
∂u ∂u
w.

(iii) x + y = 5u.
∂x ∂y
Order of a differential equation
ww

The order of a differential equation is the order of the highest derivative that
occurs in it.
Degree of a differential equation
The degree of a differential equation is the degree of the highest derivative

Visit For More : www.LearnEngineering.in


Visit For More : www.LearnEngineering.in

310 Engineering Mathematics - I

occurring in it after the equation has been reduced to a form free from radicals
and fractions as far as the derivatives are concerned.
Examples
d2 x
(i) For the differential equation + a2 x = 0, the order is 2 and the degree is 1
dt2
 !2  23 2
 dy  = c d y , the order is 2 and degree is 2,

(ii) For the differential equation 1 +

.in
dx dx2
 !2 3 2y 2
!
 dy  2 d
since it can be reduced to the form 1 +  = c .
dx  dx2

ng
Formation of a differential equation
An ordinary differential equation can be formed by eliminating certain arbitrary

eri
constants from a relation involving variables and constants. In the study of
applied mathematics, every geometrical and physical problems when translated


e
into a mathematical model will always give rise to a differential equation.

gin
Worked Examples
✍ ✌

Example 5.1. Form the differential equation of the family of straight lines passing
through the origin.
En

Solution. The equation of the family of straight lines passing through the origin
is given by
arn

y = mx. (1)

Differentiating w.r.t. x, we get


dy
Le

= m.
dx
Substituting the value of m in (1) we obtain
w.

!
dy
y= x
dx
ww

which is the required differential equation.

Example 5.2. Obtain the differential equation of the family of circles


x2 + y2 + 2ax + r2 = 0 by eliminating the arbitrary constant a.

Visit For More : www.LearnEngineering.in


Visit For More : www.LearnEngineering.in

Differential Equations 311

Solution. The given equation is

x2 + y2 + 2ax + r2 = 0. (1)

Differentiating w.r.t. x, we get


dy
2x + 2y + 2a = 0
dx
dy

.in
x+y = −a
dx !
dy
a=− x+y .
dx

ng
Substituting the value of a in (1) we obtain
!
2 2 dy
x + y − 2x x + y + r2 = 0

eri
dx
dy
x2 + y2 − 2x2 − 2xy + r2 = 0
dx
2xy
dy
e
= y2 − x2 + r2 ,
gin
dx
which is the required differential equation.

Example 5.3. Find the differential equation of the family of circles with centre
En

(a, b) and radius r.


Solution. The equation of the circle is
arn

(x − a)2 + (y − b)2 = r2 . (1)


dy
Differentiating w.r.t. x, we get 2(x − a) + 2(y − b) =0
dx
dy
Le

x − a + (y − b) = 0. (2)
dx
Again differentiating we get
w.

d2 y dy dy
1 + (y − b) + =0
dx2 dx dx  !2 
d2 y dy
ww

 
(y − b) 2 = − 1 + 
dx dx 
 dy 2
1 + dx
y−b=− 2
.
d y
dx2

Visit For More : www.LearnEngineering.in


Visit For More : www.LearnEngineering.in

312 Engineering Mathematics - I

Substituting this in (2) we get


 dy 2
1 + dx dy
x−a− 2
=0
d y dx
dx2
 dy 2
dy 1 + dx
x−a= .
dx d2 y

.in
dx2

Substituting the values of x − a and y − b in (1) we obtain


   2   dy 2 2
!2 1 + dy 2

ng
dy dx 1 + dx
2
+ 2 = r2
dx  2
d y
  2
d y
dx2 dx2

eri
  dy 2 2
1+ dx

 dy
!2 

2 1 +  = r2

d2 y dx 
dx2
e
gin
  dy 2 3
1+ dx
 2 = r2
d2 y
dx2
En

 !2  32 2

1 + dy  = r d y ,

dx dx2
arn

which is the required differential equation.

Example 5.4. Form the differential equation of the simple harmonic motion
Le

x = a cos nt.
Solution. x = a cos nt.
Differentiate w.r.t. t we get
w.

dx
= −an sin nt.
dt
Again differentiating w.r.t. t we obtain
ww

d2 x
= an2 (− cos nt) = −n2 a cos nt = −n2 x
dt2
d2 x
+ n2 x = 0,
dt2
which is the required differential equation.

Visit For More : www.LearnEngineering.in


Visit For More : www.LearnEngineering.in

Differential Equations 313

Solution of a differential equation


A solution of a differential equation is a relation between the variables that
satisfies the given differential equation. A solution of a differential equation is
also called as the integral of the equation.
Examples
d2 x

.in
1. x = a cos nt is a solution of the differential equation + n2 x = 0.
dt2
d2 y dy
2. For the differential equation 2
+ 3 + 2y = 0, y = e−x and y = e−2x are
dx dx

ng
solutions.
General Solution

eri
A general or complete solution of a differential equation is the one in which the
number of arbitrary constants is equal to the order of the differential equation.
Example. Consider
e
gin
y = A cos αx + B sin αx (1)

where A and B are the arbitrary constants.


En

Differentiating w.r.t. x we get

dy
= −Aα sin αx + Bα cos αx.
dx
arn

d2 y
= −Aα2 cos αx − Bα2 sin αx.
dx2
= −α2 (A cos αx + B sin αx) = −α2 y.
Le

d2 y
+ α2 y = 0. (2)
dx2
w.

Hence, (1) is a general solution of (2) as the number of arbitrary constants A, B is


the same as the order of (2).
ww

Particular Solution
A particular solution is a solution that can be obtained by giving particular
values to the arbitrary constants in the general solution.
Example. y = 2 cos αx + sin αx is a particular solution of (2), since it can be derived

Visit For More : www.LearnEngineering.in


Visit For More : www.LearnEngineering.in

314 Engineering Mathematics - I

dy
from the general solution by giving initial values of 0, 2 and α to x, y and
dx
respectively.
Linear independent solutions
Two solutions y1 and y2 of the differential equation

d2 y dy

.in
+ a1 (x) + a2 (x)y = 0 (1)
dx2 dx

ng
are said to be linearly independent, if there exist constants c1 and c2 such that
c1 y1 + c2 y2 = 0 implies c1 = 0 and c2 = 0.

eri
If c1 and c2 are not both zero, then the two solutions y1 and y2 are said to be linearly
dependent.
If y1 and y2 are any two solutions of (1) then the linear combination c1 y1 + c2 y2
e
where c1 and c2 are constants is also a solution of (1).
gin
Example. y1 = e−x and y2 = e−2x are two independent solutions of the differential
equation
En

d2 y dy
2
+ 3 + 2y = 0.
dx dx
arn

Then, the general solution is y = c1 e−x + c2 e−2x .


Linear Differential Equation
A linear differential equation is an equation in which the dependent variable and
Le

its derivatives occur only in the first degree and there is no product of dependent
variable and derivative or product of derivatives.
w.

Note. A differential equation which is not linear is called a non linear


differential equation.
ww

Examples
d2 y
1. x 2 + y = x2 is linear.
dx
dy
2. y + x2 = 0 is nonlinear.
dx

Visit For More : www.LearnEngineering.in


Visit For More : www.LearnEngineering.in

Differential Equations 315

5.2 Linear differential equation with constant coefficients

The general form of the nth order linear ordinary differential equation with
constant coefficients is
dn y dn−1 y dy
a0 n
+ a1 n−1
+ · · · + an−1 + an y = Q(x) (1)
dx dx dx

.in
where a0 , a1 , a2 , . . . , an are constants with a0 , 0. If Q(x) = 0 then
dn y dn−1 y dy
(1) ⇒ a0 n
+ a1 n−1
+ · · · + an−1 + an y = 0 (2)

ng
dx dx dx
which is the homogeneous equation corresponding to (1).
The general solution of (2) is called the complementary function of (1) and it is

eri
denoted by yc . The general solution of (2) contains n arbitrary constants. A solution
which contains no arbitrary constants is a particular solution.

e
If y p is a particular solution of (1), then the general solution of (1) is y = yc + y p .
gin
This is also called as the complete solution of the ordinary differential equation.
Computation of Complementary function
d d2 dn
Let = D, 2 = D2 , . . . , n = Dn .
En

dx dx dx
Then (1) becomes

(a0 Dn + a1 Dn−1 + · · · + an−1 D + an )y = Q(x) (3)


arn

The auxiliary equation is

a0 mn + a1 mn−1 + a2 mn−2 + · · · + an−1 m + an = 0.


Le

Let m1 , m2 , . . . , mn be its roots.


Case (i) If m1 , m2 , . . . , mn are real and different then
w.

yc = CF = c1 em1 x + c2 em2 x + · · · + cn emn x .


ww

Case (ii) If some of the roots (say r) are equal, where r < n
i.e m1 = m2 = · · · = mr = m, then

C.F = yc = (c1 + c2 x + c3 x2 + · · · + cr xr−1 )emx + cr+1 emr+1 x + · · · + cn emn x .

Visit For More : www.LearnEngineering.in


Visit For More : www.LearnEngineering.in

316 Engineering Mathematics - I

In particular, if two roots are equal say m1 = m2 = m then,

yc = (c1 + c2 x)emx + c3 em3 x + · · · + cn emn x .

Case (iii) If two roots are complex, say m1 = α + iβ and m2 = α − iβ and the other
roots are real and different, then

.in
yc = eαx (c1 cos βx + c2 sin βx) + c3 em3 x + · · · + cn emn x .

ng
Case (iv) If m1 = m2 = α + iβ, m3 = m4 = α − iβ and the other roots are real and
different, then

eri
yc = [(c1 + c2 x) cos βx + (c3 + c4 x) sin βx]eαx + c5 em5 x + · · · + cn emn x .

Computation of particular integral (y p )


e
gin
Let f (D) = a0 Dn + a1 Dn−1 + · · · + an−1 D + an .
Then (3) can be written as f (D)y = Q(x).

1
En

Now y p = P.I. = Q(x)


f (D)

5.2.1 Type I
arn

Let Q(x) = eαx .


Case (i) If α is not equal to any of the mi (i = 1, 2, . . . , n), then f (α) , 0. In this case
eαx
Le

y p = PI = (Replace D by α).
f (α)
Case (ii) If α is equal to some of the m′i s( say m1 = m2 = · · · = mr ) then
f (D) = (D − α)r g(D) where g(α) , 0.
w.

1 αx eαx xr
∴ y p = PI = e = .
(D − α)r g(D) g(α)r!
ww

In particular, if r = 1, then

1 eαx
yp = eαx = x.
(D − α)g(D) g(α)

Visit For More : www.LearnEngineering.in


Visit For More : www.LearnEngineering.in

Differential Equations 317

1 αx eαx x2
When r = 2, y p = e = .
(D − α)2 g(D) g(α) 2!
1 αx eαx x3
When r = 3, y p = e = etc.
(D − α)3 g(D) g(α) 3!

✎ ☞
Worked Examples
✍ ✌

.in
Example 5.5. Solve (D2 − 4)y = 1. [Dec 2013]
Solution. The auxiliary equation is

ng
m2 − 4 = 0 ⇒ m2 = 4 ⇒ m = ±2.

∴ yc = c1 e2x + c2 e−2x .

eri
1 1 0x e0x 1
yp = = e = =− .
D2 − 4 D2 − 4 0−4 4
1
Solution is y = yc + y p = c1 e2x + c2 e−2x − .
4 e
gin
Example 5.6. Solve (4D2 − 4D + 1)y = 4. [Jun 1996]
1 1
Solution. A.E is 4m2 − 4m + 1 = 0 ⇒ (2m − 1)2 = 0 ⇒ m = , .
2 2
En

1
yc = e 2 x (c1 + c2 x).
1 1 4
yp = (4) = e0x = = 4.
4D2 − 4D + 1 4D2 − 4D + 1 1
arn

x
Solution is y = yc + y p = (c1 + c2 x)e 2 + 4.

Example 5.7. Find the particular integral of (D2 − 2D + 1)y = cos hx.
Le

[Jun 2013, Jun 2005]


Solution. 1 1 e x + e−x
!
PI = cos hx =
w.

D2 − 2D + 1 (D − 1)2 2
" #
1 1 1
= 2
ex + e−x
2 (D − 1) (D − 1)2
ww

1 x x2 e−x
" #
= e +
2 2 (−1 − 1)2
" 2 x
e−x x2 e x e−x
#
1 x e
= + = + .
2 2 4 4 8

Visit For More : www.LearnEngineering.in


Visit For More : www.LearnEngineering.in

318 Engineering Mathematics - I

Example 5.8. Find the particular integral of (D2 − 4)y = cosh 2x. [May 2011]
Solution. 1
PI = cosh 2x
D2 −4 " 2x
e + e−2x
#
1
=
(D + 2)(D − 2) 2
" #
1 1 1
= e2x + e−2x

.in
2 (D + 2)(D − 2) (D + 2)(D − 2)
1 x 2x
 x −2x 
= e + e
2 4 −4
x 2x

ng
= [e − e−2x ]
8
x
= sinh 2x.
4

eri
Example 5.9. Find the particular integral of (D3 − 1)y = e2x . [May 2005]
1 e2x e2x
Solution. PI = 3
D −1
e2x = 3
2 −1
=
7
.
e
gin
Example 5.10. Find the particular integral of (D − 1)2 = sinh x. [Nov 2003]
Solution. " x
e − e−x
#
1 1
En

PI = sinh x =
(D − 1)2 (D − 1)2 2
" #
1 1 1
= 2
ex − e−x
2 (D − 1) (D − 1)2
arn

1 x2 x 1 −x
" #
= e − e
2 2 4
x 2 e−x
= ex − .
Le

4 8

d2 y dy
Example 5.11. Solve + 4 + 5y = −2 cosh x.
w.

dx 2 dx
Solution. The A.E is
ww

m2 + 4m + 5 = 0 ⇒ (m + 2)2 + 5 − 4 = 0

⇒ (m + 2)2 = −1 ⇒ m + 2 = ±i ⇒ m = −2 ± i.

yc = e−2x (c1 cos x + c2 sin x).

Visit For More : www.LearnEngineering.in


Visit For More : www.LearnEngineering.in

Differential Equations 319

2
yp = [−(e x + e−x )]
2(D2
+ 4D + 5)
−1 1
= 2 ex − 2 e−x
D + 4D + 5 D + 4D + 5
−e x e−x
= −
1+4+5 1−4+5
−e x e−x
= − .

.in
10 2

The general solution is y = yc + y p

ng
e x e−x
y = e−2x (c1 cos x + c2 sin x) − − .
10 2

eri
Example 5.12. Find the particular integral of (D + 2)(D − 1)2 y = e−2x + 2 sinh x.
[May 2006]
Solution.
PI =
1
e
[e−2x + 2 sinh x]
gin
(D + 2)(D − 1)2
1
= 2
[e−2x + e x − e−x ]
(D + 2)(D − 1)
1 1 1
e−2x + ex − e−x
En

= 2 2
(D + 2)(D − 1) (D + 2)(D − 1) (D + 2)(D − 1)2
x −2x x2 x e−x
= e + e −
9 6 1(4)
arn

2
x −2x x x e−x
= e + e − .
9 6 4

Example 5.13. Solve (D2 + 1)2 y = 0. [May 2008]


Le

Solution. A.E is (m2 + 1)2 = 0.


m2 = −1, m2 = −1 =⇒ m = ±i, m = ±i.
w.

The complex roots are repeated.


∴ yc = CF = (c1 + c2 x) cos x + (c3 + c4 x) sin x.
ww

y p = PI = 0.
Solution is y = yc + y p

y = (c1 + c2 x) cos x + (c3 + c4 x) sin x.

Visit For More : www.LearnEngineering.in


Visit For More : www.LearnEngineering.in

320 Engineering Mathematics - I

Example 5.14. Solve (D2 + 1)y = 0 given that y(0) = 0, y′ (0) = 1. [May 1996]
Solution. A.E is m2 + 1 = 0 =⇒ m2 = −1 =⇒ m = ±i.

CF = yc = e0x [c1 cos x + c2 sin x] = c1 cos x + c2 sin x.

y p = PI = 0.

.in
Solution is y = yc + y p

y = c1 cos x + c2 sin x.

ng
Given: y(0) = 0.

eri
=⇒ c1 = 0
y′ = −c1 sin x + c2 cos x.
Given: y′ (0) = 1 =⇒ c2 = 1.
Hence, solution is, y = sin x. e
gin
Example 5.15. Find the particular integral of (D2 − 4D + 4)y = 2 x . [Dec 2012]
1 1
Solution. P.I = 2 2x = e x log 2
(D − 2)2
En

D − 4D + 4
2x
= .
(log 2 − 2)2
arn

Example 5.16. Find the particular integral of (D2 − 4)y = 3 x . [May 2007]
1 x 1 x log 3 1 x log 3 3x
Solution. PI = 2 3 = 2 e = e = .
D −4 D −4 (log 3)2 − 4 (log 3)2 − 4
Le

d2 y dy
Example 5.17. Solve + 6 + 5y = e2x .
dx2 dx
Solution. (D2 + 6D + 5)y = e2x .
w.

A.E is, m2 + 6m + 5 = 0 ⇒ (m + 1)(m + 5) = 0 ⇒ m = −1, −5.

yc = c1 e−x + c2 e−5x .
ww

1 2x 1 2x e2x e2x
yp = e = e = = .
D2 + 6D + 5 (D + 1)(D + 5) 3(7) 21
e2x
Solution is, y = yc + y p = c1 e−x + c2 e−5x + .
21

Visit For More : www.LearnEngineering.in


Visit For More : www.LearnEngineering.in

Differential Equations 321

d3 y d2 y dy
Example 5.18. Solve + 2 + = e2x .
dx3 dx2 dx
Solution. The A.E is

m3 + 2m2 + m = 0 ⇒ m(m2 + 2m + 1) = 0

⇒ m(m + 1)2 = 0 ⇒ m = 0, m = −1, −1.

.in
yc = c1 e0x + (c2 + c3 x)e−x

yc = c1 + e−x (c2 + c3 x).

ng
1
yp = e2x
D3 + 2D2 + D
1

eri
= e2x
D(D + 1)2
e2x e2x
= = .
2(3)2 18
e
gin
The general solution is y = yc + y p
e2x
y = c1 + e−x (c2 + c3 x) + .
18
En

Example 5.19. Solve (D2 − 2D + 1)y = (e x + 1)2 . [ Dec 2010]


Solution. AE is m2 − 2m + 1 = 0 =⇒ (m − 1)2 = 0 =⇒ m = 1, 1.
arn

yc = e x (c1 + c2 x)
1
yp = (e x + 1)2
D2− 2D + 1
1
(e2x + 1 + 2e x )
Le

= 2
(D − 1)
1 1 1
= 2
e2x + 2
e0x + 2 ex
(D − 1) (D − 1) (D − 1)2
w.

e2x x2
= + 1 + 2 ex
1 2
ww

= e2x + 1 + x2 e x .

Solution is y = yc + y p

y = e x (c1 + c2 x) + e2x + 1 + x2 e x .

Visit For More : www.LearnEngineering.in


Visit For More : www.LearnEngineering.in

322 Engineering Mathematics - I

5.2.2 Type II

Q(x) = sin ax or cos ax where a is a constant.

1 1
y p = P.I. = sin ax or cos ax.
f (D) f (D)

Working rule. Replace D2 by −a2 and evaluate till D is eliminated.

.in
If f (D) = D2 + a2 , then f (D) = 0 when D2 is replaced by −a2 . In this case
Z
1 x −x

ng
P.I. = 2 2
sin ax = sin axdx = cos ax.
D +a 2 2a

and

eri
Z
1 x x
2 2
cos ax = cos axdx = sin ax.
D +a 2 2a
Another method
e
gin
1 1 iax
P.I. = sin ax = Imaginary Part of e
f (D) f (D)

and
En

1 1 iax
P.I. = cos ax = Real Part of e
f (D) f (D)
which can be evaluated using Type I.
arn

Note
Suppose f (D) = 0 when D2 is replaced by −a2 , then f (D) is of the form f (D2 ).
1 cos ax x cos ax
Then, P.I. = . If f (−a2 ) = 0 then P.I. = ′ if f ′ (−a2 ) , 0.
Le

2
cos ax = 2
f (D ) f (−a ) f (−a2 )
x2 cos ax
If f ′ (−a2 ) = 0, then P.I. = ′′ if f ′′ (−a2 ) , 0, and so on. In a similar way the
f (−a2 )
w.

particular integral for sin ax can be evaluated replacing cos ax.


✎ ☞
Worked Examples
✍ ✌
ww

Example 5.20. Find the particular integral of (D2 + 4)y = sin 2x.
[Dec 2009, Dec 2011]
Solution.

Visit For More : www.LearnEngineering.in


Visit For More : www.LearnEngineering.in

Differential Equations 323

Z
1 x
PI = 2
sin 2x = sin 2xdx [∵ f (−a2 ) = 0]
D +4 2
!
x − cos 2x
=
2 2
x
= − cos 2x.
4

Example 5.21. Find the particular integral of (D2 + 1)y = sin x sin 2x. [May 1997]

.in
Solution. 1
PI = (sin x sin 2x)
D2+1"

ng
#
−1 cos 3x − cos x
= 2
D +1 2
" #
−1 1 1
= cos 3x − 2 cos x

eri
2 D2 + 1 D +1
" Z #
−1 cos 3x x
= − cos xdx
2 −32 + 1 2

=−
"
1 cos 3x x
− sin x
#
e
gin
2 −8 2
cos 3x x sin x
= + .
16 4
En

Example 5.22. Find the particular


Z integral of (D2 + 1)y = sin x. [Jun 2010]
1 x −x
Solution. P.I = 2 sin x = sin xdx = cos x.
D +1 2 2
arn

Example 5.23. Find the particular integral of (D2 + 4D + 2)y = sin 3x. [May 1998]
Solution. 1 1 1
PI = sin 3x = sin 3x = sin 3x
D2 + 4D + 2 −9 + 4D + 2 4D − 7
(4D + 7) 4 cos 3x(3) + 7 sin 3x
Le

= sin 3x =
(4D + 7)(4D − 7) 16D2 − 49
12 cos 3x + 7 sin 3x
=
16(−9) − 49
w.

12 cos 3x + 7 sin 3x
=
−144 − 49
1
ww

=− [12 cos 3x + 7 sin 3x].


193

Example 5.24. Find the particular 2


Z integral of (D + 4)y = cos 2x. [May 2001]
1 x x sin 2x x sin 2x
Solution. PI = 2 cos 2x = cos 2xdx = = .
D +4 2 2 2 4

Visit For More : www.LearnEngineering.in


Visit For More : www.LearnEngineering.in

324 Engineering Mathematics - I

Example 5.25. Find the particular integral of (D2 + 1)2 y = sin 2x.
1 sin 2x sin 2x
Solution. PI = 2 2
sin 2x = 2
= .
(D + 1) (−4 + 1) 9

Example 5.26. Find the particular integral of (D2 + 1)y = sin2 x.


Solution. 1 1 1 − cos 2x
!
2
PI = 2 sin x = 2

.in
D +1 D +1 2
" #
1 1 1
= − cos 2x
2 D2 + 1 D2 + 1

ng
" #
1 1 0x cos 2x
= e −
2 D2 + 1 −4 + 1
" #
1 cos 2x

eri
= 1+
2 3
1 cos 2x
= + .
2 6

e
gin
Example 5.27. Find the particular integral of (D"2 + 1)y = sin(2x
# + 5).
1 sin(2x + 5) sin(2x + 5)
Solution. PI = 2 sin(2x + 5) = =− .
D +1 −4 + 1 3
En

Example 5.28. Find the particular integral of (D2 + 4D + 8)y = cos(2x + 3).
Solution. 1 1
PI = cos(2x + 3) = cos(2x + 3)
arn

D2 + 4D + 8 −4 + 4D + 8
1 1 1
= cos(2x + 3) = cos(2x + 3)
4D" + 4 4 D + 1#
1 (D − 1)
= cos(2x + 3)
Le

4 (D − 1)(D + 1)
" #
1 −2 sin(2x + 3) − cos(2x + 3)
=
4 D2 − 1
w.

" #
1 2 sin(2x + 3) + cos(2x + 3)
=−
4 −4 − 1
1
ww

= [2 sin(2x + 3) + cos(2x + 3)] .


20

Example 5.29. Solve (D2 − 4D + 3)y = sin 3x cos 2x. [May 2007]
Solution. The A.E is, m2 − 4m + 3 = 0 ⇒ (m − 1)(m − 3) = 0 ⇒ m = 1, 3.

Visit For More : www.LearnEngineering.in


Visit For More : www.LearnEngineering.in

Differential Equations 325

yc = c1 e x + c2 e3x .
1
yp = sin 3x cos 2x
D2− 4D + 3
1 1
= 2
(sin 5x + sin x)
2 D − 4D + 3
1 1 1 1
= 2
sin 5x + 2
sin x
2 D − 4D + 3 2 D − 4D + 3

.in
= PI1 + PI2 .
1 1 1 1
P.I1 = 2
sin 5x = sin 5x

ng
2 D − 4D + 3 2 (−25 − 4D + 3)
1 1 −1 1
= sin 5x = sin 5x
2 (−22 − 4D) 4 (11 + 2D)

eri
1 (2D − 11)
=− sin 5x
4 (2D + 11)(2D − 11)
1 2 × 5 cos 5x − 11 sin 5x
=−
4D2 − 121
4
1 10 cos 5x − 11 sin 5x
e
gin
=−
4 4(−25) − 121
1 10 cos 5x − 11 sin 5x
=−
4 −221
10 cos 5x − 11 sin 5x
En

= .
884 " #
1 1 1 1
P.I2 = sin x = sin x
2 D2 − 4D + 3 2 (−1) − 4D + 3
arn

1 1 1 1
= sin x = sin x
2 2 − 4D 4 1 − 2D
1 (1 + 2D)
= sin x
4 (1 − 4D2 )
Le

1 sin x + 2 cos x
=
4 5
sin x + 2 cos x
w.

= .
20
The general solution is y = yc + y p
ww

10 cos 5x − 11 sin 5x sin x + 2 cos x


y = c1 e x + c2 e3x + + .
884 20
Example 5.30. Solve (D2 + 1)y = sin2 x. [May 2006]
1 − cos 2x
Solution. (D2 + 1)y = .
2

Visit For More : www.LearnEngineering.in


Visit For More : www.LearnEngineering.in

326 Engineering Mathematics - I

The A.E is m2 + 1 = 0 ⇒ m2 = −1 ⇒ m = ±i

yc = e0x (c1 cos x + c2 sin x) = c1 cos x + c2 sin x


1 1 1
yp = (1 − cos 2x) = − cos 2x
2(D2 + 1) + 1) 2(D2+ 1) 2(D2
1 cos 2x 1 1

.in
= e0x − = + cos 2x.
2(D2 + 1) 2(−4 + 1) 2 6

The general solution is y = yc + y p

ng
1 1
y = c1 cos x + c2 sin x + + cos 2x.
2 6

eri
Example 5.31. Solve the equation (D2 + 16)y = cos3 x. [Dec. 2010]
Solution. A.E.is

e
gin
m2 + 16 = 0 ⇒ m2 = −16 ⇒ m = ±4i

yc = e0x (c1 cos 4x + c2 sin 4x)

yc = c1 cos 4x + c2 sin 4x.


En
arn

1
yp = cos3 x
D2+ 16 " #
1 1 cos 3θ = 4 cos3 θ − 3 cos θ
Le

= 2 (cos 3x + 3 cos x)
D + 16 4
" # 4 cos3 θ = cos 3θ + 3 cos θ
1 1 1
= cos 3x + 3 2 cos x 1
4 D2 + 16 D + 16 cos3 θ = (cos 3θ + 3 cos θ)
w.

"
1 cos 3x 3 cos x
# "
1 cos 3x 3 cos x
# 4
= + = +
4 −9 + 16 −1 + 16 4 7 15
ww

cos 3x 3 cos x
yp = + .
28 20
cos 3x 3 cos x
Solution is y = yc + y p = c1 cos 4x + c2 sin 4x + +
28 20

Visit For More : www.LearnEngineering.in


Visit For More : www.LearnEngineering.in

Differential Equations 327

Example 5.32. Solve (D2 − 3D + 2)y = 2 cos(2x + 3) + 2e x . [Dec 2011, May 2005]
Solution. The A.E is
m2 − 3m + 2 = 0 ⇒ (m − 1)(m − 2) = 0 ⇒ m = 1, 2.
yc = c1 e x + c2 e2x .
1 1
yp = 2 cos(2x + 3) + 2 2e x
D2 − 3D + 2 D − 3D + 2

.in
= PI1 + PI2 .
1 1
PI1 = 2 cos(2x + 3) = 2 cos(2x + 3)

ng
D2 − 3D + 2 −4 − 3D + 2
1 1
=2 cos(2x + 3) = −2 cos(2x + 3)
−2 − 3D 2 + 3D

eri
(2 − 3D)
= −2 cos(2x + 3)
4 − 9D2
2 cos(2x + 3) + 6 sin(2x + 3)
= −2
1
4 + 36
e
gin
= − 2[cos(2x + 3) + 3 sin(2x + 3)]
20
1
= − [cos(2x + 3) + 3 sin(2x + 3)].
10
1 ex
e x = 2x = −2xe x .
En

PI2 = 2
(D − 1)(D − 2) −1
The general solution is y = yc + y p
1
y = c1 e x + c2 e2x − [cos(2x + 3) + 3 sin(2x + 3)] − 2xe x .
arn

10
Example 5.33. Solve (D2 + 16)y = e−3x + cos 4x.
Solution. The A.E is
Le

m2 + 16 = 0 ⇒ m2 = −16 ⇒ m = ±4i.
yc = e0x (c1 cos 4x + c2 sin 4x) = c1 cos 4x + c2 sin 4x.
w.

e−3x
Z
1 1 x
yp = 2 e−3x + 2 cos 4x = + cos 4xdx
D + 16 D + 16 9 + 16 2
e−3x x sin 4x e −3x x sin 4x
ww

= + = + .
25 2 4 25 8
The general solution is y = yc + y p .
e−3x x sin 4x
y = c1 cos 4x + c2 sin 4x + + .
25 8

Visit For More : www.LearnEngineering.in


Visit For More : www.LearnEngineering.in

328 Engineering Mathematics - I

5.2.3 Type III

Q(x) = xm where m is a positive integer.


xm xm 1
PI = = = [1 ± g(D)]−1 xm .
f (D) k[1 ± g(D)] k
Expand [1 ± g(D)]−1 using binomial series expansion upto Dm and operate on xm
term by term, we obtain the required particular integral.

.in
Z
1 1
Note. It is important to note that f (x) = f (x)dx. 2 f (x) means integrate
D D
f (x) w.r.t x twice and so on.

ng
The following results will be useful for our discussion.
(i) (1 + x)−1 = 1 − x + x2 − x3 + x4 − · · ·

eri
(ii) (1 + x)−2 = 1 − 2x + 3x2 − 4x3 + · · ·
(iii) (1 − x)−1 = 1 + x + x2 + x3 + x4 + · · ·
(iv) (1 − x)−2 = 1 + 2x + 3x2 + 4x3 + · · · .
e
gin
Example 5.34. Find the particular integral of (D2 + D)y = x2 + 2x + 4. [Jun 2000]
Solution. 1 1
PI = (x2 + 2x + 4) = (x2 + 2x + 4)
En

D2
+D D(D + 1)
1 1
= (1 + D)−1 (x2 + 2x + 4) = (1 − D + D2 − · · · )(x2 + 2x + 4)
D D
1 2 1
arn

= (x + 2x + 4 − 2x − 2 + 2) = (x2 + 4)
D
Z D
2 1
= (x + 4)dx [∵ stands for integration]
D
x3
Le

= + 4x.
3

Example 5.35. Find the particular integral of (D2 + 5D + 1)y = x2 . [May 2002]
w.

1
Solution. PI = 2 x2 = (1 + (D2 + 5D))−1 x2
D + 5D + 1
ww

= (1 − (D2 + 5D) + (D2 + 5D)2 + · · · )x2 = (1 − D2 − 5D + 25D2 + · · · )x2

= (1 − 5D + 24D2 )x2 = x2 − 5(2x) + 24(2)

= x2 − 10x + 48.

Visit For More : www.LearnEngineering.in


Visit For More : www.LearnEngineering.in

Differential Equations 329

Example 5.36. Solve (D3 − 3D2 − 6D + 8)y = x. [Dec 2006]


Solution. A.E is m3 − 3m2 − 6m + 8 = 0
(m − 1)(m + 2)(m − 4) = 0 ⇒ m = 1, −2, 4.
yc = c1 e x + c2 e−2x + c3 e4x .
1
yp = x
D3 − 3D2 − 6D + 8

.in
1
= h 3 2
ix
8 1 + D −3D8 −6D

ng
#−1
D3 − 3D2 − 6D
"
1
= 1+ x
8 8
D3 − 3D2 − 6D
! !
1

eri
= 1− + ··· x
8 8
" # " #
1 6 1 6
= 1 + D + ··· x = x+
8 8 8 8
1
"
3
#
e
gin
= x+ .
8 4
The general solution is y = yc "+ y p #
1 3
y = c1 e x + c2 e−2x + c3 e4x + x+ .
En

8 4
Example 5.37. Solve (D3 − D2 − D + 1)y = 1 + x2 . [Apr 2007]
Solution. The A.E is m3 − m2 −m+1=0⇒ m2 (m − 1) − (m − 1) = 0
arn

⇒ (m − 1)(m2 − 1) = 0 ⇒ m = 1, 1, −1.
yc = c1 e−x + e x (c2 + c3 x).
Le

1
yp = (1 + x2 )
D3
− −D+1 D2
1 1
= 3 2
e0x + 3 2
x2
w.

D −D −D+1 D −D −D+1
x2
=1+ = 1 + [1 − (D + D2 − D3 )]−1 x2
1 − (D + D2 − D3 )
ww

= 1 + [1 + (D + D2 − D3 ) + (D + D2 − D3 )2 + · · · ]x2

= 1 + [1 + D + D2 + D2 + · · · ]x2

= 1 + [x2 + 2x + 4] = x2 + 2x + 5.

Visit For More : www.LearnEngineering.in


Visit For More : www.LearnEngineering.in

330 Engineering Mathematics - I

The general solution is y = yc + y p


y = c1 e−x + e x (c2 + c3 x) + x2 + 2x + 5.

Example 5.38. Solve (D2 + D)y = x2 + 2x + 4. [Dec 2008]


Solution. The A.E is

.in
m2 + m = 0 ⇒ m(m + 1) = 0 ⇒ m = 0, −1.
yc = c1 + c2 e−x .

ng
1 1
yp = (x2 + 2x + 4) = (x2 + 2x + 4)
D2 + D D(1 + D)

eri
1
= (1 + D)−1 (x2 + 2x + 4)
D
1
= [1 − D + D2 − · · · ](x2 + 2x + 4)
D
1
= [x2 + 2x + 4 − (2x + 2) + 2] e
gin
D
1
= [x2 + 4]
D
Z
(x2 + 4)dx
En

x3
= + 4x.
3
arn

The Solution is y = yc + y p
Le

x3
y = c1 + c2 e−x + + 4x.
3
w.

5.2.4 Type IV
ww

Q(x) = eax g(x) where g(x) may be xm or sin ax or cos ax. In this case
1 ax ax g(x)
P.I = e g(x) = e . This will fall into any of the previous three types
f (D) f (D + a)
which can be evaluated by the known methods.

Visit For More : www.LearnEngineering.in


Visit For More : www.LearnEngineering.in

Differential Equations 331

5.2.5 Type V

Q(x) = xm cos ax or xm sin ax. In this case

1 m 1 m
P.I = x cos ax or x sin ax.
f (D) f (D)
1 iax m 1 m iax
= R.P e x or I.P x e .

.in
f (D) f (D)
1 1
= R.P eiax xm or I.P eiax xm
f (D + ia) f (D + ia)

ng
which can be evaluated by earlier methods.
✎ ☞
Worked Examples
✍ ✌

eri
Example 5.39. Find the particular integral of (D2 + 4D + 4)y = xe−2x . [May 2005]
Solution. 1 1
PI =
D2+ 4D + 4
xe−2x =
e
(D + 2)2
xe−2x
gin
1 1
= e−2x 2
x = e−2x 2 x
(D − 2 + 2) D
Z
1 1 1
= e−2x x = e−2x xdx
DD D
En

2 e−2x
! Z
−2x 1 x
=e = x2 dx
D 2 2
e−2x x3 x3 e−2x
arn

= = .
2 3 6

Example 5.40. Find the particular integral of (D2 + 1)y = xe x . [Jun 2003]
1 1 1
Le

Solution. P.I = xe x = e x x = ex 2 x
2
D +1 2
(D + 1) + 1 D + 2D + 1 + 1
1 1
= ex 2 x = ex h 2
ix
D + 2D + 2 2 1 + D +2D
w.

2
#−1
ex D2 + 2D
"
= 1+ x
2 2
ww

ex D2 + 2D ex
" #
= 1− + · · · x = [1 − D · · · ]x
2 2 2
ex
= [x − 1].
2

Visit For More : www.LearnEngineering.in


Visit For More : www.LearnEngineering.in

332 Engineering Mathematics - I

Example 5.41. Find the particular integral of y′′ + 2y′ + 5y = e−x cos 2x.[May 2005]
Solution. P.I = 1 1
2
e−x cos 2x = e−x 2
cos 2x
D + 2D + 5 (D − 1) + 2(D − 1) + 5
1 1
= e−x 2 cos 2x = e−x 2 cos 2x
DZ− 2D + 1 + 2D − 2 + 5 D +4
x x sin 2x
= e−x cos 2xdx = e−x
2 2 2

.in
e−x x sin 2x
= .
4
Example 5.42. Find the particular integral of (D2 − 2D + 2)y = e x cos x. [Dec 2010]

ng
Solution. P.I = 1
e x cos x.
D2 − 2D + 2
1

eri
= ex · 2
cos x.
(D + 1) − 2(D + 1) + 2
1
= ex · 2 ✟+ 1 −✟ ✟− 2
cos x.
D +✟ 2D 2D ✁+2 ✁
= ex · 2
1
cos x. = e x · x e
cos x
gin
D
Z +1 2D
xe x xe x sin x
= cos xdx = .
2 2
Example 5.43. Find the particular integral of (D2 − 2D + 4)y = e x cos x. [Dec 2001]
En

Solution. 1 1
PI = e x cos x = e x cos x
D2 − 2D + 4 2
(D + 1) − 2(D + 1) + 4
arn

1 1
= ex 2 cos x = e x 2 cos x
D + 2D + 1 − 2D − 2 + 4 D +3
cos x e x cos x
= ex = .
−1 + 3 2
Le

Example 5.44. Find the particular integral of (D − 1)2 y = e x sin x.


[May 2003, Jun 2012]
1 1
w.

Solution. PI = e x sin x = e x sin x


(D − 1) 2 (D + 1 − 1)2
1 1 1
= e x 2 sin x = e x sin x
ww

DZ DD
1 1
= ex sin xdx = e x (− cos x)
D D
Z
= −e x cos xdx = −e x sin x.

Visit For More : www.LearnEngineering.in


Visit For More : www.LearnEngineering.in

Differential Equations 333

Example 5.45. Solve (D2 + 5D + 4)y = e−x sin 2x. [Dec 2012, May 2011]
Solution. A.E is m2 + 5m + 4 = 0
(m + 1)(m + 4) = 0 ⇒ m = −1, −4.

yc = c1 e−x + c2 e−4x .
1
e−x sin 2x.

.in
yp =
D2+ 5D + 4
−x 1
=e · 2
sin 2x.
(D − 1) + 5(D − 1) + 4

ng
1
= e−x 2 sin 2x.
D − 2D + 1 + 5D − 5 + 4
1
= e−x · 2 sin 2x.

eri
D + 3D
1
= e−x · sin 2x.
−4 + 3D
= e−x ·
3D + 4
(3D + 4)(3D − 4) e
sin 2x.
gin
3D + 4
= e−x · sin 2x.
9D2 − 16
3 × 2 cos 2x + 4 sin 2x
= e−x
En

9 × (−4) − 16
6 cos 2x + 4 sin 2x
= e−x
−52
e−x
arn

=− (6 cos 2x + 4 sin 2x).


−52

Solution is y = yc + y p
e−x
y = c1 e−x + c2 e−4x −
Le

(6 cos 2x + 4 sin 2x)


52

Example 5.46. Solve (D2 − 2D + 2)y = e2x x2 + 5 + e−2x . [Jun 2009]


w.

Solution. A.E. is

m2 − 2m + 2 = 0 ⇒ (m − 1)2 + 2 − 1 = 0
ww

(m − 1)2 + 1 = 0 ⇒ (m − 1)2 = −1

(m − 1) = ±i ⇒ m = 1 ± i.

Visit For More : www.LearnEngineering.in


Visit For More : www.LearnEngineering.in

334 Engineering Mathematics - I

yc = e x (c1 cos x + c2 sin x).


1
yp = (e2x x2 + 5 + e−2x )
D2
− 2D + 2
1 5 1
= 2 e2x x2 + 2 + 2 e−2x
D − 2D + 2 D − 2D + 2 D − 2D + 2
= PI1 + PI2 + PI3 .

.in
1 1
PI1 = e2x x2 = e2x x2
D2
− 2D + 2 2
(D + 2) − 2(D + 2) + 2
1 1
= e2x 2 x2 = e2x 2 x2

ng
D + 4 + 4D − 2D − 4 + 2 D + 2D + 2
1 e2x  D2 + 2D −1 2
= e2x 2
x 2
= 1 + x
2(1 + D +2D ) 2 2
2

eri
2x 2
!2
e D + 2D D2 + 2D
= [1 − + + . . . ]x2
2 2 2
e2x 2 1
=
2
[x − .2 − 2x + 2]
2 e
gin
e2x 2 e2x 2
= [x − 1 − 2x + 2] = (x − 2x + 1).
2 2
5 5
PI2 = 2 e0x = .
2
En

D − 2D + 2
1 e−2x e−2x
PI3 = 2 e−2x = = .
D − 2D + 2 4+4+2 10
arn

The solution is y = yc + y p
e2x 2 5 e−2x
y = e x (c1 cos x + c2 sin x) + (x − 2x + 1) + + .
2 2 10
d2 y
Le

Example 5.47. Solve − 4y = x sinh x. [Apr 2006]


dx2
Solution. (D2 − 4)y = x sinh x
A.E. is
w.

m2 − 4 = 0 ⇒ m2 = 4 ⇒ m = ±2.
yc = c1 e2x + c2 e−2x
ww

1 1
y p = PI = x sinh x = x(e x − e−x )
D2 − 4 2(D2 − 4)
1 1 1 1
= xe x − xe−x
2 D2 − 4 2 D2 − 4

Visit For More : www.LearnEngineering.in


Visit For More : www.LearnEngineering.in

Differential Equations 335

ex 1 e−x 1
= x − x
2 (D + 1)2 − 4 2 (D − 1)2 − 4
ex 1 e−x 1
= 2
x − 2
x
2 D + 2D + 1 − 4 2 D − 2D + 1 − 4
ex 1 e−x 1
= x − x
2 D2 + 2D − 3 2 D2 − 2D − 3
ex 1 e−x 1
= x− x

.in
2
2 (−3)(1 − D +2D ) 2 (−3)(1 − D2 −2D )
3 3
ex  D2 + 2D −1 e−x  D2 − 2D −1
= 1− x+ 1− x
−6 3 6 3

ng
−e x h D2 + 2D i e−x h D2 − 2D i
= 1+ + ··· x + 1+ + ··· x
6 3 6 3
−e x h 2 i e−x h 2i

eri
P.I. = x+ + x− .
6 3 6 3
The solution is y = yc + y p

y = c1 e2x + c2 e−2x −
ex h
x+ e
2 i e−x h
+ x−
2i
gin
6 3 6 3
d2 y dy e−x
Example 5.48. Solve + 2 + y = . [Jun 2013, May 1989 ]
dx2 dx x2
Solution. A.E is m2 + 2m + 1 = 0 ⇒ (m + 1)2 = 0 ⇒ m = −1, −1.
En

yc = e−x (c1 + c2 x).


1 e−x 1 1
arn

yp = 2 2
= e−x
(D + 1) x (D − 1 + 1) x2
2
! Z
1 1 1
= e−x 2 2 = e−x x−2 dx
D x D
Le

h −1 i Z
−x 1 x −x 1
=e = −e dx
D −1 x
y p = −e−x log x.
w.

The general solution is y = yc + y p


ww

y = e−x (c1 + c2 x) − e−x log x.

Example 5.49. Solve (D4 − 1)y = e x cos x. [Apr 2009]


Solution. A.E is m4 − 1 = 0 ⇒ (m2 − 1)(m2 + 1) = 0

Visit For More : www.LearnEngineering.in


Visit For More : www.LearnEngineering.in

336 Engineering Mathematics - I

⇒ (m − 1)(m + 1)(m2 + 1) = 0 ⇒ m = 1, −1, m2 = −1 ⇒ m = ±i.

yc = C1 e x + C2 e−x + C3 cos x + C4 sin x.


1 1 1
yp = e x cos x = e x cos x = e x 2 cos x
D4−1 4
(D + 1) − 1 (D + 2D + 1)2 − 1
1
= ex 4 cos x
D + 4D + 1 + 4D3 + 2D2 + 4D − 1
2

.in
1
= ex 4 cos x
D + 4D + 6D2 + 4D
3
1 cos x
= ex cos x = e x

ng
1 − 4D − 6 + 4D −5
e x cos x
yp = − .
5

eri
The general solution is y = yc + y p .

e x cos x
e
y = C1 e x + C2 e−x + C3 cos x + C4 sin x −
5
.
gin
Example 5.50. Solve (D + 2)2 y = e−2x sin x. [Jun 2009]
Solution. (D + 2)2 y = e−2x sin x
En

(D2 + 4D + 4)y = e−2x sin x.


A.E. is (m + 2)2 = 0 ⇒ m = −2, −2.
arn

yc = e−2x (c1 + c2 x).


1
yp = e−2x sin x
(D + 2)2
Le

1
= e−2x sin x
(D − 2 + 2)2
1
= e−2x 2 sin x
w.

D
sin x
= e−2x
−1
ww

= −e−2x sin x.

The general solution is y = yc + y p


y = e−2x (c1 + c2 x) − e−2x sin x.

Visit For More : www.LearnEngineering.in


Visit For More : www.LearnEngineering.in

Differential Equations 337

Example 5.51. Solve (D2 + 4D + 3)y = e−x sin x + xe3x . [Jun 2010, Jan 2002]
Solution. A.E is m2 + 4m + 3 = 0 ⇒ (m + 1)(m + 3) = 0 ⇒ m = −1, −3.

yc = c1 e−x + c2 e−3x .
1
yp = (e−x sin x + xe3x )
D2
+ 4D + 3
1 1

.in
= e−x sin x + xe3x
(D + 1)(D + 3) (D + 1)(D + 3)
e−x e3x
= sin x + x

ng
(D − 1 + 1)(D − 1 + 3) (D + 3 + 1)(D + 3 + 3)
e−x e3x
= sin x + x
D(D + 2) (D + 4)(D + 6)

eri
= PI1 + PI2 .
1 1
PI1 = e−x sin x = e−x sin x
D2
+ 2D 2D − 1
= e−x
(2D + 1)
sin x = e−x e
2 cos x + sin x
gin
2
4D − 1 −5
−e−x
= (2 cos x + sin x).
5
1
PI2 = e3x
En

x
(D + 4)(D + 6)
1
= 2 x
D + 10D + 24
arn

e3x 1
=  x
24 1 + D2 +10D
24
e3x  2
D + 10D −1
= 1+ x
Le

24 24
e3x h D2 + 10D  D2 + 10D 2 i
= 1− + − ··· x
24 24 24
w.

e3x h 10 i
= x−
24 24
e3x h 5i
= x− .
ww

24 12

The general solution is y = yc + y p


e−x e3x h 5i
y = c1 e−x + c2 e−3x − (2 cos x + sin x) + x− .
5 24 12

Visit For More : www.LearnEngineering.in


Visit For More : www.LearnEngineering.in

338 Engineering Mathematics - I

Example 5.52. Solve (D2 − 4D + 3)y = e x cos 2x. [Jun 2012]


Solution. A.E. is

m2 − 4m + 3 = 0

(m − 1)(m − 3) = 0

m = 1, 3.

.in
yc = c1 e x + c2 e3x .
1
e x cos 2x.

ng
yp =
D2 − 4D + 3
1
= ex · 2
cos 2x
(D + 1) − 4(D + 1) + 3

eri
1
= ex · 2 cos 2x
D + 2D + 1 − 4D − 4 + 3
1
= ex 2 cos 2x.
D − 2D
e
gin
1
= ex cos 2x
−4 − 2D
e x 1
=− cos 2x
2 D+2
ex
En

D−2
=− cos 2x
2 (D − 2)(D + 2)
e x −2 sin 2x − 2 cos 2x
!
=−
2 −4 − 4
arn

e x
= − 2(sin 2x + cos 2x)
16
ex
= − (sin 2x + cos 2x).
Le

8
Solution is y = yc + y p
ex
= c1 e x + c2 e3x −
w.

(sin 2x + cos 2x).


8
Example 5.53. Solve (D2 − 3D + 2)y = x cos x. [Jan 2007]
ww

Solution. The A.E. is m2 − 3m + 2 = 0 ⇒ (m − 2)(m − 1) = 0 ⇒ m = 1, 2.


yc = C1 e x + C2 e2x .
1 1
yp = x cos x = 2 x( R.P. eix )
D2 − 3D + 2 D − 3D + 2

Visit For More : www.LearnEngineering.in


Visit For More : www.LearnEngineering.in

Differential Equations 339

1
= R.P. eix x
i)2
(D + − 3(D + i) + 2
1
= R.P. eix 2 x
D − 1 + 2Di − 3D − 3i + 2
1
= R.P. eix 2 x
D − 3D + 2Di + 1 − 3i
1
= R.P. eix h ix
D2
+ D(2i−3)

.in
(1 − 3i) 1 + 1−3i 1−3i
eix h D(2i − 3) D2 i−1
= R.P. 1+ + x
1 − 3i 1 − 3i 1 − 3i

ng
eix h D(2i − 3) i
= R.P. 1− x
1 − 3i 1 − 3i
eix h 2i − 3 i

eri
= R.P. x−
1 − 3i 1 − 3i
eix h i
= R.P. x(1 − 3i) − (2i − 3)
(1 − 3i)2

= R.P.
eix h
e
x − 3ix − 2i + 3
i
gin
1 + 9 − 6i
eix h i
= R.P. x + 3 − i(2 + 3x)
10 − 6i
(10 + 6i)eix h
En

i
= R.P. x + 3 − i(2 + 3x)
136
(10 + 6i)(cos x + i sin x)(x + 3 − i(2 + 3x))
= R.P.
136
arn

(10 cos x − 6 sin x + i(10 sin x + 6 cos x))(x + 3 − i(2 + 3x))


= R.P.
136
(10 cos x − 6 sin x)(x + 3) + (10 sin x + 6 cos x)(2 + 3x))
= .
136
Le

The solution is y = yc + y p .
w.

(10 cos x − 6 sin x)(x + 3) + (10 sin x + 6 cos x)(2 + 3x))


y = C1 e x + C2 e2x + .
136
ww

Example 5.54. Solve (D2 + 1)2 y = x2 cos x. [May 2002]


Solution. A.E. is (m2 + 1)2 = 0 ⇒ m2 = −1, −1 ⇒ m = ±i, ±i.

yc = (C1 + C2 x) cos x + (C3 + C4 x) sin x.

Visit For More : www.LearnEngineering.in


Visit For More : www.LearnEngineering.in

340 Engineering Mathematics - I

1 1
yp = x2 cos x = 2 x2 (R.P. eix )
(D2+ 1) 2 (D + 1)2
1
= R.P. eix x2
((D + i)2 + 1)2
1
= R.P. eix 2 x2
(D − 1 + 2Di + 1)2
1
= R.P. eix 2 x2

.in
(D + 2Di)2
1
= R.P. eix  x
2
D 2

2
(2Di) 1 + 2i

ng
D −2
 
1 + 2i
= R.P. eix x2
−4D2

eri
−eix h D D2 i
= R.P. 2
1 − 2 + 3 + · · · x2
4D 2i −4
−e ix h 3D2 i
x2
= R.P.
4D2
1 + iD
e

4
+ · · ·
gin
eix h 2 3 i
= −R.P. x + i2x − 2
4D2 4
eix h 2 3i
= −R.P. x + 2ix −
4D2 2
En

ix
e 1
Z 
3
= −R.P. x2 + 2ix − dx
4 D 2
eix 1  x3 3 
+ ix2 − x
arn

= −R.P.
4 D 3 2
eix
Z  3
x 3 
= −R.P. + ix2 − x dx
4 3 2
ix
e x 4 x 3 2
3x i
Le

h
= −R.P. +i −
4 12 3 4
 cos x + i sin x h x4 x3 3x2 i
= −R.P. +i −
4 12 3 4
w.

1 hh x4 3x2 i x3 i
yp = − − cos x − sin x .
4 12 4 3
ww

The solution is y = yc + y p

1 hh x4 3x2 i x3 i
y = (C1 + C2 x) cos x + (C3 + C4 x) sin x − − cos x − sin x .
4 12 4 3

Visit For More : www.LearnEngineering.in


Visit For More : www.LearnEngineering.in

Differential Equations 341

d2 y dy
Example 5.55. Solve 2
− 2 + y = xe x sin x. [Dec 2013, May 2010]
dx dx
2 2
Solution. A.E. is m − 2m + 1 = 0 ⇒ (m − 1) = 0 ⇒ m = 1, 1.

yc = e x (C1 + C2 x).
1
yp = xe x sin x
(D − 1)2

.in
1
= ex x sin x
(D + 1 − 1)2
1
= e x 2 x sin x

ng
DZ
x1
=e x sin xdx
D

eri
Z
1
= ex xd(− cos x)
D
Z
1h i
= ex
D
− x cos x − − cos xdx
e
gin
1h i
= ex − x cos x + sin x
D
h Z Z i
x
=e − x cos xdx + sin xdx
En

h Z i
x
=e − xd(sin x) − cos x
h ( Z ) i
x
= e − x sin x − sin xdx − cos x
arn

h i
= e x − x sin x − cos x − cos x
h i
= −e x x sin x + 2 cos x .
Le

The solution is y = yc + y p
w.

h i
y = e x (C1 + C2 x) − e x x sin x + 2 cos x .
ww

Example 5.56. Solve (D2 − 4D + 4)y = x2 e2x cos 2x. [Jan 2007]
Solution. A.E. is (m − 2)2 = 0 ⇒ m = 2, 2.

yc = e2x (C1 + C2 x).

Visit For More : www.LearnEngineering.in


Visit For More : www.LearnEngineering.in

342 Engineering Mathematics - I

1 1
yp = 2
x2 e2x cos 2x = e2x x2 cos 2x
(D − 2) (D + 2 − 2)2
Z
1 1
= e2x 2 x2 cos 2x = e2x x2 cos 2xdx
DZ D
1  sin 2x 
= e2x x2 d
D 2
2 Z
2x 1 x sin 2x sin 2x
h i
=e − 2xdx

.in
D 2 2
1 h x2 sin 2x Z i
= e2x − x sin 2xdx
D 2

ng
2 Z
2x 1 x sin 2x
h  − cos 2x i
=e − xd
D 2 2
1 h x2 sin 2x  −x cos 2x Z − cos 2x i

eri
= e2x − − dx
D 2 2 2
1 h x2 sin 2x x cos 2x 1 sin 2x i
= e2x + −
D 2 2 2 2
= e2x
1 h x2 sin 2x x cos 2x sin 2x i
e
gin
+ −
D 2 2 4
e2x h
Z Z Z
2 1 i
= x sin 2xdx + x cos 2xdx − sin 2xdx
2 2
e2x h
Z  − cos 2x  Z  sin 2x  1  − cos 2x i
En

= x2 d + xd −
2 2 2 2 2
e2x h Z Z
cos 2x i
= − x2 cos 2x + cos 2x(2x)dx + x sin 2x − sin 2xdx +
4 2
arn

e2x h
Z  sin 2x  cos 2x cos 2x i
= − x2 cos 2x + 2 xd + x sin 2x + +
4 2 2 2
e2x h Z i
= − x2 cos 2x + x sin 2x − sin 2xdx + x sin 2x + cos 2x
Le

4
e2x h cos 2x i
= − x2 cos 2x + x sin 2x + + x sin 2x + cos 2x
4 2
w.

e2x h 3 i
= − x2 cos 2x + 2x sin 2x + cos 2x
4 2
e2x h i
4x sin 2x + (3 − 2x2 ) cos 2x .
ww

yp =
8
The solution is y = yc + y p
e2x h i
y = e2x (C1 + C2 x) + 4x sin 2x + (3 − 2x2 ) cos 2x .
8

Visit For More : www.LearnEngineering.in


Visit For More : www.LearnEngineering.in

Differential Equations 343

Example 5.57. Solve (D2 − 2D)y = x2 e x cos x. [Jan 2002]


Solution. A.E. is m2 − 2m = 0 ⇒ m(m − 2) = 0 ⇒ m = 0, 2.

yc = C1 + C2 e2x .
1 1
yp = x2 e x cos x = e x x2 cos x
D2 − 2D (D + 1)2 − 2(D + 1)

.in
1 1
= ex 2 x2 cos x = e x 2 x2 cos x
D + 2D + 1 − 2D − 2 D −1
= ex (R.P. eix )x2
D2 − 1

ng
1
= R.P. e x eix x2
(D + i)2 − 1
1

eri
= R.P. e x eix 2 x2
D − 1 + 2Di − 1
1
= R.P. e x eix 2 x2
D − 2 + 2Di
= R.P. e x eix
1
e  x2
gin
 2
(−2) 1 − D +2Di2
x
e e ix  2
D + 2Di −1 2
= -R.P. 1− x
2 2
En

e x eix  D2 + 2Di  D2 + 2Di 2 


= -R.P. 1+ + + · · · x2
2 2 2
e x eix  2 1 
= -R.P. x + (2 + 4ix) + (−2)
2 2
arn

e x eix  2 
= -R.P. x + 1 + 2ix − 2
2
e x eix  2 
= -R.P. x − 1 + 2ix
Le

2
ex  
= -R.P. (cos x + i sin x) x2 − 1 + 2ix
2
−e x
w.

= [cos x(x2 − 1) − 2x sin x]


2
ex
= [(1 − x2 ) cos x + 2x sin x].
2
ww

The solution is y = yc + y p
ex
y = C1 + C2 e2x + [(1 − x2 ) cos x + 2x sin x].
2

Visit For More : www.LearnEngineering.in


Visit For More : www.LearnEngineering.in

344 Engineering Mathematics - I

Example 5.58. Solve (D2 − 4D + 4)y = 8(e2x + sin 2x + x2 ).


Solution. A.E is m2 − 4m + 4 = 0 ⇒ (m − 2)2 = 0 ⇒ m = 2, 2.

yc = e2x (c1 + c2 x).


1
P.I = (8e2x + 8 sin 2x + 8x2 )
(D − 2)2
1 1 1

.in
= 2
8e2x + 2
8 sin 2x + 8x2
(D − 2) (D − 2) (D − 2)2
= PI1 + PI2 + PI3 .

ng
1 2x x2 2x
PI1 = 8e = 8 e = 4x2 e2x .
(D − 2)2 2
1 1

eri
PI2 = 2 8 sin 2x = 8 sin 2x
D − 4D + 4 −4 − 4D + 4
1
= − 8 sin 2x
4DZ
= −2 sin 2xdx =
2 cos 2x
e= cos 2x.
gin
2
1 8 1 D −2 2
PI3 = 8x2 = x2 = 2 1 − x
(D − 2)2 4 1 − D 2 2
2
D D2
En

+ . . . x2

=2 1+2 +3
2 4
2 3 
= 2 x + 2x + 2
4
arn

= 2x2 + 4x + 3.

The general solution is y = yc + y p


Le

y = e2x (c1 + c2 x) + 4x2 e2x + cos 2x + 2x2 + 4x + 3.

Example 5.59. Solve (D2 + 4)2 y = cos 2x.


w.

Solution. The A.E is (m2 + 4)2 = 0 ⇒ m = ±2i, ±2i.


ww

yc = (c1 + c2 x) cos 2x + (c3 + c4 x) sin 2x.


1
yp = cos 2x
(D2
+ 4)2
1
=x 2
cos 2x[ See note on Page 134]
2(D + 4)2D

Visit For More : www.LearnEngineering.in


Visit For More : www.LearnEngineering.in

Differential Equations 345

x 1
= cos 2x
4 D(D2 + 4)
x 1
= 3
cos 2x
4 D + 4D
x x
= cos 2x
4 (3D2 + 4)
x2 cos 2x
!
=

.in
4 −12 + 4
−x2
= cos 2x.
32

ng
The general solution is y = yc + y p
x2
y = (c1 + c2 x) cos 2x + (c3 + c4 x) sin 2x − 32 cos 2x.

eri
An important result to remember
1
Let us evaluate f (x).

Let
1
D−a
f (x) = y. e
gin
D−a
Operating on both sides by D − a we obtain
⇒ (D − a)y = f (x) ⇒ Dy − ay = f (x).
En

This is a linear equation of the first order, where P = −a, Q = f (x).


R R
pdx −adx
I.F. = e =e = e−ax .
The solution is
arn

Z
y × I.F = Q × I.Fdx
Z
ye−ax = f (x)e−ax dx
Le

Z
ax
y=e e−ax f (x)dx.
w.

d2 y
Example 5.60. Solve + a2 y = tan ax.
dx2
Solution. A.E is m2 + a2 = 0 ⇒ m = ±ai.
ww

yc = c1 cos ax + c2 sin ax.


1 1
yp = tan ax = tan ax
D2 +a2 (D + ai)(D − ai)

Visit For More : www.LearnEngineering.in


Visit For More : www.LearnEngineering.in

346 Engineering Mathematics - I

1 h 1 1 i
= − tan ax
2ai D − ai D + ai
1 h 1 1 i
= tan ax − tan ax .
2ai D − ai D + ai
1
PI1 = tan ax
D −Zai Z
aix −iax
h 1 ax
i
=e tan axe dx ∵ f (x) = e f (x)e−ax dx
D−a

.in
Z
= eaix tan ax(cos ax − i sin ax)dx

sin2 ax
Z

ng
iax
=e (sin ax − i )dx
cos ax
1 − cos2 ax 
 − cos ax Z
= eiax −i dx

eri
a cos ax
 − cos ax i sin ax 
= eiax − log(sec ax + tan ax) + i
a a a
−eiax 
=
a e
cos ax − i sin ax + i log(sec ax + tan ax)

gin
−1  iax −iax 
= e e + ieiax log(sec ax + tan ax)
a
−1  
= 1 + ieiax log(sec ax + tan ax) .
a
En

changing i into −i we get


1 −1  
tan ax = 1 − ie−iax log(sec ax + tan ax)
arn

D + ai a
−1 1 1 1
yp = (1 + ieiax log(sec ax + tan ax)) + (1 − ie−iax log(sec ax + tan ax))
2ai a 2ai a
−1 1
Le

= 2 (−i + eiax log(sec ax + tan ax)) + 2 (−i − e−iax log(sec ax + tan ax))
2a 2a
1 h iax
i
= 2 (i − e log(sec ax + tan ax) − i − e−iax log(sec ax + tan ax))
2a
w.

1
= − 2 [(cos ax + i sin ax) log(sec ax + tan ax)
2a
+ (cos ax − i sin ax) log(sec ax + tan ax)]
ww

−1
= log(sec ax + tan ax)2 cos ax
2a2
−1
y p = 2 cos ax log(sec ax + tan ax).
a

Visit For More : www.LearnEngineering.in


Visit For More : www.LearnEngineering.in

Differential Equations 347

Solution is y = yc + y p .

−1
y = c1 cos ax + c2 sin ax + cos ax log(sec ax + tan ax).
a2

5.3 Linear differential equations with variable coefficients

.in
5.3.1 Cauchy’s homogeneous linear differential equation

The general form of the nth order linear D.E with variable coefficients is

ng
dn y dn−1 y dy
a0 xn n
+ a1 xn−1 n−1 + · · · + an−1 x + an y = Q(x)
dx dx dx

eri
where a0 , a1 , . . . , an are constants with a0 , 0.
This can be reduced to a linear D.E with constant coefficients as follows.
Let x = ez , ⇒ z = log x.
e
gin
dy dy dz dy 1
= =
dx dz dx dz x
dy dy
x =
dx dz
En

d2 y d  dy  d  1 dy 
= =
dx2 dx dx dx x dz
arn

−1 dy 1 d  dy 
= 2 +
x dz x dx dz
−1 dy 1 d  dy  dz
=− 2 +
x dz x dz dz dx
Le

−1 dy 1 d2 y 1
=− 2 +
x dz x dz2 x
−1 dy 1 d2 y
w.

=− 2 + 2 2
x dz x dz
d 2y d 2y dy
x2 2 = 2 − .
ww

dx dz dz

d 2 d2
Let θ = ,θ = 2.
dz dz
dy dy
Now, θy = =x = xDy
dz dx

Visit For More : www.LearnEngineering.in


Visit For More : www.LearnEngineering.in

348 Engineering Mathematics - I

Then xD = θ.
d2 y
x2 = θ2 y − θy = θ(θ − 1)y
dx2
x2 D2 y = θ(θ − 1)y.

Similarly x3 D3 = θ(θ − 1)(θ − 2)y etc.

.in
On substitution in (1), it will be reduced to a D.E with constant coefficients, which
can be evaluated as usual.
✎ ☞

ng
Worked Examples
✍ ✌
d2 y dy
Example 5.61. Reduce x2 − 3x + 3y = x into a differential equation with

eri
dx2 dx
constant coefficients. [May 2007]
Solution. The given equation is

(x2 D2 − 3xD + 3)y = x. e (1)


gin
Let x = ez or z = log x.
d
Then, xD = θ and x2 D2 = θ(θ − 1) where θ = .
En

dz
Now (1) gives (θ(θ − 1) − 3θ + 3)y = ez .

i.e, (θ2 − θ − 3θ + 3)y = ez


arn

(θ2 − 4θ + 3)y = ez
d2 y dy
− 4 + 3y = ez ,
Le

dz 2 dz
which is a linear second order differential equation with constant
coefficients.
w.

d2 y dy
Example 5.62. Solve x + = 0. [Jun 2006]
dx2 dx
d2 y dy
ww

Solution. The given equation is x 2 + = 0.


dx dx
Multiplying by x, we get
d2 y dy
x2 2
+x = 0.
dx dx

Visit For More : www.LearnEngineering.in


Visit For More : www.LearnEngineering.in

Differential Equations 349

Let x = ez or z = log x.
d d2 d
Now x = θ, then x2 2 = θ(θ − 1) where θ = .
dx dx dz
The given equation becomes

(θ(θ − 1) + θ) y = 0

(θ2 − θ + θ)y = 0

.in
θ2 y = 0.

The A.E is m2 = 0 =⇒ m = 0, 0

ng
∴ yc = e0z (c1 + c2 z) = c1 + c2 z

eri
yc = c1 + c2 log x.

y p = 0.

The general solution is y = yc + y p


e
gin
y = c1 + c2 log x.
d2 y dy
Example 5.63. Solve x2 2
+ 4x + 2y = 0. [Jun 2013, May 2008]
dx dx
En

Solution. Let x = ez or z = log x.


d d2 d
Let x = θ. Then, x2 2 = θ(θ − 1) where θ = .
dx dx dz
The given equation is reduced to
arn

(θ(θ − 1) + 4θ + 2)y = 0
(θ2 − θ + 4θ + 2)y = 0
Le

(θ2 + 3θ + 2)y = 0.
A.E is m2 + 3m + 2 = 0
(m + 1)(m + 2) = 0, m = −1, m = −2.
w.

yc = c1 e−z + c2 e−2z
ww

= c1 e− log x + c2 e−2 log x


−1 ) −2 c1 c2
= c1 elog(x + c2 elog x = c1 x−1 + c2 x−2 = + 2.
x x
y p = 0.

Visit For More : www.LearnEngineering.in


Visit For More : www.LearnEngineering.in

350 Engineering Mathematics - I

The general solution is y = yc + y p


c1 c2
y= + 2.
x x
2
d y dy
Example 5.64. Solve x2 2 − x + y = 0. [Jun 2009, May 2003]
dx dx
Solution. Let x = ez or z = log x.
dy d
Define x = θy where θ = .

.in
dx dz
d2 y
Then, x2 2 = θ(θ − 1)y.
dx

ng
∴ The given equation becomes
θ(θ − 1)y − θy + y = 0
(θ2 − θ − θ + 1)y = 0

eri
(θ2 − 2θ + 1)y = 0
A.E is m2 − 2m + 1 = 0 ⇒ (m − 1)(m − 1) = 0 ⇒ m = 1, 1.

yc = (c1 + c2 z)ez . e
gin
= (c1 + c2 log x)x.

y p = 0.
En

The solution y = yc + y p
i.e., y = (c1 + c2 log x)x.
arn

Example 5.65. Transform the equation x2 y′′ + xy′ = x into a linear differential
equation with constant coefficients. [May 2011, Dec 2011, Jun 2010]
Solution. Let x = ez or z = log x.
Le

d d2 d
Let x = θ. Then, x2 2 = θ(θ − 1) where θ = .
dx dx dz
The given equation is reduced to
w.

(θ(θ − 1) + θ)y = ez

(θ2 − θ + θ)y = ez
ww

θ2 y = ez
d2 y
= ez ,
dz2

Visit For More : www.LearnEngineering.in


Visit For More : www.LearnEngineering.in

Differential Equations 351

which is the required linear equation.

Example 5.66. Reduce the equation (x2 D2 + xD + 1)y = log x into an ordinary
differential equation with constant coefficients. [Dec 2010]
Solution. Let x = ez or z = log x.
d d2
Let x = θ. Then, x2 2 = θ(θ − 1) where θ = d
dz .
dx dx

.in
The given equation is reduced to

(θ(θ − 1) + θ + 1)y = z

ng
(θ2 − θ + θ + 1)y = z

(θ2 + 1)y = z

eri
d2 y
+ y = z,
dz2
which is the required linear equation.
e
gin
2
Example 5.67. Convert (x2 D2 + xD + 7)y = into an equation with constant
x
coefficients. [Dec 2009]
Solution. Let x = ez or z = log x.
En

d d
Define D = ,θ = .
dx dz
d 2
2d
arn

Then x = θ. x = θ(θ − 1).


dx dx2
The given equation is reduced to
2
(θ(θ − 1) + θ + 7)y = = 2e−z
Le

ez
(θ2 − θ + θ + 7)y = 2e−z

(θ2 + 7)y = 2e−z


w.

d2 y
+ 7y = 2e−z ,
dz2
ww

which is the required differential equation with constant coefficients.


2
Example 5.68. Convert (3x2 D2 + 5xD + 7)y = x log x into an equation with constant
coefficients [Dec 2013]

Visit For More : www.LearnEngineering.in


Visit For More : www.LearnEngineering.in

352 Engineering Mathematics - I

Solution. Let x = ez or z = log x.


d d
Let D = ,θ= .
dx dz
d d2
Then, x = θ, x2 2 = θ(θ − 1).
dx dx
The given equation is reduced to
2
(3 · θ(θ − 1) + 5θ + 7)y = ·z

.in
ez
(3θ2 − 3θ + 5θ + 7)y = 2e−z z

(3θ2 + 2θ + 7)y = 2e−z z

ng
d2 y dy
3 2
+ 2 + 7y = 2e−z z,
dz dz

eri
which is the required equation.

Example 5.69. Solve (x2 D2 + xD + 1)y = 0. [Jun 2005, May 2002]


Solution. Let x = ez or z = log x.
e
gin
d d
Define D = ,θ = .
dx dz
d d2
Then, x = θ. x2 2 = θ(θ − 1)
dx dx
En

The given equation is reduced to

(θ(θ − 1) + θ + 1)y = 0
arn

(θ2 − θ + θ + 1)y = 0

(θ2 + 1)y = 0.

A.E is m2 + 1 = 0.
Le

m2 = −1

m = ±i.
w.

yc = e0z (c1 cos z + c2 sin z) = c1 cos(log x) + c2 sin(log x)


ww

y p = 0.

The general solution is y = yc + y p

y = c1 cos(log x) + c2 sin(log x).

Visit For More : www.LearnEngineering.in


Visit For More : www.LearnEngineering.in

Differential Equations 353

Example 5.70. Solve x2 y′′ − xy′ + y = x.


Solution. (x2 D2 − xD + 1)y = x.
Let x = ez or z = log x.
d d
Define D = ,θ = .
dx dz
Then, xD = θ, x2 D2 = θ(θ − 1).

.in
Now, the given equation is reduced to

(θ(θ − 1) − θ + 1)y = ez

ng
(θ2 − θ − θ + 1)y = ez

(θ2 − 2θ + 1)y = ez .

eri
A.E is m2 − 2m + 1 = 0 ⇒ (m − 1)2 = 0 ⇒ m = 1, 1.

e
yc = ez (c1 + c2 z) = x(c1 + c2 log x).
gin
1 z 1 z z2 z
yp = e = e = e
θ2 − 2θ + 1 (θ − 1)2 2
x(log x)2
=
En

2
The general solution isy = yc + y p
x
i.e., y = x(c1 + c2 log x) + (log x)2 .
arn

d2 y dy
Example 5.71. Solve x2 2
+ 4x + 2y = x log x. [Dec 2006, May 2003]
dx dx
Le

Solution. Let x = ez , or z = log x.


d d
Define D = ,θ = .
dx dz
Then, xD = θ, x2 D2 = θ(θ − 1).
w.

Now, the given equation is reduced to


ww

(θ(θ − 1) + 4θ + 2)y = ez z

(θ2 − θ + 4θ + 2)y = ez z

(θ2 + 3θ + 2)y = ez z.

Visit For More : www.LearnEngineering.in


Visit For More : www.LearnEngineering.in

354 Engineering Mathematics - I

A.E is, m2 + 3m + 2 = 0 ⇒ (m + 1)(m + 2) = 0 ⇒ m = −1, −2.

c1 c2
yc = c1 e−z + c2 e−2z = + 2.
x x
1 1
yp = ez z = ez z
θ2
+ 3θ + 2 2
(θ + 1) + 3(θ + 1) + 2
1 1
= ez 2 z = ez 2 z
θ + 2θ + 1 + 3θ + 3 + 2 θ + 5θ + 6

.in
1 ez  5θ + θ2 −1
= ez  2
 z = 1 + z
6 1 + 5θ+θ 6 6
6

ng
ez 5θ + θ2  ez  5θ 
= 1− + ··· z = 1− z
6 6 6 6
z
e  5θ  e  z 5  x 5
z − .1 = log x − .

eri
= z− z =
6 6 6 6 6 6

Solution is y = yc + y p

c1 c2 x 
i.e., y =+ 2 + log x − .
5
e
gin
x x 6 6
 log x 2
Example 5.72. Solve (x2 D2 − xD + 1)y = . [Jun 2005]
x
z
Solution. Let x = e , or z = log x.
En

d d
Define D = ,θ = .
dx dz
Then, xD = θ, x2 D2 = θ(θ − 1).
arn

Now, the given equation is reduced to

(θ(θ − 1) − θ + 1)y = (ze−z )2


Le

(θ2 − θ − θ + 1)y = z2 e−2z

(θ2 − 2θ + 1)y = z2 e−2z


w.

(θ − 1)2 y = z2 e−2z .

A.E is m2 − 2m + 1 = 0 ⇒ (m − 1)2 = 0 ⇒ m = 1, 1.
ww

yc = ez (c1 + c2 z) = x(c1 + c2 log x).


1 1 1
yp = 2
z2 e−2z = e−2z 2
z2 = e−2z z2
(1 − θ) (1 − (θ − 2)) (1 − θ + 2)2

Visit For More : www.LearnEngineering.in


Visit For More : www.LearnEngineering.in

Differential Equations 355

1 2 e−2z z2 e−2z  θ −2 2


= e−2z z = = 1 − z
(3 − θ)2 9 1− θ 2
  9 3
3
e−2z  2 θ2 e−2z  2 2 1
= 1 + θ + 3 + · · · z2 = z + 2z + 2
9 3 9 9 3 3
e−2z  4 2 
= z2 + z +
9 3 3
1  4 2

.in
y p = 2 (log x)2 + log x + .
9x 3 3
The solution is y = yc + y p .

ng
1  4 2
y = x(c1 + c2 log x) + 2 (log x)2 + log x + .
9x 3 3
d2 y

eri
dy 1
Example 5.73. Solve x2 2
+ 4x + 2y = x2 + 2 . [Jun 2013, May 2008]
dx dx x
Solution. Let x = ez , or z = log x.
d d
Define D =
dx
,θ = .
dz
e
gin
Then, xD = θ, x2 D2 = θ(θ − 1).
Now, the given equation is reduced to

(θ(θ − 1) + 4θ + 2)y = e2z + e−2z


En

(θ2 − θ + 4θ + 2)y = e2z + e−2z

(θ2 + 3θ + 2)y = e2z + e−2z .


arn

A.E is m2 + 3m + 2 = 0 ⇒ (m + 1)(m + 2) = 0 ⇒ m = −1, −2.


c1 c2
yc = c1 e−z + c2 e−2z = + 2.
Le

x x
1
yp = (e2z + e−2z )
θ2
+ 3θ + 2
1 1
w.

= e2z + e−2z
(θ + 1)(θ + 2) (θ + 1)(θ + 2)
e2z ze−2z
= +
ww

12 −1
x 2 log x
yp = − 2 .
12 x
c1 c2 x2 log x
Solution is y = yc + y p = + 2+ − 2 .
x x 12 x

Visit For More : www.LearnEngineering.in


Visit For More : www.LearnEngineering.in

356 Engineering Mathematics - I

Example 5.74. Solve (x2 D2 − 3xD + 4)y = x2 given that y(1) = 1, y′ (1) = 0.
Solution. Let x = ez , or z = log x.
d d
Define D = dx , θ = dz .
Then, xD = θ, x2 D2 = θ(θ − 1).
Now, the given equation is reduced to

.in
(θ(θ − 1) − 3θ + 4)y = e2z

(θ2 − θ − 3θ + 4)y = e2z

ng
(θ2 − 4θ + 4)y = e2z

(θ − 2)2 y = e2z .

eri
A.E is m2 − 4m + 4 = 0 ⇒ (m − 2)2 = 0 ⇒ m = 2, 2.

yc = e2z (c1 + c2 z)
e
gin
= x2 (c1 + c2 log x).
1 2z z2 2z (log x)2 2
yp = e = e = x .
(θ − 2)2 2 2
En

Solution is y = yc + y p .

x2
y = x2 (c1 + c2 log x) + (log x)2 .
arn

2
When x = 1, y = 1 which gives c1 = 1
1 x2 1
y′ = x2 c2 + (c1 + c2 log x)2x + (log x)2 x + 2 log x .
Le

x 2 x
When x = 1, y′ = 0. which implies
w.

0 = c2 + 2

c2 = −2
ww

Solution is
x2
y = x2 (1 − 2 log x) + (log x)2 .
2

Visit For More : www.LearnEngineering.in


Visit For More : www.LearnEngineering.in

Differential Equations 357

Example 5.75. Solve the differential equation (x2 D2 − xD + 4)y = x2 sin(log x)


[Jun 2012]
Solution. Let x = ez or z = log x.
d d
Let D = ,θ= .
dx dz
d d2
Then, x = θ, x2 2 = θ(θ − 1).
dx dx

.in
The given equation is reduced to

(θ(θ − 1) − θ + 4)y = e2z sin z

ng
(θ2 − θ − θ + 4)y = e2z sin z

(θ2 − 2θ + 4)y = e2z sin z.

eri
The A.E. is

m2 − 2m + 4 = 0
e
gin
(m − 1)2 + 4 − 1 = 0

(m − 1)2 + 3 = 0
En

(m − 1)2 = −3

m − 1 = ± 3i

arn

m = 1 ± 3i.

√ √
yc = ez (c1 cos3z + c2 sin 3z)
√ √
Le

yc = x(c1 cos 3 log x + c2 sin 3 log x).


1
yp = e2z sin z
θ2− 2θ + 4
w.

2z 1
=e 2
sin z
(θ + 2) − 2(θ + 2) + 4
1
ww

= e2z 2 sin z
θ + 4θ + 4 − 2θ − 4 + 4
1
= e2z 2 sin z
θ + 2θ + 4
1
= e2z sin z
−1 + 2θ + 4

Visit For More : www.LearnEngineering.in


Visit For More : www.LearnEngineering.in

358 Engineering Mathematics - I

1
= e2z sin z
2θ + 3
2θ − 3
= e2z sin z
(2θ − 3)(2θ + 3)
2θ − 3
= e2z 2 sin z
4θ − 9
2 cos z − 3 sin z
= e2z
−4 − 9

.in
−e 2z −x2 
= (2 cos z − 3 sin z) = 2 cos(log x) − 3 sin(log x) .
13 13

ng
Solution is y = yc + y p
√ √ x2 
= x(c1 cos 3 log x + c2 sin 3 log x) − 2 cos(log x) − 3 sin(log x) .
13

eri
Example 5.76. Solve (x2 D2 − 3xD + 4)y = x2 cos log x [Dec 2010]
Solution. Let x = ez or z = log x.
d d
e
gin
Let D = ,θ= .
dx dz
d d2
Then, x = θ, x2 2 = θ(θ − 1).
dx dx
The given equation is reduced to
En

(θ(θ − 1) − 3θ + 4)y = e2z cos z


arn

(θ2 − θ − 3θ + 4)y = e2z cos z

(θ2 − 4θ + 4)y = e2z cos z

(θ − 2)2 y = e2z cos z.


Le

The A.E. is (m − 2)2 = 0

m = 2, 2.
w.

yc = e2z (c1 + c2 z)

= x2 (c1 + c2 log x).


ww

1
yp = e2z cos z
(θ − 2)2
1
= e2z cos z
(θ + 2 − 2)2

Visit For More : www.LearnEngineering.in


Visit For More : www.LearnEngineering.in

Differential Equations 359

1
= e2z cos z
θ2Z
1
= e2z cos zdz
θ
1
= e2z sin z
θZ
= e2z sin zdz

.in
= −e2z cos z = −x2 cos(log x).

Solution is y = yc + y p

ng
= x2 (c1 + c2 log x) − x2 cos(log x).

eri
Example 5.77. Solve (x2 D2 − 2xD − 4)y = x2 + 2 log x. [Jun 2010]
Solution. Let x = ez or z = log x.
d d
Define D =
dx
,θ= .
dz e
gin
d 2
2 d
Then, x = θ, x = θ(θ − 1).
dx dx2
The given equation is now reduced to
En

(θ(θ − 1) − 2θ − 4)y = e2z + 2z.

(θ2 − θ − 2θ − 4)y = e2z + 2z.


arn

(θ2 − 3θ − 4)y = e2z + 2z.

The A.E. is
Le

m2 − 3m − 4 = 0
w.

(m − 4)(m + 1) = 0.

m = −1, 4.
ww

yc = c1 e−z + c2 e4z
c1
= + c2 x4 .
x
1
yp = 2 (e2z + 2z)
θ − 3θ − 4

Visit For More : www.LearnEngineering.in


Visit For More : www.LearnEngineering.in

360 Engineering Mathematics - I

1 2
= e2z + 2 z
θ2 − 3θ − 4 θ − 3θ − 4
e2z 2
= +  z
4 − 6 − 4 −4 1 − θ2 −3θ
4
2z 2
!−1
e 1 θ − 3θ
= − 1− z
−6 2 4
−e2z 1 θ2 − 3θ
!

.in
= − 1+ + ··· z
6 2 4
−e2z 1
" #
3
= − z−

ng
6 2 4
2
" #
−x 1 3
= − log x − .
6 2 4

eri
Solution is y = yc + y p
x2 1
" #
c1 4 3
= + c2 x − − log x − .
x 6 2
e
4
gin
d2 y dy
Example 5.78. Solve (x2 2
+ 4x + 2y) = e x
dx dx
(or)
En

d2 y dy log x
x2 2
+ 4x + 2y = ee .
dx dx
[Dec 2013]
arn

Solution. Let x = ez , or z = log x.


d d
Define D = ,θ = .
dx dz
Then, xD = θ, x2 D2 = θ(θ − 1).
Le

Now, the given equation is reduced to


z
(θ(θ − 1) + 4θ + 2)y = ee
w.

z
(θ2 − θ + 4θ + 2)y = ee
z
(θ2 + 3θ + 2)y = ee .
ww

A.E is m2 + 3m + 2 = 0 ⇒ (m + 1)(m + 2) = 0 ⇒ m = −1, −2.

yc = (c1 e−z + c2 e−2z )


c1 c2
= + 2.
x x

Visit For More : www.LearnEngineering.in


Visit For More : www.LearnEngineering.in

Differential Equations 361

!
1 ez (θ + 2) − (θ + 1) ez 1 1 z
yp = e = e = − ee .
(θ + 1)(θ + 2) (θ + 1)(θ + 2) θ+1 θ+2
1 ez 1 ez
= e − e
θ+1 θ+2
y p = PI1 − PI2 .
1 ez 1 z
PI1 = e = ee
θ+1 θ − (−1)

.in
Z Z
z ez z
=e−z
e e dz = e −z
ee d(ez )
ex

ng
z
= e−z ee = .
x
1 ez 1 z
PI2 = e = ee
θ+2 θ − (−2)

eri
Z Z
2z ez z
=e−2z
e e dz = e −2z
ez ee ez dz.

Let ez = t.
e
gin
ez dz = dt.
Z i Z h
t
PI2 = e −2z
te dt = e td(et ) −2z
En

" Z #
=e−2z
te − e dt = e−2z [tet − et ]
t t

z z 1
= e−2z [ez ee − ee ] = [xe x − e x ]
arn

x2
x − 1 x ex ex
= e = − 2.
x2 x x
ex ex ex ex
Le

Now P.I = PI1 − PI2 = − + 2 = 2.


x x x x
Solution is y = yc + y p .
c1 c2 e x
w.

y= + 2 + 2.
x x x

5.3.2 Legendre’s linear differential equation


ww

The general form of the Legendre’s linear differential equation is

dn y n−1 d
n−1 y dy
(ax + b)n + a1 (ax + b) + · · · + an−1 (ax + b) + an y = Q(x)
dxn dx n−1 dx

Visit For More : www.LearnEngineering.in


Visit For More : www.LearnEngineering.in

362 Engineering Mathematics - I

where a, b, ai ’s are constants.


d
It can be reduced to the previous type by the substitution ez = ax + b and θ = .
dz

⇒ z = log(ax + b)

dy
(ax + b) = aθy ⇒ (ax + b)D = aθ.
dx

.in
d2 y
Then, (ax + b)2 = a2 θ(θ − 1)y ⇒ (ax + b)2 D2 = a2 θ(θ − 1) etc.
dx2

ng
✎ ☞
Worked Examples
✍ ✌

eri
Example 5.79. Transform the equation (2x + 3)2 y′′ − (2x + 3)y′ + 2y = 6x into a
differential equation with constant coefficients. [May 2005]
Solution. Let 2x + 3 = ez or z = log(2x + 3).
Define θ = .
d
e
gin
dz
Then, (2x + 3)D = 2θ and (2x + 3)2 D2 = 4θ(θ − 1).
Now, the given differential equation is reduced to
En

(ez − 3)
(4θ(θ − 1) − 2θ + 2)y = 6
2
(4θ2 − 4θ − 2θ + 2)y = 3(ez − 3)
arn

(4θ2 − 6θ + 2)y = 3(ez − 3)


d2 y dy
4 2
− 6 + 2y = 3(ez − 3),
Le

dz dz

which is the required equation.


w.

d2 y dy
Example 5.80. Transform the equation (2x + 3)2 2
− 2(2x + 3) − 12y = 6x into a
dx dx
differential equation with constant coefficients. [May 2015, Jun 2012]
ww

Solution. Let 2x + 3 = ez or z = log(2x + 3).


d
Define θ = .
dz
d d2
Then, (2x + 3) = 2θ and (2x + 3)2 2 = 4θ(θ − 1).
dx dx

Visit For More : www.LearnEngineering.in


Visit For More : www.LearnEngineering.in

Differential Equations 363

Now, the given differential equation is reduced to


(ez − 3)
(4θ(θ − 1) − 2(2)θ − 12)y = 6
2
(4θ2 − 4θ − 4θ − 12)y = 3(ez − 3)

(4θ2 − 8θ − 12)y = 3(ez − 3)


d2 y

.in
dy
4 − 8 − 12y = 3(ez − 3),
dz2 dz
which is the required equation.

ng
d2 y dy
Example 5.81. Solve (x + 2)2 2
− (x + 2) + y = 3x + 4. [Jun 2008]
dx dx
Solution. Let x + 2 = ez ⇒ z = log(x + 2).

eri
d
Define θ = .
dz
d
Now (x + 2) =θ
dx
d 2
e
gin
and (x + 2)2 2 = θ(θ − 1).
dx
The given equation is now reduced to

(θ(θ − 1) − θ + 1)y = 3(ez − 2) + 4 = 3ez − 2


En

(θ2 − θ − θ + 1)y = 3ez − 2

(θ2 − 2θ + 1)y = 3ez − 2


arn

(θ − 1)2 y = 3ez − 2.

A.E is, (m − 1)2 = 0 ⇒ m = 1, 1.


Le

yc = ez (c1 + c2 z) = (x + 2)(c1 + c2 log(x + 2)).


1
w.

yp = (3ez − 2)
(θ − 1)2
1 1
=3 ez − 2 e0z
ww

(θ − 1) 2 (θ − 1)2
z2
= 3 ez − 2
2
3
y p = (log(x + 2))2 (x + 2) − 2.
2

Visit For More : www.LearnEngineering.in


Visit For More : www.LearnEngineering.in

364 Engineering Mathematics - I

The general solution is y = yc + y p .

3
y = (x + 2)(c1 + c2 log(x + 2)) + (log(x + 2))2 (x + 2) − 2.
2

Example 5.82. Solve (2x + 3)2 y′′ − (2x + 3)y′ − 12y = 6x. [Dec 2009]
d
Solution. Let 2x + 3 = ez and θ =

.in
dz
Now (2x + 3)D = 2θ
and (2x + 3)2 D2 = 4θ(θ − 1).

ng
The given equation is now reduced to

eri
1
(4θ(θ − 1) − 2θ − 12)y = 6 (ez − 3)
2
(4θ2 − 4θ − 2θ − 12)y = 3(ez − 3)

(4θ2 − 6θ − 12)y = 3(ez − 3)


e
gin
2(2θ2 − 3θ − 6)y = 3(ez − 3)
3
(2θ2 − 3θ − 6)y = (ez − 3).
2
En

A.E is, 2m2 − 3m − 6 = 0


arn

3 3 9
2(m2 − m − 3) = 0 ⇒ (m − )2 − 3 − =0
2 4 16


Le

3 2 57 3 57
m− = ⇒m− =±
4 16 4 4
√ √ √
w.

3 57 3 + 57 3 − 57
m= ± ⇒m= , .
4 4 4 4
ww

 √   √ 
3+ 57 3− 57
4 z 4 z
yc = c1 e + c2 e

 √   √ 
3+ 57 3− 57
4 4
yc = c1 (2x + 3) + c2 (2x + 3) .

Visit For More : www.LearnEngineering.in


Visit For More : www.LearnEngineering.in

Differential Equations 365

!
1 3 z 1 3 z 9
PI = 2 (e − 3) = 2 e −
2θ − 3θ − 6 2 2θ − 3θ − 6 2 2
9
3 1
= 2
ez − 2 2 e0z
2 2θ − 3θ − 6 2θ − 3θ − 6
9
3 ez 2
= −
2 2 − 3! − 6 0 − 0 − 6
3 ez 9 3 3

.in
= + = − (2x + 3) + .
2 −7 2(6) 14 4
Solution is y = yc + y p .

ng
 √   √ 
3+ 57
4
3− 57
4
3 3
y = c1 (2x + 3) + c2 (2x + 3) − (2x + 3) + .
14 4

eri
d2 y dy
Example 5.83. Solve (3x + 2)2 + 3(3x + 2) − 36y = 3x2 + 4x + 1. [Jun 2013]
dx2 dx
Solution. Let 3x + 2 = ez or z = log(3x + 2).
Define D =
d
dx
,θ= .
d
dz e
gin
d d2
Then, (3x + 2) = 3θ, (3x + 2)2 2 = 9θ(θ − 1)
dx dx
The given equation is now reduced to
En

!2
ez − 2 ez − 2
!
(9θ(θ − 1) + 3 · 3θ − 36)y = 3 +4 +1
3 3
3 4
(9θ2 − 9θ + 9θ − 36)y = (e2z + 4 − 4ez ) + (ez − 2) + 1
arn

9 3
1
(9θ2 − 36)y = [e2z + 4 − 4ez + 4ez − 8] + 1
3
1
2
9(θ − 4)y = [e2z − 4] + 1
Le

3
1 4 1
(θ2 − 4)y = e2z − +
27 27 9
1 2z 1
w.

= e − .
27 27
The A.E. is m2 − 4 = 0
ww

m2 = 4

m = ±2.

yc = c1 e2z + c2 e−2z

Visit For More : www.LearnEngineering.in


Visit For More : www.LearnEngineering.in

366 Engineering Mathematics - I

c2
= c1 (3x + 2)2 + .
(3x + 2)2
1
yp = (e2z − 1)
27(θ2 − 4)
" #
1 1 2z 1 0z
= · e − · e
27 θ2 − 4 θ2 − 4
" #
1 1 2z 1
= ·e −

.in
27 (θ − 2)(θ + 2) −4
" 2z #
1 ze 1
= +
27 4 4

ng
2
(3x + 2) log(3x + 2) 1
= +
108 108

eri
Solution is y = yc + y p
c2 1 h i
y = c1 (3x + 2)2 + + (3x + 2) 2
log(3x + 2) + 1
(3x + 2)2 108

d2 y e
dy
gin
Example 5.84. Solve (1 + x)2 2
+ (1 + x) + y = 4 cos[log(1 + x)].
dx dx
[Dec 2014, Dec 2011]
Solution. Let 1 + x = ez or z = log(1 + x).
En

d d
Define D = , θ= .
dx dz
d d2
Then, (1 + x) = θ, (1 + x)2 2 = θ(θ − 1).
dx dx
arn

The given equation is now reduced to

(θ(θ − 1) + θ + 1)y = 4 cos z.


Le

(θ2 − θ + θ + 1)y = 4 cos z

(θ2 + 1)y = 4 cos z.


w.

The A.E. is m2 + 1 = 0
ww

m2 = −1

m = ±i.

yc = e0z (c1 cos z + c2 sin z)

Visit For More : www.LearnEngineering.in


Visit For More : www.LearnEngineering.in

Differential Equations 367

= c1 cos log(1 + x) + c2 sin log(1 + x).


4
yp = cos z
θ2
+1
z
= 4 cos z
2θZ
= 2z cos zdz

.in
= 2z sin z

= 2 log(1 + x) sin log(1 + x).

ng
Solution is y = yc + y p

y = c1 cos log(1 + x) + c2 sin log(1 + x) + 2 log(1 + x) sin log(1 + x).

eri
5.4 Method of variation of parameters
e
gin
Consider the equation
d2 y dy
2
+ P(x) + Q(x)y = R(x). (1)
dx dx
En

Let C.F = c1 y1 (x) + c2 y2 (x) where c1 and c2 are arbitrary constants, then y1 (x) and
y2 (x) are two independent solutions of

d2 y dy
arn

+ P(x) + Q(x)y = 0. (2)


dx2 dx

By the method of variation of parameters, y p is evaluated by y p = u(x)y1 + v(x)y2


Le

where u(x) and v(x) are evaluated by the following way.


Define the Wronskian of y1 and y2 by
w.


y1 y2
W = , 0,
y′ y′
1 2
ww

Z Z
y2 R(x) y1 R(x)
Then, u(x) = − dx and v(x) = dx.
W W
Then, the general solution is given by y = yc + y p .
✎ ☞
Worked Examples
✍ ✌

Visit For More : www.LearnEngineering.in


Visit For More : www.LearnEngineering.in

368 Engineering Mathematics - I

Example 5.85. Find the Wronskian of y1 , y2 of y′′ − 2y′ + y = e x log x. [Dec 2012]
Solution. The A.E. is
m2 − 2m + 1 = 0
(m − 1)2 = 0
m = 1, 1.

.in
yc = e x (c1 + c2 x).

∴ y1 = e x , y2 = xe x .

ng
y′1 = e x y′2 = xe x + e x = e x (x + 1).

y1 y2 e x xe x

eri

W[y1 , y2 ] = =
y′ y′ e x e x (x + 1)
1 2

1 x
= e x × e x
1 x + 1

e
gin
= e2x (x + 1 − x) = e2x

d2 y
+ a2 y = sec ax.
En

Example 5.86. Solve [Jun 2012]


dx2
Solution. A.E is m2 + a2 = 0 ⇒ m2 = −a2 ⇒ m = ±ai.
arn

CF = yc = C1 cos ax + C2 sin ax.

y1 = cos ax, y2 = sin ax.

R(x) = sec ax.


Le


y1 y2 cos ax sin ax
W = =
y′ y′ −a sin ax a cos ax
w.

1 2

= a cos2 ax + a sin2 ax = a.
ww

Now, y p = u(x)y1 + v(x)y2 .


Z Z
y2 R(x) sin ax sec ax
u(x) = − dx = − dx
W a
Z
1 1 log(sec ax)
=− tan axdx = −
a a a

Visit For More : www.LearnEngineering.in


Visit For More : www.LearnEngineering.in

Differential Equations 369

1
= − 2 log sec ax.
Za Z
y1 R(x) cos ax sec ax 1
v(x) = dx = dx = x.
W a a
−1 x
Hence, y p = u(x)y1 + v(x)y2 = 2 log sec ax cos ax + sin ax.
a a

The solution is y = yc + y p

.in
cos ax x
y = C1 cos ax + C2 sin ax − 2
log sec ax + sin ax.
a a

ng
d2 y
Example 5.87. Solve by the method of variation of parameters, + y = x sin x.
dx2
[Jun 2010, May 2002]

eri
Solution. A.E is m2 + 1 = 0 ⇒ m2 = −1 ⇒ m = ±i.

yc = c1 cos x + c2 sin x.
e
gin
y1 = cos x, y2 = sin x.

Now, y p = u(x)y1 + v(x)y2 .



y1 y2 cos x sin x
En

W = = = cos2 x + sin2 x = 1.
y′ y′ − sin x cos x
1 2
arn

Z
y2 R(x)
u(x) = − dx
W
Z Z
sin x × x sin xdx
=− = − x sin2 xdx
Le

1
Z Z Z
x(1 − cos 2x) 1
=− dx = − [ xdx − x cos 2xdx]
2 2
1  x2
Z
w.

sin 2x 
=− − xd
2 2 2
1  x2  x sin 2x
Z
sin 2x 
=− − − dx
ww

2 2 2 2
1  x2 x sin 2x cos 2x 
=− − − .
Z2 2 2 Z 4
y1 R(x)
v(x) = dx = cos x × x sin xdx
W

Visit For More : www.LearnEngineering.in


Visit For More : www.LearnEngineering.in

370 Engineering Mathematics - I

Z Z
sin 2x 1 cos 2x 
= x dx = xd −
2 2 2
Z
1  x cos 2x cos 2x  1  x cos 2x sin 2x 
=− − dx = − − .
2 2 2 2 2 4
Hence, y p = u(x)y1 + v(x)y2
1  x2 x sin 2x cos 2x  1  x cos 2x sin 2x 
= − cos x − − − sin x − .

.in
2 2 2 4 2 2 4
Solution is y = yc + y p .
x2 x sin 2x cos 2x
! !

ng
1 1 x cos 2x sin 2x
y = c1 cos x + c2 sin x − cos x − − − sin x − .
2 2 2 4 2 2 4
Example 5.88. Solve by the method of variation of parameters

eri
d2 y
+ y = cosecx cot x. [Dec 2007]
dx2
Solution. A.E is m2 + 1 = 0 ⇒ m2 = ± − 1 ⇒ m = ±i.

yc = c1 cos x + c2 sin x. e
gin
y1 = cos x, y2 = sin x.

y1 y2 cos x sin x
W = = = cos2 x + sin2 x = 1.
En

y′ y′ − sin x cos x
1 2

y p = u(x)y1 + v(x)y2 , R(x) = cosecx cot x.


arn

Z Z
y2 R(x)
u(x) = − dx = − sin xcosecx cot xdx = − log(sin x).
W
Z Z
y1 R(x)
Le

v(x) = dx = cos xcosecx cot xdx


W
cos2 x
Z Z
= 2
dx = cot2 xdx
sin x
w.

Z Z Z
2 2
= (cosec x − 1)dx = cosec xdx − dx = − cot x − x.
ww

y p = u(x)y1 + v(x)y2 = −(log sin x) cos x + (− cot x − x) sin x.

Solution is, y = yc + y p .

y = c1 cos x + c2 sin x − (log sin x) cos x + (− cot x − x) sin x.

Visit For More : www.LearnEngineering.in


Visit For More : www.LearnEngineering.in

Differential Equations 371

Example 5.89. Solve by the method of variation of parameters (D2 + a2 )y = tan ax.
[Dec 2014, Jun 2009]
Solution. A.E is m2 + a2 = 0 ⇒ m = ±ai.

yc = c1 cos ax + c2 sin ax.

.in
y1 = cos ax, y2 = sin ax.

R(x) = tan ax.


ng
y1 y2 cos ax sin ax
W = = = a.
y′ y′ −a sin ax a cos ax
1 2

eri
y p = u(x)y1 + v(x)y2 .
Z Z
y2 R(x) sin ax tan ax
u(x) = − dx = − dx
W a

=−
1
Z
sin2 ax
dx = −
1
Z
e1 − cos2 ax
dx
gin
a cos ax a cos ax
Z Z
1 1
=− sec axdx + cos axdx
a a
1 log(sec ax + tan ax) 1
En

=− + 2 sin ax.
Za a Z a
y1 R(x) cos ax tan ax
v(x) = dx = dx
W a
arn

Z
1 1
= sin axdx = − 2 cos ax.
a a
y p = u(x)y1 + v(x)y2
Le

1 cos ax sin ax
=− 2
(log(sec ax + tan ax) − sin ax) cos ax − .
a a2
w.

Solution is y = yc + y p .

1 cos ax sin ax
y = c1 cos ax + c2 sin ax − (log(sec ax + tan ax) − sin ax) cos ax − .
ww

a2 a2

d2 y
Example 5.90. Solve by the method of variation of parameters 2 + 8y = tan 2x.
dx2
[Dec 2013]

Visit For More : www.LearnEngineering.in


Visit For More : www.LearnEngineering.in

372 Engineering Mathematics - I

Solution. The given equation can be written as

d2 y
!
2 + 4y = tan 2x
dx2
d2 y tan 2x
+ 4y = .
dx2 2

.in
The A.E. is m2 + 4 = 0

m2 = −4

ng
m = ±2i.

eri
yc = c1 cos 2x + c2 sin 2x.

∴ y1 = cos 2x, y2 = sin 2x.


tan 2x
R(x) =
2
.
e
gin

y1 y2 cos 2x sin 2x
W = =
y′ y′ −2 sin 2x 2 cos 2x
1 2

= 2 cos2 2x + 2 sin2 2x
En

= 2(cos2 2x + sin2 2x)

= 2.
arn

y p = u(x)y1 + v(x)y2 .
Z
y2 R(x)
u(x) = − dx
Le

W
Z
sin 2x · tan 2x
=− dx.
2×2
Z
1 sin 2x
w.

=− sin 2x · dx
4 cos 2x
sin2 2x
Z
1
=− dx
ww

4 cos 2x
1 − cos2 2x
Z
1
=− dx
4 cos 2x
Z
1
=− (sec 2x − cos 2x)dx
4

Visit For More : www.LearnEngineering.in


Visit For More : www.LearnEngineering.in

Differential Equations 373

" #
1 log(sec 2x + tan 2x) sin 2x
=− −
4 2 2
−1  
= log(sec 2x + tan 2x) − sin 2x
Z8 Z
y1 R(x) tan 2x
v(x) = dx = cos 2x · dx.
W 2×2
Z
1 sin 2x
= cos 2x · dx

.in
4 cos 2x
Z
1
= sin 2xdx.
4

ng
!
1 cos 2x cos 2x
= − =−
4 2 8
∴ y p = u(x)y1 + v(x)y2

eri
1 cos 2x sin 2x
= − [log(sec 2x + tan 2x) − sin 2x] cos 2x −
8 8
Solution is y = yc + y p
e
gin
1 sin 4x
y = c1 cos 2x + c2 sin 2x − [log(sec 2x + tan 2x) − sin 2x] cos 2x − .
8 16

d2 y dy e−x
En

Example 5.91. Solve the differential equation + 2 + y = by the method


dx2 dx x2
of variation of parameters. [Jun 2013]
Solution. The A.E. is
arn

m2 + 2m + 1 = 0
Le

(m + 1)2 = 0

m = −1, −1.
w.

yc = e−x (c1 + c2 x)

y1 = e−x , y2 = xe−x .
ww

e−x
R(x) = 2
.
x
y1 y2 e−x xe−x
W= =
y′ y′ −e−x −xe−x + e−x
1 2

Visit For More : www.LearnEngineering.in


Visit For More : www.LearnEngineering.in

374 Engineering Mathematics - I


e−x xe−x
=
−e−x e−x (1 − x)

1 x
= e−x × e−x
−1 1 − x

= e−2x (1 − x + x) = e−2x .

.in
y p = u(x)y1 + v(x)y2 .
Z
y2 R(x)
u(x) = − dx

ng
W
xe−x · e−x
Z
=− 2 −2x
dx.
Z x e

eri
1
=− dx = − log x.
x
Z Z −x −x
y1 R(x) e e
v(x) = dx = dx
W
e
x2 e−2x
gin
x−1
Z Z
1 −2 1
= 2
dx = x dx = =− .
x −1 x
Now y p = u(x)y1 + v(x)y2
En

1
= − log x(e−x ) − xe−x
x
= −e−x (log x + 1)
arn

Solution is y = yc + y p

y = e−x (c1 + c2 x) − e−x (log x + 1).


Le

Example 5.92. Solve by the method of variation of parameters the differential


d2 y
equation 2 + y = cosecx. [Dec 2012, May 2011]
dx
w.

Solution. The auxiliary equation is

m2 + 1 = 0
ww

m2 = −1

m = ±i.

yc = c1 cos x + c2 sin x.

Visit For More : www.LearnEngineering.in


Visit For More : www.LearnEngineering.in

Differential Equations 375

y1 = cos x, y2 = sin x.

R(x) = cosecx.

y1 y2 cos x sin x
W = =
y′ y′ − sin x cos x
1 2

= cos2 x + sin2 x = 1.

.in
y p = u(x)y1 + v(x)y2 .
Z
y2 R(x)
Now, u(x) = −

ng
dx
W
Z
= − sin x · cosecxdx.

eri
Z
= − dx = −x.
Z
y1 R(x)
v(x) =
W
dx
e
gin
Z
= cos x · cosecxdx
Z
= cot xdx = log sin x.
En

∴ y p = u(x)y1 + v(x)y2

= −x cos x + log(sin x) · sin x.


arn

Solution is y = yc + y p

y = c1 cos x + c2 sin x − x cos x + sin x log sin x.


Le

5.5 Simultaneous linear differential equations with constant


w.

coefficients
ww

We have seen so far, the method of solving a single differential equation involving
one independent variable x and one dependent variable y. Quite often we come
across linear differential equations in which there will be two or more dependent
variables and a single independent variable. Such equations are known as

Visit For More : www.LearnEngineering.in


Visit For More : www.LearnEngineering.in

376 Engineering Mathematics - I

simultaneous linear equations. In this section, we consider linear differential


equations with one independent variable t and two dependent variables x and y.
We need two differential equations to solve for x and y. Hence, we will be given a
system of two linear differential equations which need not be of the same order.
We shall consider here only first order linear differential equations with constant

.in
coefficients and we consider three types of equations.

5.5.1 Type I

ng
We consider simultaneous equations of the form

eri
dx dy
a1 + b1 y = f (t), a2 + b2 x = g(t).
dt dt

e
First we eliminate one of the dependent variables from the two equations which
gin
results in a second order linear differential equation with constant coefficients in
the other dependent variable and the independent variable t.
✎ ☞
Worked Examples
✍ ✌
En

dy dx
Example 5.93. Solve for x and y if = x, = y. [May 2004]
dt dt
Solution. The given equations are
arn

dy
=x (1)
dt
dx
= y. (2)
dt
Le

Differentiating (2) w. r. t. x we get

d2 x dy
= = x[from (1)]
w.

dt2 dt
d2 x
−x=0
dt2
ww

d
(D2 − 1)x = 0where D = .
dt
A.E is m2 − 1 = 0

m2 = 1

Visit For More : www.LearnEngineering.in


Visit For More : www.LearnEngineering.in

Differential Equations 377

m = ±1

∴ x = c1 et + c2 e−t .

dx
Now from (2) we have y = = c1 et − c2 e−t .
dt
dx dy
Example 5.94. Eliminate y from the system + 2y = − sin t and − 2x = cos t.

.in
dt dt
Solution. Differentiating the first equation w.r.t. t we get

d2 x dy

ng
2
+ 2 = − cos t.
dt dt
d2 x
+ 2[2x + cos t] = − cos t.
dt2

eri
d2 x
+ 4x + 2 cos t + cos t = 0
dt2
d2 x
dt2
+ 4x = −3 cos t,
e
gin
which is the required equation.
dx
Example 5.95. Eliminate y from the simultaneous equations + y = sin t and
En

dt
dy
+ x = cos t.
dt
Solution. Differentiating the first equation w.r.t. t we get
arn

d2 x dy
+ = cos t.
dt2 dt
d2 x
+ cos t − x = cos t.
Le

dt2
d2 x
− x = 0,
dt2
w.

which is the required equation.


dx dy
+ x = t2 .
ww

Example 5.96. Solve − y = t, [May 2011, Jun 2006]


dt dt
Solution. The equations are
dx
− y = t. (1)
dt
and

Visit For More : www.LearnEngineering.in


Visit For More : www.LearnEngineering.in

378 Engineering Mathematics - I

dy
+ x = t2 . (2)
dt
Differentiating (1) w. r. t. t we get
d2 x dy
− =1
dt2 dt
2
d x
− [−x + t2 ] = 1 [ from (2)]
dt2
d2 x
+ x = 1 + t2 .

.in
dt 2
This is a second order differential equation.
Now, A.E is m2 + 1 = 0 ⇒ m = ±i

ng
xc = CF = c1 cos t + c2 sin t.

eri
1
xp = 1 + t2 = (1 + D2 )−1 (1 + t2 )
1 + D2
= (1 − D2 + D4 − · · · )(1 + t2 ) = 1 + t2 − 2 = t2 − 1.

e
Solution is x = xc + x p = c1 cos t + c2 sin t + t2 − 1.
gin
From (1) we get
dx
y= − t = −c1 sin t + c2 cos t + 2t − t
dt
En

y = c2 cos t − c1 sin t + t.
The required solution is x = c1 cos t + c2 sin t + t2 − 1.
arn

y = c2 cos t − c1 sin t + t.
dx dy
Example 5.97. Solve + y = sin t, + x = cos t given that x = 2, y = 0 when t = 0.
dt dt
Le

[Dec 2009]
Solution. The given equations are
dx
+ y = sin t. (1)
w.

dt
dy
+ x = cos t. (2)
dt
Differentiating (1) w.r.t t we get
ww

d2 x dy
+ = cos t
dt2 dt
d2 x
− x + cos t = cos t [from (2)]
dt2

Visit For More : www.LearnEngineering.in


Visit For More : www.LearnEngineering.in

Differential Equations 379

d2 x
− x = 0.
dt2
This is a second order linear differential equation with constant coefficients.
A.E is m2 − 1 = 0 ⇒ m2 = 1 ⇒ m = ±1.
The solution is x = c1 et + c2 e−t .
dx
From (1) we obtain y = − + sin t

.in
dt
i.e., y = c1 et − c2 e−t + sin t.

ng
Given, x = 2 when t = 0

eri
=⇒ c1 + c2 = 2. (3)

y = 0 when t = 0.

=⇒ c1 − c2 = 0.
e (4)
gin
(3) + (4) =⇒ 2c1 = 2 ⇒ c1 = 1 ⇒ c2 = 1.

The solution is x = et + e−t


En

y = et − e−t + sin t.
dx dy
Example 5.98. Solve + y = et , x − = t. [Dec 2012]
arn

dt dt
Solution. The equations are
dx
+ y = et . (1)
dt
Le

dy
x− = t. (2)
dt
w.

Differentiating(1) w.r.t. t we get


d2 x dy
+ = et
dt2
ww

dt
d2 x
+ x − t = et [from (2)]
dt2
d2 x
+ x = t + et .
dt2

Visit For More : www.LearnEngineering.in


Visit For More : www.LearnEngineering.in

380 Engineering Mathematics - I

The auxiliary equation is

m2 + 1 = 0 ⇒ m2 = −1

m = ±i.

xc = c1 cos t + c2 sin t
1
(t + et )

.in
xp =
D2 +1
1 1
= 2 t+ 2 et
D +1 D +1

ng
1
= (1 + D ) t + et
2 −1
2
et
= (1 − D2 + D4 · · · )t +

eri
2
et
=t+ .
2
The solution for x is
e
gin
x = xc + x p
et
.
x = c1 cos t + c2 sin t + t +
2
Substituting in (1) we get
En

dx
y = et −
"dt
et
#
arn

t
= e − −c1 sin t + c2 cos t + 1 +
2
et
= et + c1 sin t − c2 cos t − 1 −
2
Le

et
y = + c1 sin t − c2 cos t − 1.
2
dx dy
Example 5.99. Solve the simultaneous equations + 2y = sin 2t, − 2x = cos 2t.
w.

dt dt
[Jun 2012]
Solution. The given equations are
ww

dx
+ 2y = sin 2t (1)
dt
dy
− 2x = cos 2t. (2)
dt

Visit For More : www.LearnEngineering.in


Visit For More : www.LearnEngineering.in

Differential Equations 381

Differentiating (1) w.r.t.t we get


d2 x dy
2
+ 2 = 2 cos 2t.
dt dt
d2 x
+ 2[2x + cos 2t] = 2 cos 2t. [ f rom(2)]
dt2
d2 x
+ 4x + 2 cos 2t = 2 cos 2t.
dt2

.in
d2 x
+ 4x = 0.
dt2

ng
The auxiliary equation is

m2 + 4 = 0 ⇒ m2 = −4

eri
m = ±2i.

xc = c1 cos 2t + c2 sin 2t.

x p = 0.
e
gin
The solution for x is
x = xc + x p = c1 cos 2t + c2 sin 2t.
En

Substituting in (1) we get


dx
2y = sin 2t −
dt
arn

= sin 2t − [−2c1 sin 2t + 2c2 cos 2t]

= sin 2t + 2c1 sin 2t − 2c2 cos 2t.


sin 2t
Le

y= + c1 sin 2t − c2 cos 2t.


2
dx dy
Example 5.100. Solve − y = t and + x = t2 given that x(0) = y(0) = 2.
dt dt
w.

[Dec 2011]
Solution. Refer Example 2.104.
ww

The solution is

x = c1 cos t + c2 sin t + t2 − 1

y = c2 cos t − c1 sin t + t.

Visit For More : www.LearnEngineering.in


Visit For More : www.LearnEngineering.in

382 Engineering Mathematics - I

Given: when t = 0, x = 2
∴ c1 − 1 = 2 ⇒ c1 = 3.
Also, when t = 0, y = 2.
∴ c2 = 2.
The solutions are

.in
x = 3 cos t + 2 sin t + t2 − 1

y = 2 cos t − 3 sin t + t.

ng
dx dy
Example 5.101. Solve + 2y = − sin t, − 2x = cos t given, x = 1 and y = 0 at
dt dt

eri
t = 0.
[Dec 2010]
Solution. The given equations are
e
gin
dx
+ 2y = − sin t (1)
dt
dy
− 2x = cos t. (2)
dt
En

Differentiating (1) w.r.t.t we get

d2 x dy
arn

+ 2 = − cos t
dt2 dt
d2 x
+ 2[2x + cos t] = − cos t [from (2)].
dt2
d2 x
Le

+ 4x + 2 cos t = − cos t.
dt2
d2 x
+ 4x = −3 cos t.
dt2
w.

The auxiliary equation is


ww

m2 + 4 = 0 ⇒ m2 = −4

m = ±2i.

xc = c1 cos 2t + c2 sin 2t.

Visit For More : www.LearnEngineering.in


Visit For More : www.LearnEngineering.in

Differential Equations 383

1
xp = (−3 cos t)
D2 +4
cos t
= −3 ·
−1 + 4
cos t
= −3 = − cos t.
3
∴ The solution for x is

.in
x = xc + x p
x = c1 cos 2t + c2 sin 2t − cos t.

ng
Substituting in (1) we get
dx
2y = − sin t −
dt

eri
= − sin t − [−c1 2 sin 2t + 2c2 cos 2t + sin t]

= − sin t + 2c1 sin 2t − 2c2 cos 2t − sin t

e
= 2c1 sin 2t − 2c2 cos 2t − 2 sin t.
gin
y = c1 sin 2t − c2 cos 2t − sin t.

Given: when t = 0, x = 1.
En

∴ c1 − 1 = 1 ⇒ c1 = 2.
When t = 0, y = 0.
−c2 = 0 ⇒ c2 = 0.
arn

∴ The solutions are


x = 2 cos 2t − cos t
Le

y = 2 sin 2t − sin t.

5.5.2 Type II
w.

We consider a system of first order linear differential equations of the form


ww

dx dx
+ b1 x + c1 y = f (t) and a2
a1 + b2 x + c2 y = g(t).
dt dt
d
Procedure. Replace by D and rewrite the given equations involving D.
dt
Eliminating either x or y from the two equations we obtain a second order

Visit For More : www.LearnEngineering.in


Visit For More : www.LearnEngineering.in

384 Engineering Mathematics - I

differential equation in the other variable for which the solution can be obtained
by earlier methods. Substituting this value in any one of the given equations, we
obtain the solution for the other variable.
✎ ☞
Worked Examples
✍ ✌
dx dy
Example 5.102. Solve = 3x + 8y, = −x − 3y, x(0) = 6, y(0) = −2. [May 2007]

.in
dt dt
d
Solution. Let = D. The given equations are reduced to
dt
dx

ng
= 3x + 8y
dt
(D − 3)x − 8y = 0. (1)

eri
dy
= −x − 3y
dt
(D + 3)y + x = 0. (2)
e
gin
Operating (1) by D + 3 we obtain

(D + 3)(D − 3)x − 8(D + 3)y = 0


En

(D2 − 9)x + 8x = 0

(D2 − 9 + 8)x = 0
arn

(D2 − 1)x = 0

m2 − 1 = 0.

m = ±1.
Le

x =c1 et + c2 e−t .
" #
dx 1 dx
w.

From (1) we have 8y = − 3x =⇒ y = − 3x


dt 8 dt
1n o 1 −1
[c1 et − c2 e−t ] − 3[c1 et + c2 e−t ] = [−2c1 et − 4c2 e−t ] = [c1 et + 2c2 e−t ].
ww

y=
8 8 4
When t = 0, x = 6.

=⇒ c1 + c2 = 6. (3)

Visit For More : www.LearnEngineering.in


Visit For More : www.LearnEngineering.in

Differential Equations 385

When t = 0, y = −2.
c1 + 2c2
⇒ = −2
−4
c1 + 2c2 = 8. (4)
(4) − (3) ⇒ c2 = 2 ⇒ c1 = 4.
Solution is x = 4et + 2e−t
1
y = − [4et + 4e−t ] = −[et + e−t ].

.in
4
dx dy
Example 5.103. Solve + 2x − 3y = t, − 3x + 2y = e2t . [Dec 2014, Jun 2006]

ng
dt dt
d
Solution. Let = D. The given equations are reduced to
dt

eri
(D + 2)x − 3y = t. (1)

−3x + (D + 2)y = e2t . (2)

(1) × 3 =⇒ 3(D + 2)x − 9y = 3t


e (3)
gin
Operating (2) by D + 2 we obtain

−3(D + 2)x + (D + 2)2 y = (D + 2)e2t = 2e2t + 2e2t = 4e2t .


En

(4)

(3) + (4) =⇒ ((D + 2)2 − 9)y = 4e2t + 3t.

(D2 + 4D + 4 − 9)y = 4e2t + 3t


arn

(D2 + 4D − 5)y = 4e2t + 3t.


Le

This is a second order linear differential equation.


A.E is, m2 + 4m − 5 = 0 ⇒ (m + 5)(m − 1) = 0 ⇒ m = −5, m = 1.
w.

C.F. = yc = c1 e−5t + c2 et .
1 1
yp = 4e2t +  3t
D2 2
ww

+ 4D − 5 −5 1 − D +4D
5
e2t 3  D2 + 4D −1
=4 − 1− t
7 5 5
4 3 D2 4 
= e2t − 1 + + D + ··· t
7 5 5 5

Visit For More : www.LearnEngineering.in


Visit For More : www.LearnEngineering.in

386 Engineering Mathematics - I

4 3 4
PI = e2t − t +
7 5 5
4 3 4
y = c1 e−5t + c2 et + e2t − t + .
7 5 5

From (2) we have


dy
3x = + 2y − e2t
dt

.in
" #
4 3 4 3 4
= −5c1 e−5t + c2 et + 2e2t − + 2 c1 e−5t + c2 et + e2t − (t + ) − e2t
7 5 7 5 5
9 6 39
3x = −3c1 e−5t + 3c2 et + e2t − t − .

ng
7 5 25
−5t t 3 2t 2 9
x = −c1 e + c2 e + e − t − .
7 5 5

eri
Example 5.104. Solve (D + 2)x + 3y = 0, 3x + (D + 2)y = 2e2t . [May 2003]
Solution. The given equations are

(D + 2)x + 3y = 0. e (1)
gin
3x + (D + 2)y = 2e2t . (2)
En

Operate (1) by (D + 2) we get

(D + 2)2 x + 3(D + 2)y = 0. (3)


arn

(2) × 3 ⇒ 9x + 3(D + 2)y = 6e2t . (4)

(3) − (4) ⇒ (D + 2)2 − 9 x = −6e2t




D2 + 4D + 4 − 9 x = −6e2t

Le

D2 + 4D − 5 x = −6e2t .

w.

A.E is, m2 + 4m − 5 = 0 ⇒ (m + 5)(m − 1) = 0.

xc = c1 e−5t + c2 et .
ww

e2t
x p = −6
D2 + 4D − 5
e2t −6 2t
= −6 = e .
7 7

Visit For More : www.LearnEngineering.in


Visit For More : www.LearnEngineering.in

Differential Equations 387

The solution for x is

x = xc + x p .
6
x = c1 e−5t + c2 et − e2t .
7

From (1) we have

.in
3y = −(D + 2)x = −Dx − 2x
12 2t 12

ng
= 5c1 e−5t − c2 et + e − 2c1 e−5t − 2c2 et + e2t
7 7
24
3y = 3c1 e−5t − 3c2 et + e2t
7

eri
1 24  8
y = 3c1 e − 3c2 e + e2t = c1 e−5t − c2 et + e2t .
−5t t
3 7 7

Example 5.105. Solve


dx
+ 4x + 3y = t;
dy
e
+ 2x + 5y = e2t . [Dec 2013]
gin
dt dt
Solution. The given equations are

(D + 4)x + 3y = t. (1)
En

2x + (D + 5)y = e2t . (2)


arn

Operate (1) by D + 5, we get

(D + 4)(D + 5)x + 3(D + 5)y = (D + 5)t = 1 + 5t.


Le

(2) × 3 ⇒
6x + 3(D + 5)y = 3e2t .
w.

Subtracting, we get
ww

(D + 4)(D + 5)x − 6x = 1 + 5t − 3e2t

(D2 + 9D + 20 − 6)x = 1 + 5t − 3e2t

(D2 + 9D + 14)x = 1 + 5t − 3e2t .

Visit For More : www.LearnEngineering.in


Visit For More : www.LearnEngineering.in

388 Engineering Mathematics - I

The auxiliary equation is

m2 + 9m + 14 = 0

(m + 2)(m + 7) = 0

m = −2, − 7.

.in
xc = c1 e−2t + c2 e−7t .
1
xp = (1 + 5t − 3e2t )
D2 + 9D + 14

ng
1 3
= 2
(1 + 5t) − 2 e2t
D + 9D + 14 D + 9D + 14
1 3e2t

eri
=  2
 (1 + 5t) −
14 1 + 9D+D 4 + 18 + 14
14
!−1
1 9D + D2 3e2t
= · 1+ (1 + 5t) −
14
e 14 36
gin
2 e2t
!
1 9D + D
= 1− · · · (1 + 5t) −
14 14 12
2t
" #
1 9 e
= 1 + 5t − ·5 −
14 14 12
En

2t
" #
1 45 e
= 1 + 5t − −
14 14 12
e2t
" #
1 14 + 70t − 45
arn

= −
14 14 12
70t − 31 e 2t
= − .
196 12
Le

The solution for x is


x = xc + x p
w.

70t − 31 e2t
x = c1 e−2t + c2 e−7t + − .
196 12
Substituting in (1) we get
ww

3y = t − (D + 4)x.
70t − 31 e2t
!
−2t −7t
= t − (D + 4) c1 e + c2 e + −
196 12

Visit For More : www.LearnEngineering.in


Visit For More : www.LearnEngineering.in

Differential Equations 389

e2t 70t − 31 e2t


!
−2t −7t 70
= t − −2c1 e − 7c2 e + − + 4c1 e−2t + 4c2 e−7t + −
196 6 49 3
70 e2t 70t − 31 e 2t
= t − 2c1 e−2t + 3c2 e−7t − + − + .
196 6 49 3
70t 31 70 e2t
=t− + − − 2c1 e−2t + 3c2 e−7t + .
49 49 196 2
21t 54 e2t
− 2c1 e−2t + 3c2 e−7t +

.in
3y = − + .
49 196 2
e2t
!
1 21t 54 −2t −7t
y= − + − 2c1 e + 3c2 e + .
3 49 196 2

ng
5.5.3 Type III

eri
Here, we consider a system of linear first order differential equations of the form
dx dy
a1 + b1 + c1 x = f (t)
dt dt
dx dye
gin
a2 + b2 + c2 y = g(t).
dt dt
d
As in Type II, replace by D and eliminate any one variable and solve for the
dt
other variable by obtaining a second order linear differential equation and finally
En

on substitution of this value in any one of the equations, we obtain the solution for
the other variable.
arn

dx dy dx dy
Example 5.106. Solve − + 2y = cos 2t, + − 2x = sin 2t. [Dec 2008]
dt dt dt dt
d
Solution. Let = D. The given equations can be written as
dt
Le

Dx − (D − 2)y = cos 2t (1)

(D − 2)x + Dy = sin 2t (2)


w.

Operating (1) by D we get


ww

D2 x − D(D − 2)y = −2 sin 2t (3)

Operating (2) by (D − 2) we obtain

(D − 2)2 x + D(D − 2)y = (D − 2) sin 2t = 2 cos 2t − 2 sin 2t (4)

Visit For More : www.LearnEngineering.in


Visit For More : www.LearnEngineering.in

390 Engineering Mathematics - I

(3)+(4)=⇒

(D2 + (D − 2)2 )x = 2 cos 2t − 4 sin 2t

(D2 + D2 − 4D + 4)x = 2 cos 2t − 4 sin 2t

(2D2 − 4D + 4)x = 2 cos 2t − 4 sin 2t

(D2 − 2D + 2)x = cos 2t − 2 sin 2t.

.in
A.E is, m2 − 2m + 2 = 0

ng
(m − 1)2 + 2 − 1 = 0

(m − 1)2 = −1

eri
m − 1 = ±i ⇒ m = 1 ± i.

e
xc = et (c1 cos t + c2 sin t).
gin
1
xp = (cos 2t − 2 sin 2t)
D2 − 2D + 2
1 1
= 2 cos 2t − 2 2 sin 2t
D − 2D + 2 D − 2D + 2
En

1 1
= cos 2t − 2 sin 2t
−4 − 2D + 2 −4 − 2D + 2
1 1
= cos 2t − 2 sin 2t
arn

−2 − 2D −2 − 2D
−1 1 1
= cos 2t + sin 2t
2 D+1 D+1
−1 (D − 1) (D − 1)
= cos 2t + 2 sin 2t
Le

2 D2 − 1 D! − 1
−1 −2 sin 2t − cos 2t 2 cos 2t − sin 2t
= +
2 −5 −5
w.

!
2 sin 2t + cos 2t 2 cos 2t − sin 2t
= +
−10 −5
−1 −1
= (2 sin 2t + cos 2t + 4 cos 2t − 2 sin 2t) = (5 cos 2t).
ww

10 10
−1
xp = (cos 2t)
2
1
The solution is x = xc + x p = et (c1 cos t + c2 sin t) − (cos 2t).
2

Visit For More : www.LearnEngineering.in


Visit For More : www.LearnEngineering.in

Differential Equations 391

dx
(1) + (2) ⇒ 2 + 2y − 2x = cos 2t + sin 2t
dt
dx
2y = 2x − 2 + cos 2t + sin 2t.
" dt #
t cos 2t
= 2 e (c1 cos t + c2 sin t) −
2
" !#
1 − sin 2t

.in
t t
− 2 e (−c1 sin t + c2 cos t) + e (c1 cos t + c2 sin t) − + cos 2t + sin 2t
2 2
sin 2t
= 2et ((c1 sin t − c2 cos t) +
2

ng
t sin 2t
y = e ((c1 sin t − c2 cos t) + .
4

e eri
gin
En
arn
Le
w.
ww

Visit For More : www.LearnEngineering.in

You might also like